You are on page 1of 416

fiziks

Institute for NET/JRF, GATE, IIT‐JAM, M.Sc. Entrance, JEST, TIFR and GRE in Physics

MATHEMATICAL PHYSICS SOLUTIONS


NET/JRF (JUNE-2011)
Q1. The value of the integral  dz z 2 e z , where C is an open contour in the complex z -plane as
C
lm z
shown in the figure below, is:
5 5
(a) e (b) e  0,1
e e
5 5 C
(c) e (d)  e
e e
Re z
Ans. : (c)  1,0  1,0
Solution: If we complete the contour, then by Cauchy integral theorem
1 1
e   dzz e  0   dzz e    dzz 2 e z   z 2 e z  2 ze z  2e z  1 
1 5
 dzz e
2 z 2 z 2 z

1 C C 1
e
Q2. Which of the following matrices is an element of the group SU 2  ?
1 i 1 
 
 1 1
(a)   (b)  3 3
 0 1  1 1 i 
 
 3 3
 1 3
 
2  i i 
(c)   (d)  2 2 
 3 1 i  3 1 
 
 2 2 
Ans. : (b)
   
Solution: SU 2  is a group defined as following: SU 2   
2 2
 :  ,   C ;     1
    
1 i 1 1 i 1
clearly (b) hold the property of SU 2  .   ,  and   ,  .
3 3 3 3
Note: SU 2  has wide applications in electroweak interaction covered in standard model
of particle physics.
  
Q3. Let a and b be two distinct three dimensional vectors. Then the component of b that is

perpendicular to a is given by
  
(a)
a ba  
(b)
  
b  a b 
(c)
 
a b b  (d)
 
b a a    
a2 b2 b2 a2
Ans. : (a)
H.No. 40-D, Ground Floor, Jia Sarai, Near IIT, Hauz Khas, New Delhi-110016
Phone: 011-26865455/+91-9871145498
Website: www.physicsbyfiziks.com | Email: fiziks.physics@gmail.com
1
fiziks
Institute for NET/JRF, GATE, IIT‐JAM, M.Sc. Entrance, JEST, TIFR and GRE in Physics

 
Solution: a  b  ab sin  nˆ where n̂ is perpendicular to plane containing
 
a and b and pointing upwards. b b sin  kˆ
   
  
a  a  b  ab sin  a  nˆ    a 2 b sin  kˆ
     
b sin  kˆ 
 a  a  b  
 b sin  kˆ 
a  ba
.
  a

a 2
a2
Q4. Let p n  x  (where n  0,1, 2, ...... ) be a polynomial of degree n with real coefficients,
4
defined in the interval 2  n  4 . If  pn  x  pm  x dx   nm , then
2

1 3 1
(a) p 0 x   and p1  x    3  x  (b) p0  x   and p1  x   3 3  x 
2 2 2

1 3 1 3
(c) p0  x   and p1  x   3  x  (d) p 0  x   and p1  x   3  x 
2 2 2 2
Ans. : (d)
Solution: For n not equal to m kroneker delta become zero. One positive and one negative term
1
can make integral zero. So answer may be (c) or (d). Now take n  m  0 so p0  x  
2
and then integrate. (d) is correct option because it satisfies the equation Check by
integration and by orthogonal property of Legendre polynomial also.
Q5. Which of the following is an analytic function of the complex variable z  x  iy in the

domain z  2 ?

(a) 3  x  iy  (b) 1  x  iy  7  x  iy 
7 4 3

(c) 1  x  iy   7  x  iy  (d) x  iy  1
4 3 1/ 2

Ans. : (b)
Solution: Put z  x  iy . If z  x  iy appears in any of the expressions then that expression is
1
non-analytic. For option (d) we have a branch point singularity as the power is which
2
is fractional. Hence only option (b) is analytic.

H.No. 40-D, Ground Floor, Jia Sarai, Near IIT, Hauz Khas, New Delhi-110016
Phone: 011-26865455/+91-9871145498
Website: www.physicsbyfiziks.com | Email: fiziks.physics@gmail.com
2
fiziks
Institute for NET/JRF, GATE, IIT‐JAM, M.Sc. Entrance, JEST, TIFR and GRE in Physics

 1 1 1
Q6. Consider the matrix M  1 1 1
 1 1 1
 
A. The eigenvalues of M are
(a) 0, 1, 2 (b) 0, 0, 3 (c) 1, 1, 1 (d) – 1, 1, 3
Ans. : (b)
1   1 1 
Solution: For eigen values  1 1  1   0
 1 1 1   

1   1   2  1  1    1  11  1     0
1   1  2  2  1      0  2  2  3  22  2  0
3  32  0  2   3  0    0, 0, 3
For any n  n matrix having all elements unity eigenvalues are 0, 0, 0,..., n .
B. The exponential of M simplifies to (I is the 3  3 identity matrix)
 e3  1  M2
(a) e  I  
M
 M (b) e  I  M 
M

 3  2!

(c) e M  I  33 M (d) e M  e  1M


Ans. : (a)
Solution: For e M , let us try to diagonalize matrix M using similarity transformation.

 2 1 1   x1  0

For   3 ,  1  2 1   x 2   0
 1 1  2  x3  0

  2 x1  x2  x 3  0 , x1  2 x 2  x3  0 , x1  x 2  2 x3  0

 3x 2  3x3  0 or x2  x3  x1  x 2  x3  k .

1
1 
Eigen vector is 1 , where k  1 .
3 
1

For   0 ,

H.No. 40-D, Ground Floor, Jia Sarai, Near IIT, Hauz Khas, New Delhi-110016
Phone: 011-26865455/+91-9871145498
Website: www.physicsbyfiziks.com | Email: fiziks.physics@gmail.com
3
fiziks
Institute for NET/JRF, GATE, IIT‐JAM, M.Sc. Entrance, JEST, TIFR and GRE in Physics

1 1 1  x1  0
1 1 1  x  0  x  x  x  0
  2   1 2 3

1 1 1  x3  0

 k1  1
  1  
Let x1  k1 , x2  k2 and x3   k1  k2  . Eigen vector is  k2    1 where k1  k 2  1 .
 k 1  k 2   2
   1 

1
1  
Let x1  k1 , x 2  k 2 and x3  k1  k 2  . Other Eigen vector 0 where k1  1, k 2  1 .
2 
 1

 0 1 1 1  2 1 
S   1 0 1  S  2  1  1  D  S 1 MS , M  SDS 1 .
  1

 1  1 1 1  1 1 

1 0 0 
eM  Se D S 1  e D  0 1 0   e M  1 
e3  1 M  
3
0 0 e 3 

NET/JRF (DEC-2011)
Q7. An unbiased dice is thrown three times successively. The probability that the numbers of
dots on the uppermost surface add up to 16 is
1 1 1 1
(a) (b) (c) (d)
16 36 108 216
Ans. : (b)
Solution: We can get sum of dice as 16 in total six ways i.e. three ways (6, 5, 5) and three ways
(6, 6, 4).
Total number of ways for 3 dice having six faces  6  6  6
6 1
 
6  6  6 36

Q8. The generating function F x, t    Pn  x t n for the Legendre polynomials Pn  x 
n 0


is F x, t   1  2 xt  t 2 
1
2
. The value of P3  1 is
(a) 5 / 2 (b) 3 / 2 (c)  1 (d)  1
H.No. 40-D, Ground Floor, Jia Sarai, Near IIT, Hauz Khas, New Delhi-110016
Phone: 011-26865455/+91-9871145498
Website: www.physicsbyfiziks.com | Email: fiziks.physics@gmail.com
4
fiziks
Institute for NET/JRF, GATE, IIT‐JAM, M.Sc. Entrance, JEST, TIFR and GRE in Physics

Ans. : (d)

Solution: P3 
1
2
  1
2
 1

5 x 3  3x  P3  1  5 1  3 1   5  3  1
3

2
Q9. The equation of the plane that is tangent to the surface xyz  8 at the point 1, 2, 4  is
(a) x  2 y  4 z  12 (b) 4 x  2 y  z  12
(c) x  4 y  2  0 (d) x  y  z  7
Ans. : (b)
Solution: To get a normal at the surface, lets take the gradient

 xyx   yziˆ  zxˆj  kˆxy  8iˆ  4 ˆj  2kˆ

We want a plane perpendicular to this so: r  r0    8iˆ64 4 16ˆj 2k4ˆ  0 .


x 1iˆ   y  2 ˆj  z  4kˆ 8iˆ  4 ˆj  2kˆ  0  4 x  2 y  z  12 .
Q10.    
A 3  3 matrix M has Tr M   6, Tr M 2  26 and Tr M 3  90 . Which of the following
can be a possible set of eigenvalues of M ?
(a) 1,1, 4 (b)  1, 0, 7 (c)  1, 3, 4 (d) 2, 2, 2
Ans. : (c)
  2 2
 
Solution: Tr M 2   1  3  4 also Tr M 3   1  3  4  90 .
2 3 3 3

Q11. Let x1 t  and x2 t  be two linearly independent solutions of the differential equation

d 2x dx t  dx t 
 2  f t x  0 and let wt   x1 t  2  x2 t  1 . If w0   1, then w1 is
dx
2
dt dt dt dt
given by
(a) 1 (b) e 2 (c) 1 / e (d) 1 / e 2
Ans. : (d)
Solution: W t  is Wronskian of D.E.

W  e   e 2t  W 1  e 2 since P  2 .


 Pdt

H.No. 40-D, Ground Floor, Jia Sarai, Near IIT, Hauz Khas, New Delhi-110016
Phone: 011-26865455/+91-9871145498
Website: www.physicsbyfiziks.com | Email: fiziks.physics@gmail.com
5
fiziks
Institute for NET/JRF, GATE, IIT‐JAM, M.Sc. Entrance, JEST, TIFR and GRE in Physics

1 for 2n  x  2n  1
Q12. The graph of the function f  x   
0 for 2n  1  x  2n  2
~
where n  0,1, 2,...... is shown below. Its Laplace transform f s  is
f x 
1  es 1  es
(a) (b)
s s 1
1 1
(c) (d) x
s 1  e  s  s 1  e  s  0 1 2 3 4 5

Ans. : (c)
 1 2 3
Solution: L f  x    e  sx
f  x  dx   e  sx
 1dx   e  sx
 0dx   e  sx  1dx  ......
0 0 1 2

1 3
 e  sx   e  sx 
   0     ...... 
1 s
e 1 
1 3 s

e  e  2 s  ......   
  s 0   s 2 s s


1
s
 1
 
 1  e  s  e 2 s  e 3 s  ........  1  e  s  e  2 s  e 3s  ....
s

a 1 1 
Since S   where r  e  s and a  1  S    .
1 r s  1  e  s 

Q13. The first few terms in the Taylor series expansion of the function f  x   sin x around

x are:
4

1     1   1   
2 3

(a)  
1  x     x     x   .....
2   4  2!  4  3!  4 

1   
2 3
 1  1 
(b) 1
  x     x     x   .....
2   4  2!  4  3!  4 

   1   
3

(c)  x     x   .....
 4  3!  4 

1  x 2 x3 
(d) 1  x   .....
2  2! 3! 

Ans. : (b)
Solution: f  x   sin x

H.No. 40-D, Ground Floor, Jia Sarai, Near IIT, Hauz Khas, New Delhi-110016
Phone: 011-26865455/+91-9871145498
Website: www.physicsbyfiziks.com | Email: fiziks.physics@gmail.com
6
fiziks
Institute for NET/JRF, GATE, IIT‐JAM, M.Sc. Entrance, JEST, TIFR and GRE in Physics

  1    1    1
f   , f     cos  , f      sin  
4 2 4 4 2 4 4 2

1   
2 3
 1  1 
So Taylor’s series is given by 1   x     x     x   .....
2   4  2!  4  3!  4 

NET/JRF (JUNE-2012)

Q14. A vector perpendicular to any vector that lies on the plane defined by x  y  z  5 , is

(a) iˆ  ˆj (b) ˆj  kˆ (c) iˆ  ˆj  kˆ (d) 2iˆ  3 ˆj  5kˆ


Ans. : (c)
   
Solution: Let   x  y  z  5     iˆ  ˆj  kˆ  x  y  z  5  iˆ  ˆj  kˆ .
 x y z 

1 2 3 
 
Q15. The eigen values of the matrix A   2 4 6  are
3 6 9 
 
(a) 1, 4, 9  (b) 0, 7, 7  (c) 0,1,13 (d) 0, 0,14 
Ans. : (d)
1   2 3 
Solution: For eigenvalues A  I  0   2 4 6   0
 3 6 9   

1   4   9     36  229     18  312  34     0


1   4   9     361     49     36  9  0
3  142  0  2   14  0    0, 0, 14 .
1
Q16. The first few terms in the Laurent series for in the region 1  z  2 and
z  1z  2
around z  1 is

1  z  z 2  ....1  z  z  z  ....
2 3
1 1
 z  1  z   1  z   ....
2 3
(a) (b)
2  2 4 8  1 z

1  1 1  2 4 
(d) 2 z  1  5 z  1  7z  1  ....
2 3
(c)
z2 1  z  z 2  .... 1  z  z 2  ....

H.No. 40-D, Ground Floor, Jia Sarai, Near IIT, Hauz Khas, New Delhi-110016
Phone: 011-26865455/+91-9871145498
Website: www.physicsbyfiziks.com | Email: fiziks.physics@gmail.com
7
fiziks
Institute for NET/JRF, GATE, IIT‐JAM, M.Sc. Entrance, JEST, TIFR and GRE in Physics

Ans. : (b)
1 1 1 1 1 1
 1  1  z 
1
Solution:     
z  1z  2 z  2 z  1 1  z  z  1  1 1  z

1 
 1  1  z  
 1 2  1  z 2   1 2  3 1  z 3 ...
     
1 z  2! 3! 


1
1 z

 z  1  z   1  z   ....
2 3

1 2
Q17. Let u  x, y   x  x  y 2  be the real part of analytic function f z  of the complex
2
variable z  x  i y . The imaginary part of f  z  is

(a) y  xy (b) xy (c) y (d) y 2  x 2


Ans. : (a)

Solution: u  x, y   x 
1 2
2
 
x  y 2 , v  x, y   ?

u v u v
Check  and  .
x y y x
u v v
  ,  1 x , v  y  xy  f  x 
x y y
u v v
   y, v  yx  f  y 
y x x

y  xy  f  x   yx  f  y 

If f  x   0, f  y  y
v  xy  y
Q18. Let y  x  be a continuous real function in the range 0 and 2 , satisfying the

d2y dy  
inhomogeneous differential equation: sin x 2
 cos x  x  
dx dx  2
The value of dyldx at the point x   / 2
(a) is continuous (b) has a discontinuity of 3
(c) has a discontinuity of 1/3 (d) has a discontinuity of 1
Ans. : (d)

H.No. 40-D, Ground Floor, Jia Sarai, Near IIT, Hauz Khas, New Delhi-110016
Phone: 011-26865455/+91-9871145498
Website: www.physicsbyfiziks.com | Email: fiziks.physics@gmail.com
8
fiziks
Institute for NET/JRF, GATE, IIT‐JAM, M.Sc. Entrance, JEST, TIFR and GRE in Physics

d2y dy  
Solution: After dividing by sin x , 2
 cot x  cosec x    x  
dx dx  2

dy  dy   
Integrating both sides,   cot x dx   cosec x  x  dx
dx  dx   2
dy
 cot x  y   cosec 2 x  ydx  1
dx

Using Dirac delta property:  f x  x  x   f x  (it lies with the limit).


0 0

dy cos x
 y   y cosec 2 xdx  1 , at x   ; sin x  0 . So this is point of discontinuity.
dx sin x
Q19. A ball is picked at random from one of two boxes that contain 2 black and 3 white and 3
black and 4white balls respectively. What is the probability that it is white?
(a) 34 / 70 (b) 41 / 70 (c) 36 / 70 (d) 29 / 70
Ans. : (b)
Solution: Probability of picking white ball
2 B 3W 3B 4W
3 4
From box I  and from box II 
5 7  
1  3 4  41
Probability of picking a white ball from either of the two boxes is   
2  5 7  70
Q20. The eigenvalues of the antisymmetric matrix,
 0  n3 n2 
 
A   n3 0  n1 
  n2 0 
 n1

where n1 , n 2 and n3 are the components of a unit vector, are


(a) 0, i,  i (b) 0,1,  1 (c) 0,1  i,  1,  i (d) 0, 0, 0
Ans. : (a)
 0 n3 n2   0  n3 n2 
Solution: A   n3 0 n1    A   n3
 T
0  n1 
 n2 n1 0    n2 n1 0 

0    n3 n2 
 A   I   0,  n3 0  n1   0
  n2 n1 0   

H.No. 40-D, Ground Floor, Jia Sarai, Near IIT, Hauz Khas, New Delhi-110016
Phone: 011-26865455/+91-9871145498
Website: www.physicsbyfiziks.com | Email: fiziks.physics@gmail.com
9
fiziks
Institute for NET/JRF, GATE, IIT‐JAM, M.Sc. Entrance, JEST, TIFR and GRE in Physics

 1  0  2    n12  n22  n32  3   n12  n22  n32

but n12  n22  n32  1

so, 1  0 , 2  i , 3  i

A   AT (Antisymmetric). Eigenvalues are either zero or purely imaginary.


Q21. Which of the following limits exists?
 N 1   N 1 
(a) lim    ln N  (b) lim    ln N 
N 
 m 1 m  N 
 m 1 m 
 N
1  N
1
(c) lim    ln N  (d) lim 
N 
 m 1 m  N 
m 1 m

Ans. : (b)
Q22. A bag contains many balls, each with a number painted on it. There are exactly n balls
which have the number n (namely one ball with 1, two balls with 2, and so on until N on
them). An experiment consists of choosing a ball at random, noting the number on it and
returning it to the bag. If the experiment is repeated a large number of times, the average
value the number will tend to
2N  1 N N 1 N  N  1
(a) (b) (c) (d)
3 2 2 2
Ans. : (a)
N  N  1
Solution: Total number of balls 1  2  3  4  .....  N 
2
k
The probability for choosing a k th ball at random 
N  N  1
2
2 k 2 2 N  N  12 N  1
Average of it is given by k  k  P   
N  N  1 N N  1 6
2N 1 N  N  12 N  1
 where k 2  .
3 6

H.No. 40-D, Ground Floor, Jia Sarai, Near IIT, Hauz Khas, New Delhi-110016
Phone: 011-26865455/+91-9871145498
Website: www.physicsbyfiziks.com | Email: fiziks.physics@gmail.com
10
fiziks
Institute for NET/JRF, GATE, IIT‐JAM, M.Sc. Entrance, JEST, TIFR and GRE in Physics

Q23. Consider a sinusoidal waveform of amplitude 1V and frequency f 0 . Starting from an


1
arbitrary initial time, the waveform is sampled at intervals of . If the corresponding
2 f0

Fourier spectrum peaks at a frequency f and an amplitude A , them

(a) f  2 f 0 and A  1V (b) f  2 f 0 and 0  A  1 V


f0 1
(c) f  0 and A  1V (d) f  and A  V
2 2
Ans. : (b)
Solution: y  1sin 2 f 0t  . y
The Fourier transform is:
1 IV
F y    f  f 0     f  f 0 
2
t
1 0 T
In Fourier domain f  f 0 , A  .
2

NET/JRF (DEC-2012)
 a b c 
Q24. The unit normal vector of the point  , ,  on the surface of the ellipsoid
 3 3 3
x2 y 2 z 2
   1 is
a2 b2 c2
bciˆ  caˆj  abkˆ aiˆ  bˆj  ckˆ
(a) (b)
a2  b2  c2 a2  b2  c2

biˆ  cˆj  akˆ iˆ  ˆj  kˆ


(c) (d)
a2  b2  c2 3
Ans. : All the options given are incorrect.
x2 y 2 z 2
Solution: Here     1.
a 2 b2 c2


Unit normal vector is .


H.No. 40-D, Ground Floor, Jia Sarai, Near IIT, Hauz Khas, New Delhi-110016
Phone: 011-26865455/+91-9871145498
Website: www.physicsbyfiziks.com | Email: fiziks.physics@gmail.com
11
fiziks
Institute for NET/JRF, GATE, IIT‐JAM, M.Sc. Entrance, JEST, TIFR and GRE in Physics

      x2 y 2 z 2  2 xiˆ 2 yˆj 2 zkˆ


So,    i  ˆj  kˆ    2  2  2  1  2  2  2
 x y z   a b c  a b c

2 ˆ 2 ˆ 2 ˆ
  a b c 
 i j k
 , ,
 3 3 3
 a 3 b 3 c 3

4 4 4 2 b 2c 2  a 2c 2  a 2c 2
    
3a 2 3b 2 3c 2 3 a 2b 2 c 2
2 ˆ 2 ˆ 2 ˆ
i j k
 a 3 b 3 c 3 bciˆ  caˆj  abkˆ
 
  a b c 
2 b 2c 2  c 2 a 2  a 2b 2 b 2c 2  c 2 a 2  a 2b 2
 , , 
 3 3 3 3 abc

Q25. Given a 2  2 unitary matrix U satisfying U †U  UU †  1 with det U  e i , one can


construct a unitary matrix V V †V  VV †  1 with det V  1 from it by

(a) multiplying U by e i / 2


(b) multiplying any single element of U by e  i
(c) multiplying any row or column of U by e i / 2
(d) multiplying U by e  i
Ans. : (a)
Q26. The graph of the function f x  shown below is best described by
1.00
(a) The Bessel function J 0  x 
0.25
0.50
(b) cos x
0.25
f (x)

0.00
(c) e  x cos x
 0.25
 0.50
1
(d) cos x  0.75
x  1.00
0 1 2 3 4 5 6 7 8 9 10
Ans. : (a) x
Q27. In a series of five Cricket matches, one of the captains calls “Heads” every time when the
toss is taken. The probability that he will win 3 times and lose 2 times is
(a) 1 / 8 (b) 5 / 8 (c) 3 / 16 (d) 5 / 16
Ans. : (d)

H.No. 40-D, Ground Floor, Jia Sarai, Near IIT, Hauz Khas, New Delhi-110016
Phone: 011-26865455/+91-9871145498
Website: www.physicsbyfiziks.com | Email: fiziks.physics@gmail.com
12
fiziks
Institute for NET/JRF, GATE, IIT‐JAM, M.Sc. Entrance, JEST, TIFR and GRE in Physics

3 53 2
1  1 5! 1 1 5!
Solution: P    1      .
2  2 3!5  3! 8  2  3!5  3!
1 5  4  3! 20 5 5
    
32 3! 2! 32  2 8  2 16

The probability of getting exactly k successes in n trials is given by probability mass


n!
p k  1  p  , k  successes, n  trials.
nk
function 
k !n  k !

Q28. The Taylor expansion of the function ln cosh x  , where x is real, about the point x  0
starts with the following terms:
1 2 1 4 1 2 1 4
(a)  x  x  .... (b) x  x  ....
2 12 2 12
1 2 1 4 1 2 1 4
(c)  x  x  .... (d) x  x  ....
2 6 2 6
Ans. : (b)
e x  ex
Solution: cosh x  .Tailor’s series expansion of f  x  about x  a
2
f  a  f ''  a  f '''
f  x  f a   x  a   x  a  x  a   ... . Here a  0 .
2 3

1 ! 2 ! 3 !

e x  ex  1 e x  e x e x  e x
f  x   log    0 , f   x  x 0  x  x   x  tanh x  0
 2  x 0 e e 2 e  e x
2

f ' ' x  
e x
  
 ex e x  ex  e x  ex e x  ex    e x
 ex   e
2 x
 ex 
2

 1 tanh 2 x
e x
e x 2
 e x
e 
x 2

At x  0, f ''  x   1, f '''  x   0, f   x   2,...

1 2 1 4
 f x   x  x  .......
2 12
z 3 dz
Q29. The value of the integral  , where C is a closed contour defined by the
C z 2
 5z  6 
equation 2 z  5  0, traversed in the anti-clockwise direction, is

(a)  16 i (b) 16 i (c) 8 i (d) 2 i


Ans. : (a)
H.No. 40-D, Ground Floor, Jia Sarai, Near IIT, Hauz Khas, New Delhi-110016
Phone: 011-26865455/+91-9871145498
Website: www.physicsbyfiziks.com | Email: fiziks.physics@gmail.com
13
fiziks
Institute for NET/JRF, GATE, IIT‐JAM, M.Sc. Entrance, JEST, TIFR and GRE in Physics

Solution: z 2  5 z  6  0  z 2  2 z  3z  6  0  z  z  2   3 z  2  0  z  3, 2

2 z  5  z  2.5 , only 2 will be inside.

z3 8 z 3 dz
Residue   z  2    8   2  2 i  8  16 i
z  3z  2 z  2 2  3 c z  5z  6

NET/JRF (JUNE-2013)


tn
 H n x 
2
Q30. Given that  e t  2tx
n 0 n!
the value of H 4 0  is
(a) 12 (b) 6 (c) 24 (d) – 6
Ans. : (a)
 
tn tn t4 t6
 H n x   e t  2tx   H n 0   e t  1  t 2  
2 2
Solution:
n 0 n! n 0 n! 2! 3!

H 4 0  4 t 4 4!
 t   H 4 0   12 .
4! 2! 2!
Q31. A unit vector n̂ on the xy -plane is at an angle of 120 o with respect to iˆ . The angle
 
between the vectors u  a iˆ  b nˆ and v  anˆ  b iˆ will be 60 o if

(a) b  3a / 2 (b) b  2a / 3 (c) b  a / 2 (d) b  a


Ans. : (c)

Solution: u  aiˆ  bnˆ , v  anˆ  biˆ

  
 u  v  aiˆ  bnˆ  anˆ  biˆ  u v cos 60  a 2 iˆ  nˆ  ab  ba  b 2 nˆ.iˆ

a 2
 b 2  2ab cos120   cos 60  a
2
2
cos120  2ab  b 2 cos120

 2 2 1
2
1 2
2
2 1
 1
 a  b  2ab    cos 60   a  b  2ab   a 2  b 2      a 2  b 2   2ab
ab
 2 2 2

5ab a
 a2  b2  b .
2 2

H.No. 40-D, Ground Floor, Jia Sarai, Near IIT, Hauz Khas, New Delhi-110016
Phone: 011-26865455/+91-9871145498
Website: www.physicsbyfiziks.com | Email: fiziks.physics@gmail.com
14
fiziks
Institute for NET/JRF, GATE, IIT‐JAM, M.Sc. Entrance, JEST, TIFR and GRE in Physics

Q32. With z  x  iy, which of the following functions f  x, y  is NOT a (complex) analytic
function of z ?

(a) f  x, y    x  iy  8 4  x 2  y 2  2ixy 
3 7

(b) f  x, y    x  iy  1  x  iy 
7 3

(c) f  x, y   x 2  y 2  2ixy  3
5

(d) f  x, y   1  x  iy  2  x  iy 
4 6

Ans. : (d)
Solution: f  x, y   1  x  iy  2  x  iy   1   x  iy  2  x  iy 
4 6 4 6

Due to present of z   x  iy 
Q33. The solution of the partial differential equation
2 2
u  x , t   u  x, t   0
t 2 x 2
satisfying the boundary conditions u 0, t   0  u L, t  and initial conditions

u  x,0   sin x / L  and u x, t  t 0  sin 2x / L  is
t

(a) sin  x / L  cos t / L   sin 2x / L  cos2 t / L 


L
2
(b) 2 sin x / L  cos t / L   sin x / L  cos2 t / L 

(c) sin x / L  cos2 t / L   sin 2x / L sin  t / L 


L

(d) sin x / L  cos t / L   sin 2x / L sin 2 t / L 


L
2
Ans. : (d)
 2u  2u x u 2 x
Solution:  2  0 , u  x,0   sin and  sin
t 2
x L t t 0 L
This is a wave equation
 an t an  t   n x 
So solution is given by u  x, t     An cos  Bn sin  sin  
n  L L   L 

H.No. 40-D, Ground Floor, Jia Sarai, Near IIT, Hauz Khas, New Delhi-110016
Phone: 011-26865455/+91-9871145498
Website: www.physicsbyfiziks.com | Email: fiziks.physics@gmail.com
15
fiziks
Institute for NET/JRF, GATE, IIT‐JAM, M.Sc. Entrance, JEST, TIFR and GRE in Physics

n x n x
L L
2 2
with An   f  x  sin dx, Bn   g  x  sin dx
L0 L an 0 L

 2u  2u x 2 x
Comparing a 2  2 , We have a  1 and f  x   sin , g  x   sin ,
t 2
x L L
 2 x 
 1  cos
x n x x L  dx  2  L  1 (let n  1 )
L L L
2 2 2
An   sin sin dx   sin 2 dx    
L0 L L L0 L L 0 2  L 2
 
2 x n x
L
2
Putting n  2 , Bn  
an 0
sin
L
 sin
L
dx

 4 x 
 1  cos
2 2 x L  dx  2  L  L
L L
2 2
2 0 2 0 
 sin dx   
L 2  2 2 2
 
Q34. The solution of the differential equation
dx
 x2
dt
with the initial condition x0   1 will blow up as t tends to
1
(a) 1 (b) 2 (c) (d) 
2
Ans. : (a)
dx dx x 21 1
Solution:  x 2   2   dt  t C  t C
dt x  2 1 x
1 1 1
 x0  1   0  C  C  1   t 1  x  as t  1 , x blows up
1 x 1 t
1
Q35. The inverse Laplace transforms of is
s s  1
2

1 2 t 1 2
(a) t e (b) t  1  e t
2 2

t 1  e t 
1 2
(c) t 1  e  t (d)
2
Ans. : (c)

H.No. 40-D, Ground Floor, Jia Sarai, Near IIT, Hauz Khas, New Delhi-110016
Phone: 011-26865455/+91-9871145498
Website: www.physicsbyfiziks.com | Email: fiziks.physics@gmail.com
16
fiziks
Institute for NET/JRF, GATE, IIT‐JAM, M.Sc. Entrance, JEST, TIFR and GRE in Physics

 1  t t
  e dt   e t 0   e t  1
1
Solution: f s    f t   e t  L1 
t

s 1  ss  1  0
 1  t
 
   e  1 dt  e  t  
t
 L1  2 t t
 et  t  1 .
 s  s  1  0 0

Q36. The approximation cos   1 is valid up to 3 decimal places as long as  is less than:

(take 180 o /   57.29 o )


(a) 1.280 (b) 1.810 (c) 3.280 (d) 4.010
Ans. : (b)
2 4 2
Solution: cos  1    .......  1 
2! 4! 2!

cos   1 when   1.81o   0.0314
100

NET/JRF -(DEC-2013)

Q37. If A  iˆyz  ˆjxz  kˆxy , then the integral  A  dl (where C is along the perimeter of a
C

rectangular area bounded by x  0, x  a and y  0, y  b ) is

(a) 
1 3
2
a  b3  (b)  ab 2  a 2 b  (c)  a 3  b 3  (d) 0

Ans. : (d)

 A  d l     A.d a  0 since   A  0 .
C S

Q38. If A, B and C are non-zero Hermitian operators, which of the following relations must
be false?
(a) A, B   C (b) AB  BA  C (c) ABA  C (d) A  B  C
Ans. : (a)
Solution:  A, B   C  AB  BA  C  ( AB  BA)†  C †

(( AB)†  ( BA)† )  C †  ( B † A† )  ( A† B † )  C †
Hence A,B and C are hermitian then
BA  AB  C  A, B   C

H.No. 40-D, Ground Floor, Jia Sarai, Near IIT, Hauz Khas, New Delhi-110016
Phone: 011-26865455/+91-9871145498
Website: www.physicsbyfiziks.com | Email: fiziks.physics@gmail.com
17
fiziks
Institute for NET/JRF, GATE, IIT‐JAM, M.Sc. Entrance, JEST, TIFR and GRE in Physics

Q39. Which of the following functions cannot be the real part of a complex analytic function
of z  x  iy ?

(a) x 2 y (b) x 2  y 2 (c) x 3  3 xy 2 (d) 3x 2 y  y  y 3


Ans. : (a)
Solution: Let x 2 y be real part of a complex function. Use Milne Thomson’s method to write
analytic complex function. The real part of that function should be (1) but that is not the
case. So this cannot be real part of an analytic function. Also,
z 2  x  iy   x 2  y 2  2ixy , Real part option (2)
2

z 3   x  iy   x 3  iy 3  3ixy  x  iy 
3

 x 3  iy 3  3ix 2 y  3xy 2 , Real part option (3)


Q40. The expression
 2 2 2 2  1
 2  2  2  2  2
 x
 1 x 2 x3 x 4  1 
 x  x2  x2  x2
2 3 4 
is proportional to
(a)   x1  x 2  x3  x 4  (b)   x1  x 2  x3   x 4 

(c) x12  x 22  x32  x 42  (d) x12  x 22  x32  x 42 


3 / 2 2

Ans. : (b)
   1   2 x1
Solution:   2  
2 
 x1  x1  x 2  x3  x 4  x1  x2  x3  x4 
 2 2 2 2 2 2 2

 2
    
2
2 2 2
 2 x
 1  x2  x3  x4 1  2  2 x1  x1 x12  x22  x32  x42
 2  
x12
 
4
 x12  x22  x32  x42 
 

 2 3 4 1  1 2 3
 x 2  x 2  x 2  x 2 2  4 x 2  8x 2  2 x 2  x 2  x 2  x 2
 2  1  1 4 
 
x1  x 2  x3  x 4
2 2 2 2 3
  
x1  x 2  x3  x 4
2 2 2 2 3

Now similarly solving all and add up then we get
 2 2 2  2  1 1 1 1 
 2  2  2  2  2  2  2  2 
 x1 x 2 x3 x 4  x1 x 2 x3 x 4 

H.No. 40-D, Ground Floor, Jia Sarai, Near IIT, Hauz Khas, New Delhi-110016
Phone: 011-26865455/+91-9871145498
Website: www.physicsbyfiziks.com | Email: fiziks.physics@gmail.com
18
fiziks
Institute for NET/JRF, GATE, IIT‐JAM, M.Sc. Entrance, JEST, TIFR and GRE in Physics

8  x12  x22  x32  x42   8  x12  x22  x32  x42 


 0
 x12  x22  x32  x42 
3

also if all x1 , x2 , x3 , x4 becomes zero it should be infinity.


 2 2 2 2  1
 x  x 2  x 2  x 2    x1     x 2     x3     x 4 

So  2  2  2  2  2
 x1 x 2 x3 x 4  1  2 3 4 
 
dx  dx
Q41. Given that the integral  2
0 y  x
2

2y
, the value of  y
0
2
 x2 
2
is

   
(a) (b) 3
(c) (d)
y 3
4y 8y 3
2 y3
Ans. : (b)
 
1
Solution:  y
dx
 
dx
, pole is of 2nd order at x  iy , residue  1/ 4iy 3  
 x2   
2 2
0
2 2  y  x
2 2

1 1 
Integral     2 i  3 
2 4iy 4 y3  
Q42. The Fourier transform of the derivative of the Dirac  - function, namely   x  , is
proportional to
(a) 0 (b) 1 (c) sin k (d) ik
Ans. : (d)
Solution: Fourier transform of   x 

H K     x e ikx dx  ike k 0   ik


Q43. Consider an n  nn  1 matrix A , in which Aij is the product of the indices i and j

(namely Aij  ij ). The matrix A

(a) has one degenerate eigevalue with degeneracy n  1


(b) has two degenerate eigenvalues with degeneracies 2 and n  2 
(c) has one degenerate eigenvalue with degeneracy n
(d) does not have any degenerate eigenvalue
Ans. : (a)

H.No. 40-D, Ground Floor, Jia Sarai, Near IIT, Hauz Khas, New Delhi-110016
Phone: 011-26865455/+91-9871145498
Website: www.physicsbyfiziks.com | Email: fiziks.physics@gmail.com
19
fiziks
Institute for NET/JRF, GATE, IIT‐JAM, M.Sc. Entrance, JEST, TIFR and GRE in Physics

1 2
Solution: If matrix is 2  2 let   then eigen value is given by
 2 4
1   2 
   0  (1   )(4   )  4  0    0,5
 2 4

 1 2 3
 
If If matrix is 3  3 let  2 4 6  then eigen value is given by
 3 6 9
 

1   2 3 
 
 2 4 6 0
 3 6 9   

1     4    9     36  2 18  2  9     3 12  3  4     0


1     2  13  36  36  2 18  18  2   312  12  3   0
 2  13   3  13 2  13  0   3  14 2  0    0, 0,   14
i.e. has one degenerate eigenvalue with degeneracy 2.
Thus one can generalized that for n dimensional matrix has one degenerate eigevalue
with degeneracy n  1 .
Q44. Three sets of data A, B and C from an experiment, represented by  , and  , are
plotted on a log-log scale. Each of these are fitted with straight lines as shown in the
figure. 1000

100
C
B
10

1 

 A

0.1 1 10 100 1000
0.1

The functional dependence y  x  for the sets A, B and C are respectively


x 1 1
(a) x , x and x 2 (b)  , x and 2 x (c) , x and x 2 (d) , x and x 2
2 x2 x
Ans. : (d)

H.No. 40-D, Ground Floor, Jia Sarai, Near IIT, Hauz Khas, New Delhi-110016
Phone: 011-26865455/+91-9871145498
Website: www.physicsbyfiziks.com | Email: fiziks.physics@gmail.com
20
fiziks
Institute for NET/JRF, GATE, IIT‐JAM, M.Sc. Entrance, JEST, TIFR and GRE in Physics

NET/JRF -(JUNE-2014)
Q45. Consider the differential equation
d 2x dx
2
2 x0
dt dt
with the initial conditions x0   0 and x 0   1 . The solution xt  attains its maximum
value when t is
(a) 1/2 (b) 1 (c) 2 (d) 
Ans. : (b)
d 2x dx
 2  x  0  m 2  2m  1  0   m  1  0  m  1,  1
2
Solution: 2
dt dt
 x   c1  c2 t  e  t , since x  0   0  0  c1  x  c2te t

 x  c2  te t  e  t 

Since x  0   1  1  c2  x  te t

For maxima or minima x  0  x  te t  e t  0  x  e t 1  t 

 e  t  0, 1  t  0  t  , t  1
x  e  t  1  1  t  e  t  1  e  t   t  1 e  t  
 x 1  e 1  0e  t  0
Q46. Consider the matrix
 0 2i 3i 
 
M    2i 0 6i 
  3i  6i 0 
 
The eigenvalues of M are
(a)  5,  2, 7 (b)  7, 0, 7 (c)  4i, 2i, 2i (d) 2, 3, 6
Ans. : (b)
 0 2i 3i   0 2i 3i 
    
Solution: M   2i 0 6i  , M   2i 0 6i 
 3i 6i 0   3i 6i 0 
   
M  M
Matrix is Hermitian so roots are real and trace = 0.
1  2  3  0, 1  2  3  0    7, 0, 7

H.No. 40-D, Ground Floor, Jia Sarai, Near IIT, Hauz Khas, New Delhi-110016
Phone: 011-26865455/+91-9871145498
Website: www.physicsbyfiziks.com | Email: fiziks.physics@gmail.com
21
fiziks
Institute for NET/JRF, GATE, IIT‐JAM, M.Sc. Entrance, JEST, TIFR and GRE in Physics

1
Q47. If C is the contour defined by z  , the value of the integral
2
dz
 C sin 2 z

is
(a)  (b) 2 i (c) 0 (d)  i
Ans. : (c)
1  1
Solution: f  z   2 z  
sin z  2
3 5
z z 1 1
sin z  z   ....  2
 2
3 5 sin z  z3 z5 
 z   .... 
 3 5 
2
1 1  z2 z4  dz
  2 
1   ....   0
2
sin z z  3 5  C sin 2 z

Q48. Given 

n 0

Pn  x t n  1  2 xt  t 2 1 / 2
, for t  1 , the value of P5  1 is

(a) 0.26 (b) 1 (c) 0.5 (d) 1


Ans. : (d)
Solution: Pn  1  1 if n is odd  P5  1  1

Q49. The graph of a real periodic function f  x  for the range  ,   is f x 


shown in the figure.
Which of the following graphs represents the real part of its Fourier x
transform?
Re f k  Re f k 
(a) (b)

k k

(c) Re f k  (d) Re f k 

k k

H.No. 40-D, Ground Floor, Jia Sarai, Near IIT, Hauz Khas, New Delhi-110016
Phone: 011-26865455/+91-9871145498
Website: www.physicsbyfiziks.com | Email: fiziks.physics@gmail.com
22
fiziks
Institute for NET/JRF, GATE, IIT‐JAM, M.Sc. Entrance, JEST, TIFR and GRE in Physics

Ans. : (b)
Solution: This is cosine function
A
f  x   A cos x  F  k     k  k0     k  k0  
2
NET/JRF (DEC-2014)
 
Q50. Let r denote the position vector of any point in three-dimensional space, and r  r .

Then
      
(a)   r  0 and   r  r / r (b)   r  0 and  2 r  0
       
(c)   r  3 and  2 r  r / r 2 (d)   r  3 and   r  0
Ans. : (d)

Solution: r  xxˆ  yyˆ  zzˆ
  x y z
  r      111  3
x y z

xˆ yˆ zˆ
   z y   x z   y x 
  r   / x  / y  / z  xˆ     yˆ     zˆ     0
x y z  y z   z x   x y 

a 0 0 1
   
Q51. The column vector  b  is a simultaneous eigenvector of A   0 1 0  and
a 1 0 0
   

0 1 1
 
B   1 0 1  if
1 1 0
 
(a) b  0 or a  0 (b) b  a or b  2a
(c) b  2a or b  a (d) b  a / 2 or b  a / 2
Ans. : (b)
Solution: Let b  a

 0 0 1  a   a   0 1 1  a  a
         
 0 1 0  a    a  and  1 0 1  a   2  a 
 1 0 0  a   a   1 1 0  a  a
         

H.No. 40-D, Ground Floor, Jia Sarai, Near IIT, Hauz Khas, New Delhi-110016
Phone: 011-26865455/+91-9871145498
Website: www.physicsbyfiziks.com | Email: fiziks.physics@gmail.com
23
fiziks
Institute for NET/JRF, GATE, IIT‐JAM, M.Sc. Entrance, JEST, TIFR and GRE in Physics

Let b  2a

0 0 1 a   a   0 1 1   a   a   a 
           
 0 1 0   2a    2a  and  1 0 1   2a    2a   1 2a 
1 0 0 a   a   1 1 0   a   a   a 
           
For other combination above relation is not possible.
sin 2 x 

Q52. The principal value of the integral  x3
dx is

(a)  2 (b)   (c)  (d) 2


Ans. : (a)
ei 2 z
Solution: Let f  z  
z3
ei 2 z
lim  z  0  f  z   lim  z  0  3  1 finite and  0   z  0 is pole of order 3.
3 3

z 0 z 0 z

1 d2  3 e
i2z

Residue R  lim 2   z  0  3 
 2
2! z  0 dz  z 
 
 f  x  dx   iR   i  2   2 i  Im. Part  2   f  x  dx   2
 

Q53. The Laurent series expansion of the function f  z   e z  e1 / z about z  0 is given by

zn  n 1 1
n n! for all z   
 
(a) (b) n 0
z  n  only if 0  z  1
 z  n!

 1 1 zn
n0  z n  z n n n! only if z  1
 
(c)  for all 0  z   (d)
 n!
Ans. : (c)
 z2   zn 1 1 1 
1
Solution: e  1  z   ....   
z
and e  1  
1/ z
2
 ....   n
 2!  n 0 n ! z 2! z n 0 z n !


 11
 f  z    e z  e1/ z     z n
  , for all 0  z  
n 0 z n  n!

H.No. 40-D, Ground Floor, Jia Sarai, Near IIT, Hauz Khas, New Delhi-110016
Phone: 011-26865455/+91-9871145498
Website: www.physicsbyfiziks.com | Email: fiziks.physics@gmail.com
24
fiziks
Institute for NET/JRF, GATE, IIT‐JAM, M.Sc. Entrance, JEST, TIFR and GRE in Physics

Q54. Two independent random variables m and n , which can take the integer values
0, 1, 2, ...,  , follow the Poisson distribution, with distinct mean values  and 
respectively. Then
(a) the probability distribution of the random variable l  m  n is a binomial distribution.
(b) the probability distribution of the random variable r  m  n is also a Poisson
distribution.
(c) the variance of the random variable l  m  n is equal to   
(d) the mean value of the random variable r  m  n is equal to 0.
Ans. : (c)
Solution:  l2   m2   n2   
1
Q55. Consider the function f z  
ln1  z  of a complex variable
z
z  re i r  0,       . The singularities of f  z  are as follows:
(a) branch points at z  1 and z   ; and a pole at z  0 only for 0    2
(b) branch points at z  1 and z   ; and a pole at z  0 for all  other than 0    2
(c) branch points at z  1 and z   ; and a pole at z  0 for all 
(d) branch points at z  0, z  1 and z   .
Ans. : None of the above is correct

1 1 z 2 z3  z z2
Solution: For f  z   ln 1  z     z    .....   1    .....
z z 2 3  2 3
There is no principal part and when z  0 , f  z   1 . So there is removable singularity

at z  0 . Also z  1 and z   is Branch point.

 1 2 n 1
n

x
Q56. The function f  x      , satisfies the differential equation
n 0 n ! n  1 ! 2 

(a) x
d2 f
2

dx 2
x
df
dx

 x2 1 f  0  (b) x
d2 f
dx
2
2
 2x
df
dx

 x2 1 f  0 

(c) x
d2 f
2

dx 2
x
df
dx

 x2 1 f  0  (d) x
d2 f
dx
2
2
x
df
dx

 x2 1 f  0 
Ans. : (c)

H.No. 40-D, Ground Floor, Jia Sarai, Near IIT, Hauz Khas, New Delhi-110016
Phone: 011-26865455/+91-9871145498
Website: www.physicsbyfiziks.com | Email: fiziks.physics@gmail.com
25
fiziks
Institute for NET/JRF, GATE, IIT‐JAM, M.Sc. Entrance, JEST, TIFR and GRE in Physics

 1  x 
2 n 1
n

Solution: f  x     is generating function (Bessel Function of first kind)
n  0 n ! n  1 !  2 

d2 f df
 x  x 2  n 2 f  0 , put n  1 .
which satisfies the differential equation x 2
dx 2
dx
 
Q57. Let  and  be complex numbers. Which of the following sets of matrices forms a
group under matrix multiplication?
   1 
(a)   (b)   , where   1
0 0   
  *     2 2
(c)  *
 , where  * is real (d)   , where   
*
1
      
*

Ans. : (d)
  2 2
Solution:      1
 
* *

  
 x , p , Lk  (where i j k is the Levi-Civita symbol, x , p, L are
3
Q58. The expression i jk i j
i , j , k 1

the position, momentum and angular momentum respectively, and A, B represents the
Poisson Bracket of A and B ) simplifies to
  
(a) 0 (b) 6 
(c) x , p  L   
(d) x  p
Ans. : (b)
NET/JRF (JUNE-2015)
 dx
Q59. The value of integral  1  x 4

 
(a) (b) (c) 2 (d) 2
2 2
Ans. (a)
 dz
Solution:  1 z4
z R


 2 n 1
Now, pole z  e 4

i
1 1 1 1
n  0,  z0  e 4  i , n  2  z2  i
2 2 2 2

H.No. 40-D, Ground Floor, Jia Sarai, Near IIT, Hauz Khas, New Delhi-110016
Phone: 011-26865455/+91-9871145498
Website: www.physicsbyfiziks.com | Email: fiziks.physics@gmail.com
26
fiziks
Institute for NET/JRF, GATE, IIT‐JAM, M.Sc. Entrance, JEST, TIFR and GRE in Physics

i 3
1 1 1 1
n  1  z1  e 4
 i , n  3  z3   i
2 2 2 2
only z0 and z1 lies in contour
i
  1 1 1 
i.e., residue at  z  e 4    i 
  4 2 2
i 3
  1 1 1 
residue at  z  e 4   i 
  4 2 2
 dx 
now  x 1
4
 2 i Re S 
2
d 2x dx
Q60. Consider the differential equation 2
 3  2 x  0 . If x  0 at t  0 and x  1 at t  1 ,
dt dt
the value of x at t  2 is
(a) e 2  1 (b) e 2  e (c) e  2 (d) 2e
Ans. (b)
Solution: D 2  3D  2  0
 D  1 D  2   0  D  1, 2  x  c1e2t  c2et
using boundary condition x  0, t  0  c1  c2

again using boundary condition x  1, t  1

1 1 e2t 1 t
c2  , c1   x   e
ee 2
e e
2
e  e e  e2
2

again using t  2 then x  e 2  e


Q61. The Laplace transform of 6t 3  3sin 4t is
36 12 36 12
(a)  2 (b)  2
s 4
s  16 s 4
s  16
18 12 36 12
(c)  2 (d)  2
s 4
s  16 s 3
s  16
Ans. (a)
n 1
Solution: L 6t 3  3sin 4t   L t n  
s n 1

H.No. 40-D, Ground Floor, Jia Sarai, Near IIT, Hauz Khas, New Delhi-110016
Phone: 011-26865455/+91-9871145498
Website: www.physicsbyfiziks.com | Email: fiziks.physics@gmail.com
27
fiziks
Institute for NET/JRF, GATE, IIT‐JAM, M.Sc. Entrance, JEST, TIFR and GRE in Physics

a
 L sin at  
 s  a2 
2

6  4 3 4 36 12
L 6t 3  3sin 4t    2  4  2
s 4
s  16 s s  16
2 f 2  f
2
Q62. Let f  x, t  be a solution of the wave equation v in 1 -dimension. If at
t 2 x 2
f
t  0, f  x, 0   e  x and  x, 0   0 for all x , then f  x, t  for all future times t  0 is
2

t
described by

 x 2 v 2t 2 
(b) e  x vt 
2
(a) e
1  x vt 2 3  x  vt 2 1   x vt 2  x  vt  
2
(c) e  e (d) e  e
4 4 2  

Ans. (d)
2 f 2  f
2
Solution: For  v
t 2 x 2
f
 x, 0   0 and f  x, 0   e x
2

t
1 1
 
2
 f  x  vt   f  x  vt   . Therefore, solution is f  e  x vt   e  x  vt 
2
f 
2 2
NET/JRF (DEC-2015)
Q63. In the scattering of some elementary particles, the scattering cross-section  is found to
depend on the total energy E and the fundamental constants h (Planck’s constant) and c
(the speed of light in vacuum). Using dimensional analysis, the dependence of  on
these quantities is given by
2
hc hc  hc  hc
(a) (b) 3 / 2 (c)   (d)
E E E E
Ans. : (c)
Solution: The dimension of  is dimension of “Area”
h  Joul  sec
c  m / sec
E  Joul

H.No. 40-D, Ground Floor, Jia Sarai, Near IIT, Hauz Khas, New Delhi-110016
Phone: 011-26865455/+91-9871145498
Website: www.physicsbyfiziks.com | Email: fiziks.physics@gmail.com
28
fiziks
Institute for NET/JRF, GATE, IIT‐JAM, M.Sc. Entrance, JEST, TIFR and GRE in Physics

2
 hc 
   m dimension of area
2

E
1
Q64. If y  , then x is
tanh  x 

 y 1   y 1  y 1 y 1
(a) ln   (b) ln   (c) ln (d) ln
 y 1   y 1  y 1 y 1
Ans. : (d)
1
Solution: y 
tanh x
e x  e x e2 x  1
y 
e x  e x e2 x  1
ye 2 x  y  e 2 x  1  ye 2 x  e 2 x  1  y  e 2 x  y  1  1  y 
1
 y 1  1  y 1  y  1 2
2 x  ln    x  ln    ln  
 y 1  2  y 1  y 1
z
Q65. The function of a complex variable z has
sin  z 2
(a) a simple pole at 0 and poles of order 2 at  n for n  1, 2,3...

(b) a simple pole at 0 and poles of order 2 at  n and i n for n  1, 2,3...

(c) poles of order 2 at  n , n  0,1, 2,3...


(d) poles of order 2 at  n , n  0,1, 2,3...
Ans. : (b)
z z
Solution: f  z   
sin  z 2
sin  z 2
 z2
 z2
sin  z 2
at z  0 , it is a simple pole since, lim 1
z 0  z2
Also, sin  z 2  sin n   z 2   n , z   n ,  i n

 
2 z
lim z  n . , exists. So its pole of order 2 .
z n sin  z 2

H.No. 40-D, Ground Floor, Jia Sarai, Near IIT, Hauz Khas, New Delhi-110016
Phone: 011-26865455/+91-9871145498
Website: www.physicsbyfiziks.com | Email: fiziks.physics@gmail.com
29
fiziks
Institute for NET/JRF, GATE, IIT‐JAM, M.Sc. Entrance, JEST, TIFR and GRE in Physics


Q66. The Fourier transform of f  x  is f  k    dxeikx f  x  .


If f  x     x      x      x  , where   x  is the Dirac delta-function (and prime

denotes derivative), what is f  k  ?

(a)   i  k  i k 2 (b)    k   k 2

(c)   i  k   k 2 (d) i   k  i k 2
Ans. : (c)

Solution: f  k    dx e   x      x      x  
ikx



   x  e dx  
ikx




 


    x  eikx dx   eikx  x     ike   x  dx   i  k
ikx

  

    x  e dx   k 2
ikx



dx
Q67. The solution of the differential equation  2 1  x 2 , with initial condition x  0 at
dt
t  0 is
   
 sin 2t , 0  t  4 sin 2t , 0  t  2
(a) x   (b) x  
sinh 2t ,   1, 
t t
 4  2

 
sin 2t , 0  t  4
(c) x   (d) x  1  cos 2t , t  0
 1, 
t
 4
Ans. : (c)
dx dx
Solution:  2 1  x2 ,  2dt , sin 1 x  2t  c , x  0, t  0 so, c  0  x  sin 2t
dt 1 x 2

x should not be greater than 1 at x  1

H.No. 40-D, Ground Floor, Jia Sarai, Near IIT, Hauz Khas, New Delhi-110016
Phone: 011-26865455/+91-9871145498
Website: www.physicsbyfiziks.com | Email: fiziks.physics@gmail.com
30
fiziks
Institute for NET/JRF, GATE, IIT‐JAM, M.Sc. Entrance, JEST, TIFR and GRE in Physics

 
1  sin 2t , sin  sin 2t , t  .
2 4
 
sin 2t , 0t 
So, x 4
1 
, t
 4
Q68. The Hermite polynomial H n  x  , satisfies the differential equation

d 2Hn dH n
2
 2x  2nH n  x   0
dx dx
The corresponding generating function

1
G t, x    H n  x  t n , satisfies the equation
n 0 n !

 2G G G  2G G G
(a)  2x  2t 0 (b)  2x  2t 2 0
x 2
x t x 2
x t
 2G G G  2G G  2G
(c)  2x 2 0 (d)  2 x  2 0
x 2
x t x 2 x xt
Ans. : (a)
1 1 1
Solution: G  H n  x  t n , G   H n  x  t n , G   H n  x  t n
n! n! n!
G 1
 H n  x  n t n 1
t n!
Let’s check the options one by one
G G G
 2x  2t 0
x 2
x t
1 1 1
H n  x  t n  2 x H n  x  t n  2t H n  x  nt n 1
n! n! n!

H n  x   2 xH n  x   2 xH n  x   0 , which is Hermite Differential Equation.


8 1 1
Q69. The value of the integral  0 x 5
2
dx , valuated using Simpson’s rule with h  2 is
3
(a) 0.565 (b) 0.620 (c) 0.698 (d) 0.736

H.No. 40-D, Ground Floor, Jia Sarai, Near IIT, Hauz Khas, New Delhi-110016
Phone: 011-26865455/+91-9871145498
Website: www.physicsbyfiziks.com | Email: fiziks.physics@gmail.com
31
fiziks
Institute for NET/JRF, GATE, IIT‐JAM, M.Sc. Entrance, JEST, TIFR and GRE in Physics

Ans. : (a)
2 1
 y0  4  y1  y2   2 y3  y4 
x
Solution: I  y
3 x 5
2

2 1 1 1 1 1 1
   4    2   0
3 5 9 4 21 69  5
2 1
2 1 
   0.5734  0.09523  0.0145 9
3 5  4 1
2 21
  0.2  0.5734  0.09523  0.0145 1
3 6
31
2 8 1
  0.8831  0.5887
3 69
Q70. Consider a random walker on a square lattice. At each step the walker moves to a nearest
neighbour site with equal probability for each of the four sites. The walker starts at the
origin and takes 3 steps. The probability that during this walk no site is visited more than
one is
(a) 12 / 27 (b) 27 / 64 (c) 3 / 8 (d) 9 /16
Ans. : (d)
Solution: Total number of ways  4  4  4
Number of preferred outcome  4  3  3
( Any four option in step-1 and only 3 option in step 2 &3 because he can not go to
previous position)
4  3 3 9
probability  
4  4  4 16

H.No. 40-D, Ground Floor, Jia Sarai, Near IIT, Hauz Khas, New Delhi-110016
Phone: 011-26865455/+91-9871145498
Website: www.physicsbyfiziks.com | Email: fiziks.physics@gmail.com
32
fiziks
Institute for NET/JRF, GATE, IIT‐JAM, M.Sc. Entrance, JEST, TIFR and GRE in Physics

NET/JRF (JUNE-2016)
1
Q71. The radius of convergence of the Taylor series expansion of the function
cosh  x 

around x  0 , is

(a)  (b)  (c) (d) 1
2
Ans. : (c)
1 e4 z  1
Q72. The value of the contour integral  cosh  z   2sinh  z  dz around the unit circle C
2 i 
C

traversed in the anti-clockwise direction, is


8 1
(a) 0 (b) 2 (c) (d)  tanh  
3 2
Ans. : (c)
e4 z  1 e4 z  1 e4 z  1
Solution: f  z    z z 
cosh z  2sinh z e  e ez 3
2
 e z  e z   e z
2 2
 

 f z 

2e z e 4 z  1   2 e  e 
5z z

3  e 2z
 3  e  2z

ln 3
For pole at z  z0 ,3  e 2 z0  0  e 2 z0  3  z0 
2
It has simple pole at z0

Re  z0   lim  z  z0  f  z   lim  z  z0 

2 e5 z  e z 
z  z0 z  z0 3e 2z

 z  z0   2  5e5 z  e z   2  e5 z  e z  1  e5 z0  e z0 
 lim   
z  z0 2e 2 z  e
2 z0


 3  3 
5
9 3 3 8
         
 3   3  3
 
1 1 8
f  z  dz   2 i  Residue  
2 i  2 i 3

H.No. 40-D, Ground Floor, Jia Sarai, Near IIT, Hauz Khas, New Delhi-110016
Phone: 011-26865455/+91-9871145498
Website: www.physicsbyfiziks.com | Email: fiziks.physics@gmail.com
33
fiziks
Institute for NET/JRF, GATE, IIT‐JAM, M.Sc. Entrance, JEST, TIFR and GRE in Physics

Q73. The Gauss hypergeometric function F  a, b, c, z  , defined by the Taylor series expansion
around z  0 as F  a, b, c, z  
 a  a  1 ...  a  n  1 b  b  1 ...  b  n  1

n 0 c  c  1 ...  c  n  1 n !
zn ,

satisfies the recursion relation


d c
(a) F  a, b, c; z   F  a  1, b  1, c  1; z 
dz ab
d c
(b) F  a, b, c; z   F  a  1, b  1, c  1; z 
dz ab
d ab
(c) F  a, b, c; z   F  a  1, b  1, c  1; z 
dz c
d ab
(d) F  a, b, c; z   F  a  1, b  1, c  1; z 
dz c
Ans. : (d)
dF  a  a  1 ...  a  n  1 b  b  1 ...  b  n  1 n 1
Solution:  nz
dz n 0 c  c  1 ...  c  n  1 n !
 a  a  1 ...  a  n  1 b  b  1 ...  b  n  1 z n 1

n 0 c  c  1 ...  c  n  1 n 1

ab  a  1 ...  a  n  1 b  1 ...  b  n  1 z n 1



c
  c  1 ...  c  n  1 n 1

ab   a  1 ...  a   n  1  1  b  1 ... b   n  1  1 z n 1


 
c n 0  c  1 ... c   n  1  1 n 1

dF ab
 F  a  1, b  1, c  1, z 
dz c
Q74. Let X and Y be two independent random variables, each of which follow a normal
distribution with the same standard deviation  , but with means   and   ,
respectively. Then the sum X  Y follows a
(a) distribution with two peaks at   and mean 0 and standard deviation  2
(b) normal distribution with mean 0 and standard deviation 2
(c) distribution with two peaks at   and mean 0 and standard deviation 2

(d) normal distribution with mean 0 and standard deviation  2

H.No. 40-D, Ground Floor, Jia Sarai, Near IIT, Hauz Khas, New Delhi-110016
Phone: 011-26865455/+91-9871145498
Website: www.physicsbyfiziks.com | Email: fiziks.physics@gmail.com
34
fiziks
Institute for NET/JRF, GATE, IIT‐JAM, M.Sc. Entrance, JEST, TIFR and GRE in Physics

Ans. : (d)
Solution:     x   y      0

 12   x2   y2   2   2

   2
Q75. Using dimensional analysis, Planck defined a characteristic temperature TP from powers

of the gravitational constant G , Planck’s constant h , Boltzmann constant k B and the

speed of light c in vacuum. The expression for TP is proportional to

hc 5 hc3 G hk B2
(a) (b) 2 (c) (d)
k B2G kBG hc 4 k B2 Gc 3

Ans. : (a)
E ML2T 2
Solution: E  h  h    ML2T 1
 T 1
E ML2T 2
E  k BT  k B    ML2T 2TP1
T TP

m1m2 MLT 2  L2
F G 2 G  2
 G  M 1 L3T 2
r M

 
5
hc 5 ML2T 1  LT 1 ML7T 6
   TP2  TP
k B2G M 2 L4T 4TP2  M 1 L3T 2 7 6 2
ML T TP

Q76. What is the Fourier transform  dxeikx f  x  of



dn
f  x    x   n
  x
n 1 dx

where   x  is the Dirac delta-function?

1 1 1 1
(a) (b) (c) (d)
1 i k 1 i k k i k i
Ans. : (b)
  
dn dn
Solution: f  x     x    n   x    n   x      n   x 
n 1 dx n  0 dx n 0

 F   x    1  F   n   x     ik  F   x     ik 


n n

H.No. 40-D, Ground Floor, Jia Sarai, Near IIT, Hauz Khas, New Delhi-110016
Phone: 011-26865455/+91-9871145498
Website: www.physicsbyfiziks.com | Email: fiziks.physics@gmail.com
35
fiziks
Institute for NET/JRF, GATE, IIT‐JAM, M.Sc. Entrance, JEST, TIFR and GRE in Physics


 f  x      n  x 
n 0

1 1
 F  f  x      ik   1  ik   ik    ik   .... 
n 2 3

n 0 1   ik  1  ik

Q77. In finding the roots of the polynomial f  x   3 x 3  4 x  5 using the iterative Newton-

Raphson method, the initial guess is taken to be x  2 . In the next iteration its value is
nearest to
(a) 1.671 (b) 1.656 (c) 1.559 (d) 1.551
Ans. : (b)
Solution: f  x   3 x 3  4 x  5 ; f   x   9 x 2  4

3xn3  4 xn  5 3 x03  4 x0  5
xn 1  xn   x1  x 0 
9 xn2  4 9 x02  4
3 8  4  2  5 11
Let x0  2  x1  2   2  x1  1.656
9 4  4 32

NET/JRF (DEC-2016)
1 3 2
 
Q78. The matrix M   3 1 0  satisfies the equation
0 0 1
 
(a) M 3  M 2  10M  12 I  0 (b) M 3  M 2  12 M  10 I  0
(c) M 3  M 2  10M  10 I  0 (d) M 3  M 2  10 M  10 I  0
Ans. : (c)
Solution: The characteristic equation is

1    3 2
3  1    0 0
0 0 1   
 1    1   1      3  3 1     0
    2  1    1  9 1     0   3  10   2  10  0

Thus the matrix M satisfies the equation


M 3  M 2  10 M  10 I  0 then the correct option is (c)
H.No. 40-D, Ground Floor, Jia Sarai, Near IIT, Hauz Khas, New Delhi-110016
Phone: 011-26865455/+91-9871145498
Website: www.physicsbyfiziks.com | Email: fiziks.physics@gmail.com
36
fiziks
Institute for NET/JRF, GATE, IIT‐JAM, M.Sc. Entrance, JEST, TIFR and GRE in Physics

Q79. The Laplace transform of


t
 , 0t T
f t   T
 1 t T

is
 1  e  sT  1  e   sT
1  e   sT
1  e  sT

(a) (b) (c) (d)


s 2T s 2T s 2T s 2T
Ans. : (b)
Solution: we can write
t t t t
f  t   u0  t   uT  t    uT  t   1  uT  t    uT  t    uT  t   uT  t 
T T T T
Hence the transform of f  t  is

t     t  T   T  
L  f  t   L    L uT  t      L uT  t 
T    T  

1 e  sT  1 T  e
 sT
1  e  sT
     
s 2T T  s2 s  s s 2T
 1
Q80. The Fourier transform  dxf  x eikx of the function f  x   is
 x 2
2

 2k    2k
(a) 2 e (b) 2 e  2k
(c) e 2k
(d) e
2 2
Ans. : (d)
1 1  a k
Solution: Fourier transform of f  x   , a  0 is x eikx dx  e
x  a2
2 2
a 2
a
1  
x
2k
Hence eikx dx  e
2
a 2
2
Q81. Given the values sin 450  0.7071, sin 500  0.7660, sin 550  0.8192 and sin 600  0.8660 ,

the approximate value of sin 520 , computed by Newton’s forward difference method, is
(a) 0.804 (b) 0.776 (c) 0.788 (d) 0.798
Ans. : (c)

H.No. 40-D, Ground Floor, Jia Sarai, Near IIT, Hauz Khas, New Delhi-110016
Phone: 011-26865455/+91-9871145498
Website: www.physicsbyfiziks.com | Email: fiziks.physics@gmail.com
37
fiziks
Institute for NET/JRF, GATE, IIT‐JAM, M.Sc. Entrance, JEST, TIFR and GRE in Physics

Solution: Given -
x y  f  x f  x  2 f  x  3 f  x 

450 0  7071

500 0  7660 0  0589 0  0057 0  0007

550 0  8192 0  0532 0  0064

600 0  8660 0  0468

h  5 , 520  x0  uh  450  uh

52  45 52  45 7
u  
h 5 5
From table we have
y0  0  7071, y0  0  0589 ,  2 y0  0  0057

 3 y0  0  0007 ,  4 y0  0,.....
y0  2 y0  3 y0
 sin  520   y0  u u  u  1  u  u  1 u  2   
1 2 3

7  0  0057  7  7   0  0007  7  7   7 
 0  7071   0  0589     1      1   2   0
5 2 55   6  5  5  5 
0  0399 0  0049
 0  7071  0  0825  
25 125
 0  7071  0  0825  0  0016  0  0000  0  7880
f 2 f
Q82. Let f  x, t  be a solution of the heat equation  D 2 in one dimension. The initial
t x
condition at t  0 is f  x, 0   e  x for   x   . Then for all t  0, f  x, t  is given by
2

 

2
[Useful integral: dx e  x  ]
 
2 2
1 
x
1 
x
(a) e 1 Dt (b) e 1 2 Dt
1  Dt 1  2 Dt
2
x2
1 
x

(c) e 1 4 Dt (d) e 1 Dt

1  4 Dt
Ans. : (c)
f 2 f
Solution: D 2 , t 0
t x
H.No. 40-D, Ground Floor, Jia Sarai, Near IIT, Hauz Khas, New Delhi-110016
Phone: 011-26865455/+91-9871145498
Website: www.physicsbyfiziks.com | Email: fiziks.physics@gmail.com
38
fiziks
Institute for NET/JRF, GATE, IIT‐JAM, M.Sc. Entrance, JEST, TIFR and GRE in Physics

f 2 f
Let D  c 2 ,  c2 2
t x

Initial condition f  x , 0   e  x  g  x  ,    x  
2

   
2
 x  2 cz t
Now, g x  2cz t  e

The solution f  x , t  is given as


 
1   1  e  z dz
2
 x 2  4 c 2 z 2t  4 cxz t
f  x,t 
 x  2 cz t
e e
 z2 2
e dz 
   

  x 2 
 
2
e x   4 c 2 z 2t  4 cxz t  z 2  dz  e  
 1 4 c 2t z 2  4 cxz t

 e
  e

dz

e x 2   2  2cx t   2cx t  


2 2

 exp   1  4c t   z  2  z  1  4c 2t    1  4c 2t    1  4c 2t   dz
cx t
 2 2

       

 2 cx t 
2
4 c 2 x 2t  2 cx t 
2

e x
2   
 1 4 c 2t  z  2 1 4c t  dz  e 2  x2 4 c 2 x 2t 
 
 1 4 c 2t  z  2

 e

 1 4 tc 
e

e 1 4 c 2t
e

 1 4 tc 
dz

 x2

e 1 4 c 2t
1 
  2 
 2 1  4c 2t
2
x
1
f  x, t  
1  4 Dt
e1 4 Dt c 2
 D

3
Q83. A stable asymptotic solution of the equation xn 1  1  is x  2 . If we take
1  xn
n 1
xn  2 n and xn 1  2 n 1 , where n and n1 are both small, the ratio is
n
approximately
1 1 1 2
(a)  (b)  (c)  (d) 
2 4 3 3
Ans. : (c)

H.No. 40-D, Ground Floor, Jia Sarai, Near IIT, Hauz Khas, New Delhi-110016
Phone: 011-26865455/+91-9871145498
Website: www.physicsbyfiziks.com | Email: fiziks.physics@gmail.com
39
fiziks
Institute for NET/JRF, GATE, IIT‐JAM, M.Sc. Entrance, JEST, TIFR and GRE in Physics
Q84. The 2  2 identity matrix I and the Pauli matrices  x ,  y ,  z do not form a group under matrix multiplication. The minimum number of 2  2
matrices, which includes these four matrices, and form a group (under matrix multiplication) is
(a) 20 (b) 8 (c) 12 (d) 16
Ans. (d)
 I I i I i  I x  x i x i x y  y i y i y z  z i z i z

I I I i I i  I x  x i x i x y  y i y i y z  z i z i z

I I I i  I i I  x x i x i x  y y i y i y  z z i z i z

i I i I i  I I I i x i x  x x i y i y  y y i z i z  z z


i  I i  I i I I I i x i x x  x i y i y y  y i z i z z  z

x x  x i x i x I I i I i  I i z i z  z z i y i y y  y

 x  x x i x i x I I i  I i I i z i z z  z i y i y  y y

i x i x i x  x x i I i  I I I  z z i z i z y  y i y i y

i x i x i x x  x i  I i I I I z  z i z i z  y y i y i y

y y  y i y i y i z i z z  z I I i I i  I i x i x  x x

 y  y y i y i y i z i z  z z I I i  I i I i x i x x  x

i y i y i y  y y z  z i z i z i I i  I I I  x x i x i x

i y i y i y y  y  z z i z i z i  I i I I I x  x i x i x

z z  z i z i z i y i y  y y i x i x x  x I I i I i  I

H.No. 40‐D, Ground Floor, Jia Sarai, Near IIT, Hauz Khas, New Delhi‐110016
Phone: 011‐26865455/+91‐9871145498
Website: www.physicsbyfiziks.com | Email: fiziks.physics@gmail.com
40
fiziks
Institute for NET/JRF, GATE, IIT‐JAM, M.Sc. Entrance, JEST, TIFR and GRE in Physics
 z  z z i z i z i y i y y  y i x i x  x x I I i  I i I

i z i z i z  z z  y y i y i y x x i x i x i I i  I I I

i z i z i z z  z y  y i y i y x x i x i x i  I i I I I

H.No. 40‐D, Ground Floor, Jia Sarai, Near IIT, Hauz Khas, New Delhi‐110016
Phone: 011‐26865455/+91‐9871145498
Website: www.physicsbyfiziks.com | Email: fiziks.physics@gmail.com
41
fiziks
Institute for NET/JRF, GATE, IIT‐JAM, M.Sc. Entrance, JEST, TIFR and GRE in Physics

NET/JRF (JUNE-2017)
Q85. Which of the following can not be the eigen values of a real 3  3 matrix
(a) 2i, 0,  2i (b) 1,1,1 (c) ei , e  i ,1 (d) i, 1, 0
Ans. : (d)
Solution: If the matrix is real then the complex eigen values always occurs with its complex
conjugate. In option (d) if i is an eigen value then i must also be an eigen value. But
i is not given in option, hence option (d) is incorrect.
Q86. Let u  x, y   e ax cos  by  be the real part of a function f  z   u  x, y   iv  x, y  of the

complex variable z  x  iy , where a, b are real constants and a  0 . The function f  z 

is complex analytic everywhere in the complex plane if and only if


(a) b  0 (b) b   a (c) b  2 a (d) b  a  2
Ans. : (b)
Solution: The function f  z  will be analytic everywhere in the complex plane if and only if it

satisfies the Cauchy Riemann equation in that region.


u v u v
  and 
x y y x
v
Hence ae ax cos  by   (i)
y
v
and be ax sin  by   (ii)
x
From equation (i)
ae ax sin  by 
v  x, y    c y (iii)
b
Differentiating partially with x gives

v a e sin  by 
2 ax

 (iv)
x b
From equation (iii) and (iv)
a 2 e ax sin  by 
be sin  by  
ax

b
 b2  a 2  b   a

H.No. 40-D, Ground Floor, Jia Sarai, Near IIT, Hauz Khas, New Delhi-110016
Phone: 011-26865455/+91-9871145498
Website: www.physicsbyfiziks.com | Email: fiziks.physics@gmail.com
42
fiziks
Institute for NET/JRF, GATE, IIT‐JAM, M.Sc. Entrance, JEST, TIFR and GRE in Physics

zei z / 2
Q87. The integral   z 2  1 dz along the closed contour  shown in the figure is
y

1 1 x

(a) 0 (b) 2 (c) 2 (d) 4 i


Ans. : (c)
zeiz / 2
Solution: f  z  
 z  1 z  1
For z  1 anti-clockwise
zei z / 2 2 i i / 2
I  2 i lim  e   iei / 2
z 1  z  1 2

For z  1

I  2 i lim
zei z / 2
 2 i 
 1 e  i / 2

  ie  i / 2
z 1  z  1  2 

Integral   i
e i / 2
 e  i / 2 
 2i  2 i 2 sin

 2
2i 2
dy cos  x
Q88. The function y  x  satisfies the differential equation x  2y  . If y 1  1 , the
dx x
value of y  2  is

(a)  (b) 1 (c) 1/ 2 (d) 1/ 4


Ans. : (d)
Solution: The given differential equation can be written as
dy 2 cos  x
 y
dx x x2
2
This is a linear differential equation with Integrating factor  e  x  x 2
dx

cos  x sin  x c
Hence y.x 2   x 2 . dx  c  y   2
x 2
 x2 x

H.No. 40-D, Ground Floor, Jia Sarai, Near IIT, Hauz Khas, New Delhi-110016
Phone: 011-26865455/+91-9871145498
Website: www.physicsbyfiziks.com | Email: fiziks.physics@gmail.com
43
fiziks
Institute for NET/JRF, GATE, IIT‐JAM, M.Sc. Entrance, JEST, TIFR and GRE in Physics

sin  x 1
when x  1, y  1 hence c  1  y   2
 x2 x
1
hence, when x  2, y 
4
Q89. The random variable x    x    is distributed according to the normal distribution
x2
1 
P  x  e 2 2
. The probability density of the random variable y  x 2 is
2 2

1 2 1 2
(a) e  y / 2 , 0  y   (b) e  y / 2 , 0  y  
2 y 2
2 2 y 2

1 2 1 2
(c) e  y / 2 , 0  y   (d) e y /  , 0  y  
2 2
2 y 2

Ans. : (a)
x2
1 
Solution: p  x   e 2 2
,   x  
2 2
  x2
1 

 p  x  dx  1  
  2 2
e 2 2
dx  1

 x2  x2
1  2 1 
2 e 2 2
dx  1  e 2 2
dx  1
0 2 2 2 2 0

1
put x 2  y dy  2 xdx , dx 
2 y
 y  y
1 1 1 1 2 2
2
2 2
e
0
2 2

2 y
dy 
2 2

0 y
e dy 

y
1 
f  y  e 2 2
, 0  y  
2 y 2

H.No. 40-D, Ground Floor, Jia Sarai, Near IIT, Hauz Khas, New Delhi-110016
Phone: 011-26865455/+91-9871145498
Website: www.physicsbyfiziks.com | Email: fiziks.physics@gmail.com
44
fiziks
Institute for NET/JRF, GATE, IIT‐JAM, M.Sc. Entrance, JEST, TIFR and GRE in Physics

Q90. The Green’s function satisfying


d2
g  x, x0     x  x0 
dx 2
with the boundary conditions g   L, x0   0  g  L, x0  , is

 1  1
 2 L  x0  L  x  L  ,  L  x  x0  2 L  x0  L  x  L  ,  L  x  x0
(a)  (b) 
 1  x  L  x  L  , x0  x  L  1  x  L  x  L  , x0  x  L
 2 L 0  2 L 0

 1
 2 L  L  x0  x  L  ,  L  x  x0
1
(c)  (d)  x  L  x  L  , L  x  L
 1  x  L  L  x  , x0  x  L
2L
 2 L 0
Ans. : (a)
d2
Solution: 2 g  x, x0     x  x0 
dx
boundary conditions:
g   L, x0   0  g  L, x0 

The homogeneous equation for Green’s function is


d2
g  x, x0   0
dx 2
Solution of above equation is
 Ax  B, x  x0
g  x, x0   
Cx  D, x  x0

Applying boundary condition


g   L, x0   0   AL  B  0  AL  B

g  L, x0   0  CL  D  0  CL  D

 A  x  L  , x  x0
Hence, g  x, x0   
C  x  L  , x  x0

From continuity of Green’s function at x  x0 we have

A  x0  L   C  x0  L 

H.No. 40-D, Ground Floor, Jia Sarai, Near IIT, Hauz Khas, New Delhi-110016
Phone: 011-26865455/+91-9871145498
Website: www.physicsbyfiziks.com | Email: fiziks.physics@gmail.com
45
fiziks
Institute for NET/JRF, GATE, IIT‐JAM, M.Sc. Entrance, JEST, TIFR and GRE in Physics

 x L
AC 0 
 x0  L 
g
From discontinuity of derivative of Green’s function at x  x0
x
We have
g g
 1
x x  x0 x x  x0

g g
 C, A
x x  x0 x x  x0

 C  A  1  C  A 1
Thus, the required solution of Green’s function is given by
  x0  L  x  L 
 , x  x0
g  x, x0    2 L
  x0  L  x  L  , x  x0
 2L
 i z 
Q91. Let  x , y , z be the Pauli matrices and x x  y y  z  z  exp  
 2 
 i z 
 x x  y y  z z  exp   2 .
 
Then the coordinates are related as follows
 x   cos   sin  0  x   x   cos  sin  0  x 
         
(a)  y    sin  cos  0  y  (b)  y     sin  cos  0  y 
 z   0 0 1   z   z   0 0 1   z 
     

       
 cos 2 sin 0  cos 2  sin 0
 x   2  x   2
 x  x
           
(c)  y     sin cos 0  y  (d)  y    sin cos 0  y 
 2 2   2 2 
 z    z   z    z 
  0 0 1     0 0 1  
   
   
Ans. : (b)
0 1  0 i  1 0 
Solution:  x    , y    and  z   
1 0 i 0   0 1 
H.No. 40-D, Ground Floor, Jia Sarai, Near IIT, Hauz Khas, New Delhi-110016
Phone: 011-26865455/+91-9871145498
Website: www.physicsbyfiziks.com | Email: fiziks.physics@gmail.com
46
fiziks
Institute for NET/JRF, GATE, IIT‐JAM, M.Sc. Entrance, JEST, TIFR and GRE in Physics

 z x  iy 
Hence, x x  y y  z z   
 x  iy z 

 z' x1  iy ' 
x ' x  y ' y  z ' z   
 x ' iy ' z ' 
i / 2  i / 2
 i z  e 0   i z  e 0 
exp  
   i / 2 
and exp  
  
 z   0 e   2   0 ei / 2 

 z x  iy   ei / 2 0  z x  iy   e i / 2 0 


Hence,     
 x  iy  z   0 e i / 2   x  iy z   0 ei / 2 

 z x  iy   z ei  x  iy  
 
  
 x  iy  z    e  i  x  iy  z 
Hence, z   z and x  iy  ei  x  iy 

Thus x  iy   cos   x   sin   y   i  cos   y   sin   x 

Thus x   cos   x   sin   y

And y '    sin   x   cos   y

 x   cos  sin  0  x 
    
Thus,  y     sin  cos  0  y 
 z   0 0 1   z 
  
Q92. Which of the following sets of 3  3 matrices (in which a and b are real numbers) forms
a group under matrix multiplication?
 1 0 a    1 a 0  
     
(a)  0 1 0  ; a, b    (b)  0 1 b  ; a, b   
 b 0 1     
    0 0 1  

 1 0 a    1 a 0  
     
(c)  0 1 b  ; a, b    (d)  b 1 0  ; a, b   
 0 0 1     
    0 0 1  
Ans. : (c)

H.No. 40-D, Ground Floor, Jia Sarai, Near IIT, Hauz Khas, New Delhi-110016
Phone: 011-26865455/+91-9871145498
Website: www.physicsbyfiziks.com | Email: fiziks.physics@gmail.com
47
fiziks
Institute for NET/JRF, GATE, IIT‐JAM, M.Sc. Entrance, JEST, TIFR and GRE in Physics

Solution: In order to form the group the required matrix must satisfy the following conditions.
(a) For any three matrices A, B, C  G A  BC    AB  C

(b) There must exist an identity element I such that AI  IA  A


(c) There must exist an inverse element for each element belonging to G.
(d) For any two matrices A, B  G , the matrix A B  G (closer property).
All the given matrices satisfy property (a), if we put a  0 and b  0, we see that
property (b) also holds for all the given matrices.
If we put a  1 and b  1, in option (a) , two rows becomes identical and matrix in option
(a) is non-invertible. If we put a  1 and b  1, in option (d), two rows becomes identical
and the matrix is non-invertible.
Now only option (b) and (c) remains. For option (b) take two matrices and multiply
 1 a1 0   1 a2 0   1 a1  a2 a1b2 
    
 0 1 b1   0 1 b2    0 1 b1  b2 
0 0 1 0 0 
1  0  0 1 
 
we see that the resulting matrix does not satisfy closure property. For option (3) take two
matrices and multiply
 1 0 a1   1 0 a2   1 0 a2  a1 
    
 0 1 b1   0 1 b2    0 1 b2  b1 
0 0 1 0 0 1  0 0 1 
   
we see that the resulting matrix satisfy the closure property. Hence the correct option is (c)
Q93. A random variable n obeys Poisson statistics. The probability of finding n  0 is 106 .
The expectation value of n is nearest to
(a) 14 (b) 106 (c) e (d) 102
Ans. : (a)
n
Solution: In Poisson’s statistics the probability of finding the value n is given by P  n   e 
n!
The mean of Poisson’s statistics is  . From the question
0
P  0   10  10 
6 6
e    e    106
0!
Talking Log of both sides,    6 ln10    6 ln10
Hence the expectation value of n is   6  2.30  13.8  14

H.No. 40-D, Ground Floor, Jia Sarai, Near IIT, Hauz Khas, New Delhi-110016
Phone: 011-26865455/+91-9871145498
Website: www.physicsbyfiziks.com | Email: fiziks.physics@gmail.com
48
fiziks
Institute for NET/JRF, GATE, IIT‐JAM, M.Sc. Entrance, JEST, TIFR and GRE in Physics

NET/JRF (DEC - 2017)


Q94. Let A be a non-singular 3  3 matrix, the columns of which are denoted by the vectors
     
a , b and c , respectively. Similarly, u , v and w denote the vectors that form the

 
1
corresponding columns of AT . Which of the following is true?
           
(a) u  a  0, u  b  0, u  c  1 (b) u  a  0, u  b  1, u  c  0
           
(c) u  a  1, u  b  0, u  c  0 (d) u  a  0, u  b  0, u  c  0
Ans. : (c)
Solution: We can take any 3  3 non singular matrix in order to avoid long calculation.
1 0 0 1 0 0 
0 2 0   0 1/ 2 0 
  
 
1
Take A   0 0 3   AT   0 0 1/ 3
   
 

    
 a 
c   u v 
w 
 b

We see that

u .a  1.1  0.0  0.0  1

u .b  1.0  0.2  0.0  0

u .c  1.0  0.0  0.3  0
Q95.  
Consider the real function f  x   1/ x 2  4 . The Taylor expansion of f  x  about x  0

converges
(a) for all values of x (b) for all values of x except x  2
(c) in the region 2  x  2 (d) for x  2 and x  2
Ans. : (c)
1 1
Solution: f  x   
x 4
2
 x2 
4 1  
 4 
1
1  x2 
Thus the Taylor’s series of f  x  is times the binomial series of 1  
4  4

x2
 1  x  4   x  2  x  2   0
2
Now, the binomial series converges if
4

H.No. 40-D, Ground Floor, Jia Sarai, Near IIT, Hauz Khas, New Delhi-110016
Phone: 011-26865455/+91-9871145498
Website: www.physicsbyfiziks.com | Email: fiziks.physics@gmail.com
49
fiziks
Institute for NET/JRF, GATE, IIT‐JAM, M.Sc. Entrance, JEST, TIFR and GRE in Physics

Since x  2 is always greater than 0 ,

Hence x  2  0   2  x  2

Q96. Consider the matrix equation


1 1 1  x  0
    
1 2 3  y   0
 2 b 2c   z   0 
    
The condition for existence of a non-trivial solution and the corresponding normalised
solution (upto a sign) is
1
(a) b  2c and  x, y, z   1, 2, 1
6
1
(b) c  2b and  x, y, z   1, 1, 2 
6
1
(c) c  b  1 and  x, y, z    2, 1, 1
6
1
(d) b  c  1 and  x, y, z   1, 2, 1
6
Ans. : (d)
Solution: We know that the matrix equation, AX  0 , where A is the given matrix and X is a
column vector has a non-zero solution if and only if A  0

1 1 1
1 2 3  0  4c  3b  2c  6  b  4  0
2 b 2c

 2c  2b  2  0  b  c  1
we do not need to perform further calculation.
dy
Q97. Consider the differential equation  ay  e  bt with the initial condition y  0   0 .
dt
Then the Laplace transform Y  s  of the solution y  t  is

1 1 1 e a  eb
(a) (b) (c) (d)
 s  a  s  b  b s  a a  s  b ba

Ans. : (a)
H.No. 40-D, Ground Floor, Jia Sarai, Near IIT, Hauz Khas, New Delhi-110016
Phone: 011-26865455/+91-9871145498
Website: www.physicsbyfiziks.com | Email: fiziks.physics@gmail.com
50
fiziks
Institute for NET/JRF, GATE, IIT‐JAM, M.Sc. Entrance, JEST, TIFR and GRE in Physics

dy
Solution: Given  ay  e  bt
dt
Taking Laplace transform of both sides
We obtain
 dy  1
 dt 
 
L    aL  y  t   L e  bt  sY  s   y  0   aY  s  
sb

Since, y  0   0 , we obtain

1 1
 s  a Y  s    Y s 
sb  s  a  s  b 
Q98. The number of linearly independent power series solutions, around x  0 , of the second
d 2 y dy
order linear differential equation x 2   xy  0 , is
dx dx
(a) 0 (this equation does not have a power series solution)
(b) 1
(c) 2
(d) 3
Ans. : (b)
Q99. Consider an element U   of the group SU  2  , where  is any one of the parameters

of the group. Under an infinitesimal change      , it changes as

U    U     U    1  X    U  gj  . To order  , the matrix X   should

always be
(a) positive definite (b) real symmetric (c) hermitian (d) anti-hermitian
Ans. : (d)
Solution: Since, U    e im

U   d   e  im .e  im d 

 U     U    1  X  d   U  

 1  X  d   e  im d  or, 1  X  d   1  im  d   ...

or, Xd    im  d 

H.No. 40-D, Ground Floor, Jia Sarai, Near IIT, Hauz Khas, New Delhi-110016
Phone: 011-26865455/+91-9871145498
Website: www.physicsbyfiziks.com | Email: fiziks.physics@gmail.com
51
fiziks
Institute for NET/JRF, GATE, IIT‐JAM, M.Sc. Entrance, JEST, TIFR and GRE in Physics

here, m is one of the Pauli spin matrices, since Pauli matrices are hermitian,
taken complex conjugate, so matrix should anti-hermitian.
Hence correct option is (d)
dy  x 
Q100. The differential equation   x 2 , with the initial condition y  0   0 , is solved
dx
using Euler’s method. If yE  x  is the exact solution and y N  x  the numerical solution

obtained using n steps of equal length, then the relative error


 y  x   y  x 
N E
is
yE  x 
proportional to
1 1 1 1
(a) 2 (b) (c) (d)
n n3 n4 n
Ans. : (d)
dy
Solution:   x2 , y  0  0
dx
 x3
yE  , but x  n 
3
 n3 h3
Exact solution, yE 
3
Numerically, f  x, y    x 2

Euler’s method, yi  yi 1  hf  xi 1 , yi 1 

y1  0 , y2   h3 y3  5 h3

yn 
 n  1 n  2n  1  h3
6
Since, 0,5,14,30,...different from square terms
At, x0  0 x1  x0  h  h x2  x0  2h  2h x3  x0  3h  3h

xn 1  x0   n  1 h   n  1 h . Now, xn  nh

f  x0 , y0   0 , f  x1 , y1    h 2 , f  x2 , y2   4 h 2

f  xn 1 , yn 1     n  1 h 2
2

 n  1 n  2n  1  h3   n3h3
 y N  yE   6 3
yE  n3 h3
3
H.No. 40-D, Ground Floor, Jia Sarai, Near IIT, Hauz Khas, New Delhi-110016
Phone: 011-26865455/+91-9871145498
Website: www.physicsbyfiziks.com | Email: fiziks.physics@gmail.com
52
fiziks
Institute for NET/JRF, GATE, IIT‐JAM, M.Sc. Entrance, JEST, TIFR and GRE in Physics

yN  yE 1
By solving, 
yE n

Q101. The interval  0,1 is divided into n parts of equal length to calculate the integral
1

e
i 2 x
dx using the trapezoidal rule. The minimum value of n for which the result is
0

exact, is
(a) 2 (b) 3 (c) 4 (d) 
Ans. : (a)
1
Solution:  ei 2 nx dx  0 , exact value
0

1
Now, nh  1  0, h 
n
1
y  f  x   ei 2 n x , Let n  2 , then x0  0, y0  1 , x1  , y1  1 and x2  1, y2  1
2
h 1
I  y0  2  y1  ....  yn 1   yn   1  2  1  1
2 4
I  0. So, n  2
Q102. The generating function G  t , x  for the Legendre polynomials Pn  t  is

1
G t, x     x n Pn  t , for x  1
1  2 xt  x 2 n 0

x
If the function f  x  is defined by the integral equation  f  x dx  xG 1, x  , it can be
0

expressed as
 
1
(a) 
n ,m 0
x nm
Pn 1 Pm  
2
(b) 
n ,m 0
x n  m Pn 1 Pm 1

 
(c) 
n ,m 0
x n  m Pn 1 Pm 1 (d) 
n ,m 0
x n  m Pn  0  Pm 1

Ans. : (b)

1
Solution: G  t , x     x n Pn  t  for x  1
1  2 xt  x 2 n0

H.No. 40-D, Ground Floor, Jia Sarai, Near IIT, Hauz Khas, New Delhi-110016
Phone: 011-26865455/+91-9871145498
Website: www.physicsbyfiziks.com | Email: fiziks.physics@gmail.com
53
fiziks
Institute for NET/JRF, GATE, IIT‐JAM, M.Sc. Entrance, JEST, TIFR and GRE in Physics


1
G 1, x     x n Pn 1
1  2 x  x2 n0


1 1
   x n Pn 1  Since x  1
1  x 
2
n 0 1 x

x
1
 f  x   dx 
1  x 0
Now, x 

Differentiating both sides,


d x 1
f  x  
dx 1  x 1  x 2

NET/JRF (JUNE-2018)
Q103. Consider the following ordinary differential equation
2
d 2 x 1  dx  dx
    0
dt 2 x  dt  dt

with the boundary conditions x  t  0   0 and x  t  1  1 . The value of x  t  at t  2 is

(a) e 1 (b) e2  1 (c) e 1 (d) e2  1


Ans. : (c)
Solution: The given equation can be written as
1 d  dx  dx d  dx  dx
x   0  x  x 0
x dt  dt  dt dt  dt  dt
dx
putting y  x gives
dt
dy
 y  0  lny  t  lnc1  y  c1 et
dt
dx
Since x  c1 et hence by integrating
dt
x2
 c1 et  c2 (i)
2
Using boundary conditions we obtain
1
c1  c2  0 and c1 e  c2 
2
H.No. 40-D, Ground Floor, Jia Sarai, Near IIT, Hauz Khas, New Delhi-110016
Phone: 011-26865455/+91-9871145498
Website: www.physicsbyfiziks.com | Email: fiziks.physics@gmail.com
54
fiziks
Institute for NET/JRF, GATE, IIT‐JAM, M.Sc. Entrance, JEST, TIFR and GRE in Physics

1 1
Solving these equations we obtain c1  and c2  
2  e  1 2  e  1

x2 1 1
Thus,  et 
2 2  e  1 2  e  1

When t  2, we obtain, x 2 
e2

1

 e2  1  e  1
 e  1  e  1  e  1
Therefore, x  2   e  1

Q104. What is the value of a for which f  x, y   2 x  3  x 2  y 2   2i  3 xy  ay  is an analytic

function of complex variable z  x  iy


(a) 1 (b) 0 (c) 3 (d) 2
Ans. : (a)
Solution: f  x, y   2 x  3  x 2  y 2   2i  3 xy   y 

u  2 x  3  x 2  y 2  , v  2  3 xy   y 

C-R conditions: u x  v y , u y  vx ,

2  3  2 x   2  3 x       1   6 y  6 y
  
Q105. Consider the three vectors v1  2iˆ  3kˆ, v2  iˆ  2 ˆj  2kˆ and v3  5iˆ  ˆj  akˆ where iˆ, ˆj

and k̂ are the standard unit vectors in a three-dimensional Euclidean space. These vectors
will be linearly dependent if the value of a is
31 23 27
(a) (b) (c) (d) 0
4 4 4
Ans. : (a)
Solution: Given vector will be linearly dependent if the determinant of the matrix formed by
taking these vectors as column is zero.
2 1 5
0 2 1  0  2  2 a  2     3   5  6   0
3 2 a
31
 4a  4  3  30  0  4a  31  0  a 
4

H.No. 40-D, Ground Floor, Jia Sarai, Near IIT, Hauz Khas, New Delhi-110016
Phone: 011-26865455/+91-9871145498
Website: www.physicsbyfiziks.com | Email: fiziks.physics@gmail.com
55
fiziks
Institute for NET/JRF, GATE, IIT‐JAM, M.Sc. Entrance, JEST, TIFR and GRE in Physics


 dxf  x  eikx of the function f  x   e
x
Q106. The Fourier transform


2 1 2 2
(a)  (b)  (c) (d)
1 k 2 2 1  k 2  1 k 2 2  k 2 
Ans. : (c)
 

  dx e
x x
Solution: dxe eikx  cos kxdx odd function sin kx vanishes
 

e x
2 
 cos kx  k sin kx 0

 2 e  x cos kx dx  2
0
1 k
e ax
 e ax cos bxdx   a cos bx  b sin bx 
a 2  b2

e0 2
 2 e cos kxdx  2
x

0
1 k 2
1 k 2

Q107. The value of the integral


 /2 1
 dx  dy.  sin 2 x    x  y  is
 /2 1

1 1
(a) 0 (b) (c) (d) 1
2 2
Ans. : (b)
 /2 1  /2 1
Solution: I   dx  dy  sin 2 x    x  y    dx   sin 2 x     y  x  dy
 / 2 1  / 2 1

If we assume that x lies between 1 and 1 then the second integral is 1 and the given
integral becomes
 /2
I
 /2
   sin 2 x dx
 n 

 x 
 2 
now   sin 2 x   
n  n
2 cos 2
2
 /2
1 1 1
Therefore, I     x  dx  .1 
2  / 2 2 2

H.No. 40-D, Ground Floor, Jia Sarai, Near IIT, Hauz Khas, New Delhi-110016
Phone: 011-26865455/+91-9871145498
Website: www.physicsbyfiziks.com | Email: fiziks.physics@gmail.com
56
fiziks
Institute for NET/JRF, GATE, IIT‐JAM, M.Sc. Entrance, JEST, TIFR and GRE in Physics

Q108. Which of the following statements is true for a 3  3 real orthogonal matrix with
determinant 1 ?
(a) the modulus of each of its eigenvalues need not be 1 , but their product must be 1
(b) at least one of its eigenvalues is 1
(c) all of its eigenvalues must be real
(d) none of its eigenvalues must be real
Ans. : (b)
Solution: The characteristic equation of any 3  3 matrix is of thee form  3  a 2  b  c  0
which implies that at least one of the eigenvalues must be real. It is a proven fact that
modulus of each eigenvalues of an orthogonal matrix is 1.
If all eigenvalues of 3  3 orthogonal matrix are real then only possibilities for
eigenvalues are
1  1, 2  1 and 3  1 or 1  1, 2  1, 3  1 or 1  1, 2  1, 3  1
Thus we see that at least one eigenvalue is 1 . Suppose one eigenvalues is real and other
two eigenvalues are complex conjugates. Now
12 3  1

 1  a  ib  a  ib   1  1  a 2  b 2   1

Since a 2  b 2 is always positive hence 1  1 .


In this case also we see that at least one eigenvalue must be 1

Q109. In the function Pn  x  e  x of a real variable x , Pn  x  is polynomial of degree n . The


2

maximum number of extrema that this function can have is


(a) n  2 (b) n  1 (c) n  1 (d) n
Ans. : (c)

Solution: y  Pn  x  e  x  Pn  x  e  x  Pn  x  e x  2 x   0  Pn  x   2 xPn  x   0


2 2 2

P0  x   1, P1  x   2  P0  x   2 xP0  x   0  0  2 x.1  0

x  0, 1 extrema

P1 x   2 xP1  x   0

H.No. 40-D, Ground Floor, Jia Sarai, Near IIT, Hauz Khas, New Delhi-110016
Phone: 011-26865455/+91-9871145498
Website: www.physicsbyfiziks.com | Email: fiziks.physics@gmail.com
57
fiziks
Institute for NET/JRF, GATE, IIT‐JAM, M.Sc. Entrance, JEST, TIFR and GRE in Physics

1
1  2 x.x  0  x   i.e., 2 extrema.
2
Thus in general there are  n  1 extrema.

d 2 y  x
Q110. The Green’s function G  x, x  for the equation  y  x   f  x  , with the
dx 2
 
boundary values y  0   y    0 , is
2
   
 x  x  2  , 0  x  x 
2
  
(a) G  x, x    
 x    x , 0  x  x 


 
2 2

 
  cos x  sin x, 0  x  x   2
(b) G  x, x    
  sin x  cos x, 0  x   x  
 2

 
cos x  sin x, 0  x  x 
2
(c) G  x, x    
sin x  cos x, 
0  x  x 
 2

   
 x  2  x  , 0  x  x 
2
  
(d) G  x, x    
 x    x  , 0  x  x 

  2 
 2
Ans. : (b)
d2y
Solution: 2
 y  0  m2  1  0  m  0  i
dx
y1  x   sin x  at  , y1  x   cos x

  
y2  x   cos x  at  , y2  x    sin x
 2
 
A  P  x   y2  x  y1  x1   y1  x   y2  x 

 A   sin x sin x  cos cos x  P  x   1  A  1

H.No. 40-D, Ground Floor, Jia Sarai, Near IIT, Hauz Khas, New Delhi-110016
Phone: 011-26865455/+91-9871145498
Website: www.physicsbyfiziks.com | Email: fiziks.physics@gmail.com
58
fiziks
Institute for NET/JRF, GATE, IIT‐JAM, M.Sc. Entrance, JEST, TIFR and GRE in Physics

 
 sin cos , 0  x  x 
 Ay  x  y2  x  , x  x  x x
Thus G  x, x    1  2
 Ay2  x  y1  x  , x  x  cos x sin x, 0  x  x  
 2
1 1
Q111. The fractional error in estimating the integral 0
xdx using Simpson’s
3
rule, using a step

size 0.1 , is nearest to


(a) 104 (b) 0 (c) 102 (d) 3 104
Ans. : (b)
h
Solution: I   y0  2  y2  y4  ....  4  y1  y3  y6  ...  yn 
3
0.1
 0  2  0.2  0.4  0.6  0.8   4  0.1  0.3  0.5  0.7  0.9  1 
3 
1 15
  4  10  1   0.5
30 30
I 0.5  0.5
fractional error   0
I true 0.5
y0 0

y1 0.1

y2 0.2

y3 0.3

y4 0.4

y5 0.5

y6 0.6

y7 0.7

y8 0.8

y9 0.9

y10 1.0

H.No. 40-D, Ground Floor, Jia Sarai, Near IIT, Hauz Khas, New Delhi-110016
Phone: 011-26865455/+91-9871145498
Website: www.physicsbyfiziks.com | Email: fiziks.physics@gmail.com
59
fiziks
Institute for NET/JRF, GATE, IIT‐JAM, M.Sc. Entrance, JEST, TIFR and GRE in Physics

NET/JRF (DEC-2018)
a 0 0
Q112. One of the eigenvalues of the matrix e is e , where A   0 0 a  . The product of the
A a

0 a 0
 
other two eigenvalues of e A is
(a) e 2 a (b) e  a (c) e 2 a (d) 1
Ans. : (d)
Solution: Eigenvalues of matrix A are a, a and  a . The product of two other eigenvalues of A

are ea a  a  1
Alternativety
eTraceA  e1  2  3  det e A
 e1 .e2  3  det e A  e a .e2 .e3  e a
 e2 .e3  1
Q113. The polynomial f  x   1  5 x  3 x 2 is written as linear combination of the Legendre

polynomials
 1 
 2
2
 

 P0  x   1, P1  x  , P2  x   3 x  1  as f  x    n cn Pn  x  . The value of c0 is

1 1
(a) (b) (c) 2 (d) 4
4 2
Ans. : (c)
Solution: f  x   1  5 x  3 x 2

1  P0  x  x  P1  x 

1
x2 
3
 2P2  x   1
f  x   P0  x   5 P1  x   2 P2  x   P0  x 

 2 P0  x   5 P1  x   2 P2  x 

 c0 P0  x   c1 P1  x   c2 P2  x  c0  2

H.No. 40-D, Ground Floor, Jia Sarai, Near IIT, Hauz Khas, New Delhi-110016
Phone: 011-26865455/+91-9871145498
Website: www.physicsbyfiziks.com | Email: fiziks.physics@gmail.com
60
fiziks
Institute for NET/JRF, GATE, IIT‐JAM, M.Sc. Entrance, JEST, TIFR and GRE in Physics

dz tanh 2 z 
Q114. The value of the integral  C z sin  z
, where C is a circle of radius , traversed
2
counter-clockwise, with centre at z  0 , is
(a) 4 (b) 4i (c) 2i (d) 0
Ans. : (b)
dz tanh 2 z
Solution: C z sin  z
dz i / 2

 i  i i / 4
z  0,1, 1, ,
4 4
 /2 1 0 1  /2
1 2
2 z   2 z    2 z  ....
3 5
 i / 4
f z  3 15
  z
3 3

z  z   ....   i / 2
 3! 

  z 
2 2
2  1 2
 1  z  ...  1   ... 
z 2  2! 
2
b1 

tanh 2 tanh 2
As Re z  1 , and Re z  1 ,
 
i 1 2 
Re z     2cosec h 
4  4 

i 1 2 
Re z     2cosec h 
4  4 

I  2 iR  4i only when 0 lies inside, otherwise wrong question.


Q115. The integral I   e z dz is evaluated from the point  1, 0  to Im z
C

1, 0  along the contour C , which is an arc of the parabola  1, 0  1, 0 
Re z
y  x 2  1 , as shown in the figure. C
The value of I is
(a) 0 (b) 2sinh1 (c) e2i sinh1 (d) e  e1
Ans. : (b)

H.No. 40-D, Ground Floor, Jia Sarai, Near IIT, Hauz Khas, New Delhi-110016
Phone: 011-26865455/+91-9871145498
Website: www.physicsbyfiziks.com | Email: fiziks.physics@gmail.com
61
fiziks
Institute for NET/JRF, GATE, IIT‐JAM, M.Sc. Entrance, JEST, TIFR and GRE in Physics

Solution:  f  z  dz  2 iR
C

1

 f  z  dz   e dx  0
x
C
1

1 1
 e  e   2  2sinh1
1 1

 f  z  dz    e dx   e dx 
x x
C
1 1
2
Q116. In terms of arbitrary constants A and B , the general solution to the differential equation
d2y dy
x2 2
 5 x  3 y  0 is
dx dx
A B
(a) y   Bx 3 (b) y  Ax 
x x3
A B
(c) y  Ax  Bx 3 (d) y  
x x3
Ans. : (d)
Solution: The given equation is Euler-Cauchy differential equation. The characteristic equation
of
d2y dy
x2 2
 5x  6 y  0
dx dx
is, m 2  4m  6  0  m  3 or m  1
1 1
Thus, y1  x 1  and y2  x 2  3
x x
Therefore the general solution is
A B
y 
x x3
d 2 y  x
Q117. The Green’s function G  x, x  for the equation  f  x  , with the boundary values
dx 2
y  0   0 and y 1  0 , is
1
 x 1  x  , 0  x  x  1  x  x  1 , 0  x  x  1
(a) G  x, x    2 (b) G  x, x   
1
 x 1  x  0  x  x  1  x 1  x  0  x  x  1
2

H.No. 40-D, Ground Floor, Jia Sarai, Near IIT, Hauz Khas, New Delhi-110016
Phone: 011-26865455/+91-9871145498
Website: www.physicsbyfiziks.com | Email: fiziks.physics@gmail.com
62
fiziks
Institute for NET/JRF, GATE, IIT‐JAM, M.Sc. Entrance, JEST, TIFR and GRE in Physics

 1
  x 1  x  , 0  x  x  1  x  x  1 , 0  x  x  1
(c) G  x, x    2 (d) G  x, x   
1
 x 1  x  0  x  x  1  x  x  1 0  x  x  1
 2
Ans. : (d)
d2y
Solution:  f  x
dx 2
p  x   1

x1  1, y2  x

x 1 x
y1  x, y2  1  x w  1
1 1

A  1

 A y1 y2  x
G  x, x    
 x  1 0  x  x  1
 A y1 y2  x  x  1 0  x  x  1

Q118. A 4  4 complex matrix A satisfies the relation A† A  4 I , where I is the 4  4 identity


matrix. The number of independent real parameters of A is
(a) 32 (b) 10 (c) 12 (d) 16
Ans. : (d)
1 †
Solution: Given that A† A  4 I 
4

A A I 
Let A  2 B then
A†  2 B †
Therefore, B † B  I
This shows that B is a unitary matrix. The number of independent real parameters
needed to specify an n  n unitary matrix is n 2 . Thus, the number of independent
parameter needed to specify matrix B is 42  16 .
Now, the number of independent parameters needed to specify matrix A is same as that
of matrix B .
Thus the number of independent parameters needed to specify A is 16

H.No. 40-D, Ground Floor, Jia Sarai, Near IIT, Hauz Khas, New Delhi-110016
Phone: 011-26865455/+91-9871145498
Website: www.physicsbyfiziks.com | Email: fiziks.physics@gmail.com
63
fiziks
Institute for NET/JRF, GATE, IIT‐JAM, M.Sc. Entrance, JEST, TIFR and GRE in Physics

Q119. The contour C of the following integral

 z  1 z  3
 dz
z 
3
C
2
 25
in the complex z plane is shown in the figure below.
C

1 3

This integral is equivalent to an integral along the contours

(a) (b)
5 1 3 5 5 1 3 5

(c) (d)
5 1 3 5 5 1 3 5

Ans. : (c)
Solution: z  1,3 are branch points  is not a branch point 1 branch cut 3
1
Q120. The value of the integral 0
x 2 dx , evaluated using the trapezoidal rule with a step size of

0.2 , is
(a) 0.30 (b) 0.39 (c) 0.34 (d) 0.27
Ans. : (c)
0.2
Solution: I  0  2  0.04  0.16  0.36  0.64   1 x f  x
2 
x0 0 0
 0.1 2.4  1  0.34 x1 0.2 0.04
x2 0.4 0.16
x3 0.6 0.36
x4 0.8 0.64
x5 1.0 1.00

H.No. 40-D, Ground Floor, Jia Sarai, Near IIT, Hauz Khas, New Delhi-110016
Phone: 011-26865455/+91-9871145498
Website: www.physicsbyfiziks.com | Email: fiziks.physics@gmail.com
64
fiziks
Institute for NET/JRF, GATE, IIT‐JAM, M.Sc. Entrance, JEST, TIFR and GRE in Physics

CLASSICAL MECHANICS SOLUTIONS

NET/JRF (JUNE-2011)
b
Q1. A particle of unit mass moves in a potential V  x   ax 2 , where a and b are positive
x2
constants. The angular frequency of small oscillations about the minimum of the potential
is
(a) 8b (b) 8a (c) 8a / b (d) 8b / a
Ans. : (b)
1
b V 2b  b 4
Solution: V  x   ax 2  2   0  2ax  3  0  ax 4  b  0  x0    .
x x x a
k  2V
Since   , m  1 and k  2 where x0 is stable equilibrium point.
m x x  x0
1
 2V 6b 6b  b 4
Hence k  2  2a  4  2a   8a at x  x0    .
x x0 b a
a
Thus,   8a .
Q2. The acceleration due to gravity  g  on the surface of Earth is approximately 2.6 times
that on the surface of Mars. Given that the radius of Mars is about one half the radius of
Earth, the ratio of the escape velocity on Earth to that on Mars is approximately
(a) 1.1 (b) 1.3 (c) 2.3 (d) 5.2
Ans. : (c)
Solution: Escape velocity = 2 gR
Escape velocity of Earth ge R e R g
  2.3 where e  2 and e  2.6.
Escape velocity of Mass gm R m Rm gm

Q3. The Hamiltonian of a system with n degrees of freedom is given by H q1 ,.....q n ; p1 ,....... p n ; t  ,

with an explicit dependence on the time t . Which of the following is correct?


(a) Different phase trajectories cannot intersect each other.
(b) H always represents the total energy of the system and is a constant of the motion.
(c) The equations q i  H / pi , p i  H / qi are not valid since H has explicit time
dependence.
(d) Any initial volume element in phase space remains unchanged in magnitude under
time evolution.
H.No. 40-D, Ground Floor, Jia Sarai, Near IIT, Hauz Khas, New Delhi-110016
Phone: 011-26865455/+91-9871145498
Website: www.physicsbyfiziks.com | Email: fiziks.physics@gmail.com
65
fiziks
Institute for NET/JRF, GATE, IIT‐JAM, M.Sc. Entrance, JEST, TIFR and GRE in Physics

Ans. : (a)
Q4. The Lagrangian of a particle of charge e and mass m in applied electric and magnetic
1 2   
fields is given by L  mv  eA  v  e , where A and  are the vector and scalar
2
potentials corresponding to the magnetic and electric fields, respectively. Which of the
following statements is correct?
 
(a) The canonically conjugate momentum of the particle is given by p  mv

p2 e  
(b) The Hamiltonian of the particle is given by H   A  p  e
2m m
(c) L remains unchanged under a gauge transformation of the potentials
(d) Under a gauge transformation of the potentials, L changes by the total time derivative
Ans. : (d)
L 
Solution: 2 V  
t 0

Q5. Consider the decay process         in the rest frame of the   . The masses of the

  ,   and   are M  , M  and zero respectively.

A. The energy of   is

(a)
M 2
 
 M 2 c 2
(b)
M 2
 
 M 2 c 2
(c)  M   M   c 2 (d) M M  c2
2M  2M
Ans. : (b)
Solution:        
From conservation of energy M  c2  E   E  .

E2  p 2 c 2  M 2 c 4 and E2  p 2 c 2 since momentum of   and   is same.

M 2 c 4
M c 2  E  E , M  2 c 4  E2  E2  E  E 
M c 2

E  E 
M 2 c 2
and E  E  M c  E 
2
M2  M 2 c 2
.
 
M 2M
B. The velocity of   is

(a)
M  M 2 c
2

(b)
M  M 2 c
2

(c)
M c
(d)
M c
M 2  M 2 M 2  M 2 M M
Ans. : (a)
H.No. 40-D, Ground Floor, Jia Sarai, Near IIT, Hauz Khas, New Delhi-110016
Phone: 011-26865455/+91-9871145498
Website: www.physicsbyfiziks.com | Email: fiziks.physics@gmail.com
66
fiziks
Institute for NET/JRF, GATE, IIT‐JAM, M.Sc. Entrance, JEST, TIFR and GRE in Physics

Solution: Velocity of  

E 
M 2
  M 2  c 2

M c 2  v2 
 1  2  
4 M 2 M 2
2M   c   M 2  M 2 
2
v2
1
c2
v2 4M 2 M 2 v 2 M 4  M 4  2 M 2 M 2  4 M 2 M 2  M 2  M 2 
  1     v   2 2 
c.
c2  M 2  M 2  c M 2  M 2   M  M 
2 2 2
 

Q6. The Hamiltonian of a particle of unit mass moving in the xy -plane is given to be:
1 2 1 2
H  xp x  yp y  x  y in suitable units. The initial values are given to be
2 2

x0, y0  1,1 and  p x 0, p y 0   1 , 1  . During the motion, the curves traced out
2 2
by the particles in the xy-plane and the p x p y – plane are

(a) both straight lines


(b) a straight line and a hyperbola respectively
(c) a hyperbola and an ellipse, respectively
(d) both hyperbolas
Ans. : (d)
1 2 1 2
Solution: H  xpx  yp y  x  y
2 2
Solving Hamiltonion equation of motion
H H
  p x  px  x   p x and   p y   p y  y   p y .
x y
H H
 x  x  x and  y   y  y .
px p y

After solving these four differential equation and eliminating time t and using boundary
1 1 1
condition one will get  x  and px 
y 2 py

H.No. 40-D, Ground Floor, Jia Sarai, Near IIT, Hauz Khas, New Delhi-110016
Phone: 011-26865455/+91-9871145498
Website: www.physicsbyfiziks.com | Email: fiziks.physics@gmail.com
67
fiziks
Institute for NET/JRF, GATE, IIT‐JAM, M.Sc. Entrance, JEST, TIFR and GRE in Physics

NET/JRF (DEC-2011)
Q7. A double pendulum consists of two point masses m attached by strings of length l as
shown in the figure: The kinetic energy of the pendulum is

(a)
2

1 2 2 2
ml  1   2 
l
(b)
2

ml 21   2  212 cos1   2 
1 2 2 2
 1

(c)
1 2 2
2

ml 1  222  212 cos1   2   m

l
(d) 
ml 2 1   2  212 cos 1   2 
1 2 2 2
2
 2

Ans. : (b) m

Solution: Let co-ordinate  x1 , y1  and  x 2 , y 2  . K .E. 


1
2
 1
 
m x12  y 12  m x 22  y 22
2

x1  l sin  1 , y1  l cos 1  x1  l cos 11 , y1  l sin 11

x 2  l sin  1  l sin  2 , y 2  l cos 1  l cos  2

 
 x 2  l cos 11  l cos  2 2 , y 2  l  sin 11  l  sin  2 2
Put the value of x1 , y1 , x 2 , y 2 in K.E equation, one will get
1 2 2 2
T ml  21   2  212 cos 1   2   .
2
Q8. A constant force F is applied to a relativistic particle of rest mass m. If the particle starts
from rest at t = 0, its speed after a time t is

(a) Ft / m
 Ft 
(b) c tanh  
(c) c 1  e  Ft / mc  (d)
Fct
 mc  F 2t 2  m 2c 2
Ans. : (d)
dp
Solution:  F  p  Ft  c . At t  0, p  0 so, c  0
dt
F
mu  t Fct
 p  Ft   Ft  u  m  .
u2  Ft 
2
F 2 t 2  m2 c 2
1 2 1  
c  mc 

H.No. 40-D, Ground Floor, Jia Sarai, Near IIT, Hauz Khas, New Delhi-110016
Phone: 011-26865455/+91-9871145498
Website: www.physicsbyfiziks.com | Email: fiziks.physics@gmail.com
68
fiziks
Institute for NET/JRF, GATE, IIT‐JAM, M.Sc. Entrance, JEST, TIFR and GRE in Physics

Q9. The potential of a diatomic molecule as a function of the distance r between the atoms is
a b
given by V r     12 . The value of the potential at equilibrium separation between
r 6
r
the atoms is:
(a)  4 a 2 / b (b)  2 a 2 / b (c)  a 2 / 2b (d)  a 2 / 4b
Ans. : (d)
a b V 1  12b 
Solution: V r   
a 12b
 , for equilibrium  0     6    0  6 a  0
r 6 r 12 r r 7
r 13
r 
7
r 6 
1 1
12b  12b  6  2b  6
 6a  6  0  r    r  
r  6a   a 
 1

 2b  a b a2 a2 a2
V r      
6
      .
  a    2b   2b 2 2b 4b 4b
     
 a   a 
Q10. Two particles of identical mass move in circular orbits under a central potential

V r  
1 2
kr . Let l1 and l2 be the angular momenta and r1 , r2 be the radii of the orbits
2
l1 r
respectively. If  2 , the value of 1 is:
l2 r2

(a) 2 (b) 1 / 2 (c) 2 (d) 1 / 2


Ans. : (a)
l2 1
Solution: Veff   kr 2 , where l is angular momentum.
2mr 2
2
Veff l2
Condition for circular orbit  0   3  kr  0  l 2  r 4  l  r 2 .
r mr
2
l r  r l r l
Thus 1   1   1  1  1  2 since 1  2 .
l2  r2  r2 l2 r2 l2

Q11. A particle of mass m moves inside a bowl. If the surface of the bowl is given by the

equation z 
1
2
 
a x 2  y 2 , where a is a constant, the Lagrangian of the particle is

(a)
1
2

m r 2  r 2 2  gar 2  (b)
1
2
 
m 1  a 2 r 2 r 2  r 2 2 
(c)
1
2

m r 2  r 2 2  r 2 sin 2  2  gar 2  (d)
1
2
 
m 1  a 2 r 2 r 2  r 2 2  gar 2 
H.No. 40-D, Ground Floor, Jia Sarai, Near IIT, Hauz Khas, New Delhi-110016
Phone: 011-26865455/+91-9871145498
Website: www.physicsbyfiziks.com | Email: fiziks.physics@gmail.com
69
fiziks
Institute for NET/JRF, GATE, IIT‐JAM, M.Sc. Entrance, JEST, TIFR and GRE in Physics

Ans. : (d)

Solution: L 
1
2
  1

m x 2  y 2  z 2  mgz , where z  a x 2  y 2 .
2

a r2  .
1
It has cylindrical symmetry. Thus x  r cos  , y  r sin  , z 
2
x  r cos   r sin  , y  r sin   r cos  and z  a  rr  .

m 1  a 2 r 2  r 2  r 22  gar 2  .


1
So, L 
2
Q12. A planet of mass m moves in the inverse square central force field of the Sun of mass
M . If the semi-major and semi-minor axes of the orbit are a and b , respectively, the
total energy of the planet is:
GMm 1 1
(a)  (b)  GMm  
ab a b
GMm  1 1   ab 
(c)     (d)  GMm 

a b a  a  b 
2

Ans. : (a)
Solution: Assume Sun is at the centre of elliptical orbit.
1 2 GMm 1 2 GMm
Conservation of energy mv1   mv2 
2 a 2 b v2

Conservation of momentum L  mv1a  mv2b b


v1
s a
 a
v2  v1  
 b
1 2 1 2 GMm GMm 1  2 2 a 
2
ba
mv1  mv2    m  v1  v1 2   GMm  
2 2 a b 2  b   ab 

1 2  b2  a 2   b  a 1 b 1
mv1    GMm    mv12  GMm   
 b   ab   a  b  a 
2
2 2

1 2 GMm b 1 GMm
E mv1   GMm 
2 a a b  a  a

GMm  b  GMm  b  b  a  GMm


   1     
a  b  a    a  b  a   b  a 

H.No. 40-D, Ground Floor, Jia Sarai, Near IIT, Hauz Khas, New Delhi-110016
Phone: 011-26865455/+91-9871145498
Website: www.physicsbyfiziks.com | Email: fiziks.physics@gmail.com
70
fiziks
Institute for NET/JRF, GATE, IIT‐JAM, M.Sc. Entrance, JEST, TIFR and GRE in Physics

Q13. An annulus of mass M made of a material of uniform density has inner and outer radii a
and b respectively. Its principle moment of inertia along the axis of symmetry
perpendicular to the plane of the annulus is:
1 b 4  a 4 
(a) M 2
2 b  a 2 
(b)
1
2

M b 2  a 2 
(c) M b 2  a 2   
1 1
(d) M b 2  a 2
2 2
Ans. : (d)
Q14. The trajectory on the zpz - plane (phase-space trajectory) of a ball bouncing perfectly
elastically off a hard surface at z = 0 is given by approximately by (neglect friction):

PZ PZ
(a) (b)

z z

PZ PZ

(c) (d)

z z

Ans. : (a)
Pz2 P2
Solution: H   mgz and E  z  mgz .
2m 2m

H.No. 40-D, Ground Floor, Jia Sarai, Near IIT, Hauz Khas, New Delhi-110016
Phone: 011-26865455/+91-9871145498
Website: www.physicsbyfiziks.com | Email: fiziks.physics@gmail.com
71
fiziks
Institute for NET/JRF, GATE, IIT‐JAM, M.Sc. Entrance, JEST, TIFR and GRE in Physics

NET/JRF (JUNE-2012)

Q15. The bob of a simple pendulum, which undergoes small oscillations, is immersed in water.
Which of the following figures best represents the phase space diagram for the pendulum?

p p
(a) (b)

x x

p p
(c) (d)

x x

Ans. : (d)
Solution: When simple pendulum oscillates in water it is damped oscillation so amplitude
continuously decrease and finally it stops.
Q16. Two events separated by a (spatial) distance 9  109 m , are simultaneous in one inertial
frame. The time interval between these two events in a frame moving with a constant
speed 0.8 c (where the speed of light c  3  108 m / s ) is
(a) 60 s (b) 40 s (c) 20 s (d) 0 s
Ans. : (b)
Solution: x2'  x1'  9  10 9 m and t 2'  t1'  0 . Then

   
 t '  v x'   t1  v x' 
 2 c2 2
t 2  t1  
 

1 1 ' '
 '

c 2   t  t  t 2  t1  v x 2  x1  v x 2  x1 .
' ' '
 
  2 
2 1
v2 c c2
2
 1 v
2
  1 v  v2 v2
    1  1  1 
 c2   c2  c2 c2 c2
Put v  0.8c  t 2  t1  40 sec

H.No. 40-D, Ground Floor, Jia Sarai, Near IIT, Hauz Khas, New Delhi-110016
Phone: 011-26865455/+91-9871145498
Website: www.physicsbyfiziks.com | Email: fiziks.physics@gmail.com
72
fiziks
Institute for NET/JRF, GATE, IIT‐JAM, M.Sc. Entrance, JEST, TIFR and GRE in Physics

x 2
Q17. If the Lagrangian of a particle moving in one dimensions is given by L   V  x  the
2x
Hamiltonian is
x 2 p2
xp  V  x   V x  x  V  x   V x 
1 2 1 2
(a) (b) (c) (d)
2 2x 2 2x
Ans. : (a)
L x
Solution: Since H  p x x  L and  p x   p x  x  p x x .
x x

 px x 
2
x 2 p x2 x
H  p x x   V  x   H  px  px x   Vx  H   V x  .
2x 2x 2
Q18. A horizontal circular platform mutes with a constant angular velocity  directed
vertically upwards. A person seated at the centre shoots a bullet of mass m horizontally
with speed v. The acceleration of the bullet, in the reference frame of the shooter, is
(a) 2v  to his right (b) 2v  to his left
(c) v  to his right (d) v  to his left
Ans. : (a)

Solution: Velocity of bullet = vˆj , Angular velocity= k̂ . There will be coriolis

 
force F  2m v   .

F  2mviˆ  a  2viˆ .
 
Q19. The Poisson bracket  r , p  has the value
 
(a) r p (b) rˆ  pˆ (c) 3 (d) 1

Ans. : (b)
 
Solution: r  xiˆ  yjˆ  zkˆ , r   x 2  y 2  z 2  , p  p xiˆ  p y ˆj  p z kˆ ,
1/ 2


p   px2  p y2  pz2 
1/ 2

           
   r  p  r  p   r  p  r  p   r  p  r  p 
 r , p  =  x  p  p  x    y  p  p  y    z  p  p  y 
 x x   y y   z z 
 
x px y p y z pz rp
              rˆ  pˆ 
r p r p r p r p

H.No. 40-D, Ground Floor, Jia Sarai, Near IIT, Hauz Khas, New Delhi-110016
Phone: 011-26865455/+91-9871145498
Website: www.physicsbyfiziks.com | Email: fiziks.physics@gmail.com
73
fiziks
Institute for NET/JRF, GATE, IIT‐JAM, M.Sc. Entrance, JEST, TIFR and GRE in Physics

Q20. Consider the motion of a classical particle in a one dimensional double-well

potential V  x   
1 2
4
2

x  2 . If the particle is displaced infinitesimally from the minimum

on the x-axis (and friction is neglected), then


(a) the particle will execute simple harmonic motion in the right well with an angular

frequency   2

(b) the particle will execute simple harmonic motion in the right well with an angular

frequency   2

(c) the particle will switch between the right and left wells

(d) the particle will approach the bottom of the right well and settle there

Ans. : (b)

Solution: V  x   x  2 2  V  2 x 2  2  2 x  0  x  0 , x   2 .
1 2
 
4 x 4
 2V  2V
 3 x 2
 2 . At x  0 ,  0 so V is maximum. Thus it is unstable point
x 2 x 2

 2V
 2V x 2 x  x0
 4 and it is stable equilibrium point with   2    1.
x 2 x 2

Q21. What is proper time interval between the occurrence of two events if in one inertial frame
events are separated by 7.5 108 m and occur 6.5 s apart?
(a) 6.50 s (b) 6.00 s (c) 5.75 s (d) 5.00 s
Ans. : (b)
Solution: Proper time interval
2

 6.5     6 sec.
r2 7.5
t   t    2 
2 2

c  3 

H.No. 40-D, Ground Floor, Jia Sarai, Near IIT, Hauz Khas, New Delhi-110016
Phone: 011-26865455/+91-9871145498
Website: www.physicsbyfiziks.com | Email: fiziks.physics@gmail.com
74
fiziks
Institute for NET/JRF, GATE, IIT‐JAM, M.Sc. Entrance, JEST, TIFR and GRE in Physics

NET/JRF (DEC-2012)

Q22. A solid cylinder of height H, radius R and density ρ, floats vertically on the surface of a
liquid of density  0 . The cylinder will be set into oscillatory motion when a small
instantaneous downward force is applied. The frequency of oscillation is

g  g g 0 g
(a) (b) (c) (d)
0 H 0 H 0H H
Ans. : (d)
Solution: From Newton’s law of motion ma  mg  0 Agh where A is area of cross section,

m   AH .

0 gh 0 g
  AHa   AHg  0 Agh  a  1  
H H
Q23. Three particles of equal mass (m) are connected by two identical massless springs of
stiffness constant (K) as shown in the figure
K K

m m m
If x1, x2 and x3 denote the horizontal displacement of the masses from their respective
equilibrium positions the potential energy of the system is

(a)
1
2

K x12  x 22  x32  (b)
1
2

K x12  x 22  x32  x 2  x1  x3  
(c)
1
2

K x12  2 x 22  x32  2 x 2  x1  x3  (d)
1
2
 
K x12  2 x 22  2 x 2  x1  x3 
Ans. : (c)
1 1
K  x2  x1   K  x3  x2  ,
2 2
Solution: V 
2 2

K  x22  x12  2 x2 x1   K  x32  x22  2 x3 x2   V  K  x12  2 x22  x32  2 x2  x1  x3  


1 1 1
V
2 2 2
Q24. A planet of mass m moves in the gravitational field of the Sun (mass M ). If the semi-
major and semi-minor axes of the orbit are a and b respectively, the angular momentum
of the planet is

2GMm 2 ab 2GMm 2 ab
(a) 2GMm 2 a  b  (b) 2GMm 2 a  b  (c) (d)
ab ab

H.No. 40-D, Ground Floor, Jia Sarai, Near IIT, Hauz Khas, New Delhi-110016
Phone: 011-26865455/+91-9871145498
Website: www.physicsbyfiziks.com | Email: fiziks.physics@gmail.com
75
fiziks
Institute for NET/JRF, GATE, IIT‐JAM, M.Sc. Entrance, JEST, TIFR and GRE in Physics

Ans. : (d)
Solution: Assume Sun is at the centre of elliptical orbit.
1 2 GMm 1 2 GMm
Conservation of energy mv1   mv2  v2
2 a 2 b
Conservation of momentum L  mv1a  mv2b b
v1
s a
 a
v2  v1  
 b

1 2 1 2 GMm GMm 1  2 2 a 
2
 b  a
mv1  mv2    m  v1  v1 2   GMm 
2 2 a b 2  b   ab 

1 2  b2  a 2   b  a 1  b 1
mv1    GMm    mv12  GMm   
2  b 2
  ab  2  a  b  a

 b 1
v1  2GM   
 a  b  a

 b  1  2GMab 2GMm 2 ab
L  mv1 a  m 2GM      a  m L
 a  b  a b  a ab

Q25. The Hamiltonian of a simple pendulum consisting of a mass m attached to a massless


p2
string of length l is H   mgl 1  cos   . If L denotes the Lagrangian, the value of
2ml 2
dL
is:
dt
2g g
(a)  p sin  (b)  p sin 2
l l
g
(c) p cos  (d) lp2 cos 
l
Ans. : (a)
dL L p2
Solution:  L, H   where H   mgl 1  cos   .
dt t 2ml 2
H p ml 2 2
L   pi qi  H  p   H ,  
    , L  mgl 1  cos   .
i P ml 2 2

H.No. 40-D, Ground Floor, Jia Sarai, Near IIT, Hauz Khas, New Delhi-110016
Phone: 011-26865455/+91-9871145498
Website: www.physicsbyfiziks.com | Email: fiziks.physics@gmail.com
76
fiziks
Institute for NET/JRF, GATE, IIT‐JAM, M.Sc. Entrance, JEST, TIFR and GRE in Physics

Hence we have to calculate L, H  which is only defined into phase space i.e. p and  .

p2
Then  L   mgl 1  cos  
2ml 2

L, H   L  H 
L H
 
2g
p sin  and
L
0
dL

2g
p sin 
 p p  l t dt l
Q26. Two bodies of equal mass m are connected by a massless rigid rod of length l lying in the
xy-plane with the centre of the rod at the origin. If this system is rotating about the z-axis
with a frequency ω, its angular momentum is
(a) ml 2 / 4 (b) ml 2 / 2 (c) ml 2 (d) 2ml 2
Ans. : (b)
Solution: Since rod is massless i.e. M  0 .
l
Moment of inertia of the system I  m1r12  m2 r22 , m1  m2  m and r1  r2 
2
ml 2 ml 2 ml 2 ml 2
I  I . Angular momentum, J  I and J  .
4 4 2 2
Q27. Which of the following set of phase-space trajectories is not possible for a particle
obeying Hamilton’s equations of motion?
(a) (b)
P P

x x
(c) (d)
P P

x x
Ans. : (b)
Solution: Phase curve does not cut each other
H.No. 40-D, Ground Floor, Jia Sarai, Near IIT, Hauz Khas, New Delhi-110016
Phone: 011-26865455/+91-9871145498
Website: www.physicsbyfiziks.com | Email: fiziks.physics@gmail.com
77
fiziks
Institute for NET/JRF, GATE, IIT‐JAM, M.Sc. Entrance, JEST, TIFR and GRE in Physics

Q28. The muon has mass 105 MeV / c 2 and mean life time 2.2  s in its rest frame. The mean
distance traversed by a muon of energy 315 MeV before decaying is approximately,

(a) 3  105 km (b) 2.2 cm (c) 6.6  m (d) 1.98 km


Ans. : (d)
MeV
Solution: Since E  315MeV and m0  105 .
c2
m0 c 2 m0 c 2 105
E  mc 2  E   315   315   v  0.94c .
v2 v2 v2
1 2 1 2 1 2
c c c
t0 2.2  10 6
Now, t  , t 0  2.2 s  t   t  6.6  s
v2 8
1 2 1
c 9

Now the distance traversed by muon is vt  0.94c  6.6  10 6  1.86 km .

NET/JRF (JUNE-2013)

Q29. The area of a disc in its rest frame S is equal to 1 (in some units). The disc will appear
distorted to an observer O moving with a speed u with respect to S along the plane of the
disc. The area of the disc measured in the rest frame of the observer O is ( c is the speed
of light in vacuum)
1/ 2 1 / 2 1
 u2   u2   u2   u2 
(a) 1  2  (b) 1  2  (c) 1  2  (d) 1  2 
 c   c   c   c 
Ans. : (a)
Solution: Area of disc from S frame is 1 i.e.  a 2  1 or  a  a  1
u2 u2 u2
Area of disc from S  frame is  a  b   a  a 1   1  1   1 
c2 c2 c2
u2
where b  a 1  .
c2

H.No. 40-D, Ground Floor, Jia Sarai, Near IIT, Hauz Khas, New Delhi-110016
Phone: 011-26865455/+91-9871145498
Website: www.physicsbyfiziks.com | Email: fiziks.physics@gmail.com
78
fiziks
Institute for NET/JRF, GATE, IIT‐JAM, M.Sc. Entrance, JEST, TIFR and GRE in Physics

Q30. A planet of mass m and an angular momentum L moves in a circular orbit in a potential,
V r   k / r , where k is a constant. If it is slightly perturbed radially, the angular
frequency of radial oscillations is

(a) mk 2 / 2 L3 (b) mk 2 / L3 (c) 2mk 2 / L3 (d) 3mk 2 / L3


Ans. : (b)
Veff
L2 k Veff L2 k
Solution: Veff   . For circular orbit    2 0
2mr 2
r r mr 3
r

L2 k L2 k
  2 . Thus r  r0    ,
mr 3
r mk m
r
d 2Veff 3L2
2k 3L 2
2k 3m k 3
2m k
4
m k43 4 3
k   3      6
dr 2
mr 4
r r  r0  L2 
4
 L2 
3
L6 L6 L
r  r0
m   
 mk   mk 

d 2V
dr 2 r  r0 mk 2
   .
m L3
Q31. The number of degrees of freedom of a rigid body in d space-dimensions is
(a) 2d (b) 6 (c) d d  1 / 2 (d) d!
Ans. : (c)
Q32. A system is governed by the Hamiltonian
1
 p x  ay 2  1  p x  bx 2
H 
2 2
where a and b are constants and p x , p y are momenta conjugate to x and y respectively.

For what values of a and b will the quantities  p x  3 y  and  p y  2 x  be conserved?

(a) a  3, b  2 (b) a  3, b  2


(c) a  2, b  3 (d) a  2, b  3
Ans. : (d)
Solution: Poisson bracket  px  3 y, H   0 and
 p y  2 y , H   0
p y (b  3)  x(3b  b 2 )  0 and p x ( a  2)  y (2 a  a 2 )  0

 a  2, b  3

H.No. 40-D, Ground Floor, Jia Sarai, Near IIT, Hauz Khas, New Delhi-110016
Phone: 011-26865455/+91-9871145498
Website: www.physicsbyfiziks.com | Email: fiziks.physics@gmail.com
79
fiziks
Institute for NET/JRF, GATE, IIT‐JAM, M.Sc. Entrance, JEST, TIFR and GRE in Physics

Q33. The Lagrangian of a particle of mass m moving in one dimension is given by


1 2
L mx  bx
2
where b is a positive constant. The coordinate of the particle xt  at time t is given by: (in
following c1 and c 2 are constants)
b 2
(a)  t  c1t  c 2 (b) c1t  c 2
2m
 bt   bt   bt   bt 
(c) c1 cos   c 2 sin   (d) c1 cosh   c 2 sinh  
m m m m
Ans. : (a)
d  L  L d
Solution: Equation of motion    0  mx   b  0  mx  b  0  mx  b
dt  x  x dt

d 2x b dx b b t2
     t  c1  x    c1t  c2
dt 2 m dt m m 2

NET/JRF (DEC-2013)
Q34. Let A, B and C be functions of phase space variables (coordinates and momenta of a
mechanical system). If ,  represents the Poisson bracket, the value of
A, B, C  A, B, C is given by
(a) 0 (b) B, C , A (c) A, C , B (d) C , A, B
Ans. : (d)
Solution: We know that Jacobi identity equation
A, B, C  B, C , A  C , A, B  0
Now A, B, C   A, B, C   B, C , A  C , A, B

z2
Q35. A particle moves in a potential V  x 2  y 2  . Which component(s) of the angular
2
momentum is/are constant(s) of motion?
(a) None (b) Lx , L y and L z (c) only L x and Ly (d) only L z

Ans. : (d)

H.No. 40-D, Ground Floor, Jia Sarai, Near IIT, Hauz Khas, New Delhi-110016
Phone: 011-26865455/+91-9871145498
Website: www.physicsbyfiziks.com | Email: fiziks.physics@gmail.com
80
fiziks
Institute for NET/JRF, GATE, IIT‐JAM, M.Sc. Entrance, JEST, TIFR and GRE in Physics

z2
Solution: A particle moves in a potential V  x 2  y 2 
2
r2
V r ,  ,    r 2 sin 2  cos 2   r 2 sin 2  sin 2   cos 2 
2
r2
V r ,  ,    r sin   cos 2 
2 2

2
Now  is cyclic-co-ordinate  p  i.e Lz is constant of motion.

Q36. The Hamiltonian of a relativistic particle of rest mass m and momentum p is given

by H  p 2  m 2  V x  , in units in which the speed of light c  1 . The corresponding


Lagrangian is

(a) L  m 1  x 2  V  x  (b) L  m 1  x 2  V  x 

(c) L  1  mx 2  V  x  mx  V  x 


1 2
(d) L 
2
Ans. : (b)
H
Solution: H  p 2  m 2  V x  
p
 x 
1 2p
1

 x p 2  m 2 
1/ 2
p
2
p 2
m 
2 2


xm
 p
1  x 2

Now L   xp  H  xp  H  xp


  p 2  m2  V  x 


xm
Put value p   L  m 1  x 2  V  x 
1  x 2

Q37. A pendulum consists of a ring of mass M and radius R suspended by a massless rigid
rod of length l attached to its rim. When the pendulum oscillates in the plane of the ring,
the time period of oscillation is

lR 2
(a) 2
g
(b) l 2
 R2 1/ 4

2 R 2  2 Rl  l 2 2
(c) 2
g R  l 
(d) 2R 2
 2 Rl  l 2  1/ 4

g
Ans. : (c)
Solution: The moment of inertia about pivotal point is given by

H.No. 40-D, Ground Floor, Jia Sarai, Near IIT, Hauz Khas, New Delhi-110016
Phone: 011-26865455/+91-9871145498
Website: www.physicsbyfiziks.com | Email: fiziks.physics@gmail.com
81
fiziks
Institute for NET/JRF, GATE, IIT‐JAM, M.Sc. Entrance, JEST, TIFR and GRE in Physics

I  I c.m  Md 2  MR 2  M (l  R) 2

If ring is displaced by angle  then potential energy is  Mg (l  R ) cos 


The Lagrangian is given by
1 2 1
L I  V ( ) = ( MR 2  M (l  R) 2 ) 2  Mg (l  R) cos 
2 2
d  L   L  2 
   0  ( MR  M (l  R) )  Mg (l  R ) sin   0
2

dt      

For small oscillation sin     ( MR 2  M (l  R) 2 )  Mg (l  R)  0

2 R 2  2 Rl  l 2
Time period is given by 2 .
g R  l 
Q38. Consider a particle of mass m attached to two identical springs y
each of length l and spring constant k (see the figure). The
equilibrium configuration is the one where the springs are
o
unstretched. There are no other external forces on the system. If the
particle is given a small displacement along the x -axis, which of
the following describes the equation of motion for small x
o
oscillations?
kx 3 kx 2
(a) mx  0 (b) mx  kx  0 (c) mx  2kx  0 (d) mx  0
l2 l
Ans. : (a)
Solution: The lagrangian of system is given by y
1 2
L mx  V ( x)
2 x
o
The potential energy is given by
2 2
k 1
 k 1

2

V ( x)   x 2  l 2  2

 l    x2  l 2  2  l
 2  o x
2
 1


V ( x)  k  x 2  l 2  2  l
 
For small oscillation one can approximate potential by Taylor expansion

H.No. 40-D, Ground Floor, Jia Sarai, Near IIT, Hauz Khas, New Delhi-110016
Phone: 011-26865455/+91-9871145498
Website: www.physicsbyfiziks.com | Email: fiziks.physics@gmail.com
82
fiziks
Institute for NET/JRF, GATE, IIT‐JAM, M.Sc. Entrance, JEST, TIFR and GRE in Physics

2
 1
 2
2 
 x 
2 2
  1 x 2 1 x 4  
V ( x)  kl  1  2   1  V ( x)  kl 1  2  4   1
2
 
 l   2 l 8 l  
 
2
kl 2  x   x 4
2
V ( x)     V ( x )  k  2 .
4  l2   4l 

1 2  x 4
So Lagrangian of system is given by L  mx  k  2 
2  4l 

d  L   L  kx 3
The Lagranges equation of motion      0  m
x   0.
dt  x   x  l2

NET/JRF (JUNE-2014)

Q39. The time period of a simple pendulum under the influence of the acceleration due to
gravity g is T . The bob is subjected to an additional acceleration of magnitude 3 g in
the horizontal direction. Assuming small oscillations, the mean position and time period
of oscillation, respectively, of the bob will be
(a) 0 o to the vertical and 3T (b) 30 o to the vertical and T / 2

(c) 60 o to the vertical and T / 2 (d) 0 o to the vertical and T / 3


Ans. : (c)
l
Solution: T  2
g  T

g   3g 2  g 2  4g 2  2 g
3g
l l 1 T 
T   2  T   2   T  g
2g g 2 2 g

T cos  mg , T sin   3 mg  tan   3    60o

H.No. 40-D, Ground Floor, Jia Sarai, Near IIT, Hauz Khas, New Delhi-110016
Phone: 011-26865455/+91-9871145498
Website: www.physicsbyfiziks.com | Email: fiziks.physics@gmail.com
83
fiziks
Institute for NET/JRF, GATE, IIT‐JAM, M.Sc. Entrance, JEST, TIFR and GRE in Physics

1 
Q40. A particle of mass m and coordinate q has the Lagrangian L  mq 2  qq 2 , where 
2 2
is a constant. The Hamiltonian for the system is given by
p 2 qp 2 p2
(a)  (b)
2m 2 m 2 2m  q 

p2 qp 2 pq
(c)  (d)
2m 2m  q 2 2

Ans. : (b)
1 2  2
Solution: H   qp
  L where L  mq  qq
2 2
L p
 p  mq   qq  p  q  m   q   q 
q m  q

 H  qp
 L 
p2 1
 m
p2


q 
 
p2
 m   q  2  m   q 2 2  m   q 2
p2 p2
 H  qp
 L    m  q 
 m   q  2  m   q 2
p2 p2 p2
 H  qp
 L   H 
 m  q  2  m  q  2 m  q 

Q41. The coordinates and momenta xi , pi i  1, 2, 3 of a particle satisfy the canonical Poisson

bracket relations xi , p j    ij . If C1  x2 p3  x3 p2 and C2  x1 p2  x2 p1 are constants of

motion, and if C3  C1 , C 2   x1 p3  x3 p1 , then

(a) C2 , C3   C1 and C3 , C1  C2 (b) C2 , C3   C1 and C3 , C1  C2

(c) C2 , C3   C1 and C3 , C1   C2 (d) C2 , C3   C1 and C3 , C1  C2

Ans. : (d)
Solution: C1  x2 p3  x3 p2 , C2  x1 p2  x2 p1 , C3  x1 p3  x3 p1

 C2 C3 C2 C3   C2 C3 C2 C3   C2 C3 C2 C3 
C2 , C3         
 x1 p1 p1 x1   x2 p2 p2 x2   x3 p3 p3 x3 

C2 , C3    p2 x3    x2  p3    0  x1  0    0  x1  0  p1    p2 x3  x2 p3   C1

H.No. 40-D, Ground Floor, Jia Sarai, Near IIT, Hauz Khas, New Delhi-110016
Phone: 011-26865455/+91-9871145498
Website: www.physicsbyfiziks.com | Email: fiziks.physics@gmail.com
84
fiziks
Institute for NET/JRF, GATE, IIT‐JAM, M.Sc. Entrance, JEST, TIFR and GRE in Physics

 C3 C1 C3 C1   C3 C1 C3 C1   C3 C1 C3 C1 
C3 , C1        

 1x p1 p1 x1   x2 p2 p 2 x2   x3 p3 p3 x3 

C3 , C1   p3  0  x3  0    0  x3  0  p3    p1 x2  x1 p2     x1 p2  x2 p1   C2


Q42. The recently-discovered Higgs boson at the LHC experiment has a decay mode into a
photon and a Z boson. If the rest masses of the Higgs and Z boson are 125 GeV/c2 and

90 GeV/c2 respectively, and the decaying Higgs particle is at rest, the energy of the
photon will approximately be
(a) 35 3 GeV (b) 35GeV (c) 30GeV (d) 15GeV
Ans. : (c)
Solution: H B  PH  Z B
   
From conservation of momentum 0  P1  P2  P1   P2  P1  P2

Now EH B  EPH  EZ B  EPH  EZ B  M H B c 2

E P2H  P12 c 2  0 and EZ2B  P22 c 2  M Z2B c 4


 EZ B  EPH  E ZB 
 EPH  M Z2B c 4  P1  P2

M Z2B c 4 M Z2B c 2
 EZ B  EPH    EZ B  EPH  M H B c 2
M HB c 2
M HB

 2 EPH  M H B c  2
M z2B c 2
 EPH 
M 2
HB 
 M z2B c 2
M HB M HB

 125 125  90  90  c
4
 EPH    4  30.1GeV

 2 125  c
Q43. A canonical transformation relates the old coordinates q, p  to the new ones Q, P  by

the relations Q  q 2 and P  p / 2q . The corresponding time independent generating


function is
(a) P / q 2 (b) q 2 P (c) q 2 / P (d) qP 2
Ans. : (b)
Solution: Q  q 2 ; P  p / 2q

H.No. 40-D, Ground Floor, Jia Sarai, Near IIT, Hauz Khas, New Delhi-110016
Phone: 011-26865455/+91-9871145498
Website: www.physicsbyfiziks.com | Email: fiziks.physics@gmail.com
85
fiziks
Institute for NET/JRF, GATE, IIT‐JAM, M.Sc. Entrance, JEST, TIFR and GRE in Physics

F2 F
 p  2  P  2q  F2  q 2 P  f ( P)
q q
F2
 Q  q 2  F2  q 2 P  f (q)
P
comparing both side f (q )  f ( P)  0  F2  q 2 P

NET/JRF (DEC-2014)
Q44. The equation of motion of a system described by the time-dependent Lagrangian
1 
L  e t  mx 2  V  x   is
2 
dV dV
(a) mx  mx  0 (b) mx  mx  0
dx dx
dV dV
(c) mx  mx  0 (d) mx  0
dx dx
Ans. : (a)
1  L L V  t
Solution:  L  e t  mx 2  V  x     e t mx and  e
2  x x x
d  L  L d V  t V  t
     0   e t mx     t  mx e t 
e  mxe e 0
dt  x  x dt x x

 V   t V
 mx  m x   e  0  mx   mx  0
x x

A particle of mass m is moving in the potential V  x    ax 2  bx 4 where a, b are


1 1
Q45.
2 4
positive constants. The frequency of small oscillations about a point of stable equilibrium
is
(a) a/m (b) 2a / m (c) 3a / m (d) 6a / m
Ans. : (b)
1 1
Solution: V  x    ax 2  bx 4
2 4
1
V  a 2
 0  ax  bx3  0  x  a  bx 2   0  x     , 0
x b

H.No. 40-D, Ground Floor, Jia Sarai, Near IIT, Hauz Khas, New Delhi-110016
Phone: 011-26865455/+91-9871145498
Website: www.physicsbyfiziks.com | Email: fiziks.physics@gmail.com
86
fiziks
Institute for NET/JRF, GATE, IIT‐JAM, M.Sc. Entrance, JEST, TIFR and GRE in Physics

 2V  2V
  a  3bx 2  At x  0,  a (Negative so it is unstable point)
x 2
x 2
 2V a
  a  3b  2a (Positive so it is stable point)
x 2 b
1
 a 2
x   
 b

 2V
  x 2  2a
m m
Q46. The radius of Earth is approximately 6400 km . The height h at which the acceleration
due to Earth’s gravity differs from g at the Earth’s surface by approximately 1 % is
(a) 64 km (b) 48 km (c) 32 km (d) 16 km
Ans. : (c)
g 2h g 2h g 2h
Solution:  1  1     h  32 k .m.
g R g R g R
Q47. According to the special theory of relativity, the speed v of a free particle of mass m and
total energy E is:

mc 2 2 E  mc 2 
(a) v  c 1  (b) v  1  
E m  E 
2
 mc 2   mc 2 
(c) v  c 1    (d) v  c1  
 E   E 

Ans. : (c)
2 2
v 2  mc 2 
mc 2 v2 m2c 4  mc 2 
Solution: E   1 2      1   v  c 1   
v2 c  E  c2 E2  E 
1 2
c
p2
Q48. The Hamiltonian of a classical particle moving in one dimension is H   q 4 where
2m
 is a positive constant and p and q are its momentum and position respectively. Given
that its total energy E  E 0 the available volume of phase space depends on E 0 as

(a) E 03 / 4 (b) E 0

(c) E0 (d) is independent of E 0

H.No. 40-D, Ground Floor, Jia Sarai, Near IIT, Hauz Khas, New Delhi-110016
Phone: 011-26865455/+91-9871145498
Website: www.physicsbyfiziks.com | Email: fiziks.physics@gmail.com
87
fiziks
Institute for NET/JRF, GATE, IIT‐JAM, M.Sc. Entrance, JEST, TIFR and GRE in Physics

Ans. : (a)
V q
p2
Solution: H    q4
2m E0
Phase area   p  dq q
p
1 2mE0
E 4
A   p  dq   2mE   
    E0 / 
1/4  E0 / 1/4
AE 1/2
E
1/4
 AE 3/4  2mE0
0 0 0

p2 1
Q49. A mechanical system is described by the Hamiltonian H q, p    m 2 q 2 . As a
2m 2
Q
result of the canonical transformation generated by F q, Q    , the Hamiltonian in
q
the new coordinate Q and momentum P becomes

1 2 2 m 2 2 1 2 2 m 2 2
(a) Q P  Q (b) Q P  P
2m 2 2m 2
1 2 m 2 2 1 2 4 m 2  2
(c) P  Q (d) Q P  P
2m 2 2m 2
Ans. : (d)
p2 1 Q
Solution: H   m 2 q 2 , F  F1  q, Q   
2m 2 q
F1 Q
 p  2 p (i)
q q
F1 1 1
  P    P  q  (ii)
Q q P
1
From equation (i) and (ii)  p  QP 2 q 
P
p2 1 Q2 P4 1  1  1 2 4 1
H  m 2 q 2   m 2  2   Q P  m 2 P 2
2m 2 2m 2  P  2m 2

H.No. 40-D, Ground Floor, Jia Sarai, Near IIT, Hauz Khas, New Delhi-110016
Phone: 011-26865455/+91-9871145498
Website: www.physicsbyfiziks.com | Email: fiziks.physics@gmail.com
88
fiziks
Institute for NET/JRF, GATE, IIT‐JAM, M.Sc. Entrance, JEST, TIFR and GRE in Physics

Q50. The probe Mangalyaan was sent recently to explore the planet Mars. The inter-planetary
part of the trajectory is approximately a half-ellipse with the Earth (at the time of launch),
Sun and Mars (at the time the probe reaches the destination)
forming the major axis. Assuming that the orbits of Earth
and Mars are approximately circular with radii RE and Sun
Earth Mars
RM , respectively, the velocity (with respect to the Sun) of RE
RM
the probe during its voyage when it is at a distance
r RE  r  RM  from the Sun, neglecting the effect of Earth and Mars, is

R E  RM  R E  RM  r 
(a) 2GM (b) 2GM
r R E  R M  r  r R E  RM 

RE 2GM
(c) 2GM (d)
rRM r

Ans. : (b)
Solution: Total energy E   K / 2a where 2a major axis and 2a  RE  RM .

1 2 GMm GMm  R  RM  r 
 v  2GM E
mv  
2 r  RE  RM  r  RE  RM 

H.No. 40-D, Ground Floor, Jia Sarai, Near IIT, Hauz Khas, New Delhi-110016
Phone: 011-26865455/+91-9871145498
Website: www.physicsbyfiziks.com | Email: fiziks.physics@gmail.com
89
fiziks
Institute for NET/JRF, GATE, IIT‐JAM, M.Sc. Entrance, JEST, TIFR and GRE in Physics

NET/JRF (JUNE-2015)
Q51. A particle moves in two dimensions on the ellipse x 2  4 y 2  8 . At a particular instant it

is at the point  x, y    2,1 and the x -component of its velocity is 6 (in suitable units).

Then the y -component of its velocity is


(a) 3 (b) 2 (c) 1 (d) 4
Ans. (a)
dx dy
Solution:  x 2  4 y 2  8  2 x  8y 0
dt dt
 2 xvx  8 y v y  0  2  2  6  8 1 v y  0  v y  3

Q52. Consider three inertial frames of reference A, B and C . the frame B moves with a
c c
velocity with respect to A , and C moves with a velocity with respect to B in the
2 10
same direction. The velocity of C as measured in A is

3c 4c c 3c
(a) (b) (c) (d)
7 7 7 7
Ans. (b)
c c A S B S C u x
Solution: v  , u x 
2 10
C/2 C /10
u  v 4c
ux  x 
u vx 7
1 2
c
1 2
Q53. If the Lagrangian of a dynamical system in two dimensions is L  mx  mxy
 , then its
2
Hamiltonian is
1 1 2 1 1 2
(a) H  px p y  py (b) H  px p y  px
m 2m m 2m
1 1 2 1 1 2
(c) H  px p y  py (d) H  px p y  px
m 2m m 2m
Ans. (c)
1 2 L
Solution: L  mx  mxy
   mx  my  px (i)
2 x

H.No. 40-D, Ground Floor, Jia Sarai, Near IIT, Hauz Khas, New Delhi-110016
Phone: 011-26865455/+91-9871145498
Website: www.physicsbyfiziks.com | Email: fiziks.physics@gmail.com
90
fiziks
Institute for NET/JRF, GATE, IIT‐JAM, M.Sc. Entrance, JEST, TIFR and GRE in Physics

L py
  mx  p y or x  (ii)
y m
py px  p y
put x  in equation (i)  p y  my  px  y 
m m
1
H  px x  p y y  L  px x  p y y  mx 2  mxy

2
px p y p y2
put value of x and y  H  
m 2m
 
Q54. A particle of mass m moves in the one dimensional potential V  x   x3  x 4 where
3 4
 ,   0 . One of the equilibrium points is x  0 . The angular frequency of small
oscillations about the other equilibrium point is
2   
(a) (b) (c) (d)
3m m 12m 24m
Ans. (b)
  V 
Solution: V  x   x3  x4    x 2   x3  0  x0  
3 4 x 

 2V 2 k 
Spring constant k  2   ve     
x x  x0
 m m

Q55. A particle of unit mass moves in the xy -plane in such a way that x  t   y  t  and

y  t    x  t  . We can conclude that it is in a conservative force-field which can be

derived from the potential

(a)
2

1 2
x  y2  (b)
1 2
2

x  y2  (c) x  y (d) x  y

Ans. (a)
Solution:  x  y and y   x
 x  y   x
 y   x   y
and 
 xx 0
 y y 0
and 

that is possible for L 


1 2 1 2 1 2
2
1
mx  my  x  y 2  V  x 2  y 2
2 2 2
   

H.No. 40-D, Ground Floor, Jia Sarai, Near IIT, Hauz Khas, New Delhi-110016
Phone: 011-26865455/+91-9871145498
Website: www.physicsbyfiziks.com | Email: fiziks.physics@gmail.com
91
fiziks
Institute for NET/JRF, GATE, IIT‐JAM, M.Sc. Entrance, JEST, TIFR and GRE in Physics

1
Q56. A particle moves in one dimension in the potential V  k  t  x 2 , where k  t  is a time
2
d
dependent parameter. Then V , the rate of change of the expectation value V of the
dt
potential energy is
1 dk 2 k 1 dk 2 1
(a) x  xp  px (b) x  p2
2 dt 2m 2 dt 2m
k 1 dk 2
(c) xp  px (d) x
2m 2 dt
Ans. (a)
p2 1
Solution: H   k t  x2
2m 2

d V 1 2 p
2
1 2 x 2 k
V  V , H     k t  x ,  k t  x    V , H 
dt t 2 2m 2  2 t

d 1 xp  px x 2 k x 2 k 1
V  k t   2    k  t  xp  px
dt 2 2m 2 t 2 t 2m

Q57. Let q and p be the canonical coordinate and momentum of a dynamical system. Which
of the following transformations is canonical?
1 2 1 2
1. Q1  q and P1  p
2 2
1 1
2. Q2   p  q  and P2   p  q 
2 2
(a) neither 1 nor 2 (b) both 1 and 2
(c) only 1 (d) only 2
Ans. (d)
q2 P2
Solution: For A : Q1  , P1 
2 2
Q P Q P
Q1 , P1   1 . 1  1 . 1  1 (Not canonical)
q p p q
1 1
For B : Q2   p  q  , P2   p  q 
2 2

Q2 , p2   1 (canonical)

H.No. 40-D, Ground Floor, Jia Sarai, Near IIT, Hauz Khas, New Delhi-110016
Phone: 011-26865455/+91-9871145498
Website: www.physicsbyfiziks.com | Email: fiziks.physics@gmail.com
92
fiziks
Institute for NET/JRF, GATE, IIT‐JAM, M.Sc. Entrance, JEST, TIFR and GRE in Physics

Q58. Which of the following figures is a schematic representation of the phase space
trajectories (i.e., contours of constant energy) of a particle moving in a one-dimensional
1 2 1 4
potential V  x   x  x p
2 4
p

(a) (b)
x
x

p p

(c) (d)
x x

Ans. (a)
 x2 x4 V  x
Solution: V  x   
2 4
V x
 0  x  0, x  1
x
 2V
 ve for x  0 (unstable point)
x 2 E0
= + ve for x  1 (stable point)
E0 E0

H.No. 40-D, Ground Floor, Jia Sarai, Near IIT, Hauz Khas, New Delhi-110016
Phone: 011-26865455/+91-9871145498
Website: www.physicsbyfiziks.com | Email: fiziks.physics@gmail.com
93
fiziks
Institute for NET/JRF, GATE, IIT‐JAM, M.Sc. Entrance, JEST, TIFR and GRE in Physics

NET/JRF (DEC-2015)
Q59. Two masses m each, are placed at the points  x, y    a, a  and   a,  a  and two

masses, 2m each, are placed at the points  a,  a  and  a, a  . The principal moments of

inertia of the system are


(a) 2m 2 , 4ma 2 (b) 4ma 2 ,8ma 2 (c) 4ma 2 , 4ma 2 (d) 8ma 2 ,8ma 2
Ans. : (b)

 
Solution: I xx   mi yi2  zi2   mi yi2  zi  0
i

 I xx  ma 2  ma 2  2ma 2  2ma 2  I xx  6ma 2

Similarly, I yy  6ma 2 and I zz  12ma 2

I xz  I zx  0, I yz  I zy  0

I xy  I yx   mi  xi yi   ma 2  ma 2  2ma 2  2ma 2  I xy  I yx  2ma 2


i

Moment of inertia tensor


 6ma 2 2ma 2 0 
 
I   2ma 2 6ma 2
0 
 0 0 12ma 2 

Eigen value of matrices is principal moment of inertia, which is given by
1  4ma 2  I x , 2  8ma 2  I y , 3  12ma 2  I z

So, I x  4ma 2 and I y  8ma 2

Q60. The Lagrangian of a system is given by


1 2 5 
L mq1  2mq22  k  q12  2q22  2q1q2 
2 4 
where m and k are positive constants. The frequencies of its normal modes are

(a)
k
,
2m m
3k
(b)
k
2m

13  73  (c)
5k
,
2m m
k
(d)
k
,
2m m
6k

Ans. : (a)
1 2 5 
Solution: L  mq1  2mq22  k  q12  2q22  2q1 q2 
2 4 

H.No. 40-D, Ground Floor, Jia Sarai, Near IIT, Hauz Khas, New Delhi-110016
Phone: 011-26865455/+91-9871145498
Website: www.physicsbyfiziks.com | Email: fiziks.physics@gmail.com
94
fiziks
Institute for NET/JRF, GATE, IIT‐JAM, M.Sc. Entrance, JEST, TIFR and GRE in Physics

1 2 4 2 k 10 2 
L mq1  mq2   q1  4q22  2q1 q2  2q2 q1 
2 2 2 4 

 10 
m 0  k 2 k 
T  , V  4
 0 4m   
 2k 4k 

The secular equation V   2 m  0

 10 
 4 k  m 2 k   0 ,  10 k   2 m  4k  4 2 m  4k 2  0
2

  
 4


 
  2 k 4 k   2
4 m 
 10k 2  10 2 km  4 2 km  4 4 m 2  4k 2  0
 3k 2  7 2 km  2 4 m2  0  3k 2  6 2 km   2 km  2 4 m2  0

k 3k
  
 k  2 2 m 3k   2 m  0   
2m
, 
m
Q61. Consider a particle of mass m moving with a speed v . If TR denotes the relativistic

kinetic energy and TN its non-relativistic approximation, then the value of


TR  TN  for
TR
v  0.01 c , is

(a) 1.25 105 (b) 5.0 105 (c) 7.5 105 (d) 1.0 104
Ans. : None of the options is correct.
1 m0 c 2
Solution: TN  m0 v 2 , TR  mc 2  m0 c 2   m0 c 2 ( v  0.01 c )
2 v 2
1 2
c
 0.01
2
1 v2
TR  TN  m0 v 2
T 2 2 2
Now,  1 N  1  1  1
TR TR m0 c 2
c 2
1
1
 m0 c 2  c2
1   0.01
2
v 2
v 2
1 2 1 2
c c
TR  TN
 0.75
TR

H.No. 40-D, Ground Floor, Jia Sarai, Near IIT, Hauz Khas, New Delhi-110016
Phone: 011-26865455/+91-9871145498
Website: www.physicsbyfiziks.com | Email: fiziks.physics@gmail.com
95
fiziks
Institute for NET/JRF, GATE, IIT‐JAM, M.Sc. Entrance, JEST, TIFR and GRE in Physics

Q62. A canonical transformation  p, q    P, Q  is performed on the Hamiltonian

p2 1 1
H  m 2 q 2 via the generating function, F  m q 2 cot Q . If Q  0   0 , which
2m 2 2
of the following graphs shows schematically the dependence of Q  t  on t ?

(a) (b)

Q t  Q t 

(c) (d)
Q t  Q t 

Ans. : (d)
p2 1 1
Solution: H   m 2 q 2 , F1  m q 2 cot Q
2m 2 2
F1 F1 F1
 p,   P, KH
q Q t  t 
F1
 p  m q cot Q …..(i)
q
t
F1 1
  P   m q 2 cosec 2Q   P
Q 2
1 P
 m q 2  …..(ii)
2 cosec 2Q
From (i) and (ii)
p  2m P cos Q
F1 F1
KH ;  0
t t
p2 1
KH   m 2 q 2 put the value of p and q
2m 2
 K  P using equation of motion Q and P

H.No. 40-D, Ground Floor, Jia Sarai, Near IIT, Hauz Khas, New Delhi-110016
Phone: 011-26865455/+91-9871145498
Website: www.physicsbyfiziks.com | Email: fiziks.physics@gmail.com
96
fiziks
Institute for NET/JRF, GATE, IIT‐JAM, M.Sc. Entrance, JEST, TIFR and GRE in Physics

K
  P  0  P  0  P  constant
Q
K  K
Q    Q  Q  constant  P  constant 
P P
Q    Q  t    from boundary condition   0 
 Q  t
Therefore, option (d) is correct.
Q63. The Lagrangian of a particle moving in a plane s given in Cartesian coordinates as
L  xy
  x 2  y 2

In polar coordinates the expression for the canonical momentum pr (conjugate to the
radial coordinate r ) is
(a) r sin   r cos  (b) r cos   r sin 
(c) 2r cos   r sin 2 (d) r sin 2  r cos 2
Ans. : (d)
Solution: L  xy
  x 2  y 2  xy 
  x 2  y 2 
x  r cos  , y  r sin   x  r cos   r sin , y  r sin   r cos  

L  r 2 sin  cos   r 2 sin  cos   2  r r cos 2   rr


 sin 2 
L
Pr 
r

 2r sin  cos   r cos 2   sin 2  
 Pr  r sin 2  r cos 2

H.No. 40-D, Ground Floor, Jia Sarai, Near IIT, Hauz Khas, New Delhi-110016
Phone: 011-26865455/+91-9871145498
Website: www.physicsbyfiziks.com | Email: fiziks.physics@gmail.com
97
fiziks
Institute for NET/JRF, GATE, IIT‐JAM, M.Sc. Entrance, JEST, TIFR and GRE in Physics

NET/JRF (JUNE-2016)
Q64. Let  x, t  and  x, t   be the coordinate systems used by the observers O and O ,

respectively. Observer O moves with a velocity v   c along their common positive x -


axis. If x  x  ct and x  x  ct are the linear combinations of the coordinates, the

Lorentz transformation relating O and O takes the form

x   x x   x 1  1 
(a) x  and x  (b) x  x and x  x
1  2
1  2 1  1 

x   x x   x 1  1 
(c) x  and x  (d) x  x and x  x
1  2 1  2 1  1 

Ans. : (d)
Solution: x  x  ct 

 vx   v  v v v v
ct  2  x 1   ct 1   1 1 1
x  vt c  c c
        x c  ct c  c  x  ct 
v2 v2 v 2
v 2 v v v
1 2 1 2 1 2 1 2 1 1 1
c c c c c c c

1 
x  x
1 
 vx   v  v
ct  2  x  1   ct  1  
x  vt c   c  c
x  x  ct     
v2 v2 v2 v2
1 2 1 2 1 2 1 2
c c c c

v v
1 1
x  x c  ct c  x  1    x  ct   x  1   x
  
1
v
1
v 1  1 
c c
Q65. A ball of mass m , initially at rest, is dropped from a height of 5 meters. If the coefficient
of restitution is 0.9 , the speed of the ball just before it hits the floor the second time is
approximately (take g  9.8 m / s 2 )
(a) 9.80 m / s (b) 9.10 m / s (c) 8.91 m / s (d) 7.02 m / s
Ans. : (c)
Solution: velocity just before hitting first time is
H.No. 40-D, Ground Floor, Jia Sarai, Near IIT, Hauz Khas, New Delhi-110016
Phone: 011-26865455/+91-9871145498
Website: www.physicsbyfiziks.com | Email: fiziks.physics@gmail.com
98
fiziks
Institute for NET/JRF, GATE, IIT‐JAM, M.Sc. Entrance, JEST, TIFR and GRE in Physics

v1  2 gh  2  9.8  5  9.89 m / s

After hitting velocity will be  ev1  0.9  9.89

v2  8.9 m / s

velocity hitting before second time will be same as v2

Q66. The Hamiltonian of a system with generalized coordinate and momentum  q, p  is

H  p 2 q 2 . A solution of the Hamiltonian equation of motion is (in the following A and


B are constants)
A 2 At A 2 At
(a) p  Be 2 At , q e (b) p  Ae 2 At , q e
B B
A  At A A2t
(c) p  Ae At , q (d) p  2 Ae  A t , q
2
e e
B B
Ans. : (a)
Solution: H  p 2 q 2
From Hamilton’s equation
H dp
  p   2 p 2 q (i)
q dt
H dq
 q   2 pq 2 (ii)
p dt
from equations (i) and (ii)
dp dq

p q
Integrating both sides, ln p   ln q  ln A
pq  A (iii)
from equation (i)
dp
 2 p 2 q  2 pA
dt
dp p
  p
   2 Adt  ln B  ln  2 At  p  Be 2 At
B
A 2 At
Putting this value of p in equation (iii) gives q  e
B
Hence, the correct option is (a)

H.No. 40-D, Ground Floor, Jia Sarai, Near IIT, Hauz Khas, New Delhi-110016
Phone: 011-26865455/+91-9871145498
Website: www.physicsbyfiziks.com | Email: fiziks.physics@gmail.com
99
fiziks
Institute for NET/JRF, GATE, IIT‐JAM, M.Sc. Entrance, JEST, TIFR and GRE in Physics

Q67. A canonical transformation  q, p    Q, P  is made through the generating function

F  q, P   q 2 P on the Hamiltonian

p2 
H  q, p    q4
2 q 2
4

where  and  are constants. The equations of motion for  Q, P  are

P 4P  Q
(a) Q  and P    Q (b) Q  and P 
  2
P 2P 2 2P
(c) Q  and P    Q (d) Q  and P    Q
 Q 
Ans. : (b)
Solution: F  q, P   q 2 P

This is F2 type generating function so

F2 F
 p & 2 Q
q P
1 1
p  2qP & Q  q 2  q   Q  2 & p  2  Q  2 P

4QP 2  2 2 P 2  Q 2
H  Q, P    Q  
2 Q 4  4
H  4P H Q
  Q  Q  and   P  P  
P  Q 2
Q68. The Lagrangian of a system moving in three dimensions is

mx1  m  x22  x32   kx12  k  x2  x3 


1 2 1 1 2
L
2 2 2
The independent constants of motion is/are
(a) energy alone
(b) only energy, one component of the linear momentum and one component of the
angular momentum
(c) only energy, one component of the linear momentum
(d) only energy, one component of the angular momentum
Ans. : (a)

H.No. 40-D, Ground Floor, Jia Sarai, Near IIT, Hauz Khas, New Delhi-110016
Phone: 011-26865455/+91-9871145498
Website: www.physicsbyfiziks.com | Email: fiziks.physics@gmail.com
100
fiziks
Institute for NET/JRF, GATE, IIT‐JAM, M.Sc. Entrance, JEST, TIFR and GRE in Physics

Solution: The motion is in 3D . So don’t get confine with x1 , x2 x3 they are actually x, y, z
Langrangian is then
L L L
mx  m  y 2  z 2   kx 2  k  y  z  , when
1 2 1 1 2
L  0,  0, 0
2 2 2 x y z
So, not any component at Linear momentum is conserve.
Now transform the Lagrangian to Hamiltonian
Px2 Py
2
P2 1 1
 z  kx 2  k  y  z 
2
H 
2m 4m 4m 2 2
H
 0 so energy is conserved
t
Now let us assume Lx  yPz  zPy

dLx L
  Lx , H   x
dt t

 Lx , H    yPz  zPy , H    y, H  Pz  y  Pz , H    z, H  Py  z  Py , H 
 Py2   1 2  Pz2   1 2
  Lx , H    y,  Pz  y  Pz , k  y  z     z ,  Py  z  Py , k  y  z  
 4m   2   4m   2 

Pz  1   P   1 
 2 Py  y 0  k .2  y  z     2 Py z   z 0  k .2  y  z  
4m  2   4m   2 

 k  y 2  yz   k  z 2  yz    k  y 2  z 2   k  z 2  y 2 

dLx dLy dL
  0 . Similarly  0 and  z  0
dt dt dt
Q69. For a particle of energy E and momentum p (in a frame F ), the rapidity y is defined
1  E  p3c 
as y  ln   . In a frame F  moving with velocity v   0, 0,  c  with respect to
2  E  p3c 
F , the rapidity y will be
1  1  
(a) y  y  ln 1   2 
1
(b) y  y  ln  
2 2  1  

 1    1  
(c) y  y  ln   (d) y  y  2 ln  
 1    1  
Ans. : (b)

H.No. 40-D, Ground Floor, Jia Sarai, Near IIT, Hauz Khas, New Delhi-110016
Phone: 011-26865455/+91-9871145498
Website: www.physicsbyfiziks.com | Email: fiziks.physics@gmail.com
101
fiziks
Institute for NET/JRF, GATE, IIT‐JAM, M.Sc. Entrance, JEST, TIFR and GRE in Physics

1  E  p3c 
Solution: y  ln  
2  E  p3c 

1  E   p3c 
Then y  ln  
2  E   p3c 

  E 
Where p3    p3  v  2   E     E  vp3 
  c 

 v 
1   E  p3c    E  p3c  
Put the value of p3 and E  one will get y  ln  c

2  E p c v E p c 
 3   3 
 c 

1   E  p3 c  1     1   E  p3 c   1  1   
ln    ln    ln  
2   E  p3 c  1     2   E  p3 c   2  1   

1  1   1  1  
y  ln    y  ln  
2  1   2  1  

NET/JRF (DEC-2016)
Q70. A ball of mass m is dropped from a tall building with zero initial velocity. In addition to
gravity, the ball experiences a damping force of the form  , where  is its
instantaneous velocity and  is a constant. Given the values m  10 kg ,   10 kg / s and

g  10 m / s 2 the distance travelled (in metres) in time t in seconds, is

(a) 10  t  1  e  t  (b) 10  t  1  e t 

(c) 5t 2  1  et  (d) 5t 2

Ans. : (b)
d2x dx
Solution: m  mg  
dt 2
dt
Putting the values of m,  and g and simplifying we obtain

d 2 x dx
  10
dt 2 dt
The general solution of this equation is x  t   c1  c2t  10e  t

H.No. 40-D, Ground Floor, Jia Sarai, Near IIT, Hauz Khas, New Delhi-110016
Phone: 011-26865455/+91-9871145498
Website: www.physicsbyfiziks.com | Email: fiziks.physics@gmail.com
102
fiziks
Institute for NET/JRF, GATE, IIT‐JAM, M.Sc. Entrance, JEST, TIFR and GRE in Physics

Using the initial conditions x  0   0, x  0   0

We obtain, c1  10 and c2  10

Hence the required solution is x  t   10  t  1  e  t 

Q71. A relativistic particle moves with a constant velocity v with respect to the laboratory
frame. In time  , measured in the rest frame of the particle, the distance that it travels in
the laboratory frame is

c v2 v
(a) v (b) (c) v 1  2 (d)
v2 c v2
1 1
c2 c2
Ans. : (d)
Solution: From Particle x1  0 x2  0 tinitial  t1 t final  t2

x1  vt1 x2  vt2


x1  , x2  L
1 v / c
2 2
1  v2 / c2
v
x2  x1 v  t2  t1 
x2  x1  
1  v2 / c2 1  v2 / c2
v  t2  t1  v
x  
1  v2 / c2 1  v2 / c2
Q72. A particle in two dimensions is in a potential V  x, y   x  2 y . Which of the following

(apart from the total energy of the particle) is also a constant of motion?
(a) p y  2 px (b) px  2 p y

(c) px  2 p y (d) p y  2 px

Ans. : (a)
Solution: V  x, y   x  2 y

px2 p y
2

H   x  2y
2m 2m
d  p y  2 px  
dt
  p y  2 px , H  
t
 p y  2 px 
  p y  2 px , H    p y  2 px , x  2 y    p y , 2 y    2 px , x   2  2  0

H.No. 40-D, Ground Floor, Jia Sarai, Near IIT, Hauz Khas, New Delhi-110016
Phone: 011-26865455/+91-9871145498
Website: www.physicsbyfiziks.com | Email: fiziks.physics@gmail.com
103
fiziks
Institute for NET/JRF, GATE, IIT‐JAM, M.Sc. Entrance, JEST, TIFR and GRE in Physics

Q73. The dynamics of a particle governed by the Lagrangian


1 2 1 2
L mx  kx  kxxt
 describes
2 2
(a) an undamped simple harmonic oscillator
(b) a damped harmonic oscillator with a time varying damping factor
(c) an undamped harmonic oscillator with a time dependent frequency
(d) a free particle
Ans. : (d)
1 2 1 2
Solution: L  mx  kx  kx x t
2 2
L L
 mx  kxt ,  kx  kxt

x x
d  L  L
   0  mx  kxt   0  mx  0
  kx  kx  kxt
dt  x  x
So motion is equivalent to free particle
Q74. The parabolic coordinates  ,  are related to the Cartesian coordinates  x, y  by

x   and y  
1 2
2
   2  . The Lagrangian of a two-dimensional simple harmonic

oscillator of mass m and angular frequency  is


 
m    2   2  2   2       
1  2 1 1
(a) (b) m  2   2   2   2   2  2   2 
2 2  4 
   1 
m  2   2   2   2   2  m  2   2   2   2   2 
1 1 1
(c) (d)
2  2  2  4 
Ans. : (b)
Solution: For two dimensional Harmonic oscillation

m  x 2  y 2   m 2  x 2  y 2 
1 1
L
2 2

x   , y
2

1 2
  2 

   ,
x   y    

H.No. 40-D, Ground Floor, Jia Sarai, Near IIT, Hauz Khas, New Delhi-110016
Phone: 011-26865455/+91-9871145498
Website: www.physicsbyfiziks.com | Email: fiziks.physics@gmail.com
104
fiziks
Institute for NET/JRF, GATE, IIT‐JAM, M.Sc. Entrance, JEST, TIFR and GRE in Physics

 2 2 1 2 2 2
1  
        12 m    4     
2 2
L m    2

2 

L
1
2
 1
8

m  2 2   2 2   2 2   2 2  m 2  4   4  2 2 2 

1
 
m  2   2   2   2  m 2  2   2 
1 2

2 8
 
m  2   2   2   2   2  2   2  
1 1

2  4 
Q75. After a perfectly elastic collision of two identical balls, one of which was initially at rest,
the velocities of both the balls are non zero. The angle  between the final, velocities (in
the lab frame) is
  
(a)   (b)    (c) 0    (d)  
2 2 2
Ans. : (a)
v1
Solution: Angle between two particle 1   2  0
Conservation of momentum u 1
2
mu  mv1 cos 1  mv2 cos  2 (i)
v2
0  mv1 sin 1  mv2 sin  2 (ii)
conservation of kinetic energy
1 1 1
mu 2  mv12  mv22 (iii)
2 2 2
From (i) and (ii)
u 2  v12  v22  2v1v2  cos 1 cos  2  sin 1 sin  2 

u 2  v12  v22  2v1v2 cos 1   2  (iv)

u 2  v12  v22 (v)

v12  v22  v12  v22  2v1v2 cos 1   2 

 cos 1   2   0

 
1   2   
2 2

H.No. 40-D, Ground Floor, Jia Sarai, Near IIT, Hauz Khas, New Delhi-110016
Phone: 011-26865455/+91-9871145498
Website: www.physicsbyfiziks.com | Email: fiziks.physics@gmail.com
105
fiziks
Institute for NET/JRF, GATE, IIT‐JAM, M.Sc. Entrance, JEST, TIFR and GRE in Physics

k
Q76. Consider circular orbits in a central force potential V  r    , where k  0 and
rn
0  n  2 . If the time period of a circular orbit of radius R is T1 and that of radius 2 R is

T2
T2 , then
T1
n 2 n
n 1
(a) 2 2 (b) 2 3 (c) 2 2 (d) 2n
Ans. : (c)
J2 k Veff J2 nk
Solution: Veff   ,    n 1  0
2mr 2
r n
r mr 3
r
m 2 2 r 4 nk 1  n  2  / 2
n
1
 J  mr 2   n 1
  2
 n2
   r  T  r 2
r 3
r r
n2
n
T2  2 R  2 1
   22
T1  R 
c
Q77. Consider a radioactive nucleus that is travelling at a speed with respect to the lab
2
frame. It emits  -rays of frequency v0 in its rest frame. There is a stationary detector,

(which is not on the path of the nucleus) in the lab. If a  -ray photon is emitted when the
nucleus is closest to the detector, its observed frequency at the detector is
3 1 1 2
(a) v0 (b) v0 (c) v0 (d) v0
2 3 2 3
Ans. : (a)

v2
Solution: v  v0 1  (If detector is not in the path at nucleus)
c2

1 3
v  v0 1   v0
4 2

H.No. 40-D, Ground Floor, Jia Sarai, Near IIT, Hauz Khas, New Delhi-110016
Phone: 011-26865455/+91-9871145498
Website: www.physicsbyfiziks.com | Email: fiziks.physics@gmail.com
106
fiziks
Institute for NET/JRF, GATE, IIT‐JAM, M.Sc. Entrance, JEST, TIFR and GRE in Physics

NET/JRF (JUNE-2017)
Q78. The Hamiltonian for a system described by the generalised coordinate x and generalised
momentum p is

p2 1
H   x2 p    2 x2
2 1  2 x  2

where  ,  and  are constants. The corresponding Lagrangian is


1
 x   x 2  1  2 x    2 x 2
1 1 1
2
(a) (b) x 2   2 x 2   x 2 x
2 2 2 1  2  x  2


1 2
x   2 x  1  2 x    2 x 2
1 1 1
2
(c) (d) x 2   2 x 2   x 2 x
2 2 2 1  2  x  2

Ans. : (a)
p2 1
Solution: H  ax 2 p    2 x2 .
2 1  2 x  2

H p
 x  ax 2   p   x  ax 2  1  2  x  
p 1  2 x 
L  xP
 H

p2 1
 xP
  ax 2 P    2 x2 
1  2 x  2

 x  x   x
 x   x  1  2 x 
2 2 2
2
 1  2 x    x  x   x  1  2 x 
2 2

2 1  2  x 


 1  2  x   x   x 2   x   x 2 
 x   x2    1  2 x 2
 2  2

 1  2 x   x   x
 x   x  2 2
 x   x 
2 2
2
 2
1
  2 x 2  1  2 x 
2 2
1
  2 x2
2
Q79. An inertial observer sees two events E1 and E2 happening at the same location but 6  s

apart in time. Another observer moving with a constant velocity v (with respect to the
first one) sees the same events to be 9  s apart. The spatial distance between the events,
as measured by the second observer, is approximately
(a) 300 m (b) 1000 m (c) 2000 m (d) 2700 m
H.No. 40-D, Ground Floor, Jia Sarai, Near IIT, Hauz Khas, New Delhi-110016
Phone: 011-26865455/+91-9871145498
Website: www.physicsbyfiziks.com | Email: fiziks.physics@gmail.com
107
fiziks
Institute for NET/JRF, GATE, IIT‐JAM, M.Sc. Entrance, JEST, TIFR and GRE in Physics

Ans. : (c)
Solution: x12  x11  0 , t21  t11  6 106 , t2  t1  9 106 , x2  x1  ?

t2  t2  9 106

 1 v 1   t '  v x' 
 t2  c 2 x2   1 c 2 1 
   9 106
 1  v / c  1 v / c
2 2 2 2

 
t2  t1 6 106
 9 106   9 106
1 v / c
2 2
1 v / c
2 2

5 v2
v c  1 2  2 / 3
9 c
 x2  vt2   x1  vt1 
 x2  x1     
 1 v / c   1 v / c 
2 2 2 2

v
 t2  t1 
1  v2 / c2

c    6 106  
5 9 5 9
 x2  x1    3 108   6 106
3 6 3 6
 9  5 102  20.12 102  2000m
Q80. A ball weighing 100 gm , released from a height of 5 m , bounces perfectly elastically off
a plate. The collision time between the ball and the plate is 0.5 s . The average force on
the plate is approximately
(a) 3 N (b) 2 N (c) 5 N (d) 4 N
Ans. : (d)
100
Solution: m   0.1 kg
1000
1 2
mgh  mv v  2 gh .
2
v  10 m / sec

change in momentum during collision,  mv     mv   2k .gm / sec

P 2
f    4N
t 0.5

H.No. 40-D, Ground Floor, Jia Sarai, Near IIT, Hauz Khas, New Delhi-110016
Phone: 011-26865455/+91-9871145498
Website: www.physicsbyfiziks.com | Email: fiziks.physics@gmail.com
108
fiziks
Institute for NET/JRF, GATE, IIT‐JAM, M.Sc. Entrance, JEST, TIFR and GRE in Physics

Q81. A solid vertical rod, of length L and cross-sectional area A , is made of a material of
Young’s modulus Y . The rod is loaded with a mass M , and, as a result, extends by a
small amount  L in the equilibrium condition. The mass is then suddenly reduced to
M / 2 . As a result the rod will undergo longitudinal oscillation with an angular frequency
(a) 2YA / ML (b) YA / ML

(c) 2YA / M  L (d) YA / M  L


Ans. : (b)
Fl YAl
Solution: Y  F
Al l
YAl
For mass m , mg 
l
m YA   l 
For mass g  
2 l  2 
Equation (i) and (ii) is for equilibrium condition
Change in force will generate acceleration
 mg  YA  l  YA l
F    mg     l  
 2  l  2  2 l
m 2 l YA l

2 l 2

YA

ml
Q82. The Lagrangian of a free relativistic particle (in one dimension) of mass m is given by

L   m 1  x 2 where x  dx / dt . If such a particle is acted upon by a constant force in


the direction of its motion, the phase space trajectories obtained from the corresponding
Hamiltonian are
(a) ellipses (b) cycloids (c) hyperbolas (d) parabolas
Ans. : (c)
Solution: E 2  p 2 c 2  m02 c 4 .

dP P
For constant force  F , P  Ft  t 
dt F

H.No. 40-D, Ground Floor, Jia Sarai, Near IIT, Hauz Khas, New Delhi-110016
Phone: 011-26865455/+91-9871145498
Website: www.physicsbyfiziks.com | Email: fiziks.physics@gmail.com
109
fiziks
Institute for NET/JRF, GATE, IIT‐JAM, M.Sc. Entrance, JEST, TIFR and GRE in Physics

mu
 Ft  u 
 F / m t .
1  u 2 / c2  Ft 
2

1  
 mc 

F
  t. mc 2 
2 
dx m F t tdt  Ft 
 x    1    1
dt  Ft 
2 m 0
 Ft 
2 F   mc  
1  1   
 
 mc   mc 

  P2 
2
 Fx

 2  1  1  2 4 
 mc   mc 

P 2  F 2 x 2  2mc 2 Fx   Fx  mc 2   m 2 c 4
2

 Fx  mc  2 2
 P 2  m 2 c 4 , which is equation of hyperbola.

Q83. A Hamiltonian system is described by the canonical coordinate q and canonical

momentum p . A new coordinate Q is defined as Q  t   q  t     p  t    , where t is

the time and  is a constant, that is, the new coordinate is a combination of the old
coordinate and momentum at a shifted time. The new canonical momentum P  t  can be

expressed as
(a) p  t     q  t    (b) p  t     q  t   

1 1
(c)  p  t     q  t     (d)  p  t     q  t    
2 2
Ans. : (d)
Solution: Given  q1 p   1

Q t   q t     p t   

If P is new momentum, then Q, P   1

For option (a), Q, P    q  p, p  q    q, p   q  q, q    p, q    p, p 

 1   1  2

For option (b) Q, P    q  p, p  q   2

H.No. 40-D, Ground Floor, Jia Sarai, Near IIT, Hauz Khas, New Delhi-110016
Phone: 011-26865455/+91-9871145498
Website: www.physicsbyfiziks.com | Email: fiziks.physics@gmail.com
110
fiziks
Institute for NET/JRF, GATE, IIT‐JAM, M.Sc. Entrance, JEST, TIFR and GRE in Physics

1 2
For option (c) Q, P    q  p, p  q    1 i.e. canonical transform
2 2
1 2
For option (d) Q, P    q  p, p  q    1
2 2
Option (c) and (d) are correct. But from translation symmetry option (d) is more suitable.
Q84. The energy of a one-dimensional system, governed by the Lagrangian
1 2 1 2n
L mx  kx
2 2
where k and n are two positive constants, is E0 . The time period of oscillation 
satisfies
1 1 1 n 1 n2 1 1 n
   
(a)   k n
(b)   k 2n
E02 n (c)   k 2n
E02 n (d)   k n E02 n
Ans. : (b)
H J J
Solution:   T  , Time Period T  , where J is Action variable
J H E
V  x

J   Pdx  4 2m  E  V  x  dx
0

1/ 2 n
x
 2E 
  
 k 
1
 2E   2E 
1/ 2 n
x
   
 k 
 
 k 
k 2n  2E   2E  2
1    
J 4 
0
2m  E  kx 2 n  dn  4 2mE
 2 

0
1
2E
x dx  k   k 

Don’t try to solve integration rather try to make E independent.


1 1
1/ 2 n 1
 2E   k  2n  2E  2n
J  4 2mE   1  t dt ,  x  t  dx  
2n
 Put   dt
 k  0  2E   k 
 1 n  1
1  
J  c 4 2m E  2n 
, where c   1  t 2 n dt
k 
1/ 2 n
0

 1 n
1 n 
J 1  1. 1
T  k  E
2 n  2n 
  k  E 2n
2 n
E

H.No. 40-D, Ground Floor, Jia Sarai, Near IIT, Hauz Khas, New Delhi-110016
Phone: 011-26865455/+91-9871145498
Website: www.physicsbyfiziks.com | Email: fiziks.physics@gmail.com
111
fiziks
Institute for NET/JRF, GATE, IIT‐JAM, M.Sc. Entrance, JEST, TIFR and GRE in Physics

NET/JRF (DEC - 2017)


Q85. A light signal travels from a point A to a point B , both within a glass slab that is moving
with uniform velocity (in the same direction as the light) with speed 0.3 c with respect to
an external observer. If the refractive index of the slab is 1.5 , then the observer will
measure the speed of the signal as
(a) 0.67 c (b) 0.81 c (c) 0.97 c (d) c
Ans. : (b)
Solution: v  0.3 c ,
S S
c
u x  n  1.5
'

n
c 0  3c
0.3 c  c/n
u x'  v n
ux  
u x' v 1  c . 0.3 c
1 2
c n c2
u x  0.81 c .

Q86. A disc of mass m is free to rotate in a plane parallel to the xy plane with an angular
velocity  ẑ about a massless rigid rod suspended from the roof of a stationary car (as
shown in the figure below). The rod is free to orient itself along any direction.
The car accelerates in the positive x -direction with an acceleration a  0 . Which of the
following statements is true for the coordinates

of the centre of mass of the disc in the reference

frame of the car?
(a) only the x and the z coordinates change x̂
(b) only the y and the z coordinates change
axˆ
(c) only the x and the y coordinates change
(d) all the three coordinates change
 ẑ
Ans. : (d)

H.No. 40-D, Ground Floor, Jia Sarai, Near IIT, Hauz Khas, New Delhi-110016
Phone: 011-26865455/+91-9871145498
Website: www.physicsbyfiziks.com | Email: fiziks.physics@gmail.com
112
fiziks
Institute for NET/JRF, GATE, IIT‐JAM, M.Sc. Entrance, JEST, TIFR and GRE in Physics

Q87. A cyclist, weighing a total of 80 kg with the bicycle, pedals at a speed of 10 m / s . She
stops pedalling at an instant which is taken to be t  0 . Due to the velocity dependent
10
frictional force, her velocity is found to vary as v  t   m.s , where t is
 t 
1  
 30 
measured in seconds. When the velocity drops to 8 m / s , she starts pedalling again to
maintain a constant speed. The energy expended by her in 1 minute at this (new) speed, is
(a) 4 kJ (b) 8 kJ (c) 16 kJ (d) 32 kJ
Ans. : (b)
Solution: The acceleration of cyclist is
d  300  300
a t   
dt  t  30   t  30 2

Hence net force acting on the cyclist


300  80
F t  
 t  30 
2

Since frictional force is the only force acting on the cyclist, this net force is equal to
functional force. We can write

80  300  4
2

F t   v  t  
2
  
300  t  30  15
4 2
When the cyclist moves at a constant speed, the frictional force is F  t   8 .
15
The displacement during this interval is 8  60 . Thus the work done by frictional force
4
  64  8  60 is 8.192 KJ . Hence in order to maintain constant speed the cyclist
15
must supply an energy equal to 8.192 KJ .

H.No. 40-D, Ground Floor, Jia Sarai, Near IIT, Hauz Khas, New Delhi-110016
Phone: 011-26865455/+91-9871145498
Website: www.physicsbyfiziks.com | Email: fiziks.physics@gmail.com
113
fiziks
Institute for NET/JRF, GATE, IIT‐JAM, M.Sc. Entrance, JEST, TIFR and GRE in Physics

Q88. The spring constant k of a spring of mass ms is determined experimentally by loading


the spring with mass M and recording the time period T , for a single oscillation. If the
experiment is carried out for different masses, then the graph that correctly represents the
result is
(a) (b)
T2 T2

 0, 0  M  0, 0  M

(c) T2 (d) T2

 0, 0  M
 0, 0  M

Ans. : (a)
Solution: The Langragian of system.
1 m 1 1 d  L  L
L   s x 2  Mx 2  kx 2 ,   0
2 3 2 2 dt  x  x
d L m 
 0   s  M  
x  kx
dt x  3 
m  ms 
M s M  3 
T  2 3  T 2  4  
k k
Q89. Consider a set of particles which interact by a pair potential V  ar 6 where r is the inter-
particle separation and a  0 is a constant. If a system of such particles has reached virial
equilibrium, the ratio of the kinetic to the total energy of the system is
1 1 3 2
(a) (b) (c) (d)
2 3 4 3
Ans. : (c)
n
Solution: V  r   ar n T 
V
2
V r  a r 6 and T  3 V

1 4 T 3
Then, E  T  V  T  T  E  T  
3 3 E 4

H.No. 40-D, Ground Floor, Jia Sarai, Near IIT, Hauz Khas, New Delhi-110016
Phone: 011-26865455/+91-9871145498
Website: www.physicsbyfiziks.com | Email: fiziks.physics@gmail.com
114
fiziks
Institute for NET/JRF, GATE, IIT‐JAM, M.Sc. Entrance, JEST, TIFR and GRE in Physics

Q90. A particle moves in one dimension in a potential V  x    k 2 x 4   2 x 2 where k and 

are constants. Which of the following curves best describes the trajectories of this system
in phase space?
p p

(a) (b)
x x

p
p

(c) (d)
x x

Ans. : (c)
Solution: V  x   k 2 x 4   2 x 2

For equation point


V 2 V  x
 0  4k 2 x 3  2 2 x  0 , x  0 or x 2  2
x 2k
dV2
x
Now,  12k 2 x 2  2 2 At, x  0
dx 2

d 2V Px
 2 2 , x  0 is minimum.
dx 2

x
d 2V 2  2
2
And,  12 k  2 2
 4 2
, at x 2

dx 2 2k 2 2k 2

2
Hence, x   is maxima.
2k 2
H.No. 40-D, Ground Floor, Jia Sarai, Near IIT, Hauz Khas, New Delhi-110016
Phone: 011-26865455/+91-9871145498
Website: www.physicsbyfiziks.com | Email: fiziks.physics@gmail.com
115
fiziks
Institute for NET/JRF, GATE, IIT‐JAM, M.Sc. Entrance, JEST, TIFR and GRE in Physics

Q91. Let  x, p  be the generalized coordinate and momentum of a Hamiltonian system. If new

variables  X , P  are defined by X  x sinh   p  and P  x cosh   p  , where  , 

and  are constants, then the conditions for it to be a canonical transformation, are
1 1 1 1
(a)      1 and      1 (b)     1 and     1
2 2 2 2
1 1 1 1
(c)      1 and      1 (d)     1 and     1
2 2 2 2
Ans. : (c)
Solution: X  x sinh   p 

P  x cosh  p
For canonical transformation
X P X P
.  . 1
x p p X

 x 1 sinh  px  sinh  p


 x 1 cosh  p.x  cosh  b  1

 x  1  sin 2 h  p  r cos 2 h  p   1

   1  0 (i)

 sin 2 h p   v cos 2 h p  cos 2 h p  sin 2 h p


equating the coefficient on both side
  1 (ii)
  1 (iii)
From (ii) and (iii)   
1
And from (i)     ,   2 (Not convinced with any answer)
2

H.No. 40-D, Ground Floor, Jia Sarai, Near IIT, Hauz Khas, New Delhi-110016
Phone: 011-26865455/+91-9871145498
Website: www.physicsbyfiziks.com | Email: fiziks.physics@gmail.com
116
fiziks
Institute for NET/JRF, GATE, IIT‐JAM, M.Sc. Entrance, JEST, TIFR and GRE in Physics

NET/JRF (JUNE-2018)
Q92. Two particles A and B move with relativistic velocities of equal magnitude v , but in
opposite directions, along the x -axis of an inertial frame of reference. The magnitude of
the velocity of A , as seen from the rest frame of B , is
2v 2v cv 2v
(a) (b) (c) 2v (d)
 v2   v2  cv v2
1  2  1  2  1
 c   c  c2
Ans. : (b)
Solution: ux  v V v

u x'  V vv 2v
ux  ux  
v v2
' 2
u xV 1 2 1 2
1
c2 c c
Q93. Which of the following figures best describes the trajectory of a particle moving in a
a
repulsive central potential V  r   ( a  0 is a constant)?
r
y y
(a) (b)

x x
O O

y
y

(c) x (d)
O x
O

Ans. : (c) y
a
Solution: The potential is V  r   which is repulsive. So there is
r
x
unbounded motion and mainly represent by scattering project O

H.No. 40-D, Ground Floor, Jia Sarai, Near IIT, Hauz Khas, New Delhi-110016
Phone: 011-26865455/+91-9871145498
Website: www.physicsbyfiziks.com | Email: fiziks.physics@gmail.com
117
fiziks
Institute for NET/JRF, GATE, IIT‐JAM, M.Sc. Entrance, JEST, TIFR and GRE in Physics

Q94. A particle moves in the one-dimensional potential V  x    x 6 , where a  0 is a constant.

If the total energy of the particle is E , its time period in a periodic motion is proportional
to
(a) E 1/ 3 (b) E 1/ 2 (c) E1/ 3 (d) E1/ 2
Ans. : (a)
Solution: J   Pdx
1/ 6
E dE
J   2mE    J 1/ 2    E1/ 3  T  E 1/ 3
1/ 2
   E  J 3/ 2
a dJ
1 1
Q95. A particle of mass m , kept in potential V  x    kx 2   x 4 (where k and  are
2 4
positive constants), undergoes small oscillations about an equilibrium point. The
frequency of oscillations is

1 2 1 k 1 2k 1 
(a) (b) (c) (d)
2 m 2 m 2 m 2 m
Ans. : (c)
1 1
Solution: V   kx 2   x 4
2 4
dV
0 kx   x3  0
dx
k k
x  0, x 2   x  x0 
 
d 2V
 k at x  0 so x  0 is unstable part
dx 2
d 2V k k
 2k at x0  so x0  is stable equation point
dx 2
 x

d 2V
dx 2 x  x0 2k 1 2k
  f 
m m 2 m

H.No. 40-D, Ground Floor, Jia Sarai, Near IIT, Hauz Khas, New Delhi-110016
Phone: 011-26865455/+91-9871145498
Website: www.physicsbyfiziks.com | Email: fiziks.physics@gmail.com
118
fiziks
Institute for NET/JRF, GATE, IIT‐JAM, M.Sc. Entrance, JEST, TIFR and GRE in Physics

k
Q96. A particle of mass m moves in a central potential V  r    in an elliptic orbit
r
a 1  e 2 
r    , where 0    2 and a and e denote the semi-major axis and
1  e cos 
k
eccentricity, respectively. If its total energy is E   , the maximum kinetic energy is
2a
 e  1  e  1
(a) E 1  e 2  (b) E (c) E / 1  e 2  (d) E
 e  1  e  1
Ans. : (b)
Solution: E  T  V T  E V
k k k k
T   T   1  cos  
2a r 2a a 1  e 2 

T . maximum cos   1
k k 1  e  k k 1  e 
Tmax     
2a a 1  e 
2
2a a 1  e 1  e 

k 1 1  k  1 e   1 e 
        E 
a  2 1  e   2a  1  e   1 e 

xp 2 1
Q97. The Hamiltonian of a one-dimensional system is H   kx , where m and k are
2m 2
positive constants. The corresponding Euler-Lagrange equation for the system is
(a) mx  k  0 (b) mx  2 x  kx 2  0
(c) 2mxx  mx 2  kx 2  0 (d) mxx  2mx 2  kx 2  0
Ans. : (c)
xp 2 1
Solution: H   kx
2m 2
H xp mx
 x   x p
p m x

xp 2 1 mx 2 mx 2 1 mx 2 1


L  xp
  H  L  xp
   kx    kx   kx
2m 2 x 2x 2 2x 2
Eular Lagrangas equation is given by

H.No. 40-D, Ground Floor, Jia Sarai, Near IIT, Hauz Khas, New Delhi-110016
Phone: 011-26865455/+91-9871145498
Website: www.physicsbyfiziks.com | Email: fiziks.physics@gmail.com
119
fiziks
Institute for NET/JRF, GATE, IIT‐JAM, M.Sc. Entrance, JEST, TIFR and GRE in Physics

d  L  L
  0
dr  x  x
mx mxx
 1
 2  k 0
x 2x 2
2 xmx  mx 2  kx 2  0
2
Q98. The energy of a free relativistic particle is E  p c 2  m 2 c 4 , where m is its rest mass,

p is its momentum and c is the speed of light in vacuum. The ratio vg / v p of the group

velocity vg of a quantum mechanical wave packet (describing this particle) to the phase

velocity v p is
  2 
(a) p c / E (b) p mc 3 / E 2 (c) p c 3 / E 2 (d) p c / 2 E

Ans. : (c)
dE E
Solution: E 2  p 2 c 2  m 2 c 4 and vg  , vp 
dp p
dE E dE
2E  2 pc 2   c2
dp p dp
vg c2 vg c3 p 2
  2
v p v 2p vp E

H.No. 40-D, Ground Floor, Jia Sarai, Near IIT, Hauz Khas, New Delhi-110016
Phone: 011-26865455/+91-9871145498
Website: www.physicsbyfiziks.com | Email: fiziks.physics@gmail.com
120
fiziks
Institute for NET/JRF, GATE, IIT‐JAM, M.Sc. Entrance, JEST, TIFR and GRE in Physics

Q99. An inertial frame K  moves with a constant speed v with respect to another inertial
frame K along their common x - direction. Let  x, ct  and  x , ct   denote the space-

time coordinates in the frames K and K  , respectively. Which of the following space-
time diagrams correctly describes the t  - axis  x   0 line  and the x  - axis  t   0 line 

in the x-ct plane? (In the following figures tan   v / c )


ct ct  ct ct 

(a)

(c)
 x

x  x

x

ct ct  ct
ct 

(c) x
(d)
 

 x x

x
Ans. : (b)
 ct    cosh   sinh  0 0   ct 
    
 x    sinh  cos  0 0 x 
Solution: 
 y   0 0 1 0 y 
 z    0  
1   z 
   0 0

ct ct 

 x

 x

Where v  cosh  ,  v  sinh    tanh 

H.No. 40-D, Ground Floor, Jia Sarai, Near IIT, Hauz Khas, New Delhi-110016
Phone: 011-26865455/+91-9871145498
Website: www.physicsbyfiziks.com | Email: fiziks.physics@gmail.com
121
fiziks
Institute for NET/JRF, GATE, IIT‐JAM, M.Sc. Entrance, JEST, TIFR and GRE in Physics

NET/JRF (DEC - 2018)


Q100. A particle of mass m , moving along the x - direction, experiences a damping force  v 2 ,
where  is a constant and v is its instantaneous speed. If the speed at t  0 is v0 , the

speed at time t is
 v0t
 v0 mv0 2v0
(a) v0 e m
(b) (c) (d)
 v t  m   v0t  v0t
1  ln  1  0  1 e m
 m 
Ans. : (c)
dv dv 
Solution: From Newton’s second law, m   v 2  2   dt
dt v m
1 
Integrating both sides gives    t  c
v m
where c is a constant of integration
Since v  v0 at t  0 , we obtain

1  1
   .0  c  c  
v0 m v0
1  1
Hence,    t 
v m v0
1  t 1  v0t  m mv0 mv0
    v 
v m v0 mv0  v0t  m m   v0t
k
Q101. In the attractive Kepler problem described by the central potential V  r   (where k
r
is a positive constant), a particle of mass m with a non-zero angular momentum can
never reach the centre due to the centrifugal barrier. If we modify the potential to
k 
V r     3
r r
one finds that there is a critical value of the angular momentum  c below which there is

no centrifugal barrier. This value of  c is


1/ 2 1/ 2 1/ 4 1/ 4
(a) 12km 2   (b) 12km 2   (c) 12km 2   (d) 12km 2  

Ans. : (c)

H.No. 40-D, Ground Floor, Jia Sarai, Near IIT, Hauz Khas, New Delhi-110016
Phone: 011-26865455/+91-9871145498
Website: www.physicsbyfiziks.com | Email: fiziks.physics@gmail.com
122
fiziks
Institute for NET/JRF, GATE, IIT‐JAM, M.Sc. Entrance, JEST, TIFR and GRE in Physics

L2 k L2 k L2
Solution: Veff    0     0  r0 
2mr 2 r mr 3 r 2 mk
when introduce new potential
L2 k 
Veff    3
2mr 2
r r
For critical value
Veff  L2 k 3
  
r mr 3 r 2 r 4
 2Veff 3L2 2k 12
  5 0
r 2 mr 4 r 3 r
For critical value
3L2 2k 12 3m3 k 4 2m3 x 4 12m5 x 5 
   0    0
 L2 
4
 L2 
3
 L2 
5
L6 L6 L10
m     
 mk   mk   mk 

m3 k 4  m2 k  
   
1/ 4 1/ 4
LC  12m 2 k      0  Lc  12m k 
2
 3 2 12
L6  L4 
Q102. The time period of a particle of mass m , undergoing small oscillations around x  0 , in
x
the potential V  V0 cosh   , is
L

mL2 mL2 mL2 2mL2


(a)  (b) 2 (c) 2 (d) 2
V0 2V0 V0 V0

Ans. : (c)
x
Solution: V  V0 cosh  
L
V V x
For equilibrium point  0  0 sinh    0
x L L
 2V V0
k 
x 2 x 0
L2

k 2 V0 mL2
    T  2
m T mL2 V0

H.No. 40-D, Ground Floor, Jia Sarai, Near IIT, Hauz Khas, New Delhi-110016
Phone: 011-26865455/+91-9871145498
Website: www.physicsbyfiziks.com | Email: fiziks.physics@gmail.com
123
fiziks
Institute for NET/JRF, GATE, IIT‐JAM, M.Sc. Entrance, JEST, TIFR and GRE in Physics

1
Q103. Consider the decay A  B  C of a relativistic spin - particle A . Which of the
2
following statements is true in the rest frame of the particle A ?
1
(a) The spin of both B and C may be
2
(b) The sum of the masses of B and C is greater than the mass of A
(c) The energy of B is uniquely determined by the masses of the particles
(d) The spin of both B and C may be integral
Ans. : (c)
Q104. The motion of a particle in one dimension is described by the Langrangian
1   dx  
2

L      x 2  in suitable units. The value of the action along the classical path
2   dt  

from x  0 at t  0 to x  x0 at t  t0 , is
x02 1 1 x02
(a) (b) x02 tan t0 (c) x02 cot t0 (d)
2sin 2 t0 2 2 2 cos 2 t0
Ans. : (c)
1 1
Solution: L  x 2  x 2
2 2
d  L  L
From Lagrangian equation of motion,   0
dt  x  x
xx 0

The solution is x  A sin t  B cos t
t  0, x  0 , B  0
x  A sin t
x0
t  t0 , x  x0 , A 
sin t0
x0 x
x sin t , x  0 cos t
sin t0 sin t0
t0 t0 t t t
1 0
1 1 x02 0 1 x02 0 2
A   Ldt   x 2 dt   x 2 dt 
2 sin 2 t0 0 2 sin 2 t0 0
cos 2
t dt  sin tdt
0 0
2 0
2

1 x02  t0 t  1 x02 t0 1 x02 sin 2t0


t0
x02
  t dt  0 tdt    t dt   cot t0
2 2
cos sin cos 2
2 sin 2 t0  0  2 sin t0 t
2
2 sin 2 t0 2 0 2

H.No. 40-D, Ground Floor, Jia Sarai, Near IIT, Hauz Khas, New Delhi-110016
Phone: 011-26865455/+91-9871145498
Website: www.physicsbyfiziks.com | Email: fiziks.physics@gmail.com
124
fiziks
Institute for NET/JRF, GATE, IIT‐JAM, M.Sc. Entrance, JEST, TIFR and GRE in Physics

Q105. The Hamiltonian of a classical one-dimensional harmonic oscillator is H 


1 2
2

p  x2 ,
in suitable units. The total time derivative of the dynamical variable p  2 x is 
(a) 2p  x (b) p  2 x (c) p  2 x (d) x  2 p
Ans. : (a)

Solution: H 
p2 x2
2

2

Let say dynamical variable A  p  2 x 
dA A
  A, H  
dt t
A dA
It is given 0   A, H 
t dt

dA  p2 x2   x2   p2 
  p  2 x,     p ,    2 x, 
dt  2 2  2  2 

2 x 22 p
   x  2 p  2 p  x
2 2
Q106. A relativistic particle of mass m and charge e is moving in a uniform electric field of
c
strength  . Starting from rest at t  0 , how much time will it take to reach the speed ?
2
1 mc mc mc 3 mc
(a) (b) (c) 2 (d)
3 e e e 2 e
Ans. : (a)
dp
Solution:  e
dt
p  e t  c
At t  0 , p  0 , c  0
mv
 e t
v2
1 2
c
m v c m c/2 mc mc
t Put v  , t  t 
e v2 2 e 1 3e 3eE
1 1
c2 4

H.No. 40-D, Ground Floor, Jia Sarai, Near IIT, Hauz Khas, New Delhi-110016
Phone: 011-26865455/+91-9871145498
Website: www.physicsbyfiziks.com | Email: fiziks.physics@gmail.com
125
fiziks
Institute for NET/JRF, GATE, IIT‐JAM, M.Sc. Entrance, JEST, TIFR and GRE in Physics

ELECTROMAGNETIC THEORY
NET/JRF–(JUNE-2011)
Q1. The electrostatic potential V(x, y) in free space in a region where the charge density ρ is
zero is given by V  x, y   4e 2 x  f  x   3 y 2 . Given that the x-component of the electric

field Ex, and V are zero at the origin, f  x  is

(a) 3x 2  4e 2 x  8 x (b) 3x 2  4e 2 x  16 x
(c) 4e 2 x  8 (d) 3x 2  4e 2 x
Ans. : (d)
Solution: V  4e 2 x  f  x   3 y 2 . Since   0   2V  0  16e 2 x  f  x   6  0 .
 

Since E x  0 at origin  E  V  E x   8e 2 x  f  x  
E x 0, 0  8  f 0  0  f 0   8 .

Since V 0, 0   0  4  f 0   0  f 0   4

Solve equation 16e 2 x  f  x   6  0  f  x   6  16e 2 x  f x   6 x  8e 2 x  c1 , since

f   0   8  c1  8  c1  0 .

Again Integrate f  x   6 x  8e 2 x  f  x   3 x 2  4e 2 x  c 2

since f  0   4  c2  4  c2  0 . Thus f  x   3x 2  4e 2 x
   
Q2. For constant uniform electric and magnetic field E  E 0 and B  B0 , it is possible to

choose a gauge such that the scalar potential  and vector potential A are given by
 1      1  
(a)   0 and A  B0  r
2
  
(b)    E 0  r and A  B0  r
2

    
(c)    E0  r and A  0 (d)   0 and A   E 0 t
Ans. : (a)
 
Solution: Let E  E 0  xˆ  yˆ  zˆ  and B  B0  xˆ  yˆ  zˆ  since they are constant vector.
  
Lorentz Gauge condition is   A   0 0
t
 
 
since B  r  B0  z  y  xˆ  B0  z  x  yˆ  B0  y  x  zˆ

H.No. 40-D, Ground Floor, Jia Sarai, Near IIT, Hauz Khas, New Delhi-110016
Phone: 011-26865455/+91-9871145498
Website: www.physicsbyfiziks.com | Email: fiziks.physics@gmail.com
126
fiziks
Institute for NET/JRF, GATE, IIT‐JAM, M.Sc. Entrance, JEST, TIFR and GRE in Physics

     
(a)  0 and   A  0 (b)  0, and   A  0
t t
     
(c)  0 and   A  0 (d)  0 and   A  0
t t
Q3. A plane electromagnetic wave is propagating in a lossless dielectric. The electric field is
given by

 

 
E  x, y, z, t   E0  xˆ  Azˆ  exp ik0 ct  x  3 z  ,

where c is the speed of light in vacuum, E0 , A and k0 are constant and x̂ and ẑ are

unit vectors along the x - and z -axes. The relative dielectric constant of the medium  r
and the constant A are
1 1
(a)  r  4 and A   (b)  r  4 and A  
3 3

(c)  r  4 and A  3 (d)  r  4 and A   3


Ans. : (a)

  
Solution: E  x, y, z , t   E 0  xˆ  Azˆ  exp ik 0  ct  x  3 z . 
 

Comparing with term e i k r t   k  k 0 xˆ  3 zˆ and   k 0 c .

 k0c c
Since v     Refractive index n   r  2   r  4.
k k 02  3k 02 2

  
Since k  nˆ  0  k 0 xˆ  3 zˆ   xˆ  Azˆ   0  k 0 1  A 3  0  A    1
3
A  Kr
Q4. A static, spherically symmetric charge distribution is given by  r   e where A
r
and K are positive constants. The electrostatic potential corresponding to this charge
distribution varies with r as

(a) re  Kr (b)
1  Kr
r
e (c)
1  Kr
r2
e (d)
1
r

1  e  Kr 
Ans. : (b)
Solution: since  2V    /  0

H.No. 40-D, Ground Floor, Jia Sarai, Near IIT, Hauz Khas, New Delhi-110016
Phone: 011-26865455/+91-9871145498
Website: www.physicsbyfiziks.com | Email: fiziks.physics@gmail.com
127
fiziks
Institute for NET/JRF, GATE, IIT‐JAM, M.Sc. Entrance, JEST, TIFR and GRE in Physics

A  kr 1   2 V 
 2V must be proportional to e , where  2V  2 r .
r r r  r 
Q5. The magnetic field of the TE11 mode of a rectangular waveguide of dimensions a  b as

shown in the figure is given by H z  H 0 cos  0.3  x  cos  0.4  y  , where x and y are in
x
cm.
a
z
b
y

A. The dimensions of the waveguide are


(a) a  3.33 cm, b  2.50 cm (b) a  0.40 cm, b  0.30 cm
(c) a  0.80 cm, b  0.60 cm (d) a  1.66 cm, b  1.25 cm
Ans. : (a)
Solution: Since H z  H 0 cos  0.3 x  cos  0.4 y 

m n
  0.3 where m  1 and  0.4 where n  1
a b
 a  3.33cm, b  2.50cm
B. The entire range of frequencies f for which the TE11 mode will propagate is

(a) 6.0 GHz  f  12.0 GHz (b) 7.5 GHz  f  9.0 GHz
(c) 7.5 GHz  f  12.0 GHz (d) 7.5GHz  f
Ans. : (d)
2 2
c m n c 1 1
Solution: f m , n        f1,1   2  7.5 GH z .
2  a  b 2 a 2
b
For propagation, frequency of incident wave must be greater than cutoff frequency.

H.No. 40-D, Ground Floor, Jia Sarai, Near IIT, Hauz Khas, New Delhi-110016
Phone: 011-26865455/+91-9871145498
Website: www.physicsbyfiziks.com | Email: fiziks.physics@gmail.com
128
fiziks
Institute for NET/JRF, GATE, IIT‐JAM, M.Sc. Entrance, JEST, TIFR and GRE in Physics

NET/JRF -(DEC-2011)
Q6. Consider three polarizer’s P1 , P2 and P3 placed along an axis as shown in the figure.

P1 P2 P3
(unpolarized) 
I0

The pass axis of P1 and P3 are at right angles to each other while the pass axis of P2

makes an angle  with that of P1 . A beam of unpolarized light of intensity I 0 is incident

on P1 as shown. The intensity of light emerging from P3 is

I0 I0 I0
(a) 0 (b) (c) sin 2 2 (d) sin 2 2
2 8 4
Ans. : (c)
Solution: I  I 0 cos 2  (Malus Law)

I0 I0 I0 I
 I1  , I2  cos 2  , I3  cos 2   cos 2 90     0 sin 2 2 .
2 2 2 8
Q7. Four equal point charges are kept fixed at the four vertices of a square. How many neutral
points (i.e. points where the electric field vanishes) will be found inside the square?
(a) 1 (b) 4 (c) 5 (d) 7
Ans. : (a)
Solution: Inside the square, there is only one point where field vanishes.

Q8. A static charge distribution gives rise to an electric field of the form E   1  e  r / R   rˆ ,
r2
where  and R are positive constants. The charge contained within a sphere of radius R ,
centred at the origin is
e e2 R R2
(a)  0 (b)  0 (c) 4 0 (d)  0
R2 R2 e e
Ans. : None of the options given are correct
   2
   
Solution: Qenc   0  E  da   0  1  e  r / R 2  r 2 sin ddrˆ   0    1  e  r / R sin dd

r 0 0

H.No. 40-D, Ground Floor, Jia Sarai, Near IIT, Hauz Khas, New Delhi-110016
Phone: 011-26865455/+91-9871145498
Website: www.physicsbyfiziks.com | Email: fiziks.physics@gmail.com
129
fiziks
Institute for NET/JRF, GATE, IIT‐JAM, M.Sc. Entrance, JEST, TIFR and GRE in Physics

 1
at r  R , Qenc  4 0 1   . So none of the options given are correct.
 e
Q9. In a Young’s double slit interference experiment, the slits are at a distance 2 L from each
other and the screen is at a distance D from the slits. If a glass slab of refractive index 
and thickness d is placed in the path of one of the beams, the minimum value of d for
the central fringe to be dark is
D D
(a) (b)
  1 D 2  L2   1L
 
(c) (d)
  1 2  1
Ans. : (d)
n 
Solution: For central fringe to be dark,   1d  d 
2 2  1
Q10. Consider a solenoid of radius R with n turns per unit length, in which a time dependent
current I  I 0 sin t (where  R / c  1 ) flows. The magnitude of the electric field at a
perpendicular distance r  R from the axis of symmetry of the solenoid, is
1
(a) 0 (b)  0 nI 0 R 2 cos t
2r
1 1
(c)  0 nI 0 r sin t (d)  0 nI 0 r cos t
2 2
Ans. : (d)
  
Solution: E  d l  
B
 t  d a ; B 
  0 nI t zˆ .
 dI
r
2r 2
dt r 0
 E  2r    0 n 2r d r     0 n  I 0  cos  t 
2
 1
 E     0 nI 0 r cos t
2
Q11. A constant electric current I in an infinitely long straight wire is suddenly switched on at
t  0 . The vector potential at a perpendicular distance r from the wire is given

by A 
kˆ 0 I  1
2

ln  ct  c 2 t 2  r 2
r
 . The electric field at a distance r   ct  is

H.No. 40-D, Ground Floor, Jia Sarai, Near IIT, Hauz Khas, New Delhi-110016
Phone: 011-26865455/+91-9871145498
Website: www.physicsbyfiziks.com | Email: fiziks.physics@gmail.com
130
fiziks
Institute for NET/JRF, GATE, IIT‐JAM, M.Sc. Entrance, JEST, TIFR and GRE in Physics

0 I 1 ˆ ˆ
(a) 0 (b)
2 t 2
i  j 
c 0 I c 0 I
(c) 
1 ˆ ˆ
i j  (d)  kˆ
2 c t  r
2 2 2
2 2 c t  r
2 2 2

Ans. : (d)
 
 A A   I r 1  2c 2 t 
c   .
Solution: E   
t

t
E 0 

2 ct  c 2 t 2  r 2  r  
2 c 2t 2  r 2


  c 0 I
E kˆ
2 c t  r
2 2 2

NET/JRF -(JUNE-2012)
 1   10  
Q12. The magnetic field corresponding to the vector potential A  F  r  3 r , where F is
2 r
a constant vector, is
   30   30 
(a) F (b)  F (c) F  4 r (d) F  4 r
r r
Ans. : (a)

   1   
  
 r  
Solution: B    A    F  r  10   3  . Since F is a constant vector, let
2  r 
xˆ yˆ zˆ
  
F  F0  xˆ  yˆ  zˆ  , F  r  F0 F0 F0  xˆ  z  y F0  yˆ  z  x F0  zˆ  y  x F0
x y z
xˆ yˆ zˆ
  

 F r   
x

y

z
 xˆF0  F0   yˆ  F0  F0   zˆF0  F0   2 F0  xˆ  yˆ  zˆ 
z  y F0 x  z F0
 y  x F0
  r
1   
  
   F  r  F0  xˆ  yˆ  zˆ   F ,   3  0 . Thus B  F
2 r
Q13. An electromagnetic wave is incident on a water-air interface. The phase of the
perpendicular component of the electric field, E  , of the reflected wave into the water is
found to remain the same for all angles of incidence. The phase of the magnetic field H
(a) does not change (b) changes by 3 / 2
(c) changes by  / 2 (d) changes by 
Ans. : (d)

H.No. 40-D, Ground Floor, Jia Sarai, Near IIT, Hauz Khas, New Delhi-110016
Phone: 011-26865455/+91-9871145498
Website: www.physicsbyfiziks.com | Email: fiziks.physics@gmail.com
131
fiziks
Institute for NET/JRF, GATE, IIT‐JAM, M.Sc. Entrance, JEST, TIFR and GRE in Physics

Q14. The magnetic field at a distance R from a long straight wire carrying a steady current I
is proportional to
(a) IR (b) I / R 2 (c) I 2 / R 2 (d) I / R
Ans. : (d)
Q15. Which of the following questions is Lorentz invariant?
2 2 2 2 2 2 2
(a) E  B (b) E  B (c) E  B (d) E B
Ans. : (b)
Q16. Charges Q, Q and 2Q are placed on the vertices of an equilateral triangle ABC of
sides of length a , as shown in the figure. The dipole moment of this configuration of
charges, irrespective of the choice of origin, is - 2Q
ĵ C
(a)  2aQ iˆ a
a
(b)  3aQ ˆj
A B
Q a Q
(c)  3aQ ˆj
(d) 0 iˆ
Ans. : (c)
Solution: Let coordinates of A is (l, m), then
 a   3a  ˆ 
 
   
p  qi ri   Q liˆ  mˆj  Q l  a iˆ  mˆj  2Q  l  iˆ   m   j
2  
 2  

      

p  Q liˆ  mˆj  Q l  a iˆ  mˆj  Q 2l  a iˆ  2m  3a ˆj  p   3aQˆj
mr
Q17. The vector potential A due to a magnetic moment m at a point r is given by A  .
r3

If m is directed along the positive z -axis, the x - component of the magnetic field, at the

point r , is
3myz 3mxy 3mxz 3mz 2  xy 
(a) (b)  (c) (d)
r5 r5 r5 r5
Ans. : (c)

Solution: m  mzˆ and
   m
 
 
B    A  3 2 cos rˆ  sin ˆ  3 3m  rˆ rˆ  m
r r
1

 1  
 xxˆ  yyˆ  zzˆ  r  3mxz
B  3 3mzˆ     mzˆ   Bx  5
r   r r  r
H.No. 40-D, Ground Floor, Jia Sarai, Near IIT, Hauz Khas, New Delhi-110016
Phone: 011-26865455/+91-9871145498
Website: www.physicsbyfiziks.com | Email: fiziks.physics@gmail.com
132
fiziks
Institute for NET/JRF, GATE, IIT‐JAM, M.Sc. Entrance, JEST, TIFR and GRE in Physics

NET/JRF -(DEC-2012)
Q18. Three charges are located on the circumference of a circle of radius R as shown in the
figure below. The two charges Q subtend an angle 900 at the centre Q Q
of the circle. The charge q is symmetrically placed with respect to
the charges Q . If the electric field at the centre of the circle is zero,
what is the magnitude of Q ?
q
(a) q / 2 (b) 2q (c) 2q (d) 4q
Ans. : (a)
1 Q 1 q
Solution: E1  E 2  and E3 
4 0 R 2
4 0 R 2
q
Resultant of E1 and E 2 is E  E12  E 22  2E1 , Thus E3  E  Q 
2
Q19. Consider a hollow charged shell of inner radius a and outer radius b . The volume
k
charge density is  r   ( k is constant) in the region a  r  b . The magnitude of the
r2
electric field produced at distance r  a is
k b  a 
(a) for all r  a ,
 0r 2
k b  a  kb
(b) for a  r  b and for r  b
 0r 2
 0r 2
k r  a  k b  a 
(c) for a  r  b and for r  b
 0r 2
 0r 2
k r  a  k b  a 
(d) for a  r  b and for r  b
 0a 2
 0r 2
Ans. : (c)
  1 1 1 k
Solution: For r  a :  E.da  E (4 r 2 )  Qenc    dV   2 r sin  drd d
2

0 0 0 r

4 k r 4 k  k ra
E (4 r 2 )  a dr  (r  a)  E   rˆ
0 0  0  r 2 

H.No. 40-D, Ground Floor, Jia Sarai, Near IIT, Hauz Khas, New Delhi-110016
Phone: 011-26865455/+91-9871145498
Website: www.physicsbyfiziks.com | Email: fiziks.physics@gmail.com
133
fiziks
Institute for NET/JRF, GATE, IIT‐JAM, M.Sc. Entrance, JEST, TIFR and GRE in Physics

4 k b 4 k  k  ba 
For r  b : E 4 r 2  a dr  (b  a)  E   rˆ
0 0  0  r 2 
Q20. Consider the interference of two coherent electromagnetic waves whose electric field
 
vectors are given by E1  iˆE 0 cos  t and E 2  ˆjE 0 cos t    where  is the phase

0
difference. The intensity of the resulting wave is given by E 2 , where E 2 is the
2
time average of E 2 . The total intensity is
(a) 0 (b)  0 E 02 (c)  0 E 02 sin 2  (d)  0 E 02 cos 2 
Ans. : (a)
Solution: Since waves are polarized in perpendicular direction hence there will be no
interference.
Q21. Four charges (two  q and two q ) are kept fixed at the four vertices of a square of side

a as shown. At the point P which is at a distance R from the centre  R  a  , the


q q
potential is proportional to
(a) 1/ R (b) 1/ R 2 a R P
(c) 1/ R 3
(d) 1/ R 4

q q
Ans. : (c)
Solution: Given configuration is quadrupole.
Q22. A point charges q of mass m is kept at a distance d below a grounded infinite
conducting sheet which lies in the xy - plane. For what value of d will the charge
remains stationary?
(a) q / 4 mg 0 (b) q / mg 0

(c) There is no finite value of d (d) mg 0 / q

Ans. : (a)
Solution: There is attractive force between point charge q and grounded conducting sheet that

1q2 q
can be calculate from method of images i.e.  mg  d 
4 0 2d  2
4 mg 0

H.No. 40-D, Ground Floor, Jia Sarai, Near IIT, Hauz Khas, New Delhi-110016
Phone: 011-26865455/+91-9871145498
Website: www.physicsbyfiziks.com | Email: fiziks.physics@gmail.com
134
fiziks
Institute for NET/JRF, GATE, IIT‐JAM, M.Sc. Entrance, JEST, TIFR and GRE in Physics

Q23. An infinite solenoid with its axis of symmetry along the z -direction carries a steady
current I .
 ẑ
The vector potential A at a distance R from the axis
(a) is constant inside and varies as R outside the solenoid
R
(b) varies as R inside and is constant outside the solenoid
1
(c) varies as inside and as R outside the solenoid
R
1
(d) varies as R inside and as outside the solenoid
R
Ans. : (d)
Q24. Consider an infinite conducting sheet in the xy -plane with a time dependent current

density Kt iˆ , where K is a constant. The vector potential at  x, y , z  is given


0 K 
by A  ct  z 2 iˆ . The magnetic field B is
4c
 0 Kt ˆ  0 Kz ˆ 0 K 0 K
(a) j (b)  j (c)  ct  z iˆ (d)  ct  z  ˆj
2 2c 2c 2c
Ans. : (d)
Ax  K
Solution: B    A  yˆ =  0 ct  z  ˆj
z 2c

Q25. When a charged particle emits electromagnetic radiation, the electric field E and the
 1   1 1
Poynting vector S  E  B at a larger distance r from emitter vary as n and m
0 r r
respectively. Which of the following choices for n and m are correct?
(a) n  1 and m  1 (b) n  2 and m  2
(c) n  1 and m  2 (d) n  2 and m  4
Ans. : (c)

H.No. 40-D, Ground Floor, Jia Sarai, Near IIT, Hauz Khas, New Delhi-110016
Phone: 011-26865455/+91-9871145498
Website: www.physicsbyfiziks.com | Email: fiziks.physics@gmail.com
135
fiziks
Institute for NET/JRF, GATE, IIT‐JAM, M.Sc. Entrance, JEST, TIFR and GRE in Physics

NET/JRF -(JUNE-2013)
Q26. A particle of charge e and mass m is located at the midpoint of the line joining two fixed
collinear dipoles with unit charges as shown in the figure. (The particle is constrained to
move only along the line joining the dipoles). Assuming that the length of the dipoles is
much shorter than their separation, the natural frequency of oscillation of the particle is

R R

  e, m  

2d 2d

6eR 2 6eR 6ed 2 6ed


(a) (b) (c) (d)
 0 md 5  0 md 4  0 mR 5  0 mR 4
Ans. : (d)
Solution: Let us displace the charge particle by small R R
amount x at A . Then the resultant electric field at
x
point A is given by   
 e, m A
2p  1 1  6d
E   3 
 x, 2d 2d
4 0  R  x  3
R  x    0 R 4
6ed k 6ed
F  eE   x . Then,    (where p  1 2d  2d )
 0 R 4
m  0 mR 4
Q27. A current I is created by a narrow beam of protons moving in vacuum with constant
 
velocity u . The direction and magnitude, respectively of the Poynting vector S outside the
beam at a radial distance r (much larger than the width of the beam) from the axis, are
   I2    I2
(a) S  u and S   (b) S ||  u  and S  
4 2  0 u r 2 4 2  0 u r 4
   I2    I2
(c) S || u and S   (d) S || u and S  
4 2  0 u r 2 4 2  0 u r 4

Ans. : (c)

H.No. 40-D, Ground Floor, Jia Sarai, Near IIT, Hauz Khas, New Delhi-110016
Phone: 011-26865455/+91-9871145498
Website: www.physicsbyfiziks.com | Email: fiziks.physics@gmail.com
136
fiziks
Institute for NET/JRF, GATE, IIT‐JAM, M.Sc. Entrance, JEST, TIFR and GRE in Physics

Solution: Let charge per unit length be  , hence I   u in z -direction.


0 I ˆ
The magnetic field at a distance r is B  .
2r
 I
The electric field at a distance r is E  rˆ  rˆ .
2 0 r 2 0 ur

EB I2
Hence Poynting vector S   zˆ
0 4 2  0 ur 2

Q28. If the electric and magnetic fields are unchanged when the potential A changes (in
  
suitable units) according to A  A  r̂ , where r  r t r̂ , then the scalar potential  must
simultaneously change to
(a)   r (b)   r (c)    r /  t (d)    r /  t
Ans. : (c)

Solution: A  A    A  rˆ    /  r  1    r  C
V  V   / t V   r / t
Q29. Consider an axially symmetric static charge distribution of the form,
2
r 
   0  0  e  r / r0 cos 2 
r
The radial component of the dipole moment due to this charge distribution is
(a) 2 0 r04 (b)  0 r04 (c)  0 r04 (d)  0 r04 / 2
Ans. : (a)
2
r 
Solution: p   r  r d      r    0  0  e  r  / r0 cos 2   r  2 sin dr dd
V  r 
  2

 dr  sin  d  cos 2  d  20 r04


 r  / r0
p r 0 0
2
r e
r  0 0 0

Q30. The components of a vector potential A   A0 , A1 , A2 , A3  are given by

A  k  xyz, yzt , zxt , xyt 

where k is a constant. The three components of the electric field are


(a) k  yz, zx, xy  (b) k  x, y, z  (c) 0, 0, 0  (d) k  xt , yt , zt 

H.No. 40-D, Ground Floor, Jia Sarai, Near IIT, Hauz Khas, New Delhi-110016
Phone: 011-26865455/+91-9871145498
Website: www.physicsbyfiziks.com | Email: fiziks.physics@gmail.com
137
fiziks
Institute for NET/JRF, GATE, IIT‐JAM, M.Sc. Entrance, JEST, TIFR and GRE in Physics

Ans. : (c)
Solution: A   , Ax , Ay , Az     kxyz, Ax  kyzt , Ay  kzxt , Az  kxyt

A
Since   k  yzxˆ  xzyˆ  xyzˆ  and  k  yzxˆ  xzyˆ  xyzˆ 
t

 A
E     k  yzxˆ  xzyˆ  xyzˆ   k  yzxˆ  xzyˆ  xyzˆ   0  E  0,0,0 
t
Q31. An oscillating current I t   I 0 exp it  flows in the direction of the y -axis through a

thin metal sheet of area 1.0 cm 2 kept in the xy -plane. The rate of total energy radiated
per unit area from the surfaces of the metal sheet at a distance of 100 m is

(a) I 0 / 12 0 c 3  (b) I 02 2 / 12 0 c 3 

(c) I 02 3 / 12 0 c 3  (d) I 02 4 / 12 0c 3 

Ans. : (d)
NET/JRF -(DEC-2013)
Q32. A horizontal metal disc rotates about the vertical axis in a uniform magnetic field
pointing up as shown in the figure. A circuit is made by connecting one end A of a
resistor to the centre of the disc and the other end B to its edge through a sliding contact.
The current that flows through the resistor is
B
A B

(a) zero (b) DC from A to B


(c) DC from B to A (d) AC
Ans. : (b)
Q33. The force between two long and parallel wires carrying currents I 1 and I 2 and separated
by a distance D is proportional to
(b) I 1  I 2  / D (c) I 1 I 2 / D 
2
(a) I 1 I 2 / D (d) I 1 I 2 / D 2
Ans. : (a)

H.No. 40-D, Ground Floor, Jia Sarai, Near IIT, Hauz Khas, New Delhi-110016
Phone: 011-26865455/+91-9871145498
Website: www.physicsbyfiziks.com | Email: fiziks.physics@gmail.com
138
fiziks
Institute for NET/JRF, GATE, IIT‐JAM, M.Sc. Entrance, JEST, TIFR and GRE in Physics

Q34. The electric field of an electromagnetic wave is given by



E  E 0 cos 0.3x  0.4 y  1000t kˆ .

The associated magnetic field B is
(a) 10 3 E 0 cos 0.3 x  0.4 y  1000t kˆ


(b) 10 4 E 0 cos 0.3 x  0.4 y  1000t  4iˆ  3 ˆj
(c) E cos 0.3 x  0.4 y  1000t 0.3iˆ  0.4 ˆj 
0

(d) 10 E cos 0.3 x  0.4 y  1000t 3iˆ  4 ˆj 


2
0

Ans. : (b)

Solution: k    0.3xˆ  0.4 yˆ  ,   1000 
kE
 0.3 xˆ  0.4 yˆ   E 0 cos 0.3x  0.4 y  1000t kˆ
1
B 
 
 B  10 4 E 0 cos 0.3x  0.4 y  1000t  4iˆ  3 ˆj  
Q35. A point charge q is placed symmetrically at a distance d from two perpendicularly
placed grounded conducting infinite plates as shown in the figure. The net force on the
charge (in units of 1 / 4 0 ) is
q
(a)
q2
8d 2
 
2 2  1 away from the corner
d

d
(b)
q2
8d 2
 
2 2  1 towards the corner

q2
(c) towards the corner
2 2d 2
3q 2 F3
(d) away from the corner d d
8d 2 q q
Ans. : (b) F1
F2 d
q2 q2
Solution: F 1  F 2  k and F 3  k
4d 2 8d 2 d
q 2 q q
Resultant of F 1 , F 2 is F12  F12  F22  2 2k . 2d
8d 2

H.No. 40-D, Ground Floor, Jia Sarai, Near IIT, Hauz Khas, New Delhi-110016
Phone: 011-26865455/+91-9871145498
Website: www.physicsbyfiziks.com | Email: fiziks.physics@gmail.com
139
fiziks
Institute for NET/JRF, GATE, IIT‐JAM, M.Sc. Entrance, JEST, TIFR and GRE in Physics

Net force F  k
q2
8d 2
 
2 2  1 (towards the corner)

Q36. If the electrostatic potential V r ,  ,   in a charge free region has the form

V  r ,  ,    f  r  cos  , then the functional form of f r  (in the following a and b are

constants) is:
b b b r
(a) ar 2  (b) ar  (c) ar  (d) a ln 
r r2 r b
Ans. : (b)

1   V  1   V  1   2V 
Solution:  V  2  r 2  2  sin   2 2  2 0
2

r r  r  r sin      r sin    
1   2 f  1 
 r cos    2 sin  f    sin     0
r r  r  r sin  
2

cos   2 2 f f  f
  r 2  2r   2  2sin  cos    0
r2  r r  r sin 

2 f f
 r2  2r  2 f  r   0
r
2
r
b
f r   ar  satisfy the above equation.
r2
Q37. Let four point charges q,  q / 2, q and  q / 2 be placed at the vertices of a square of
side a . Let another point charge  q be placed at the centre of the square (see the figure).

q/2 q

q

q q/2
Let V r  be the electrostatic potential at a point P at a distance r  a from the centre
of the square. Then V 2r  / V r  is
1 1 1
(a) 1 (b) (c) (d)
2 4 8
Ans. : (d)

H.No. 40-D, Ground Floor, Jia Sarai, Near IIT, Hauz Khas, New Delhi-110016
Phone: 011-26865455/+91-9871145498
Website: www.physicsbyfiziks.com | Email: fiziks.physics@gmail.com
140
fiziks
Institute for NET/JRF, GATE, IIT‐JAM, M.Sc. Entrance, JEST, TIFR and GRE in Physics

q q
Solution: According to multipole expansion Qmono   q qq 0
2 2
 q q  q q q   q q  qq q 
p  q  xˆ  yˆ     xˆ  yˆ   0  q   xˆ  yˆ    xˆ  yˆ   0
2 2  2 2 2   2 2  22 2 
1 V 2r  1
Thus, V    .
r 3
V r  8

Q38.    
Let V , A and V , A denote two sets of scalar and vector potentials, and  is a scalar
function. Which of the following transformations leave the electric and magnetic fields
(and hence Maxwell’s equations) unchanged?
 
(a) A  A   and V   V  (b) A  A   and V   V  2
t t
 
(c) A  A   and V   V  (d) A  A   and V   V 
t t
Ans. : (a)

JRF/NET–(JUNE-2014)

Q39. A time-dependent current I t   Ktzˆ (where K is a constant) is switched on at t  0 in
an infinite current-carrying wire. The magnetic vector potential at a perpendicular
distance a from the wire is given (for time t  a / c ) by

 0 K c t a  K
2 2 2
ct
ct  a 2  z 2 t
4 c  c t  a  dz a
(a) zˆ dz (b) zˆ 0
2 2 a 2  z 2 1 / 2
2
4  ct
2
 z2  1/ 2

c 2t 2  a 2
 K ct ct  a 2  z 2  K t
(c) zˆ 0  dz (d) zˆ 0  dz
4 c ct a 2  z 2 1 / 2 4  c 2t 2  a 2 a 2
 z2 
1/ 2

Ans. : (a)
  
I  tr  0 
K t  R / c  I
Solution: A  zˆ 0
4 

R
dz  zˆ
4 

R
dz
dz
z R
 0 K c t  a
2 2 2
ct  a 2  z 2
4 c  c2t 2 a2
 A  zˆ dz
  P
1/2
a2  z2 a

H.No. 40-D, Ground Floor, Jia Sarai, Near IIT, Hauz Khas, New Delhi-110016
Phone: 011-26865455/+91-9871145498
Website: www.physicsbyfiziks.com | Email: fiziks.physics@gmail.com
141
fiziks
Institute for NET/JRF, GATE, IIT‐JAM, M.Sc. Entrance, JEST, TIFR and GRE in Physics

Q40. A current i p flows through the primary coil of a transformer. The graph of i p t  as a

function of time t is shown in the figure below.


i p t 

1 2 3 t
Which of the following graphs represents the current i S in the secondary coil?
is  t  is  t 

(a) (b)
1 2 3 t

1 2 3 t

is  t  is  t 
(c) (d)

1 2 3t

1 2 3t
Ans. : (c)
di p
Solution: is  
dt
Q41. If the electrostatic potential in spherical polar coordinates is
 r    0 e  r / r
0

where  0 and r0 are constants, then the charge density at a distance r  r0 will be

 0 0 e 0 0  0 0 2e 0 0
(a) (b) (c)  (d) 
er 0
2
2r02 er 0
2
r02
Ans. : (a)

Solution:   2       0   2 
0

1   2   1   2 0  r / r0  1 0  2  r / r0
 2  2 r  2 r  e
r r  r  r r  r0
 2
r r r
r e  
 0

1 0  2 1  r / r0    1 2 
  2   r   e  2re  r / r0    0   e  r / r0  e  r / r0 
r r0 
2
r0  r0  r0 r 
H.No. 40-D, Ground Floor, Jia Sarai, Near IIT, Hauz Khas, New Delhi-110016
Phone: 011-26865455/+91-9871145498
Website: www.physicsbyfiziks.com | Email: fiziks.physics@gmail.com
142
fiziks
Institute for NET/JRF, GATE, IIT‐JAM, M.Sc. Entrance, JEST, TIFR and GRE in Physics

0  1
2 1  0  0  0 0
At a distance r  r0 ,  2    e  e    2     0   2   2
1

r0  r0 r0  r0 e  r0 e  r0 e

Q42. If A  yziˆ  zxˆj  xykˆ and C is the circle of unit radius in the plane defined by z  1 ,
 
with the centre on the z - axis, then the value of the integral  A  d  is
C

 
(a) (b)  (c) (d) 0
2 4
Ans. : (d)
iˆ ˆj kˆ
    
Solution:   A   iˆ  x  x   ˆj  y  y   kˆ  z  z   0
x y z
yz zx xy
    

Since  A  d      A  d a  0
C

S

Q43. Consider an electromagnetic wave at the interface between two homogenous dielectric
media of dielectric constants  1 and  2 . Assuming  2   1 and no charges on the surface,
 
the electric field vector E and the displacement vector D in the two media satisfy the
following inequalities
       
(a) E 2  E1 and D2  D1 (b) E 2  E1 and D2  D1
       
(c) E 2  E1 and D2  D1 (d) E 2  E1 and D2  D1

Ans. : (c)
Q44. A charge  e  is placed in vacuum at the point d ,0,0  , where d  0 . The region x  0
d 
is filled uniformly with a metal. The electric field at the point  ,0,0  is
2 
10e 10e
(a)  1, 0, 0 (b) 1, 0, 0
9 0 d 2
9 0 d 2
e e
(c) 1, 0, 0 (d)  1, 0, 0
 0 d 2  0 d 2
Ans. : (b)

Solution: E

d
E
2 P
x
e d 0 d e

H.No. 40-D, Ground Floor, Jia Sarai, Near IIT, Hauz Khas, New Delhi-110016
Phone: 011-26865455/+91-9871145498
Website: www.physicsbyfiziks.com | Email: fiziks.physics@gmail.com
143
fiziks
Institute for NET/JRF, GATE, IIT‐JAM, M.Sc. Entrance, JEST, TIFR and GRE in Physics

1 e 1 4e 1 e 1 4e
E   and E  
4 0  3d / 2  2
4 0 9d 2
4 0  d / 2  2
4 0 d 2

Thus resultant electric field at point P is


1 4e 1 4e 1 40e 1 10e  1 10e
E  E  E      E xˆ
4 0 9d 2
4 0 d 2
4 0 9d 2
9 0 d 2
9 0 d 2
Q45. A beam of light of frequency  is reflected from a dielectric-metal interface at normal
incidence. The refractive index of the dielectric medium is n and that of the metal is
n 2  n1  i  . If the beam is polarised parallel to the interface, then the phase change
experienced by the light upon reflection is
(a) tan 2 /   (b) tan 1 1 /   (c) tan 1 2 /   (d) tan 1 2  
Ans. : (c)
 1     v c/n
Solution: Since E 0 R    E0 I where   1   1  i
 1  
 v 2 c / n 1  i  
 i      ei /2     i /2   

 E0 R   E     
E e E0 I where tan   .
  4  e   4  
 2  i 
0 I i 0 I

2
 
2
 2
2 2
Thus phase change     / 2     tan   cot      tan 1  
 
Q46. A thin, infinitely long solenoid placed along the z - axis contains a magnetic flux  .
Which of the following vector potentials corresponds to the magnetic field at an arbitrary
point x, y, z  ?

   
(a) Ax , Ay , Az    
y x
, ,0 
 2 x  y 2 x  y
2 2 2 2

   
(b) Ax , Ay , Az    
y x
, ,0 
 2 x  y  z 2 x  y  z
2 2 2 2 2 2

  x y  x y 
(c) Ax , Ay , Az     , ,0 
 2 x  y 2 x  y
2 2 2 2

   
(d) Ax , Ay , Az    
x y
, ,0 
 2 x  y 2 x  y
2 2 2 2

Ans. : (a)
  
Solution: B    A  0
H.No. 40-D, Ground Floor, Jia Sarai, Near IIT, Hauz Khas, New Delhi-110016
Phone: 011-26865455/+91-9871145498
Website: www.physicsbyfiziks.com | Email: fiziks.physics@gmail.com
144
fiziks
Institute for NET/JRF, GATE, IIT‐JAM, M.Sc. Entrance, JEST, TIFR and GRE in Physics

Q47. An electromagnetically-shielded room is designed so that at a frequency   10 7 rad/s


the intensity of the external radiation that penetrates the room is 1% of the incident

 10 6 m  is the conductivity of the shielding material, its


1 1
radiation. If  
2
minimum thickness should be (given that ln 10  2.3 )
(a) 4.60 mm (b) 2.30 mm (c) 0.23 mm (d) 0.46 mm
Ans. : (b)
1  I0 
Solution: I  I 0 e 2 z  z 
2  I 
ln

I0  1 1
where  100,     106  4  107  107  103
I 2 2 2
1
z ln 100   2.30 mm
2  103
Q48. A charged particle is at a distance d from an infinite conducting plane maintained at zero
potential. When released from rest, the particle reaches a speed u at a distance d / 2 from
the plane. At what distance from the plane will the particle reach the speed 2u ?
(a) d / 6 (b) d / 3 (c) d / 4 (d) d / 5
Ans. : (d) x
d x
2
1 q d x2
A2
q 2
Solution: F  ma  m   2   2 where A  . P q
dt 2
4 0 4d 2
dt x 16 m 0
d
dv A dv A dx 1 d 2 d  A

dt
 2 v
x dt
 2
x dt

2 dt
v   
dt  x 
  0

v2 A A 1 1 d
   C at  x  d , v  0  C    v  2 A    .
2 x d x d q

 1 1 2A 1 1 d
Thus u  2 A    then 2u  2 A     x 
d /2 d  d x d 5

H.No. 40-D, Ground Floor, Jia Sarai, Near IIT, Hauz Khas, New Delhi-110016
Phone: 011-26865455/+91-9871145498
Website: www.physicsbyfiziks.com | Email: fiziks.physics@gmail.com
145
fiziks
Institute for NET/JRF, GATE, IIT‐JAM, M.Sc. Entrance, JEST, TIFR and GRE in Physics

NET/JRF–(DEC-2014)
Q49. A charged particle moves in a helical path under the influence of a constant
magnetic field. The initial velocity is such that the component along the
magnetic field is twice the component in the plane normal to the magnetic l
field.
The ratio  / R of the pitch  to the radius R of the helical path is 2R
(a)  / 2 (b) 4 (c) 2 (d) 
Ans. : (b)
Solution: v  2v

2 R 2 R l
Pitch of the helix l  vT  v  2v  4 R   4
v v R
Q50. A parallel beam of light of wavelength  is incident normally on a thin polymer film
with air on both sides. If the film has a refractive index n  1 , then second-order bright
fringes can be observed in reflection when the thickness of the film is
(a)  / 4n (b)  / 2n (c) 3 / 4n (d)  / n
Ans. : (c)

Solution: For constructive interference: 2nd cos    2m  1
2
For normal incidence   0  and second order  m  1

 3
 2nd cos 0   2 1  1 d 
2 4n
Q51. A solid sphere of radius R has a charge density, given by
 ar 
 r    0 1  
 R
where r is the radial coordinate and  0 , a and R are positive constants. If the

magnitude of the electric field at r  R / 2 is 1.25 times that at r  R , then the value of a
is
(a) 2 (b) 1 (c) 1 / 2 (d) 1 / 4
Ans. : (b)

H.No. 40-D, Ground Floor, Jia Sarai, Near IIT, Hauz Khas, New Delhi-110016
Phone: 011-26865455/+91-9871145498
Website: www.physicsbyfiziks.com | Email: fiziks.physics@gmail.com
146
fiziks
Institute for NET/JRF, GATE, IIT‐JAM, M.Sc. Entrance, JEST, TIFR and GRE in Physics

r
  1  1  ar 
 E .d a  Qenc  E  4 r 2   1   4 r dr
2
Solution:
S
0 0 0
0
 R

40  2 ar 3  4 0  r 3 ar 4  0  r ar 2 
 r 

 0 0 
 E  4 r  r 
2
 dr     E    
R   0  3 4R   0  3 4R 

0  R / 2 aR 2 / 4  0  R aR 2 
 Er  R / 2  1.25 Er  R      1.25   
0  3 4R   0  3 4R 

 1 a  5  1 a   1 a   5 5a  5a a 5 1
            
 6 16  4  3 4   6 16   12 16  16 16 12 6
4a 5  2 a 3
     a 1
16 12 4 12
Q52. The electrostatic lines of force due to a system of four point charges
is sketched here. At large distance r , the leading asymptotic
behaviour of the electrostatic potential is proportional to
(a) r (b) r 1
(c) r 2 (d) r 3
Ans. : (d)
1
Solution: The given electrostatic line of force is due to a quadrupole. So V  .
r3
Q53. A plane electromagnetic wave incident normally on the surface of a material is partially
reflected. Measurements on the standing wave in the region in front of the interface such
that the ratio of the electric field amplitude at the maxima and the minima is 5. The ratio
of the reflected intensity to the incident intensity is
(a) 4 / 9 (b) 2 / 3 (c) 2 / 5 (d) 1 / 5
Ans. : (a)
E0 I  E0 R E 2
Solution:  5  E0 I  E0 R  5  E0 I  E0 R   6 E0 R  4 E0 I  0 R 
E0 I  E0 R E0 I 3
2
I E  4
 R   0R  
I I  E0 I  9

H.No. 40-D, Ground Floor, Jia Sarai, Near IIT, Hauz Khas, New Delhi-110016
Phone: 011-26865455/+91-9871145498
Website: www.physicsbyfiziks.com | Email: fiziks.physics@gmail.com
147
fiziks
Institute for NET/JRF, GATE, IIT‐JAM, M.Sc. Entrance, JEST, TIFR and GRE in Physics

Q54. A non-relativistic particle of mass m and charge e , moving with a velocity v and

acceleration a , emits radiation of intensity I . What is the intensity of the radiation
 
emitted by a particle of mass m / 2 , charge 2e , velocity v / 2 and acceleration 2a ?
(a) 16 I (b) 8 I (c) 4 I (d) 2 I
Ans. : (a)
q 2 a 2 sin 2  I 2 q22 a22 I 2 4e 2  4a 2
Solution:  I       16  I 2  16 I
r2 I1 q12 a12 I e2 a 2

NET/JRF–(JUNE-2015)
Q55. A Plane electromagnetic wave is travelling along the positive z -direction. The maximum
electric field along the x - direction is 10 V / m . The approximate maximum values of the
power per unit area and the magnetic induction B , respectively, are
(a) 3.3  107 watts / m 2 and 10 tesla
(b) 3.3  107 watts / m 2 and 3.3  108 tesla
(c) 0.265 watts / m 2 and 10 tesla

(d) 0.265 watts / m 2 and 3.3 108 tesla


Ans. (d)
P 1 1
 c 0 E02   3 108  8.86 1012  10   0.132 W / m 2
2
Solution: E0  10V / m, I 
A 2 2
E0 10
B0    3.3  108 Tesla
c 3 10 8

 
Q56.    
Which of the following transformations V , A  V ', A ' of the electrostatic potential

V and the vector potential A is a gauge transformation?
   

(a) V   V  ax, A  A  at kˆ  
(b) V   V  ax, A  A  at kˆ 
   

(c) V   V  ax, A  A  at iˆ  
(d) V   V  ax, A  A  at iˆ 
Ans. (d)
 
Solution: V   V     ax    axt  c
t t
  
   atiˆ  0 . Thus, A  A  atxˆ
H.No. 40-D, Ground Floor, Jia Sarai, Near IIT, Hauz Khas, New Delhi-110016
Phone: 011-26865455/+91-9871145498
Website: www.physicsbyfiziks.com | Email: fiziks.physics@gmail.com
148
fiziks
Institute for NET/JRF, GATE, IIT‐JAM, M.Sc. Entrance, JEST, TIFR and GRE in Physics

Q57. Suppose the yz -plane forms a chargeless boundary between two media of permittivities
left and right where left :right  1: 2 , if the uniform electric field on the left is
 
 
Eleft  c iˆ  ˆj  kˆ (where c is a constant), then the electric field on the right Eright is


(a) c 2iˆ  ˆj  kˆ  
(b) c iˆ  2 ˆj  2kˆ 
1   1 1 
(c) c  iˆ  ˆj  kˆ  (d) c  iˆ  ˆj  kˆ 
2   2 2 
Ans. (c) y
1 2
 
Solution: E1  c ˆj  kˆ  E2

1 
D1  D2 1 E1 2 E2  E21  E1
2 x
1 ˆ  1 
 E2  ci  E2  c  iˆ  ˆj  kˆ 
2 2  z
Q58. A proton moves with a speed of 300 m / s in a circular orbit in the xy -plan in a magnetic
field 1 tesla along the positive z - direction. When an electric field of 1 V / m is applied
along the positive y -direction, the center of the circular orbit
(a) remains stationary
(b) moves at 1 m / s along the negative x  direction
(c) moves at 1 m / s along the positive z  direction
(d) moves at 1 m / s along the positive x  direction
z
Ans. (d)

Solution: Change particle will deflect in  x -direction with B
E 1 y
v  1 m/ s . 
B 1 E
x
Q59. Consider a rectangular wave guide with transverse dimensions 2 m  1 m driven with an

angular frequency   109 rad / s . Which transverse electric TE  modes will propagate

in this wave guide?


(a) TE10 , TE01 and TE20 (b) TE01 , TE11 and TE20

(c) TE01 , TE10 and TE11 (d) TE01 , TE10 and TE22

H.No. 40-D, Ground Floor, Jia Sarai, Near IIT, Hauz Khas, New Delhi-110016
Phone: 011-26865455/+91-9871145498
Website: www.physicsbyfiziks.com | Email: fiziks.physics@gmail.com
149
fiziks
Institute for NET/JRF, GATE, IIT‐JAM, M.Sc. Entrance, JEST, TIFR and GRE in Physics

Ans. (a)

m2 n2
Solution: mn  C 
a 2 b2
c 3 108  3.14
10    4.71 108 rod / sec
a 2
c 3 108  3.14
01    9.42 108 rod / sec
b 1
1 1
11  c  2  10.53 108 rod / sec
a b
2

2c
20   9.72 108 rod / sec
a
4 4
22  c  2  10.5  108 rod / sec
a b
2

Since   10 , 01 , 20

Q60. The electric and magnetic fields in the charge free region z  0 are given by
 
E  r , t   E0 e  k1z cos  k2 x  t  ˆj
  E
B  r , t   0 e  k1z  k1 sin  k2 x  t  iˆ  k2 cos  k2 x  t  kˆ 
  
where  , k1 and k2 are positive constants. The average energy flow in the x -direction is
E02 k2 2 k1z E02 k2 E02 k1 2 k1z 1
(a) e (b) e 2 k1z (c) e (d) c 0 E02 e 2 k1 z
20 0 20 2
Ans. (a)
 1   E 2 e 2 k1 z 
Solution: S 
0

EB  0
0  1
  
k cos  sin   kˆ  k2 cos 2  iˆ  , where   k2 x  t

 k E 2 e 2 k1 z E02 k2 2 k1 z
 S  2 0  e
2 0 20
Q61. A uniform magnetic field in the positive z -direction passes through a circular wire loop
of radius 1 cm and resistance 1  lying in the xy -plane. The field strength is reduced
from 10 tesla to 9 tesla in 1s . The charge transferred across any point in the wire is
approximately
(a) 3.1104 coulomb (b) 3.4 104 coulomb
(c) 4.2 104 coulomb (d) 5.2  104 coulomb

H.No. 40-D, Ground Floor, Jia Sarai, Near IIT, Hauz Khas, New Delhi-110016
Phone: 011-26865455/+91-9871145498
Website: www.physicsbyfiziks.com | Email: fiziks.physics@gmail.com
150
fiziks
Institute for NET/JRF, GATE, IIT‐JAM, M.Sc. Entrance, JEST, TIFR and GRE in Physics

Ans. (a)
d dq  1 d A  r 2
Solution:    I     dq   dB  dB
dt dt R R dt R R

 
2
3.14  102
 dq   1  3.14  104 coulomb
1
Q62. A rod of length L carries a total charge Q distributed uniformly. If this is observed in a
frame moving with a speed v along the rod, the charge per unit length (as measured by
the moving observer) is

Q  v2  Q v2 Q Q
(a) 1  2  (b) 1 2 (c) (d)
L c  L c v2  v2 
L 1 L 1  2 
c2  c 
Ans. : (c)
0 Q
Solution:   
v2
v2
1 L 1
c2 c2
NET/JRF–(DEC-2015)
Q63. A hollow metallic sphere of radius a , which is kept at a potential V0 has a charge Q at its
centre. The potential at a point outside the sphere, at a distance r from the centre, is
Q Va Q V a2 V0 a
(a) V0 (b)  0 (c)  02 (d)
4 0 r r 4 0 r r r
Ans. : (d)
Q
Solution: Let charge on conductor is Q , then V0 
4 0 a
Q Va
Now V V  0
4 0 r r
Q64. Consider a charge Q at the origin of 3 - dimensional coordinate system. The
flux of the electric field through the curved surface of a cone that has a height
h
h and a circular base of radius R (as shown in the figure) is
R Q
Q Q hQ QR
(a) (b) (c) (d)
0 2 0 R 0 2h 0
Ans. : (b)

H.No. 40-D, Ground Floor, Jia Sarai, Near IIT, Hauz Khas, New Delhi-110016
Phone: 011-26865455/+91-9871145498
Website: www.physicsbyfiziks.com | Email: fiziks.physics@gmail.com
151
fiziks
Institute for NET/JRF, GATE, IIT‐JAM, M.Sc. Entrance, JEST, TIFR and GRE in Physics

Q65. Given a uniform magnetic field B  B0 kˆ (where B0 is a constant), a possible choice for
the magnetic vector potential A is

(a) B0 yiˆ (b)  B0 yiˆ 


(c) B0 xjˆ  yiˆ  
(d) B0 xiˆ  yjˆ 
Ans. : (b)
 
Solution: (a)   A   B0 kˆ
 
(b)   A  B0 kˆ
 
(c)   A  0
 
(d)   A  0
Q66. A beam of unpolarized light in a medium with dielectric constant 1 is reflected from a

plane interface formed with another medium of dielectric constant 2  3 1 . The two

media have identical magnetic permeability. If the angle of incidence is 600 , then the
reflected light
(a) is plane polarized perpendicular to the plane of incidence
(b) is plane polarized parallel to the plane of incidence
(c) is circularly polarized
(d) has the same polarization as the incident light
Ans. : (a)
n  I
Solution:  B  tan 1  2 
1
 n1 
2
 2 
 B  tan 1 
  
  tan
1
3  
 1
  B  600 (hence reflected light is plane polarized perpendicular to plane of incidence))

Q67. A small magnetic needle is kept at  0, 0  with its moment along the x -axis. Another

small magnetic needle is at the point 1,1 and is free to rotate in the xy - plane. In

equilibrium the angle  between their magnetic moments is such that


1
(a) tan   (b) tan   0 (c) tan   3 (d) tan   1
3
Ans. : (c)
H.No. 40-D, Ground Floor, Jia Sarai, Near IIT, Hauz Khas, New Delhi-110016
Phone: 011-26865455/+91-9871145498
Website: www.physicsbyfiziks.com | Email: fiziks.physics@gmail.com
152
fiziks
Institute for NET/JRF, GATE, IIT‐JAM, M.Sc. Entrance, JEST, TIFR and GRE in Physics

0      mm
Solution: U   m  m2  3  m1  rˆ  m2  rˆ    U  0 1 3 2 cos   3cos 450 cos   450  
3  1
4 r 4 r  
For stable position energy is minimum i.e.
 r̂ y
 mm m2
U  

0 0 132
4 r   sin  
3
 
sin   450   0 r
 2  
450 x

3  sin  cos   m1
 sin       tan   3
2 2 2 
so, option (c) is correct .

Q68. A dipole of moment p , oscillating at frequency  , radiates spherical waves. The vector
potential at large distance is
   eikr 
A  r   0 i p
4 r
1  
To order   the magnetic field B at a point r  rnˆ is
r
0  2  eikr 0  2  eikr
(a)    nˆ
n
ˆ  p (b)   n
ˆ  p 
4 C r 4 C r
0 2   eikr  0  2  eikr
(c)   k  nˆ  p  p (d)  p
4 r 4 C r
Ans. : (b)
 
Solution: Let p  pzˆ , then B must be in ˆ direction.

Check nˆ  p  rˆ  zˆ  ˆ . So, correct option is (b).
Q69. The frequency dependent dielectric constant of a material is given by
A
    1  2
0   2  i
where A is a positive constant, 0 is the resonant frequency and  is the damping

coefficient. For an electromagnetic wave of angular frequency   0 , which of the


following is true? (Assume that  1 ).
0
(a) There is negligible absorption of the wave
(b) The wave propagation is highly dispersive
(c) There is strong absorption of the electromagnetic wave
(d) The group velocity and the phase velocity will have opposite sign
H.No. 40-D, Ground Floor, Jia Sarai, Near IIT, Hauz Khas, New Delhi-110016
Phone: 011-26865455/+91-9871145498
Website: www.physicsbyfiziks.com | Email: fiziks.physics@gmail.com
153
fiziks
Institute for NET/JRF, GATE, IIT‐JAM, M.Sc. Entrance, JEST, TIFR and GRE in Physics

Ans. : (a)
Solution: When   0 , there is negligible absorption of the wave.

NET/JRF–(JUNE-2016)
Q70. Four equal charges of Q , each are kept at the vertices of a square of side R . A particle
of mass m and charge Q is placed in the plane of the square at a short distance

a   R  from the centre. If the motion of the particle is confined to the plane, it will

undergo small oscillations with an angular frequency

Q2 Q2
(a) (b)
2 0 R 3 m  0 R 3m

2Q 2 Q2
(c) (d)
 0 R 3 m 4 0 R 3m

Ans. : (c)
kQ Q Q
Solution: E1  E2   
 R  R2 
2
E2 E1 y
 a    
 2 4 
a x
Resultant field E12, y  2 E1 cos 
R /2
2kQ  R 2kQ  R
E12, y  a    a   Q R /2 Q
 2  2
3 3
 R  R2 
2 2  R2  2
 a      
 2 4   2 

4 2kQ  R
E12, y  a  
R 3
 2
Q R /2 Q
kQ R
Similarly; E3  E4   2 a
 R  R 
2 2

  a     
 2  4  E3 a E4

2kQ R 
Resultant E34, y  2 E3 cos    a
2 
3
 R  R 
2 2 2

  a    Q Q
 2  4 

H.No. 40-D, Ground Floor, Jia Sarai, Near IIT, Hauz Khas, New Delhi-110016
Phone: 011-26865455/+91-9871145498
Website: www.physicsbyfiziks.com | Email: fiziks.physics@gmail.com
154
fiziks
Institute for NET/JRF, GATE, IIT‐JAM, M.Sc. Entrance, JEST, TIFR and GRE in Physics

4 2kQ  R 
 E34, y    a
R 3
2 

4 2kQ  R  R  8 2kQ
Resultant E     a     a    a
R 3
 2  2  R3

8 2 1 2 2Q
E  Qa  E   a
R 3
4 0  0 R 3

2 2Q 2 2 2Q 2
 F  QE   a
 0 R 3  0 mR 3
Q71. Two parallel plate capacitors, separated by distances x and 1.1x respectively, have a
dielectric material of dielectric constant 3.0 inserted between the plates and are
connected to a battery of voltage V . The difference in charge on the second capacitor
compared to the first is
(a) 66% (b) 20% (c) 3.3% (d) 10%
Ans. : (d)
3 0 A 3 A
Solution: Q1  C1V1  V , Q2  C2V2  0 V
x 1.1x
 1  3 A
  1  0 V
Q2  Q1 x
 100%  
1.1 
 100  9%
Q1 3  A
0
V
x
Q72. The half space region x  0 and x  0 are filled with dielectric
x0 x0
media of dielectric constants 1 and  2 respectively. There is a 2 1 
uniform electric field in each part. In the right half, the electric field 1 E1

makes an angle 1 to the interface. The corresponding angle  2 in


the left half satisfies 2
(a) 1 sin  2   2 sin 1 (b) 1 tan  2   2 tan 1 
E2
(c) 1 tan 1   2 tan  2 (d) 1 sin 1   2 sin  2
Ans. : (c)

H.No. 40-D, Ground Floor, Jia Sarai, Near IIT, Hauz Khas, New Delhi-110016
Phone: 011-26865455/+91-9871145498
Website: www.physicsbyfiziks.com | Email: fiziks.physics@gmail.com
155
fiziks
Institute for NET/JRF, GATE, IIT‐JAM, M.Sc. Entrance, JEST, TIFR and GRE in Physics

E1
tan 1 E E 
Solution:  1  1
tan  2 E2 E2
 E 
1  E2 

E2

E1  2 tan 1  2
D1  D2  1 E1   2 E2      1 tan 1   2 tan  2
E2 1

tan  2 1

Q73. The x - and z -components of a static magnetic field in a region are Bx  B0 x 2  y 2  


and Bz  0 , respectively. Which of the following solutions for its y -component is
consistent with the Maxwell equations?
(a) By  B0 xy (b) By  2 B0 xy

1 

(c) By   B0 x 2  y 2  (d) By  B0  x3  xy 2 
3 
Ans. : (b)

 
Solution: Bx  B0 x 2  y 2 , Bz  0

  B By Bz By B


  B  0  x   0   x  2 B0 x  By  2 B0 xy
x y z y x
Q74. A magnetic field B is Bzˆ in the region x  0 and zero elsewhere. A rectangular loop, in
the xy -plane, of sides l (along the x -direction) and h (along the y - direction) is
inserted into the x  0 region from the x  0 region at constant velocity v  vxˆ . Which of
the following values of l and h will generate the largest EMF?
(a) l  8, h  3 (b) l  4, h  6 (c) l  6, h  4 (d) l  12, h  2
Ans. : (b) z
Solution: m  Bhx h
l
 dm v
  Bvh  h
dt y

H.No. 40-D, Ground Floor, Jia Sarai, Near IIT, Hauz Khas, New Delhi-110016
Phone: 011-26865455/+91-9871145498
Website: www.physicsbyfiziks.com | Email: fiziks.physics@gmail.com
156
fiziks
Institute for NET/JRF, GATE, IIT‐JAM, M.Sc. Entrance, JEST, TIFR and GRE in Physics

Q75. Consider a sphere S1 of radius R which carries a uniform charge


S2
R
of density  . A smaller sphere S2 of radius a  is cut out and S1 P
2 
b 
removed from it. The centres of the two spheres are separated by r
 nRˆ
the vector b  , as shown in the figure. The electric field at a
2
point P inside S2 is

R R  R a 
(a) nˆ (b)  r  na
ˆ  (c) nˆ (d) r
3 0 3 0 a 6 0 3 0 R
Ans. : (c)
  
Solution: Electric field at P due to S1 is E1  r
3 0 S2
   S1 
Electric field at P due to S2 (assume   ) is E2  r r
3 0 
   b  P
        
Thus E  E1  E2   r  r  ;  b  r  r  r  r  b r
3 0
   R   R 
E b nˆ  b  nˆ 
3 0 6 0  2 
Q76. The value of the electric and magnetic fields in a particular reference frame (in Gaussian
units) are E  3xˆ  4 yˆ and B  3 zˆ respectively. An inertial observer moving with respect

to this frame measures the magnitude of the electric field to be E   4 . The magnitude of

the magnetic field B measured by him is

(a) 5 (b) 9 (c) 0 (d) 1


Ans. : (c)
Solution:  E 2  B 2  E 2  B2  constant   9  16   9  16  B2  B  0

Q77. A loop of radius a , carrying a current I , is placed in a uniform magnetic field B . If the
 
normal to the loop is denoted by n̂ , the force F and the torque T on the loop are
     
(a) F  0 and T   a 2 I n̂ B (b) F  0 I  B
4
        1 
(c) F  0 I  B and T  I nˆ  B (d) F  0 and T  IB
4  0 0

H.No. 40-D, Ground Floor, Jia Sarai, Near IIT, Hauz Khas, New Delhi-110016
Phone: 011-26865455/+91-9871145498
Website: www.physicsbyfiziks.com | Email: fiziks.physics@gmail.com
157
fiziks
Institute for NET/JRF, GATE, IIT‐JAM, M.Sc. Entrance, JEST, TIFR and GRE in Physics

Ans. : (a)

Solution: In uniform field F  0
   
Torque T  m  B   a 2 Inˆ  B
Q78. A waveguide has a square cross-section of side 2a . For the TM modes of wave vector k ,
the transverse electromagnetic modes are obtained in terms of a function   x, y  which

obeys the equation


 2 2   2 
 2    2  k 2    x, y   0
 x y  c
2


with the boundary condition    a, y     x,  a   0 . The frequency  of the lowest

mode is given by
 4 2   2 
(a)  2  c 2  k 2  2  (b)  2  c 2  k 2  2 
 a   a 

 2 2   2 2 
(c)   c  k  2 
2 2
(d)   c  k  2 
2 2

 2a   4a 

Ans. : (c)
Solution: c 2 k 2   2  mn
2
  2  c 2 k 2  mn
2

 m2 n2   1 1 
 mn
2
 c 2 2  2  2   112  c 2 2   
a b    2a   2a  
2 2

1 c 2 2 2 2 2 
 112  c 2 2     2
 c  k  
2a 2 2a 2  2a 2 

H.No. 40-D, Ground Floor, Jia Sarai, Near IIT, Hauz Khas, New Delhi-110016
Phone: 011-26865455/+91-9871145498
Website: www.physicsbyfiziks.com | Email: fiziks.physics@gmail.com
158
fiziks
Institute for NET/JRF, GATE, IIT‐JAM, M.Sc. Entrance, JEST, TIFR and GRE in Physics

NET/JRF -(DEC-2016)
Q79. A screen has two slits, each of width w with their centres at a distance 2 w apart. It is
illuminated by a monochromatic plane wave travelling along the x -axis.
The intensity of the interference pattern, measured on a distant screen, at an angle
n
 to the x -axis is
w
w
(a) zero for n  1, 2,3... 
w x
(b) maximum for n  1, 2,3...
1 3 5 w
(c) maximum for n  , , ...
2 2 2
(d) zero for n  0 only
Ans. : (a)
Solution: maximum for n  0 and zero for n  1, 2,3... .
Q80. The electric field of an electromagnetic wave is

E  z , t   E0 cos  kz  t  iˆ  2 E0 sin  kz  t  ˆj

where  and k are positive constants. This represents


(a) a linearly polarised wave travelling in the positive z -direction
(b) a circularly polarised wave travelling in the negative z -direction
(c) an elliptically polarised wave travelling in the negative z -direction
(d) an unpolarised wave travelling in the positive z -direction
Ans. : (c)
Solution: Amplitude along iˆ is E0 and along ĵ is 2 E0 . So resultant wave is elliptically
polarised

H.No. 40-D, Ground Floor, Jia Sarai, Near IIT, Hauz Khas, New Delhi-110016
Phone: 011-26865455/+91-9871145498
Website: www.physicsbyfiziks.com | Email: fiziks.physics@gmail.com
159
fiziks
Institute for NET/JRF, GATE, IIT‐JAM, M.Sc. Entrance, JEST, TIFR and GRE in Physics

Q81. A conducting circular disc of radius r and resistivity  rotates with an angular velocity
 in a magnetic field B perpendicular to it. A voltmeter is connected as shown in the
figure below. Assuming its internal resistance to be infinite, the reading on the voltmeter
(a) depends on  , B, r and 
B
(b) depends on  , B and r but not on 
(c) is zero because the flux through the loop is not
changing
r V
(d) is zero because a current the flows in the direction
of B
Ans. : (b)
Solution: Force experienced by charge is
  
 
F  q v  B and v  r

Q82. The charge per unit length of a circular wire of radius a in the xy -plane, with its centre at
the origin, is   0 cos  , where 0 is a constant and the angle  is measured from the

positive x -axis. The electric field at the centre of the circle is


 0  0
(a) E   iˆ (b) E  iˆ
4 0  4 0 
 0  0
(c) E   ˆj (d) E  kˆ
4 0  4 0 
Ans. : (a)
y
Solution: At centre O , direction of field is  x̂ . 0
So best option is (a)  0
o x
0

H.No. 40-D, Ground Floor, Jia Sarai, Near IIT, Hauz Khas, New Delhi-110016
Phone: 011-26865455/+91-9871145498
Website: www.physicsbyfiziks.com | Email: fiziks.physics@gmail.com
160
fiziks
Institute for NET/JRF, GATE, IIT‐JAM, M.Sc. Entrance, JEST, TIFR and GRE in Physics

Q83. A pair of parallel glass plates separated by a distance d is illuminated by white light as
shown in the figure below. Also shown in the graph of the intensity of the reflected light
I as a function of the wavelength  recorded by a spectrometer.
1
spectrometer
0.8

Intensity
partially incident 0.6
reflecting mirror white light 0.4
0.2
0
d air gap 490 500 510 520 530
glass plates    m
Assuming that the interference takes place only between light reflected by the bottom
surface of the top plate and the top surface of bottom plate, the distance d is closest to
(a) 12  m (b) 24  m (c) 60  m (d) 120  m
Ans. : (d)
 1
Solution: For constructive interference of reflected light, 2d cos   n    .
 2
 495 m
First maxima occurs at   495 m ,   00 and n  0 . Thus, d    120 m
4 4
Q84. Suppose that free charges are present in a material of dielectric constant  10 and
resistivity   1011   m . Using Ohm’s law and the equation of continuity for charge, the
1
time required for the charge density inside the material to decay by is closest to
e
(a) 106 S (b) 106 S (c) 1012 S (d) 10 S
Ans. : (d)
t
 8.8 1012  10
0r 1
Solution:  f  t    f  0  e ;   

,  10sec ,  
  10 11

Q85. A particle with charge q moves with a uniform angular velocity  in a circular orbit of
radius a in the xy - plane, around a fixed charge  q , which is at the centre of the orbit at

 0, 0, 0  . Let the intensity of radiation at the point  0, 0, R  be I1 and at  2 R, 0, 0  be ‘ I 2

I2
The ratio for R  a , is
I1
1 1
(a) 4 (b) (c) (d) 8
4 8
H.No. 40-D, Ground Floor, Jia Sarai, Near IIT, Hauz Khas, New Delhi-110016
Phone: 011-26865455/+91-9871145498
Website: www.physicsbyfiziks.com | Email: fiziks.physics@gmail.com
161
fiziks
Institute for NET/JRF, GATE, IIT‐JAM, M.Sc. Entrance, JEST, TIFR and GRE in Physics

Ans. : (c)
I 2 r13 R3 1
Solution:  3 
I1 r2  2 R  3
8

Q86. A parallel plate capacitor is formed by two circular conducting plates of radius a
separated by a distance d , where d  a . It is being slowly charged by a current that is
nearly constant. At an instant when the current is I , the magnetic induction between the
a
plates at a distance from the centre of the plate, is
2
0 I 0 I 0 I 0 I
(a) (b) (c) (d)
a 2 a a 4 a
Ans. : (d)
  Ir P
Solution: B  0 2 I a
2 a r
 I a
B  0 at r 
4 a 2
Q87. Two uniformly charged insulating solid spheres A and B , both of radius a , carry total
charges Q and Q , respectively. The spheres are placed touching each other as shown
in the figure.    
 A   B 
If the potential at the centre of the sphere A is VA and that at the
   
centre of B is VB then the difference VA  VB is

Q Q Q Q
(a) (b) (c) (d)
4 0 a 2 0 a 2 0 a 4 0 a
Ans. : (c)
3Q Q Q
Solution: VA   
8 0 a 4 0  2a  4 0 a

3Q Q Q
VB   
8 0 a 4 0  2a  4 0 a

Q
VA  VB 
2 0 a

H.No. 40-D, Ground Floor, Jia Sarai, Near IIT, Hauz Khas, New Delhi-110016
Phone: 011-26865455/+91-9871145498
Website: www.physicsbyfiziks.com | Email: fiziks.physics@gmail.com
162
fiziks
Institute for NET/JRF, GATE, IIT‐JAM, M.Sc. Entrance, JEST, TIFR and GRE in Physics

NET/JRF -(JUNE -2017)


Q88. Two long hollow co-axial conducting cylinders of radii R1 and R2  R1  R2  are placed

in vacuum as shown in the figure below.

2 R1 2 R2

The inner cylinder carries a charge   per unit length and the outer cylinder carries a
charge  per unit length. The electrostatic energy per unit length of this system is
2 2
(a)
 0
ln  R2 / R1  (b)
4 0
 R22 / R12 

2 2
(c) ln  R2 / R1  (d) ln  R2 / R1 
4 0 2 0
Ans. : (c)
  
Solution: r  R1 , E1  0 ; R1  r  R2 , E2  rˆ
2 0 r

r  Rz , E3  0

0  Rz 2
W  E 2 dz  0
2 all spce 2 R1 4 2 02 r 2
 2 rldr

W 0 2 R2 1 2 R 
l
 
2 2 02 R1 r
dr  ln  2 
4 0  R1 

Q89. A set of N concentric circular loops of wire, each carrying a steady current I in the
same direction, is arranged in a plane. The radius of the first loop is r1  a and the radius

of the n th loop is given by rn  nrn 1 . The magnitude B of the magnetic field at the centre

of the circles in the limit N   , is


(a) 0 I  e 2  1 / 4 a (b) 0 I  e  1 /  a

(c) 0 I  e 2  1 / 8a (d) 0 I  e  1 / 2a

Ans. : (d)

H.No. 40-D, Ground Floor, Jia Sarai, Near IIT, Hauz Khas, New Delhi-110016
Phone: 011-26865455/+91-9871145498
Website: www.physicsbyfiziks.com | Email: fiziks.physics@gmail.com
163
fiziks
Institute for NET/JRF, GATE, IIT‐JAM, M.Sc. Entrance, JEST, TIFR and GRE in Physics

0 I  11 1 1
Solution: B      ........ 
2  r1 r2 r3 rn 

r1  a

rn  nrn 1

r1  r0  a , r2  2r1  2a , r3  3r2  3.2a and r4  4r3  4.3.2a

0 I  1 1 1 
B 1     ....... 
2a  2 3.2 4.3.2 

0 I N
1
B  
2a  n 1 n 

xn 
1 
1 
1
e    e    1      e 1
x

n 0 n n 0 n n 1 n n 1 n

 N 1 I
lim     e  1  B  0  e  1
N 
 n l n  2a

Q90. An electromagnetic wave (of wavelength 0 in free space) travels through an absorbing

I
medium with dielectric permittivity given by    R  i I where  3 . If the skin
R
0
depth is , the ratio of the amplitude of electric field E to that of the magnetic field B ,
4
in the medium (in ohms) is
(a) 120  (b) 377 (c) 30 2  (d) 30 
Ans. : (d)
1 0 I 
Solution: d   ,  3
 4 R 
1/ 2
       4 2 4
2

   1           
  
1
2    2 0  0
 
1/ 2
   
2

K  k       1  
2 2

     
 

H.No. 40-D, Ground Floor, Jia Sarai, Near IIT, Hauz Khas, New Delhi-110016
Phone: 011-26865455/+91-9871145498
Website: www.physicsbyfiziks.com | Email: fiziks.physics@gmail.com
164
fiziks
Institute for NET/JRF, GATE, IIT‐JAM, M.Sc. Entrance, JEST, TIFR and GRE in Physics

E0   1 1
   
B0 K 2  4
1/ 2
   
2
2
2


   1      0
    
 
0 0  2 c / 0 c E c
     0
8 8 4 H0 4

E 4  107  3 108
   30
H0 4

Q91. The vector potential A  ke  at rrˆ (where a and k are constants) corresponding to an

electromagnetic field is changed to A   ke  at rrˆ . This will be a gauge transformation if
the corresponding change     in the scalar potential is

(a) akr 2 e  at (b) 2akr 2 e  at (c)  akr 2 e  at (d) 2akr 2 e  at


Ans. : (c)
Solution: Gauge Transformation
       
A  A   ,       A'  A  2ke at rrˆ    rˆ
t r

   ke at r 2   kae  at r 2
t

  '     kae  at r 2
t
Q92. An electron is decelerated at a constant rate starting from an initial velocity u (where
u  c ) to u / 2 during which it travels a distance s . The amount of energy lost to
radiation is
0 e 2u 2 0 e 2u 2 0 e 2u 0 e 2u
(a) (b) (c) (d)
3 mc 2 s 6 mc 2 s 8 mcs 16 mcs
Ans. : (d)
0 q 2 a 2
Solution: Total power radiated P 
6 c
0 e 2 a 2  e2 a 2 u
Total energy radiated in time t is E  P  t  t  0 
6 c 6 c 2a

H.No. 40-D, Ground Floor, Jia Sarai, Near IIT, Hauz Khas, New Delhi-110016
Phone: 011-26865455/+91-9871145498
Website: www.physicsbyfiziks.com | Email: fiziks.physics@gmail.com
165
fiziks
Institute for NET/JRF, GATE, IIT‐JAM, M.Sc. Entrance, JEST, TIFR and GRE in Physics

 u u 
 v  u  at  2  u  at  t  2a 

0 e2 au
E
12 c
E 2E
Fraction of initial K .E. lost due to radiation  
1
mu 2 mu 2
2
2 0e 2 au 0 e2 a
  
mu 2 12 c 6 mcu
 1 2 u 1 u2 u 2 u 2 3u 2 3u 2 
 s  ut  at  u   a      a 
 2 2a 2 4 a 2 2 a 8a 8a 8s 

0 e2 3u 2  e 2u
   0
6 mcu 8s 16 mcs
Q93. The figure describes the arrangement of slits and S1 S2
screens in a Young’s double slit experiment. The

screen
width of the slit in S1 is a and the slits in S2 are  b

of negligible width.
If the wavelength of the light is  , the value of d
d for which the screen would be dark is
2 2 2
a b a ab ab
(a) b    1 (b)   1 (c)   (d)
 2  2  
Ans. : (d)

Solution: If the path difference Op2  Op1 
2
The minima of the interference pattern produced by O will P1
fall on the maxima produced by O Now
1/ 2 a / 2 O b
 b a 
2
1 b a
2

OP2   d 2      d    a/2 O


  2 2   2d  2 2 
1/ 2 P2
 2  b a 2  1 b a
2

OP1   d      d   
  2 2   2d  2 2 
d
H.No. 40-D, Ground Floor, Jia Sarai, Near IIT, Hauz Khas, New Delhi-110016
Phone: 011-26865455/+91-9871145498
Website: www.physicsbyfiziks.com | Email: fiziks.physics@gmail.com
166
fiziks
Institute for NET/JRF, GATE, IIT‐JAM, M.Sc. Entrance, JEST, TIFR and GRE in Physics

ab
 OP2  OP1   d  b, a 
2d
 ab ab
Thus  d 
2 2d 
Q94. A constant current I is flowing in a piece of wire that is bent into a loop as shown in the
figure. y
2b

2b a a
2b
2a
2a O x
b b
The magnitude of the magnetic field at the point O is
0 I a 0 I  1 1  0 I  1  0 I  1 
(a) ln   (b)    (c)   (d)  
4 5  b  4 5  a b  4 5  a  4 5  b 
Ans. : (b)
 I
Solution: B  0  sin  2  sin 1  ˆ
4 d
Magnetic field due to left and right segment of 2a
d 1 2
 I  2a 
B2 a  0  
4 a  5a  I
Field due to upper segment of 2a
0 I  a a 
   
4  2a   5a 5a 
0 I 2 0 I 1
Net field B2 a  2    
4 a 5 4 a 5
0 I
B2 a  5  (inward)
4 a
0 I
similarly, B2b  5  (outward)
4 b
0 I 1 1
Net field B  B2 a  B2b  5  
4 a b

H.No. 40-D, Ground Floor, Jia Sarai, Near IIT, Hauz Khas, New Delhi-110016
Phone: 011-26865455/+91-9871145498
Website: www.physicsbyfiziks.com | Email: fiziks.physics@gmail.com
167
fiziks
Institute for NET/JRF, GATE, IIT‐JAM, M.Sc. Entrance, JEST, TIFR and GRE in Physics

Q95. The charge distribution inside a material of conductivity  and permittivity  at initial
time t  0 is   r , 0   0 , a constant. At subsequent times   r , t  is given by

 t  1    t 
(a) 0 exp    (b) 0 1  exp   
  2    
0 t
(c) (d) 0 cosh
   t  
1  exp    
  
Ans. : (a)
       
Solution: J f   E , .E  f , .J f   f
 E
    
 .E   f  f    f
t t 
     
  f  t   0 exp   f    f  t   0 exp  t
     

NET/JRF (DEC - 2017)


Q96. Two point charges 3Q and Q are placed at  0, 0, d  and  0, 0, 2d  respectively,

above an infinite grounded conducting sheet kept in the xy - plane. At a point  0, 0, z  ,

where z  d , the electrostatic potential of this charge configuration would


approximately be
1 d2 1 2d 1 3d 1 d2
(a) Q (b) Q (c) Q (d)  Q
4 0 z 3 4 0 z 2 4 0 z 2 4 0 z 3
Ans. : (b)
Solution: Monopole moment Qmono  Q  3Q  3Q  Q  0

Dipole moment p  3Q   dzˆ    Q    2dzˆ    3Q    dzˆ   Q   2dzˆ 
 z
p  2Qdzˆ 2d Q

1 p  rˆ 1 2Qd d 3Q
Vdip  
4 0 r 2
4 0 z 2

y
d  3Q
x 2 d Q
H.No. 40-D, Ground Floor, Jia Sarai, Near IIT, Hauz Khas, New Delhi-110016
Phone: 011-26865455/+91-9871145498
Website: www.physicsbyfiziks.com | Email: fiziks.physics@gmail.com
168
fiziks
Institute for NET/JRF, GATE, IIT‐JAM, M.Sc. Entrance, JEST, TIFR and GRE in Physics

Q97. A rectangular piece of dielectric material is inserted partially into the (air) gap between
the plates of a parallel plate capacitor. The dielectric piece will
(a) remain stationary where it is placed
(b) be pushed out from the gap between the plates
(c) be drawn inside the gap between the plates and its velocity does not change sign
(d) execute an oscillatory motion in the region between the plates
Ans. : (c)
Solution: Just like a conductor, a dielectric is attracted into an electric field. The reason is: the
bound charge tends to accumulate near the fee charge of the opposite sign.
Q98. An electromagnetic wave is travelling in free space (of permittivity  0 ) with electric field

ˆ cos q  x  ct 
E  kE 0

The average power (per unit area) crossing planes parallel to 4 x  3 y  0 will be
4 1 16
(a)  0 cE02 (b)  0 cE02 (c)  0 cE02 (d)  0 cE02
5 2 25
Ans. : (c)
x y
Solution: 4 x  3 y  0   0
3 4 y

 E
B   0 cos  qx  qct  yˆ 4
c

 1   1  E0   E02 B 90  
S
0

EB  E
0  0 c
cos 2
 

x
ˆ  S 
2 0 c

 x

 E2 E2 K 3
2
I  S .nˆ  0 cos  90     0 sin   c 0 E02 
2 0 c 2 0 c 5 E
z
4 4
 tan    sin  
3 5
1
I  0.4c 0 E02  c 0 E02
2

H.No. 40-D, Ground Floor, Jia Sarai, Near IIT, Hauz Khas, New Delhi-110016
Phone: 011-26865455/+91-9871145498
Website: www.physicsbyfiziks.com | Email: fiziks.physics@gmail.com
169
fiziks
Institute for NET/JRF, GATE, IIT‐JAM, M.Sc. Entrance, JEST, TIFR and GRE in Physics

Q99. A plane electromagnetic wave from within a dielectric medium (with   4 0 and   0 )

is incident on its boundary with air, at z  0 . The magnetic field in the medium is

 
H  ˆjH 0 cos t  kx  k 3 z , where  and k are positive constants.

The angles of reflection and refraction are, respectively,


(a) 450 and 600 (b) 300 and 900 (c) 300 and 600 (d) 600 and 900
Ans. : (b) z n  r
Solution: n  r
 y
I
k  kxˆ  k 3 zˆ
sin  I n2 1 Dielectric n1  4  2
  x
sin T n1 2 n2  1
Air T
kx 1
sin T  2sin  I  tan  I 
   I  300
kz 3
 sin T  2  sin 30  1  T  90
0 0

Q100. In an inertial frame S , the magnetic vector potential in a region of space is given by

A  az iˆ (where a is a constant) and the scalar potential is zero. The electric and

magnetic fields seen by an inertial observer moving with a velocity viˆ with respect to S ,
1
are, respectively [In the following   ]
v2
1 2
c

(a) 0 and  ajˆ (b) vakˆ and  aiˆ (c) v akˆ and v ajˆ (d) v akˆ and  ajˆ
Ans. : (d)
Solution: Ex  Ex , E y    E y  v Bz  and Ez    Ez  vBy 

 v   v 
Bx  Bx , By    By  2 Ez  and Bz    Bz  2 E y 
 c   c 

  A   
E   V    0, B    A  a ˆj
t
Ex  0, E y    0  v  0   0, Ez    0  va    va

(replace v by v )  E  v azˆ

H.No. 40-D, Ground Floor, Jia Sarai, Near IIT, Hauz Khas, New Delhi-110016
Phone: 011-26865455/+91-9871145498
Website: www.physicsbyfiziks.com | Email: fiziks.physics@gmail.com
170
fiziks
Institute for NET/JRF, GATE, IIT‐JAM, M.Sc. Entrance, JEST, TIFR and GRE in Physics

 v   v 
Bx  0, By    a  2  0    a, Bz    0  2  0   0
 c   c 

 B   a ˆj
Q101. In the rest frame S1 of a point particle with electric charge q1 another point particle with

electric charge q2 moves with a speed v parallel to the x -axis at a perpendicular

distance l . The magnitude of the electromagnetic force felt by q1 due to q2 when the

1
distance between them is minimum, is [In the following   ]
v2
1 2
c
1 q1q2 1  q1q2
(a) (b)
4 0  l 2 4 0 l 2

1  q1q2  v 2  1 q1q2  v 2 
(c) 1   (d) 1  
4 0 l 2  c 2  4 0  l 2  c 2 

Ans. : (b)
1  q1q2
Solution: Charge of q2 seen by rest frame of q1   q2 ; F
4 0 l 2
Q102. A circular current carrying loop of radius a carries a steady current. A constant electric
 
charge is kept at the centre of the loop. The electric and magnetic fields, E and B
respectively, at a distance d vertically above the centre of the loop satisfy
        
(a) E  B (b) E  0 
(c)  E  B  0  
(d)   E  B  0 
Ans. : (c)
    

Solution: E  B  0  . E  B  0 
1 1 1 
Q103. The Hamiltonian of a two-level quantum system is H     possible initial
2 1 1
state in which the probability of the system being in that quantum state does not change
with time, is

H.No. 40-D, Ground Floor, Jia Sarai, Near IIT, Hauz Khas, New Delhi-110016
Phone: 011-26865455/+91-9871145498
Website: www.physicsbyfiziks.com | Email: fiziks.physics@gmail.com
171
fiziks
Institute for NET/JRF, GATE, IIT‐JAM, M.Sc. Entrance, JEST, TIFR and GRE in Physics

       
 cos 4   cos 8   cos 2   cos 6 
(a)   (b)   (c)   (d)  
 sin    sin    sin    sin  
       
 4  8   2  6
Ans. : (b)

NET/JRF -(JUNE -2018)


Q104. Two point charges 2Q and Q are kept at point with Cartesian coordinates 1, 0, 0  ,

respectively, in front of an infinite grounded conducting plate at x  0 . The potential at


 x, 0, 0  for x  1 depends on x as

(a) x 3 (b) x 5 (c) x 2 (d) x 4


Ans. : (a)
Solution: 2 1
x
Q 2Q 0 2Q Q

Monopole moment 2Q  Q  2Q  Q  0
 
Dipole moment p  Q  2 xˆ   2Q  xˆ   2Q   xˆ   Q  2 xˆ   p  0

1
Thus V 
x3
Q105. The following configuration of three identical narrow slits are illuminated by
monochromatic light of wavelength  (as shown in the figure below). The intensity is
measured at an angle  (where  is the angle with the incident beam) at a large distance
2 d
from the slits. If   sin  , the intensity is proportional to

1 d
(a) 2 cos   2 cos 2 (b) 3  sin 2 3
2

1
(c) 3  2 cos   2 cos 2  2 cos 3 (d) 2  sin 2 3 2d
 2

Ans. : (c)
  i t     
Solution: E1  Ae   , E2  A ei ei t , E3  A ei1 ei t  Ae3i ei t

2 2
  d sin  , 1   3d sin    3
 

H.No. 40-D, Ground Floor, Jia Sarai, Near IIT, Hauz Khas, New Delhi-110016
Phone: 011-26865455/+91-9871145498
Website: www.physicsbyfiziks.com | Email: fiziks.physics@gmail.com
172
fiziks
Institute for NET/JRF, GATE, IIT‐JAM, M.Sc. Entrance, JEST, TIFR and GRE in Physics
    
E  E1  E2  E3  A 1  ei  e3i  ei t
 
E   A 1  e i  e 3i  e it
 
I  E.E   A2 1  ei  e3i  1  e i  e3i 

 ei  e  i e i 2  e  i 2 ei 3  e  i 3 
I  A2  3  2 2 2 
 2 2 2 
I  A2 3  2 cos   2 cos 2  2 cos 3 
 
Q106. The electric field E and the magnetic field B corresponding to the scalar and vector
 1
potentials, V  x, y, z , t   0 and A  x, y, z , t   kˆ0 A0  ct  x  , where A0 is a constant,
2
are
  1  1  1
(a) E  0 and B  ˆj 0 A0 (b) E   kˆ0 A0 c and B  ˆj 0 A0
2 2 2
  1  1  1
(c) E  0 and B   iˆ0 A0 (d) E  kˆ0 A0 c and B   iˆ0 A0
2 2 2
Ans. : (b)

 A 1  1
Solution: E     0 A0  c  0   kˆ   0 A0 ckˆ
t 2  2

xˆ yˆ zˆ
      A A  1
B   A   xˆ z  yˆ z  B  0 A0 ˆj
x y z y x 2
0 0 Az

Q107. The electric field of a plane wave in a conducting medium is given by



ˆ 0 e z / 3a cos  z  t  ,
E  z , t   iE  
 3a 
where  is the angular frequency and a  0 is a constant. The phase difference between
 
the magnetic field B and the electric field E is
   
(a) 300 and B lags behind B (b) 300 and B lags behind E
   
(c) 600 and E lags behind B (d) 600 and B lags behind E
Ans. : (b)

H.No. 40-D, Ground Floor, Jia Sarai, Near IIT, Hauz Khas, New Delhi-110016
Phone: 011-26865455/+91-9871145498
Website: www.physicsbyfiziks.com | Email: fiziks.physics@gmail.com
173
fiziks
Institute for NET/JRF, GATE, IIT‐JAM, M.Sc. Entrance, JEST, TIFR and GRE in Physics
 
Solution: E  z , t   iE
ˆ 0 e z cos  kz  t   E  and B  z , t   ˆjB0 e z cos  kz  t   E   

 
where   tan 1   .
k

ˆ 0 e  z / 3a cos  z  t     1 and k  1
 E  z , t   iE  
 3a  3a 3a

 1 
   tan 1    30
0

 3
Q108. A hollow waveguide supports transverse electric TE  modes with the dispersion

1
relation k   2  mn
2
, where mn is the mode frequency. The speed of flow of
c
electromagnetic energy at the mode frequency is
(a) c (b) mn / k (c) 0 (d) 
Ans. : (c)
Solution: Energy carried by the wave travels at the group velocity

d  
2

vg   c 1   mn  at   mn , vg  0
dk   
Q109. The loop shown in the figure below carries a steady current I .
I

3a
a
O
The magnitude of the magnetic field at the point O is
0 I 0 I 0 I 0 I
(a) (b) (c) (d)
2a 6a 4a 3a
Ans. : (b)
1 0 I 1 0 I
Solution: Ba  , B3a  
2 2a 2 2  3a 

0 I  1  0 I
B  Ba  B3a  1   
4a  3  6a

H.No. 40-D, Ground Floor, Jia Sarai, Near IIT, Hauz Khas, New Delhi-110016
Phone: 011-26865455/+91-9871145498
Website: www.physicsbyfiziks.com | Email: fiziks.physics@gmail.com
174
fiziks
Institute for NET/JRF, GATE, IIT‐JAM, M.Sc. Entrance, JEST, TIFR and GRE in Physics

Q110. In the region far from a source, the time dependent electric field at a point  r ,  ,   is

  sin     r 
E  r ,  ,    ˆ E0 2   cos   t   
 r    c 
where  is angular frequency of the source. The total power radiated (averaged over a
cycle) is
2 E02 4 4 E02 4 4 E02 4 2 E02 4
(a) (b) (c) (d)
3 0 c 3 0 c 3 0 c 3 0 c
Ans. : (b)
E
Solution: B 
c
 1 E 2 E02 4 Sm2   r 
S  E.B   cos 2   t   
0 0 c 0 c r 2
  c 
 1 E02 4 sin 2 
S 
2 0 c r 2
  E 2 4  2
sin 2  2
P   S .da  0  r sin  d d
S
2 0 c 0 0
r2

E02 4 4 4 E02 4
P   2 
2 0 c 3 3 0 c

NET/JRF (DEC - 2018)

Q111. Two current-carrying circular loops, each of radius R , are placed z


perpendicular to each other, as shown in the figure.
2I 0
The loop in the xy - plane carries a current I 0 while that in the
 y
xz -plane carries a current 2I 0 . The resulting magnetic field B
I0
at the origin is x
0l0  ˆ ˆ  0l0  ˆ ˆ 
(a) 2jk (b) 2jk
2R   2R  

0l0  ˆ ˆ  0l0  ˆ ˆ 
(c) 2 j  k (d) 2 j  k
2R   2R  
Ans. : (c)
H.No. 40-D, Ground Floor, Jia Sarai, Near IIT, Hauz Khas, New Delhi-110016
Phone: 011-26865455/+91-9871145498
Website: www.physicsbyfiziks.com | Email: fiziks.physics@gmail.com
175
fiziks
Institute for NET/JRF, GATE, IIT‐JAM, M.Sc. Entrance, JEST, TIFR and GRE in Physics

 I
Solution: Field due to loop in xy plane is B1  0 0 zˆ
2R
Field due to loop in xz plane is
   2I0 
B2  0   yˆ 
2R
   I
Resultant field B  B1  B2  0 0  2 yˆ  zˆ 
2R

Q112. An electric dipole of dipole moment P  qbiˆ is placed at
Y
origin in the vicinity of two charges  q and  q at  L, b 

and  L, b  , respectively, as shown in the figure.


 q ( L, b )
L 
The electrostatic potential at the point  , 0  is 
2  P A
x
O ( L /2, 0)
qb  1 2  4qbL
(a)  2 2 2 
(b)
 0  L L  4b   0  L2  4b 2 
3/ 2

 q ( L,-b)
qb 3qb
(c) (d)
 0 L2  0 L2
Ans. : (c)
1 p cos 00 1 p
Solution: Potential due to dipole V1  
4 0  L / 2  2
 0 L2

1 q
Potential due to  q charge V2 
4 0 L2 / 4  b 2
1 q
Potential due to  q charge V3  
4 0 L / 4  b2
2

1 p 1 qb
Resultant V  V1  V2  V3  V 
 0 L2
 0 L2
Hence, correct option is (c)
Q113. A monochromatic and linearly polarized light is used in a Young’s double slit experiment.
A linear polarizer, whose pass axis is at an angle 450 to the polarization of the incident
wave, is placed in front of one of the slits. If I max and I min , respectively, denote the

H.No. 40-D, Ground Floor, Jia Sarai, Near IIT, Hauz Khas, New Delhi-110016
Phone: 011-26865455/+91-9871145498
Website: www.physicsbyfiziks.com | Email: fiziks.physics@gmail.com
176
fiziks
Institute for NET/JRF, GATE, IIT‐JAM, M.Sc. Entrance, JEST, TIFR and GRE in Physics

maximum and minimum intensities of the interference pattern on the screen, the visibility,
I max  I min
defined as the ratio , is
I max  I min

2 2 2 2 2
(a) (b) (c) (d)
3 3 3 3
Ans. : (b)
  A  xˆ  yˆ  it i
ˆ 0 eit ;
Solution: E1  xA E2  0 e
2 2
    y
 
I  E1  E2  E1*  E2*  E2 A0
A0 cos 45 
 2  2     2
 I  E1  E2  E1  E2*  E2  E1* x
E1 A0
A02 A2 A2
 A02   1  1  0 e i  0 ei
4 2 2
A02 A02 ei  e  i 3 A02
 I  A02     A02 cos 
2 2 2 2
5 A02 A2 I I 2
I max  , I min  0  V  max min 
2 2 I max  I min 3
Q114. An electromagnetic wave propagates in a nonmagnetic medium with relative permittivity
  4 . The magnetic field for this wave is

H  x, y   kH 
ˆ cos t   x   3 y
0 

where H 0 is a constant. The corresponding electric field E  x, y  is

(a)
1
4
  
0 H 0 c  3iˆ  ˆj cos t   x   3 y 
(b)
1
4
0 H 0 c   
3iˆ  ˆj cos t   x   3 y 
(c)
1
4
0 H 0 c   
3iˆ  ˆj cos t   x   3 y 
(d)
1
4
  
0 H 0 c  3iˆ  ˆj cos t   x   3 y 
Ans. : (a)

H.No. 40-D, Ground Floor, Jia Sarai, Near IIT, Hauz Khas, New Delhi-110016
Phone: 011-26865455/+91-9871145498
Website: www.physicsbyfiziks.com | Email: fiziks.physics@gmail.com
177
fiziks
Institute for NET/JRF, GATE, IIT‐JAM, M.Sc. Entrance, JEST, TIFR and GRE in Physics


Solution: E  v Kˆ  Bˆ 

 K  xˆ   3 yˆ 1 3
K   xˆ   3 yˆ  K   
ˆ  xˆ  yˆ
K  2  3 2 2 2

c  xˆ  3 yˆ 
E 
r  2

 0 H 0 cos t   x   3 y zˆ  

c 0 H 0 
E
2 4    
 yˆ  3 xˆ cos t   x  3 y 
 
E
1
4
  
c0 H 0  3 xˆ  yˆ cos t   x   3 y 
 
Q115. In an inertial frame uniform electric and magnetic field E and B are perpendicular to
2 2
each other and satisfy E  B  29 (in suitable units). In another inertial frame, which

moves at a constant velocity with respect to the first frame, the magnetic field is 2 5kˆ .
In the second frame, an electric field consistent with the previous observations is

(a)
2

7 ˆ ˆ
ij  
(b) 7 iˆ  kˆ  (c) 
7 ˆ ˆ
2
i k  
(d) 7 iˆ  ˆj 
Ans. : (a)
2 2
Solution: E  B  29
 2  2
In another Frame E   B  29
  2
B  2 5kˆ  B  4  5  20  E   49
2

  
It is given E  B so E  
7 ˆ ˆ
2
ij  
Q116. Electromagnetic wave of angular frequency  is propagating in a medium in which, over
2
 
a band of frequencies the refractive index is n    1    , where 0 is a constant.
 0 
v 
The ratio g of the group velocity to the phase velocity at   0 is
vp 2
1 2
(a) 3 (b) (c) (d) 2
4 3

H.No. 40-D, Ground Floor, Jia Sarai, Near IIT, Hauz Khas, New Delhi-110016
Phone: 011-26865455/+91-9871145498
Website: www.physicsbyfiziks.com | Email: fiziks.physics@gmail.com
178
fiziks
Institute for NET/JRF, GATE, IIT‐JAM, M.Sc. Entrance, JEST, TIFR and GRE in Physics

Ans. : (a)
2
Solution: n  1 
02

c 2 / 4 3 4c
n  1  0 2   vp 
vp 0 4 3

ck 2 3
n  1  kc   
 02 02

dk 3 2 2 / 4 1 3 1 d
 .c  1  2  1  3 0 2    vg   4c
d 0 0 4 4 dk

vg 4c
Thus,  3
vp 4c / 3

Q117. A rotating spherical shell of uniform surface charge and mass density has total mass M
and charge Q . If its angular momentum is L and magnetic moment is  , then the ratio

is
L
Q 2Q Q 3Q
(a) (b) (c) (d)
3M 3M 2M 4M
Ans. : (c)
2 2
Solution: I  MR 2 , L  I   MR 2
3 3 z
 R sin 
Q

4 R 2 Rd

   2 R sin   Rd    R 2 sin 2 


dm  dI    R sin   
2

2 / 
d
dm   R sin  d 4 3
R
4 4    4 QR  2
   dm   R 4    2 
R   
3 3  4 R  3

 QR 2 / 3 QR 2 3 Q
   
L 2
MR 2 3 2MR  2M
2

H.No. 40-D, Ground Floor, Jia Sarai, Near IIT, Hauz Khas, New Delhi-110016
Phone: 011-26865455/+91-9871145498
Website: www.physicsbyfiziks.com | Email: fiziks.physics@gmail.com
179
fiziks
Institute for NET/JRF, GATE, IIT‐JAM, M.Sc. Entrance, JEST, TIFR and GRE in Physics

QUANTUM MECHANICS SOLUTIONS


NET/JRF (JUNE-2011)
 1 
Q1. The wavefunction of a particle is given by    0  i1  where 0 and 1 are the
 2 
normalized eigenfunctions with energies E0 and E1 corresponding to the ground state

and first excited state, respectively. The expectation value of the Hamiltonian in the state
 is
E0 E0 E0  2 E1 E0  2 E1
(a)  E1 (b)  E1 (c) (d)
2 2 3 3
Ans. : (d)
1  H E 0  2 E1
Solution:   0  i1 and H  
2  3

Q2. The energy levels of the non-relativistic electron in a hydrogen atom (i.e. in a Coulomb
potential V r   1 / r ) are given by E nlm  1 / n 2 , where n is the principal quantum

number, and the corresponding wave functions are given by  nlm , where l is the orbital

angular momentum quantum number and m is the magnetic quantum number. The spin
of the electron is not considered. Which of the following is a correct statement?
(a) There are exactly  2l  1 different wave functions nlm , for each Enlm .

(b) There are l  l  1 different wave functions  nlm , for each Enlm .

(c) Enlm does not depend on l and m for the Coulomb potential.

(d) There is a unique wave function  nlm for each Enlm .


Ans. : (c)
  
Q3. The Hamiltonian of an electron in a constant magnetic field B is given by H    B .

where  is a positive constant and    1 ,  2 ,  3  denotes the Pauli matrices. Let

  B /  and I be the 2  2 unit matrix. Then the operator e i H t /  simplifies to


   
 t i  B  t i  B
(a) I cos  sin (b) I cos  t  sin  t
2 B 2 B
   
i  B i  B
(c) I sin  t  cos  t (d) I sin 2 t  cos 2 t
B B
H.No. 40-D, Ground Floor, Jia Sarai, Near IIT, Hauz Khas, New Delhi-110016
Phone: 011-26865455/+91-9871145498
Website: www.physicsbyfiziks.com | Email: fiziks.physics@gmail.com
180
fiziks
Institute for NET/JRF, GATE, IIT‐JAM, M.Sc. Entrance, JEST, TIFR and GRE in Physics

Ans. : (b)
  
Solution: H   B where    1 ,  2 ,  3  are pauli spin matrices and B are constant magnetic
 
 


 
field.    1iˆ,  2 ˆj ,  3 kˆ , B  Bx iˆ  By ˆj  Bz kˆ and Hamiltonion H    B in matrices

form is given by
 Bz Bx  iBy 
H   .
 Bx  iBy  Bz 

Eigenvalue of given matrices are given by   B and   B . H matrices are not diagonals

so e i H t /  is equivalent to
 iBt 
1  e 0 
S   i  Bt 
S
 0  
 e 
where S is unitary matrices
 1 1 
 2 2 
and S 1  S   .
 1

1 
 
2 2
 1 1  i Bt  1 1 
 iBt   2  
e 0 
 2   e  0 
 2 2 
S 1   i  Bt 
S  , where   B /  .
 0   1 1    i  Bt 
  1 1 
 e      0 e    
 2 2   2 2

 cos  t i sin  t 
eiHt /    , which is equivalent to I cos  t  i x sin  t can be written
 i sin  t cos  t 
  
i  B i  B
as I cos  t  sin  t , where  x 
B B
Q4. If the perturbation H   ax , where a is a constant, is added to the infinite square well
potential
0 for 0  x  
V  x  
 otherwise.
The correction to the ground state energy, to first order in a , is
a a a
(a) (b) a (c) (d)
2 4 2
H.No. 40-D, Ground Floor, Jia Sarai, Near IIT, Hauz Khas, New Delhi-110016
Phone: 011-26865455/+91-9871145498
Website: www.physicsbyfiziks.com | Email: fiziks.physics@gmail.com
181
fiziks
Institute for NET/JRF, GATE, IIT‐JAM, M.Sc. Entrance, JEST, TIFR and GRE in Physics

Ans. : (a)
 
a2 x a 2 x
Solution: E   H ' 0 dx 
1 *
0  x sin
2
dx   0  sin .
   
0
0 0
2

Q5. A particle in one dimension moves under the influence of a potential V  x   ax 6 , where
a is a real constant. For large n the quantized energy level En depends on n as:

(a) En ~ n3 (b) En ~ n 4 / 3 (c) En ~ n6 / 5 (d) En ~ n3 / 2


Ans. : (d)
p x2 p x2
 ax 6 and p x  2mE  ax 6  2 .  
1
Solution: V  x   ax , H  6
 ax , E 
6

2m 2m
According to W.K.B approximation pdx  nh

  2m  E  ax 
1/ 2
6
dx  n

We can find this integration without solving the integration


Px
1 2mE
1/ 6
E
2 2 6 1/ 6
p p x 6
E  ax 6   1  x    at p x  0 .
x x
 E / a  E / a 
2m 2mE E / a a x
Area of Ellipse =  (semi major axis  semiminor axis)
 2mE
1
E
3
6
  2mE     n  E  n 2 .
a
1   
Q6. (A) In a system consisting of two spin particles labeled 1 and 2, let S 1   1 and
2 2

S 2    2  denote the corresponding spin operators. Here    x ,  y ,  z  and
  
2
 x ,  y ,  z are the three Pauli matrices.

In the standard basis the matrices for the operators S x1S y2  and S y1S x2  are respectively,

 2 1 0   2   1 0  2  i 0  2   i 0 
(a)  ,   (b)  ,  
4  0  1 4  0 1  4 0  i 4  0 i 
0 1 0 0  0 i 0 0
0 0 0  i 2 0 0 0  i    
2  0 0 i 0 ,   0 0 i 0  2 1 0 0 0  2 i 0 0 0
(c)  0  4 0 0 0
(d) ,
4 0  i 0 i 4 0 0 0  i 4 0 0 0 1
i 0 0 0  i
 0 0 0  
0
  
 0 i 0  0
 0 1 0 

H.No. 40-D, Ground Floor, Jia Sarai, Near IIT, Hauz Khas, New Delhi-110016
Phone: 011-26865455/+91-9871145498
Website: www.physicsbyfiziks.com | Email: fiziks.physics@gmail.com
182
fiziks
Institute for NET/JRF, GATE, IIT‐JAM, M.Sc. Entrance, JEST, TIFR and GRE in Physics

Ans. : (c)
0 0 0 i 
 
2  0 1  0 i  2  0 0 i 0 
Solution: Sx1Sy2     
4 1 0 i 0  4 0 i 0 0 
 i 
0 
 0 0

0 0 0 i 
i   0  
2
0 1  02
0 i 0 
S y1 S x 2   i  
4 0  1 0 4 0 i 0 0 
 
i 0 0 0 
(B) These two operators satisfy the relation
 
(a) S x1S y2  , S y1S x2   S z1S z2   
(b) S x1S y2  , S y1S x2   0

(c) S  S   , S  S     iS  S  
x
1
y
2
y
1
x
2
z
1
z
2
(d) S  S   , S  S     0
x
1
y
2
y
1
x
2

Ans. : (d)
Solution: We have matrix Sx1Sy2 and Sy1Sx2 from question 6(A) so commutation is given by

S   S   , S   S     0 .
x
1
y
2
y
1
x
2

NET/JRF (DEC-2011)

Q7. The energy of the first excited quantum state of a particle in the two-dimensional

potential V  x, y  
1
2
 
m 2 x 2  4 y 2 is

3 5
(a) 2 (b) 3 (c)  (d) 
2 2
Ans. : (d)
 1  1
Solution: V  x, y   m 2 x 2  4 y 2   m 2 x 2  m 4 2 y 2 , E   n x     n y  2
1 1 1
2 2 2  2  2
 1 3
For ground state energy n x  0, n y  0  E   2 
2 2 2
3 5
First exited state energy n x  1, n y  0    
2 2

H.No. 40-D, Ground Floor, Jia Sarai, Near IIT, Hauz Khas, New Delhi-110016
Phone: 011-26865455/+91-9871145498
Website: www.physicsbyfiziks.com | Email: fiziks.physics@gmail.com
183
fiziks
Institute for NET/JRF, GATE, IIT‐JAM, M.Sc. Entrance, JEST, TIFR and GRE in Physics

Q8. Consider a particle in a one dimensional potential that satisfies V  x   V  x  . Let  0

and 1 denote the ground and the first excited states, respectively, and let

   0  0   1  1 be a normalized state with  0 and  1 being real constants. The

expectation value x of the position operator x in the state  is given by

(a)  02  0 x  0   12  1 x  1 (b)  0 1   0 x  1   1 x  0 
(c)  02   12 (d) 2 0 1
Ans. : (b)
Solution: Since V  x   V  x  so potential is symmetric.

 0 x 0  0 ,  1 x 1  0

 x    0  0   1  1    0  0   1  1    01   0 x  1   1 x  0 

Q9. The perturbation H '  bx 4 , where b is a constant, is added to the one dimensional

harmonic oscillator potential V  x  


1
m 2 x 2 . Which of the following denotes the
2
correction to the ground state energy to first order in b ?
[Hint: The normalized ground state wave function of the one dimensional harmonic

 m 
1/ 4

is  0   e  m x
2
/ 2
oscillator potential  . You may use the following
  

 1
1
n
integral  x e 2n  ax 2
dx  a 2
 n   ].
  2

3b 2 3b 2 3b 2 15b 2


(a) (b) (c) (d)
4m 2  2 2m 2  2 2 m 2 2 4m 2  2
Ans. : (a)

Solution: H '  bx 4 , V  x  
1
m 2 x 2 .
2
m x 2
 m 
1/ 4

Correction in ground state is given by E   0 H ' 0 1
where 0    e 2
.
  
0

H.No. 40-D, Ground Floor, Jia Sarai, Near IIT, Hauz Khas, New Delhi-110016
Phone: 011-26865455/+91-9871145498
Website: www.physicsbyfiziks.com | Email: fiziks.physics@gmail.com
184
fiziks
Institute for NET/JRF, GATE, IIT‐JAM, M.Sc. Entrance, JEST, TIFR and GRE in Physics

1 1
mx 2 
 m  2

 m  2 2 2  mx
 
 2
E   bx  0 dx     b x e dx    b  x e
1 * 4 4 
dx
     
0 0
 


 n 1/ 2  1
 e dx    n  2 
2 n x 2
It is given in the equation x

m
Thus n  2 and  

1 1 1
 2 
 m  2 2 2  mx  m  2  m 
   1
2 2
 E 01    b  x e dx  b   2  
   
       2
1 5
 m  2  m  2 5 3 b 2
 E  b1

     
.
2 4 m 2 2
0

Q10. Let 0 and 1 denote the normalized eigenstates corresponding to the ground and first

excited states of a one dimensional harmonic oscillator. The uncertainty  p in the

state
1
0  1  , is
2

(a) p  m / 2 (b) p  m / 2

(c) p  m (d) p  2m


Ans. : (c)
m
Solution:  
1
2
 0  1  , p  i 2 a†  a  
a†  
1
2
 1 1  2 2  and a  
1
2
0  10 

p i
m
2

 a†  a   0 , p 2  
m † 2
2

a  a 2   2 N  1  
 m  †2 m  m  1 
p2  a  a2  2N  1   2N  1   2   1  m
2   2 2  2 

p  p2  p  m 
2

H.No. 40-D, Ground Floor, Jia Sarai, Near IIT, Hauz Khas, New Delhi-110016
Phone: 011-26865455/+91-9871145498
Website: www.physicsbyfiziks.com | Email: fiziks.physics@gmail.com
185
fiziks
Institute for NET/JRF, GATE, IIT‐JAM, M.Sc. Entrance, JEST, TIFR and GRE in Physics

Q11. The wave function of a particle at time t  0 is given by  0 


1
u 1  u 2  , where
2
u1 and u2 are the normalized eigenstates with eigenvalues E1 and E2

respectively, E 2  E1  . The shortest time after which  t  will become orthogonal to

 0 is

   2 2
(a) (b) (c) (d)
2E 2  E1  E 2  E1 E 2  E1 E 2  E1
Ans. : (b)

1  
 iE1t  iE2t
Solution:   0  
1
 u1  u2    t    u1 e
2

 u2 e 

2 
 iE1t  iE2t
1 1
  t  is orthogonal to   0    0   t   0  e 
 e 
0
2 2
 iE1t  iE2t  iE1t  iE2t  E2  E1 
i
e  e  0e   e  e 
 1

 cos
 E2  E1  t  cos   t  
 E2  E1
Q12. A constant perturbation as shown in the figure below acts on a particle of mass m
confined in an infinite potential well between 0 and L .

V0 V0
2
0 L/2 L

The first-order correction to the ground state energy of the particle is


V0 3V0 V0 3V0
(a) (b) (c) (d)
2 4 4 2
Ans. : (b)

H.No. 40-D, Ground Floor, Jia Sarai, Near IIT, Hauz Khas, New Delhi-110016
Phone: 011-26865455/+91-9871145498
Website: www.physicsbyfiziks.com | Email: fiziks.physics@gmail.com
186
fiziks
Institute for NET/JRF, GATE, IIT‐JAM, M.Sc. Entrance, JEST, TIFR and GRE in Physics

V0 2 2 x x
2 L
2
Solution: E   1 V p  1
1
1  sin dx   V0 sin 2 dx
0
2 L L L L L
2

V 1 2x  2V 1  2x 
2 L

E  0  1  cos
1
 dx  0  1  cos  dx
L 0 2 L  L L 2 L 
1

V0  L  2V0  L  V0 2V0 3V0


 E11    L    
2L  2  2L  2 4 4 4

NET/JRF (JUNE-2012)
Q13. The component along an arbitrary direction n̂ , with direction cosines n x , n y , n z  , of the
1
spin of a spin  particle is measured. The result is
2
(c)  n x  n y  n z 
  
(a) 0 (b)  n z (d) 
2 2 2
Ans. : (d)
 0 1  0 i   1 0 
Solution: S x    , S y    , S z   
2 1 0 2 i 0  2 0 1

n  n x ˆi  n y ˆj  n z kˆ and n x2  n y2  n z2  1 , S  S x iˆ  S y ˆj  S z kˆ

   i   
0  0    0 
  2 +n  2 
n  S  nx  2  n 
  y
 i  z  
   0  0 
2  2   2 

 
n
   z2

n x  in y 
nS  2 
 
 n x  in y 

 nz 
2 2 
 
Let  is eigen value of n  S

nz



n x  in y 
2 2
0

n x  in y  
n z  
2 2

H.No. 40-D, Ground Floor, Jia Sarai, Near IIT, Hauz Khas, New Delhi-110016
Phone: 011-26865455/+91-9871145498
Website: www.physicsbyfiziks.com | Email: fiziks.physics@gmail.com
187
fiziks
Institute for NET/JRF, GATE, IIT‐JAM, M.Sc. Entrance, JEST, TIFR and GRE in Physics

 n 2 2  2 2
n   n   
 
2
   z     z      n 2x  n 2y   0   z  2   n x  n y2  0 .
 2  2  4  4  4


2 2
4
  
n x  n y2  n z2  2  0     .
2
Q14. A particle of mass m is in a cubic box of size a . The potential inside the box
0  x  a,0  y  a,0  z  a  is zero and infinite outside. If the particle is in an

14 2  2
eigenstate of energy E  , its wavefunction is
2ma 2

3 x 5 y 6 z 7 x 4 y 3 z
3/ 2 3/ 2
2 2
(a)     sin sin sin (b)     sin sin sin
a a a a a a a a

4 x 8 y 2 z x 2 y 3 z
3/ 2 3/ 2
2 2
(c)     sin sin sin (d)     sin sin sin
a a a a a a a a
Ans. : (d)

   2 14 2  2
2
Solution: E nx ,n y ,nz  n x2  n y2  n z2 
2ma 2 2ma 2
 n x2  n y2  n z2  14  n x  1, n y  2, n z  3 .

Q15. Let  nlml denote the eigenfunctions of a Hamiltonian for a spherically symmetric

potential V r  . The wavefunction  


1
4
 
 210  5 211  10 211 is an eigenfunction

only of
(a) H , L2 and Lz (b) H and Lz (c) H and L2 (d) L2 and Lz
Ans. : (c)
Solution: H  En

L2  l l  1 2 and Lz  m .

Q16. The commutator x 2 , p 2 is 


(a) 2ixp (b) 2i ( xp  px ) (c) 2ipx (d) 2i ( xp  px )
Ans. : (b)
Solution:  x 2 , p 2   x  x, p 2    x, p 2  x  xp  x, p   x  x, p  p  p  x, p  x   x, p  px

H.No. 40-D, Ground Floor, Jia Sarai, Near IIT, Hauz Khas, New Delhi-110016
Phone: 011-26865455/+91-9871145498
Website: www.physicsbyfiziks.com | Email: fiziks.physics@gmail.com
188
fiziks
Institute for NET/JRF, GATE, IIT‐JAM, M.Sc. Entrance, JEST, TIFR and GRE in Physics

x 2

, p 2  xpi   xi  p  pi x  i  px  2ixp  px  .
Q17. A free particle described by a plane wave and moving in the positive z -direction
undergoes scattering by a potential
V , if r  R
V r    0
0 , if r  R

If V0 is changed to 2V0 , keeping R fixed, then the differential scattering cross-section, in


the Born approximation.
(a) increases to four times the original value
(b) increases to twice the original value
(c) decreases to half the original value
(d) decreases to one fourth the original value
Ans. : (a)
V0 , rR
Solution: V  r   
0, rR

Low energy scattering amplitude f  ,    


m 4
V R 3
2 0
2 3
2
d  2mV0 R 3 
And differential scattering is given by 1  f 
2

d  3 
2

d 2  2m  2V0  R 
2 2
3
 2mV0 R 3  d 1
Now V r   2V0 for r  R      4   4
d  3 2
  3
2
 d
Q18. A variational calculation is done with the normalized trial wavefunction

 x   5 / 2 a 2  x 2  for the one-dimensional potential well


15
4a
0 if x  a
V x   
 if x  a
The ground state energy is estimated to be
5 2 3 2 3 2 5 2
(a) (b) (c) (d)
3ma 2 2ma 2 5ma 2 4ma 2
Ans. : (d)

Solution:   x  
15
5
a 2
 x2 , V x   0 , x  a and V  x    , x  a
4a 2

H.No. 40-D, Ground Floor, Jia Sarai, Near IIT, Hauz Khas, New Delhi-110016
Phone: 011-26865455/+91-9871145498
Website: www.physicsbyfiziks.com | Email: fiziks.physics@gmail.com
189
fiziks
Institute for NET/JRF, GATE, IIT‐JAM, M.Sc. Entrance, JEST, TIFR and GRE in Physics

  2  2
a
E  Hdx where H 
a
2m x 2

 15 2
a
    2 d 2  15 2   15   2
a

 a  4a5/ 2
E  a   x 2

 2  5/ 2 
a  x 2   dx    a 2  x 2  2dx
  2m dx  4a  
5
 16a 2m  a
a
15 2 2 15  2 4a 3 5 2
 E 
16a 5 2m a
 a 2
 x 2
 dx 
16a 5 m 3

4ma 2

Q19. A particle in one-dimension is in the potential


 , if x  0

V  x   V0 , if 0  x  l
0 , if x  l

If there is at least one bound state, the minimum depth of potential is
 2 2  2 2 2 2 2  2 2
(a) (b) (c) (d)
8ml 2 2ml 2 ml 2 ml 2
Ans. : (a)
Solution: For bound state,  V0  E  0
Wave function in region I, I  0 ,  II  A sin kx  B cos kx , III  ce  x
  
2mV0  E  2m   E   V0 o
where k  ,  . l
2 2
Use Boundary condition at x  0 and x  l
(wave function is continuous and differential at x  0 and x  l ), one will get
k cot kl    kl cot kl  l     cot  where  l   , kl   .

2mV0l 2 
2  2 
2
1/ 2 o  3
 2mV0l 2   2 2
For one bound state     V0  . 2
 2
   8ml 2
2
2
Q20. Which of the following is a self-adjoint operator in the spherical polar coordinate
system r ,  ,   ?
i   i  
(a)  (b)  i (c)  (d)  i sin 
sin 2    sin   
Ans. : (c)
H.No. 40-D, Ground Floor, Jia Sarai, Near IIT, Hauz Khas, New Delhi-110016
Phone: 011-26865455/+91-9871145498
Website: www.physicsbyfiziks.com | Email: fiziks.physics@gmail.com
190
fiziks
Institute for NET/JRF, GATE, IIT‐JAM, M.Sc. Entrance, JEST, TIFR and GRE in Physics

i 
Solution: is Hermitian.
sin  
NET/JRF (DEC-2012)
Q21. Let v, p and E denote the speed, the magnitude of the momentum, and the energy of a
free particle of rest mass m. Then

(a) dE  constant (b) p = mv


dp

(c) v  cp (d) E = mc2


p m c
2 2 2

Ans. : (c)
mv m2v 2 p2v2
Solution: p  mv  p   m v  p  2 , m  rest mass energy
2 2 2 2

v2 v2 c
1 2 1 2
c c

 p2  p2 pc
 v 2  m2  2   p 2  v 2  2 2 v
 c  m c  p2 p  m2c 2
2

c2
Q22. The wave function of a state of the Hydrogen atom is given by,
   200  2 211  3 210  2 211
where  nlm is the normalized eigen function of the state with quantum numbers n, l , m in

the usual notation. The expectation value of Lz in the state  is

15 11 3 
(a) (b) (c) (d)
6 6 8 8
Ans. : (d)
1 2 3 2
Solution: Firstly normalize ,    200   211   210   211
16 16 16 16

P0  
1 9 10
  .
16 16 16
4 2
Probability of getting 1  i.e. P     and P      .
16 16
 Lz  
 0   1    1  
10 4 2 4 2 2
Now, Lz       
 16 16 16 16 16 16 8

H.No. 40-D, Ground Floor, Jia Sarai, Near IIT, Hauz Khas, New Delhi-110016
Phone: 011-26865455/+91-9871145498
Website: www.physicsbyfiziks.com | Email: fiziks.physics@gmail.com
191
fiziks
Institute for NET/JRF, GATE, IIT‐JAM, M.Sc. Entrance, JEST, TIFR and GRE in Physics

The energy eigenvalues of a particle in the potential V  x  


1
Q23. m 2 x 2  ax are
2
 1 a2  1 a2
(a) En   n    (b) En   n   
 2 2m 2  2 2m 2

 1 a2  1
(c) En   n    (d) En   n  
 2 m 2  2
Ans. : (a)
p x2 1
Solution: Hamiltonian  H  of Harmonic oscillator, H   m 2 x 2
2m 2
 1
Eigenvalue of this, E n   n  
 2
p 2
1 p2 1  2ax a2  a2
But here, H  x  m 2 x 2  ax  H  x  m 2  x 2    
2m 2 2m 2  m 2 m 2 4  2m 2
2
p x2 1 2 a  a2
H  m  x  
2m 2  m 2  2m 2
 1 a2
Energy eigenvalue, E n   n   
 2 2m 2
Q24. If a particle is represented by the normalized wave function


 15 a 2  x 2


, for  a  x  a
  x   4a 5 / 2
0
 , otherwise

the uncertainty p in its momentum is

(a) 2 / 5a (b) 5 / 2a (c) 10 / a (d) 5 / 2a


Ans. : (d)

  i 
Solution: p  p2  p
2
and p  x

a

15 a 2  x 2   i 
 p   4a5/ 2 4a
15  2
5 / 2 xa  x 2 dx 
a
a
 i  a 2  x 2  2 x dx  ih 2 155
   a 
a
15
 2
x  x3 dx  0 , ( odd function)
16  a
5
 a 16a a

H.No. 40-D, Ground Floor, Jia Sarai, Near IIT, Hauz Khas, New Delhi-110016
Phone: 011-26865455/+91-9871145498
Website: www.physicsbyfiziks.com | Email: fiziks.physics@gmail.com
192
fiziks
Institute for NET/JRF, GATE, IIT‐JAM, M.Sc. Entrance, JEST, TIFR and GRE in Physics

2 2
 a  x2  a  x 2 dx
a
15
p 2   2  2

16a 5 a x 2

a
 2 x3 
 
a
  2 15
  2   a 2
 x 2
dx   2

15
 2  a  x  
16a 5 a 16a 5  3  a

15  3 2a 3  2 15 2 3 1  15 2 2 5 2
 2   2  2 a        2 a 1 
 3   4a 2  3  2a 2
16a 5  3  16 a5

5 2 5
Now, p  p2  p  0 
2

2a 2 2a
Q25. Given the usual canonical commutation relations, the commutator A, B  of
A  i xp y  yp x  and B   yp z  zp y  is

(a)  xp z  p x z  (b)   xp z  p x z  (c)  xp z  p x z  (d)   xp z  p x z 


Ans. : (c)

Solution: A, B   ixp y  iyp x ,  yp z  zp y  
A, B  ixp y , yp z   i yp x , yp z   ixp y , zp y   iyp x , zp y 
A, B  ixp y , yp z   0  0  iyp x , zp y   ixp y , yp z   iyp x , zp y 
A, B  ix p y , yp z   ix, yp z  p y  iy p x , zp y   iy, zp y p x
A, B  ix p y , yp z   0  0  iy, zp y p x   
 ix p y , yp z  i y, zp y p x 
 A, B   ix   i  pz  izi  px    xpz  zpx 
A, B  xp z  p x z 
1
Q26. The energies in the ground state and first excited state of a particle of mass m  in a
2
potential V  x  are 4 and 1 , respectively, (in units in which   1 ). If the

corresponding wavefunctions are related by  1  x    0  x sinh x, then the ground state


eigenfunction is
(a)  0  x   sec hx (b)  0  x   sec hx

(c)  0  x   sec h 2 x (d)  0  x   sec h 3 x


Ans. : (c)

H.No. 40-D, Ground Floor, Jia Sarai, Near IIT, Hauz Khas, New Delhi-110016
Phone: 011-26865455/+91-9871145498
Website: www.physicsbyfiziks.com | Email: fiziks.physics@gmail.com
193
fiziks
Institute for NET/JRF, GATE, IIT‐JAM, M.Sc. Entrance, JEST, TIFR and GRE in Physics

Solution: Given that ground state energy E0  4 , first excited state energy E1  1 and  0 ,  1
are corresponding wave functions.
1
Solving Schrödinger equation (use m  and   1 )
2
  2  2 0  2 0
 V 0  E0 0   V 0  4 0 ……(1)
2m x 2 x 2
 2  2 1  2 1
 V 1  E1 1   V 1  1 1 ……..(2)
2m x 2 x 2
Put  1   0 sinh x in equation (2) one will get

  2 0  
  2 .sinh x  2 0 cosh x  0 sinh x   V 0 sinh x   0 sinh x
 x x 

  2  
  20  2 0 coth x  0   V 0   0
 x x 
  2 0   0  2 0
   V  0  2 coth x     using relation   V 0  4 0
 x x x 2
2 0 0

 0 d 0
4 0  2 coth x  0   0   2 tanh xdx   0  sec h 2 x .
x 0

NET/JRF (JUNE-2013)
Q27. In a basis in which the z - component S z of the spin is diagonal, an electron is in a spin

 1  i  / 6 
state    . The probabilities that a measurement of S z will yield the values
 
 2 / 3 
 / 2 and   / 2 are, respectively,
(a) 1/ 2 and 1/ 2 (b) 2 / 3 and 1/ 3 (c) 1/ 4 and 3 / 4 (d) 1/ 3 and 2 / 3
Ans. : (d)
1 0  
Solution: Eigen state of S z is 1    and 2    corresponds to Eigen value and 
0 1 2 2
respectively.
2 2
  1  1 i
2
2 1   2  2
P      , P   
 2   6 6 3  2   3

H.No. 40-D, Ground Floor, Jia Sarai, Near IIT, Hauz Khas, New Delhi-110016
Phone: 011-26865455/+91-9871145498
Website: www.physicsbyfiziks.com | Email: fiziks.physics@gmail.com
194
fiziks
Institute for NET/JRF, GATE, IIT‐JAM, M.Sc. Entrance, JEST, TIFR and GRE in Physics

Q28. Consider the normalized state  of a particle in a one-dimensional harmonic oscillator:

  b1 0  b2 1

where 0 and 1 denote the ground and first excited states respectively, and b1 and b2

are real constants. The expectation value of the displacement x in the state  will be a
minimum when
1 1
(a) b2  0, b1  1 (b) b2  b1 (c) b2  b1 (d) b2  b1
2 2
Ans. : (d)
Solution: x  b12 0 x 0  b22 1 x 1  2b1b2 0 x 1

Since 0 x 0  0 and 1 x 1  0  x  2b1b2 0 x 1 .

Min of x means min 2b1b2 . We know that b12  b22  1.

x min  
2

2
  2

  b1  b2   b12  b22  0 x 1  b1  b2   1 0 x 1  1  b1  b2  0 x 1 will 
be minimum and minimum value of 1   b1  b2   , there must be maximum of  b1  b2  ,
2 2

 
so  b1  b2
Q29. The un-normalized wavefunction of a particle in a spherically symmetric potential is
given by

  r   zf  r 

where f r  is a function of the radial variable r . The eigenvalue of the operator



L2 (namely the square of the orbital angular momentum) is
(a)  2 / 4 (b)  2 / 2 (c)  2 (d) 2 2
Ans. : (d)
Solution:  r   zf r   r cos f r 

  r  Y10  ,    , L2  r   L2Y10  ,   , where l  1

L2  l l  1 2  11  1 2  2 2

H.No. 40-D, Ground Floor, Jia Sarai, Near IIT, Hauz Khas, New Delhi-110016
Phone: 011-26865455/+91-9871145498
Website: www.physicsbyfiziks.com | Email: fiziks.physics@gmail.com
195
fiziks
Institute for NET/JRF, GATE, IIT‐JAM, M.Sc. Entrance, JEST, TIFR and GRE in Physics

Q30. If  nlm denotes the eigenfunciton of the Hamiltonian with a potential V  V r  then the

expectation value of the operator L2x  L2y in the state

1
  3 211  210  15 211 
5
is
(a) 39 2 / 25 (b) 13 2 / 25 (c) 2 2 (d) 26 2 / 25
Ans. : (d)
Solution: L2x  L2y  L2  L2z  L2x  L2y  L2  L2z  L2  L2z

 9 1 15 
L2  L2z = 2 2    1 2   0 2  1 2 
 25 25 25 
24 2 50  24 2 26 2
L2  L2z  2 2      
25 25 25
Q31. Consider a two-dimensional infinite square well
0, 0  x  a, 0 ya
V  x, y   
 , otherwise

2  nx  x   n y  y 
Its normalized Eigenfunctions are nx ,ny  x, y   sin   sin  ,
a  a   a 
where nx , n y  1, 2, 3, ..

 a a
V 0 x , 0 y
If a perturbation H '   0 2 2 is applied, then the correction to the
 0 otherwise

energy of the first excited state to order V0 is

V0 V0  64 
(a)
4
(b)
4 1  9 2 

V0  16  V0  32 
(c)
4 1  9 2  (d)
4 1  9 2 
Ans. : (b)
Solution: For first excited state, which is doubly degenerate
2 x 2 y 2  2 x    y 
1  sin sin , 2  sin   sin  
a a a a  a   a 

H.No. 40-D, Ground Floor, Jia Sarai, Near IIT, Hauz Khas, New Delhi-110016
Phone: 011-26865455/+91-9871145498
Website: www.physicsbyfiziks.com | Email: fiziks.physics@gmail.com
196
fiziks
Institute for NET/JRF, GATE, IIT‐JAM, M.Sc. Entrance, JEST, TIFR and GRE in Physics

2 a / 2 2   x  2 a / 2 2  2 y  1 1 V
H11  1 H 1  V0 
a 0
sin   dx  sin 
 a  a 0  a 
 dy  V0    0
2 2 4
2 a/2  x 2 x 2 a / 2 2 y y
H12  1 H 2 
a 0
 V0
sin
a
sin
a
dx  sin
a 0 a
sin
a
dy

 4  4  16 16 V
H 12  V0     V0 , H 21  2 H  1  V0 2 and H 22  2 H  2  0 .
 3  3  9 9
2
4

 V0 16V0 
   2 2

Thus  4 9 2   0   V0      16V0   0
 16V0 V0
    4   9 
2

 9 2 4 
V  16V0 V  64 
  0      0 1  2 
 4  9 2
4  9 
Q32. The bound on the ground state energy of the Hamiltonian with an attractive delta-
function potential, namely
2 d 2
H   a  x 
2m dx 2

using the variational principle with the trial wavefunction   x   A exp  bx 2 is 
 

 Note :  e t dt  a  1
t a

 0 

(a)  ma 2 / 4  2 (b)  ma 2 / 2  2 (c)  ma 2 /   2 (d)  ma 2 / 5  2


Ans. : (c)
 2b 2b  2b 2b
Solution: For given wavefunction T  and V  a  E  a
2m  2m 
d E d E 2 2 1  12 2m 2 a 2
For variation of parameter 0  a  b 0 b .
db db 2m  2  4
ma 2
 E  .
min
 2
Q33. If the operators A and B satisfy the commutation relation  A, B  I , where I is the
identity operator, then
 
(a) e A , B  e A   
(b) e A , B  e B , A
(c) e , B  e
A B

,A (d) e , B  I
A

Ans. : (a)

H.No. 40-D, Ground Floor, Jia Sarai, Near IIT, Hauz Khas, New Delhi-110016
Phone: 011-26865455/+91-9871145498
Website: www.physicsbyfiziks.com | Email: fiziks.physics@gmail.com
197
fiziks
Institute for NET/JRF, GATE, IIT‐JAM, M.Sc. Entrance, JEST, TIFR and GRE in Physics

A, B  I and e A  1  A  A 


2
Solution:  .......
 1 2 
 A2 , B   A3 , B 
 
 A A2
e , B  1  
A 
 ......., B  = 1, B    A, B    ....
 1 2  2 3

A  A, B    A, B  A A  A2 , B    A2 , B  A
e , B  0  I 
A

2!

3!
 ....

e A , B   1  A 
A2
2!
 
 ....  e A where A, B  I , A 2 , B  2 A and A3 , B  3 A 2 .  
Q34. Two identical bosons of mass m are placed in a one-dimensional potential

V x  
1
m 2 x 2 . The bosons interact via a weak potential,
2

V12  V0 exp  m x1  x 2  / 4
2

where x1 and x 2 denote coordinates of the particles. Given that the ground state
1
m x 2
 m  4 
wavefunction of the harmonic oscillator is  0  x     e
2
. The ground state
  
energy of the two-boson system, to the first order in V0 , is

V0 
(a)   2V0 (b)  

1

   2  
(c)   V0 1   (d)   V0 1  
 2   
Ans. : (c)
Solution: There are two bosons trapped in harmonic oscillator.

So, energy for ground state without perturbation is, 2    .
2
If perturbation is introduced, we have to calculate V1,2 

where V1,2  V0 exp  m  x1  x2  / 4  .


2

 

 1
m x12 m x22 
  m  2  
 is very tedious task.
But calculating V1,2  on state  0  x    2
e 2
    
e

 

H.No. 40-D, Ground Floor, Jia Sarai, Near IIT, Hauz Khas, New Delhi-110016
Phone: 011-26865455/+91-9871145498
Website: www.physicsbyfiziks.com | Email: fiziks.physics@gmail.com
198
fiziks
Institute for NET/JRF, GATE, IIT‐JAM, M.Sc. Entrance, JEST, TIFR and GRE in Physics

So lets use a trick i.e perturbation is nothing but approximation used in Taylor series. So

just expand V1,2  V0 exp   m  x1  x2  / 4  and take average value of first term
2

 
 m  x1  x2 2 
V1,2  V0 exp m  x1  x2  / 4  V0  1 
   ... 
2

   4 
 

 V0  1 

 m x12  x22  2 x1.x2 
 ... 

 4 
 

    

 m  x12    x22   2 x1 . x2     m    0 
V1,2   V0 1   ...   Vo  1   2m 2m   ...
 4   4 
   
 
1 1
   2   2
 V12   Vo (1  )  V0 1   , so E    V0 1   .
4  2   2 

NET/JRF (DEC-2013)
1 1 1  i 
Q35. A spin - particle is in the state     in the eigenbasis of S 2 and S z . If we
2 11  3 
h h
measure S z , the probabilities of getting  and  , respectively are
2 2
1 1 2 9 1 3
(a) and (b) and (c) 0 and 1 (d) and
2 2 11 11 11 11
Ans. : (b)
2
  1 1  i 1 2
Solution: P    10     2     1
 2 11  3  11 11
2
  1 1  i 9
P  
 2
 01   
11  3  11

   
i.e. probability of S z getting   and   
 2  2

H.No. 40-D, Ground Floor, Jia Sarai, Near IIT, Hauz Khas, New Delhi-110016
Phone: 011-26865455/+91-9871145498
Website: www.physicsbyfiziks.com | Email: fiziks.physics@gmail.com
199
fiziks
Institute for NET/JRF, GATE, IIT‐JAM, M.Sc. Entrance, JEST, TIFR and GRE in Physics

Q36. The motion of a particle of mass m in one dimension is described by the


p2 1
Hamiltonian H   m 2 x 2  x . What is the difference between the (quantized)
2m 2
energies of the first two levels? (In the following, x is the expectation value of x in the
ground state)
2
(a)    x (b)    x (c)   (d) 
2m 2
Ans. : (d)
p2 1 1
Solution: H   m 2 x 2   x  V  x   m 2 x 2   x
2m 2 2
1  2  1 2 2  2 2 
V  x  m 2  x 2   x  m   x  2  x    
2  m 2  2  m 2 m 2 4 m 2 4 
  2
2

V  x   m 2  x 
1
 
2  m 2  2m 2
 1 2 3 1
 En   n       E1  E0      
 2 2m 2
2 2
Q37. Let  nlm denote the eigenfunctions of a Hamiltonian for a spherically symmetric

potential V r  . The expectation value of L z in the state

 
1
6
 200  5 210  10 211  20 211 is 
5 5 5
(a)   (b)  (c)  (d) 
18 6 18
Ans. : (d)
1 5 10 20 10 5
Solution: Lz   L z  =  0   0   (1)  (1)      1
36 36 36 36 36 18
Q38. If   x   A exp x 4  is the eigenfunction of a one dimensional Hamiltonian with eigen
value E  0 , the potential V  x  (in units where   2m  1 ) is

(a) 12x 2 (b) 16x 6 (c) 16 x 6  12 x 2 (d) 16 x 6  12 x 2


Ans. : (d)
Solution: Schrodinger equation
  2  V  0 (where   2m  1 and E  0 )

2
x
  4 
 2 Ae  x  VAe  x  0   e  x  4 x 3   Ve x  0
4

x 
4


4

H.No. 40-D, Ground Floor, Jia Sarai, Near IIT, Hauz Khas, New Delhi-110016
Phone: 011-26865455/+91-9871145498
Website: www.physicsbyfiziks.com | Email: fiziks.physics@gmail.com
200
fiziks
Institute for NET/JRF, GATE, IIT‐JAM, M.Sc. Entrance, JEST, TIFR and GRE in Physics

 4

4 3x 2 e  x  x 3  4 x 3 e  x
4
 Ve  x4
 0  12 x 2 e  x  16 x 6 e  x  Ve  x  0
4 4 4

 V  16 x 6  12 x 2
Q39. A particle is in the ground state of an infinite square well potential is given by,
0 for  a  x  a
V x   
 otherwise
a a
The probability to find the particle in the interval between  and is
2 2
1 1 1 1 1 1
(a) (b)  (c)  (d)
2 2  2  
Ans. : (b)
a a
Solution: The probability to find the particle in the interval between  and is
2 2

x x x 1 1  2x  
a/2 a/2 a/2
2 2 1
   cos  cos dx   cos 2 dx     1  cos dx 
a / 2
2a 2a 2a 2a a / 2
a 2a a 2 a / 2  2a  

x 
a/2
1  1 a a a  1  2a   1 1 
  1  1 
a
  x  sin     a    
2a   a   a / 2 2a  2 2   2a    2  
Q40. The expectation value of the x - component of the orbital angular momentum L x in the

state  
1
5

3 2,1, 1  5 2,1,0  11 2,1, 1 
(where  nlm are the eigenfunctions in usual notation), is

(a) 
 10
25
 11  3 (b) 0 (c)
 10
25
 11  3 (d)  2

Ans. : (a)

Solution: L l , m  l  l  1  m  m  1 l , m  1 and L l , m  l  l  1  m  m  1 l , m  1

L  L L  L
Lx   Lx  
2 2
1
L  3 2 210  5 2 211 
5
1 1 1
 L   .3 10  110  10(3  11)
25 25 25

H.No. 40-D, Ground Floor, Jia Sarai, Near IIT, Hauz Khas, New Delhi-110016
Phone: 011-26865455/+91-9871145498
Website: www.physicsbyfiziks.com | Email: fiziks.physics@gmail.com
201
fiziks
Institute for NET/JRF, GATE, IIT‐JAM, M.Sc. Entrance, JEST, TIFR and GRE in Physics

1
L   2 5 211  2 11 210 
5
1 1
 L   .3 10  10 11
25 25
L  L 1
Lx  = 10(3  11)
2 25

 Lx  
1
25
.3 10 
1
25
10 11 = 
 10
25
 11  3
Q41. A particle is prepared in a simultaneous eigenstate of L2 and Lz . If l  l  1  2 and m

are respectively the eigenvalues of L2 and Lz , then the expectation value L2x of the

particle in this state satisfies


(a) L2x  0 (b) 0  L2x   2  2
    1  2  2     1  2
(c) 0  L2x  (d)  L2x 
2 2 2
Ans. : (d)
Solution: L2x 
1
2

l  l  1  2  m 2  2 
For max value m  0 and for min m  l
l 2 l l  1 2
 L2x 
2 2
A, B, C are Non zero Hermitian operator.
A, B  C  AB  BA  AB  Ab  0  C
but C0
if AB  BA i.e. A, B   C false (2)

H.No. 40-D, Ground Floor, Jia Sarai, Near IIT, Hauz Khas, New Delhi-110016
Phone: 011-26865455/+91-9871145498
Website: www.physicsbyfiziks.com | Email: fiziks.physics@gmail.com
202
fiziks
Institute for NET/JRF, GATE, IIT‐JAM, M.Sc. Entrance, JEST, TIFR and GRE in Physics

NET/JRF (JUNE-2014)
Q42. Consider a system of two non-interacting identical fermions, each of mass m in an
infinite square well potential of width a . (Take the potential inside the well to be zero
and ignore spin). The composite wavefunction for the system with total energy
5 2  2
E is
2ma 2
2   x1   2x 2   2x1   x 2 
(a) sin  sin   sin  sin 
a  a   a   a   a 

2   x1   2x 2   2x1   x 2 


(b) sin  sin   sin  sin 
a  a   a   a   a 

2   x1   3x 2   3x1   x 2 


(c) sin  sin   sin  sin 
a   a   2a   2a   a  

2   x1   x 2   x   x 
(d) sin  cos   sin 2  cos 2 
a  a   a   a   a 
Ans. : (a)
Solution: Fermions have antisymmetric wave function
2    x1   2 x2   2 x1    x2  
  x1 x2    sin   sin    sin    sin  
a  a   a   a   a 

5 2  2
 En   nx1  1, nx2  2
2ma 2

Q43. A particle of mass m in the potential V  x, y  


1
2
 
m 2 4 x 2  y 2 , is in an eigenstate of

5
energy E   . The corresponding un-normalized eigen function is
2
 m  m
(a) y exp  2 x 2  y 2 

(b) x exp  2 x 2  y 2 

 2   2 
 m 2  m 2
(c) y exp  x  y 2 

(d) xy exp  x  y 2 

 2   2 
Ans. : (a)

Solution: V  x, y  
1
2
  5
m 2 4 x 2  y 2 , E   
2
H.No. 40-D, Ground Floor, Jia Sarai, Near IIT, Hauz Khas, New Delhi-110016
Phone: 011-26865455/+91-9871145498
Website: www.physicsbyfiziks.com | Email: fiziks.physics@gmail.com
203
fiziks
Institute for NET/JRF, GATE, IIT‐JAM, M.Sc. Entrance, JEST, TIFR and GRE in Physics

1 1
 V  x, y   m  2  x 2  m 2 y 2
2

2 2
 1  1  1  1
Now, E n   n x   x   n y   y   n x  2   n y  
 2  2  2  2

 3
 En   2 n x  n y   
 2
5
 En   when n x  0 and n y  1 .
2
Q44. A particle of mass m in three dimensions is in the potential
0, ra
V r   
 , ra
Its ground state energy is
 2 2  2 2 3 2  2 9 2  2
(a) (b) (c) (d)
2ma 2 ma 2 2ma 2 2ma 2

Ans. : (a)
  2  d u  r  l  l  1
2

Solution:    2
  V  r  u  r   Eu  r 
 2m  dr 2mr 2

d 2u  r  2mE
  K 2u  r  K  , l  0, V  r   0
dr 2
2
u  r   A sin Kr  B cos Kr

Using boundary condition, B  0,

 22
u  r   A sin Kr , r  a, u  r   0  sin Ka  0  Ka  n  E  n 1
2ma 2
  1
Q45. Given that pˆ r  i   , the uncertainty p r in the ground state
 r r 
1
 0 r   e  r / a0 of the hydrogen atom is
a 3
0

 2  2
(a) (b) (c) (d)
a0 a0 2a 0 a0

Ans. : (a)

H.No. 40-D, Ground Floor, Jia Sarai, Near IIT, Hauz Khas, New Delhi-110016
Phone: 011-26865455/+91-9871145498
Website: www.physicsbyfiziks.com | Email: fiziks.physics@gmail.com
204
fiziks
Institute for NET/JRF, GATE, IIT‐JAM, M.Sc. Entrance, JEST, TIFR and GRE in Physics

  1 1
Solution: pˆ r  i    ,  0  r   e  r / a0 , Pr  Pr2  Pr
2

 r r   a03


1  r / a
  1  e 0 
Now Pr   e  r / a 0    i     4 r dr
2

0 a 03   r r  a 03 

4 i    r / a0   r / a0  1  1  r / a0  2 

a 03  0
  e     e  r dr 
e 
  a0  r  

4 i   1 
 
 3 
 e r dr   re  2 r / a0 dr 
 2 r / a0 2

a 0  a 0 0 0 

4 i   1  2!   1!  
    
 a03  a0   2 / a0 3    2 / a0 2  
 

4 i   a02 a02 
   0
 a03  4 4 

1

 2   2 2    r / a0 
a 03 0
4 r dr
 r / a0
Pr2     2  e 2
e
  r r r  

4  2  r / a  r / a  1  2  1  r / a   2
   e 0  e 0  2       e 0   r dr
 a03  0   a0  r  a0   

4 2  1 2 2 r / a0

2

 4 2 1 2 ! 2 1 ! 
a03 0 a02 a0 0
2 r / a0
  r e dr  re dr     2  
 a0  2 / a0  a0  2 / a0  
3 2
 a03

4 2  2 ! a03 2 a02  4 2  a0 a0  4 2  a0   2
 3  2       3      3    2
a0  a0 8 a0 4 a0  4 2 a0  4  a0

2 
 P  Pr2  Pr  0 
2
2
a0 a0

Q46. The ground state eigenfunction for the potential V  x     x  where   x  is the delta

function, is given by   x   Ae
 x
, where A and   0 are constants. If a perturbation

H   bx 2 is applied, the first order correction to the energy of the ground state will be
b b 2b b
(a) (b) (c) (d)
2 2
 2
 2
2 2
H.No. 40-D, Ground Floor, Jia Sarai, Near IIT, Hauz Khas, New Delhi-110016
Phone: 011-26865455/+91-9871145498
Website: www.physicsbyfiziks.com | Email: fiziks.physics@gmail.com
205
fiziks
Institute for NET/JRF, GATE, IIT‐JAM, M.Sc. Entrance, JEST, TIFR and GRE in Physics

Ans. : (d)

Solution: V  x     x  ,  x   Ae
 x

   1    x    e  x

E  1 H  1    e  x bx 2  e  x dx
1
1


 
 0 2 2 x 
   2  2x 
 e bx dx  b   e x dx  b   x e dx   x e dx   b 2   x e dx 
 2 x 2 2 x 2 2 2 x

    0   0 

 2!  2! b

 2 x
 e bx 2
dx  2b    2  b 
 2   8 2 2
3 3

Q47. An electron is in the ground state of a hydrogen atom. The probability that it is within the
Bohr radius is approximately equal to
(a) 0.60 (b) 0.90 (c) 0.16 (d) 0.32
Ans. : (d)
2
4
a0 a0
1
 4 r dr  3 r e
 r / a0 2 2 2 r / a0
Solution: Probability: e dr
0 a 3
0
 a0 0

4   
a0 a0 a0
 a   2 r / a0  a0  a0    2 r / a0  a0  a0  a0  
 3   r 2 e 2 r / a0   0
a0    2
    2r e
0 
 

  2   2     2e
0 
  2   2   2  
   0

 

4  2  2aa0  a0   a02  2 a0 / a0 a03 2 a0 / a0 0  a0


3

 3  a0 e 0  2   2 a0   e  e  2 e  
a0     4  4  8  

4  a03 1 a03 1 a03 a03   5 1  1 


        4   2     5  2  1
a03  4e 4  
2 2 2
 2 e 2 e 4e 4 e

  5  0.137  1   0.685  1  0.32

H.No. 40-D, Ground Floor, Jia Sarai, Near IIT, Hauz Khas, New Delhi-110016
Phone: 011-26865455/+91-9871145498
Website: www.physicsbyfiziks.com | Email: fiziks.physics@gmail.com
206
fiziks
Institute for NET/JRF, GATE, IIT‐JAM, M.Sc. Entrance, JEST, TIFR and GRE in Physics

Q48. A particle in the infinite square well potential


0 , 0 xa
V  x  
 , otherwise
is prepared in a state with the wavefunction
 3x 
 A sin 
  x   , 0  x  a
 a 
0
 , otherwise
The expectation value of the energy of the particle is
5 2 2 9 2  2 9 2  2  2 2
(a) (b) (c) (d)
2ma 2 2ma 2 10ma 2 2ma 2

Ans. : (c)
 3x 
0, 0 xa  A sin   , 0 xa 
Solution: V  x      x    a  
 , otherwise 0 
 , otherwise 
x  x 3 x
  x   A sin 3 
 a 
3
  A sin
4 a
1
 A sin
4 a
sin 3 A  3sin A  4 sin A
3

A a 2 x a 2 3 x  A a a 
   3sin  sin     x   3 1  x   3  x  
4 2 a a 2 a a  4 2 2 

a 2 a 2 10a 2 32
  1 9 A  A 1  A 1  A 
32 32 32 10a

1 a 32 a 32  3 1
  x    3. 1  x   3  x    1  x   3  x 
4 2 10a 2 10a  10 10

 22 9 2  2
Now, E1  2
, E3   E  an P  an 
2ma 2ma 2
2 2
1  9 2  1
Probability P  E1    , P  E3   
 10  10

9  2  2 1 9 2  2 9 2  2
E      E 
10 2ma 2 10 2ma 2 10ma 2

H.No. 40-D, Ground Floor, Jia Sarai, Near IIT, Hauz Khas, New Delhi-110016
Phone: 011-26865455/+91-9871145498
Website: www.physicsbyfiziks.com | Email: fiziks.physics@gmail.com
207
fiziks
Institute for NET/JRF, GATE, IIT‐JAM, M.Sc. Entrance, JEST, TIFR and GRE in Physics

NET/JRF (DEC-2014)
Q49. Suppose Hamiltonian of a conservative system in classical mechanics is H  xp , where
 is a constant and x and p are the position and momentum respectively. The
corresponding Hamiltonian in quantum mechanics, in the coordinate representation, is
  1   1
(a)  i  x   (b)  i  x  
 x 2   x 2 
 i 
(c)  ix (d)  x
x 2 x
Ans. : (b)
Solution: Classically H   xp , quantum mechanically H must be Hermitian,
 
So, H   xp  px  and  xp  px 
H 
2 2
  i  x       
 H   x  i      i    x x  
2 x x  2  x x 
i      1
 H   2x    i  x  
2  x   x 2 
Q50. Let  1 and  2 denote the normalized eigenstates of a particle with energy eigenvalues
E1 and E2 respectively, with E 2  E1 . At time t  0 the particle is prepared in a state

 t  0 
 1   2 
1
2
The shortest time T at which  t  T  will be orthogonal to  t  0  is
2   
(a) (b) (c) (d)
E 2  E1  E 2  E1  2E 2  E1  4E 2  E1 
Ans. : (b)
1  iE1T 1  iE2T
 1   2 
1
Solution:   t  0  and   t  T   e 1  e 2
2 2 2
1  iE1 T 1  iE2 T iE
 1T
iE
 2T i  E2  E1 
T

  0  T  dx  0  e  e  0  e   e   e   1
*

2 2
T   
Equate real part  cos   E2  E1   1  T  cos 1  1 
   E2  E1   E2  E1 

H.No. 40-D, Ground Floor, Jia Sarai, Near IIT, Hauz Khas, New Delhi-110016
Phone: 011-26865455/+91-9871145498
Website: www.physicsbyfiziks.com | Email: fiziks.physics@gmail.com
208
fiziks
Institute for NET/JRF, GATE, IIT‐JAM, M.Sc. Entrance, JEST, TIFR and GRE in Physics

Q51. Consider the normalized wavefunction


  a1 11  a 2 10  a3 11
where  lm is a simultaneous normalized eigenfunction of the angular momentum

operators L2 and Lz , with eigenvalues l l  1 2 and m respectively. If  is an


eigenfunction of the operator L x with eigenvalue  , then

1 1 1 1
(a) a1  a3  , a2  (b) a1  a3  , a2 
2 2 2 2
1 1 1
(c) a1  a3  , a 2   (d) a1  a 2  a3 
2 2 3
Ans. : (b)
L  L
Solution: Lx        
2
For L , L  a1 11  a2 10  a3 11   a1 0 12  a2 2 11  a3 2 10

 a2 2 11  a3 2 10

For L , L  a1 11  a2 10  a3 11   a1 2 10  a2 2 11

L  L
Given   
2
L  L 1
    a2 2 11   a1  a3  2 10  a2 2 11 
2 2
L  L
     a1 11  a2 10  a3 11  (Given)
2
a2
Thus  a1  a2  2a1
2
a1  a3 a1  a3 a22
 a2   2a1  a1  a3  a12  a22  1
2 2 2
1 1
a1  a3  , a2 
2 2

H.No. 40-D, Ground Floor, Jia Sarai, Near IIT, Hauz Khas, New Delhi-110016
Phone: 011-26865455/+91-9871145498
Website: www.physicsbyfiziks.com | Email: fiziks.physics@gmail.com
209
fiziks
Institute for NET/JRF, GATE, IIT‐JAM, M.Sc. Entrance, JEST, TIFR and GRE in Physics

Q52. Let x and p denote, respectively, the coordinate and momentum operators satisfying the
canonical commutation relation x, p   i in natural units   1 . Then the commutator

x, pe  is
p

(a) i 1  p  e p  
(b) i 1  p 2 e p (c) i 1  e  p  (d) ipe  p
Ans. : (a)
Solution:   x, p   i
 p 2 p3 
 x, pe  p    x, p  e  p  p  x, e  p   ie  p  p  x,1  p   ....
 2 3 
  p2    2ip 3ip 2 
 ie p  p  x,1   x, p    x,  ....  ie  p  p 0  i   ......
  2   2 3 
 p3 
  x, pe p   ie  p  i  p  p 2  .....  ie  p  ipe  p  i 1  p  e  p
 2 
  
Q53. Let    1 ,  2 ,  3  , where  1 ,  2 ,  3 are the Pauli matrices. If a and b are two
   

arbitrary constant vectors in three dimensions, the commutator a   , b   is equal to (in 
the following I is the identity matrix)
  
 
 
(a) a  b  1   2   3  
(b) 2i a  b  
 
 
(c) a  b I
 
(d) a b I

Ans. : (b)
 
Solution: a  a1iˆ  a2 ˆj  a3 kˆ , b  b1iˆ  b2 ˆj  b3 kˆ ,    x iˆ   y ˆj   z kˆ
   
 a   , b      a1 x  a2 y  a3 z , b1 x  b2 y  b3 z 
   
   
 a   , b     a1b1  x ,  x   a1b2  x ,  y   a1b3  x ,  z   a2b1  y ,  x   a2b2  y ,  y 
       

 a2b3  y ,  z   a3b1  z ,  x   a3b2  z ,  y   a3b3  z ,  z 

 a1b1  0  a1b2  2i z  2ia1b3 y  a2b1  2i z  0  a2b3  2i x  a3b1  2i y  a3b2  2i x  0
      

  a   , b      2i a  b  

H.No. 40-D, Ground Floor, Jia Sarai, Near IIT, Hauz Khas, New Delhi-110016
Phone: 011-26865455/+91-9871145498
Website: www.physicsbyfiziks.com | Email: fiziks.physics@gmail.com
210
fiziks
Institute for NET/JRF, GATE, IIT‐JAM, M.Sc. Entrance, JEST, TIFR and GRE in Physics

Q54. The ground state energy of the attractive delta function potential
V  x   b  x  ,
where b  0 , is calculated with the variational trial function
 x 
 A cos , for  a  x  a, 
 x    2a  is
 0, otherwise, 

mb 2 2mb 2 mb 2 mb 2
(a)  (b)  (c)  (d) 
 2 2  2 2 2 2  2 4 2  2
Ans. : (b)
 x
Solution: V  x   b  x  ; b  0 and   x    A cos ; a  x  a
 2a

2 x
Normalized   cos
2a 2a
a  2   2  2 2
T   *  2  dx 
a
 2m  x 8ma 2

2 b
V    *  b  x  dx   b   
a

a 2a a
 2 2 b  E 2 2  2 b  2  2  2 2
E       0   b  0  a 
8ma 2 a a 8ma 3 a2 4ma 4mb

b    4mb  b  4mb 
2
 22
2 2
2mb 2
Put the value of a in equation: E     2 2  2 2
8ma 2 a 8m  2  2
2
      
Q55. Let   c0 0  c1 1 (where c 0 and c1 are constants with c 02  c12  1 ) be a linear

combination of the wavefunctions of the ground and first excited states of the one-
dimensional harmonic oscillator. For what value of c 0 is the expectation value x a

maximum?

 1  1
(a) x  , c0  (b) x  , c0 
m 2 2 m 2

 1  1
(c) x  , c0  (d) x  , c0 
2m 2 m 2
Ans. : (c)
H.No. 40-D, Ground Floor, Jia Sarai, Near IIT, Hauz Khas, New Delhi-110016
Phone: 011-26865455/+91-9871145498
Website: www.physicsbyfiziks.com | Email: fiziks.physics@gmail.com
211
fiziks
Institute for NET/JRF, GATE, IIT‐JAM, M.Sc. Entrance, JEST, TIFR and GRE in Physics

Solution:   c0 0  c1 1

X   X

   
 X  2c0 c1 0 X 1   c02  c12   c0  c1   0 X 1  1   c0  c1   0 X 1
2

2

1
For max X  c0  c1  c02  c12  1  c0 
2
1 1
 X 2 0 X 1  0 X 1
2 2

 
2m
 0 aa 1

X 
2m
Q56. Consider a particle of mass m in the potential V  x   a x , a  0 . The energy eigen-

values E n n  0, 1, 2, .... , in the WKB approximation, are


1/ 3 2/3
 3a  1   3a  1 
(a)   n   (b)   n  
 4 2m  2   4 2m  2 
4/3
3a  1  3a  1 
(c) n   (d)   n  
4 2m  2  4 2m  2 
Ans. : (b)
Solution: V  x   a x , a0

2  1
According to W.K.B., 
1
pdq   n    where a1 and a2 are positive mid point
 2

P2
E  a x  P  2m  E  a x 
2m E /  E /

 1
2m  E  a x dx   n   
E/ a
 E/ a
 2

 1
2m  E  ax dx   2m  E  ax dx   n   
0 E/a
E / a 0
 2

 1
2m  E  ax dx   n   
E/a
2
0
 2

H.No. 40-D, Ground Floor, Jia Sarai, Near IIT, Hauz Khas, New Delhi-110016
Phone: 011-26865455/+91-9871145498
Website: www.physicsbyfiziks.com | Email: fiziks.physics@gmail.com
212
fiziks
Institute for NET/JRF, GATE, IIT‐JAM, M.Sc. Entrance, JEST, TIFR and GRE in Physics

2m  E  ax   t , At x  0, t  2mE; x  E / a, t  0

 2madx  dt
2 mE
2 mE  1 2  1
 2ma  t dt   n     2ma t 3/ 2
1/ 2
 n  
0
 2 3 0  2

4  1 4  1
  n     ma  2mE    n   
2 mE 3/ 2
 ma t 3/2
3 0
 2 3  2
2/3
4  1  3a  1 
  23/ 2 am5 / 2 E 3/ 2   n     E    n  
3  2  4 2m  2 

Q57. The Hamiltonian H 0 for a three-state quantum system is given by the matrix

1 0 0  0 1 0
   
H 0   0 2 0  . When perturbed by H    1 0 1  where  1 , the resulting shift
0 0 2  0 1 0
   
in the energy eigenvalue E 0  2 is

(a) ,  2  (b)  , 2  (c)   (d)  2 


Ans. : None of the answer is correct.
 1 0 0  0 1 0
Solution: H 0   0 2 0 , H  0  1 0 1
   
 0 0 2  0 1 0

 2 0  0 1
 0 2 in H 0 is not 0  1 0 in H  because H  is not in block diagonal form. So we

must diagonalised whole H  . The Eigen value at H   0,  2 0 ,  2 0 .

0 0 0 
 
After diagonalisation H  0  0 2 0  ,   0 is correction for Eigenvalue at H 0 .
 0 0  2 

So  2 0 is the correction for eigenvalue of H 0  2


Hence none of the options given is correct.

H.No. 40-D, Ground Floor, Jia Sarai, Near IIT, Hauz Khas, New Delhi-110016
Phone: 011-26865455/+91-9871145498
Website: www.physicsbyfiziks.com | Email: fiziks.physics@gmail.com
213
fiziks
Institute for NET/JRF, GATE, IIT‐JAM, M.Sc. Entrance, JEST, TIFR and GRE in Physics

NET/JRF (JUNE-2015)
Q58. The ratio of the energy of the first excited state E1 , to that of the ground state E0 , to that

L
of a particle in a three-dimensional rectangular box of side L, L and , is
2
(a) 3 : 2 (b) 2 :1 (c) 4 :1 (d) 4 : 3
Ans. (a)
 22 6 2  2
Solution: E   n 2
 n 2
 4 n 2
 , for ground state n  1, n  1, n  1  E 
2mL2 
x y z  x y z 0
2mL2
 2 2 9 2  2
2 
For first excited state nx  1, n y  2, nz  1  E  E1  1  4  4 
2mL 2mL2
E1 9 3
  
E0 6 2

Q59. If Li are the components of the angular momentum operator L , then the operator

 i 1,2,3  L, Li  equals
   
(a) L (b) 2L (c) 3L (d)  L
Ans. (b)

Solution: Let L  Lx iˆ  Ly ˆj  Lz kˆ

x  1, y  2, z  3

 L, Lx    Ly , Lx  j   Lz , Lx  kˆ  iLz ˆj  Ly kˆ i
 

  L, Lx  , Lx   i    Lz , Lx  ˆj   Ly , Lx  i   i .i Ly ˆj   i   Lz  i   Lz  i   .kˆ   2  Ly ˆj  Lz kˆ 
      

similarly,   L, Ly  Ly    2  Lx iˆ  Lz kˆ 
   

  L, Lz  Lz    2  Lx iˆ  Ly ˆj 
    

  L, Li  Li   2 2  Lx iˆ  Ly ˆj  Lz kˆ   2 L put   1
i 1,2,3
 

H.No. 40-D, Ground Floor, Jia Sarai, Near IIT, Hauz Khas, New Delhi-110016
Phone: 011-26865455/+91-9871145498
Website: www.physicsbyfiziks.com | Email: fiziks.physics@gmail.com
214
fiziks
Institute for NET/JRF, GATE, IIT‐JAM, M.Sc. Entrance, JEST, TIFR and GRE in Physics

Q60. The wavefunction of a particle in one-dimension is denoted by   x  in the coordinate


 ipx
representation and by   p     x  e 
dx in the momentum representation. If the

action of an operator T̂ on   x  is given by Tˆ  x     x  a  , where a is a constant

then Tˆ  p  is given by


 iap  iap
i  i 
(a)  ap  p  (b) e 
  p (c) e 
  p (d) 1  ap    p 
   
Ans. (c)
 ipx
Solution:   p     x e 
dx

T  x     x  a 
 ipx  ipx ipa  ip  x  a 

T   p    T  x  e 
dx    x  a  e 
dx  e 
  x  a e 
dx
ipa
 T  p   e    p 

Q61. The differential cross-section for scattering by a target is given by


d
 ,    a 2  b2 cos 2 
d
If N is the flux of the incoming particles, the number of particles scattered per unit time
is
4  1 
(a)
3

N a2  b2  (b) 4 N  a 2  b 2 
 6 

1 1   1 
(c) 4 N  a 2  b 2  (d) 4 N  a 2  b 2 
2 3   3 
Ans. (d)
d
Solution:  a 2  b 2 cos 2 
d
 2  2
2  b2 
 a   sin  d d  b  cos  sin  d  d  a .4  b .2   4  a 2  
2 2 2 2 2

0 0 0 0
3  3

 b2 
Number of particle scattered per unit time,  .N  4 N  a 2  
 3

H.No. 40-D, Ground Floor, Jia Sarai, Near IIT, Hauz Khas, New Delhi-110016
Phone: 011-26865455/+91-9871145498
Website: www.physicsbyfiziks.com | Email: fiziks.physics@gmail.com
215
fiziks
Institute for NET/JRF, GATE, IIT‐JAM, M.Sc. Entrance, JEST, TIFR and GRE in Physics

1
Q62. A particle of mass m is in a potential V  m 2 x 2 , where  is a constant. Let
2
m  ipˆ  daˆ
aˆ   xˆ   . In the Heisenberg picture is given by
2  m  dt

(a)  â (b) i aˆ (c)  â † (d) i aˆ †


Ans. : (b)
1
Solution: V  m 2 x 2
2
m  ipˆ 
aˆ   xˆ  
2  m 
daˆ 1 a a
  a, H   , 0
dt i t t

daˆ 1 m   p 2  im 2  1 m  2 p i 
   x,   pˆ , x 2   
  i   2 x  i  
dt i 2   2m  2m  i  2  2m 2 

m  p  m  ip 
   i x   i x   i aˆ
2  m  2  m 
Q63. Two different sets of orthogonal basis vectors
 1   0    1 1 1  1  
  ,    and   ,    are given for a two dimensional real vector space.
 0   1    2  1  2  1  
The matrix representation of a linear operator  in these basis are related by a unitary
transformation. The unitary matrix may be chosen to be
 0 1  0 1 1 1 1  1 1 0 
(a)   (b)   (c)   (d)  
1 0  1 0 2 1 1 2 1 1 
Ans. : (c)
1 0 1 1 1  1 1 1 
Solution: u1    , u2     u  u1  u2   
0 1 2  1 1  2  1 1 

H.No. 40-D, Ground Floor, Jia Sarai, Near IIT, Hauz Khas, New Delhi-110016
Phone: 011-26865455/+91-9871145498
Website: www.physicsbyfiziks.com | Email: fiziks.physics@gmail.com
216
fiziks
Institute for NET/JRF, GATE, IIT‐JAM, M.Sc. Entrance, JEST, TIFR and GRE in Physics
 
Q64. The Dirac Hamiltonian H  c . p   mc 2 for a free electron corresponds to the classical

relation E 2  p 2 c 2  m 2 c 4 . The classical energy-momentum relation of a piratical of

 2   q 
2

 
charge q in a electromagnetic potential  , A is  E  q   c  p  A   m 2 c 4 .
2

 c 
Therefore, the Dirac Hamiltonian for an electron in an electromagnetic field is
 e    e 
(a) c . p  A. A   mc 2  e (b) c .  p  A    mc 2  e
c  c 
  e   e 
(c) c   . p  e  A    mc 2 (d) c .  p  A    mc 2  e
 c   c 
Ans. : (d)
Solution: Electromagnetic interaction of Dirac particle
1
  qA  2 2
H   P   c  m c   q
2 2 4

 c  
Quantum mechanical Hamiltonian

    qA  
i   c  P     mc  q 
2

t   c  
put q  e
  e  
H   c .  P  A    mc 2  e 
  c  
Q65. A particle of energy E scatters off a repulsive spherical potential
V for r  a
V r    0
 0 for r  a
where V0 and a are positive constants. In the low energy limit, the total scattering cross-
2
 1  2m
section is   4 a 2  tanh ka  1 , where k 2  2 V0  E   0 . In the limit V0  
 ka  h
the ratio of  to the classical scattering cross-section off a sphere of radius a is
1
(a) 4 (b) 3 (c) 1 (d)
2
Ans. : (a)

H.No. 40-D, Ground Floor, Jia Sarai, Near IIT, Hauz Khas, New Delhi-110016
Phone: 011-26865455/+91-9871145498
Website: www.physicsbyfiziks.com | Email: fiziks.physics@gmail.com
217
fiziks
Institute for NET/JRF, GATE, IIT‐JAM, M.Sc. Entrance, JEST, TIFR and GRE in Physics

2
1 
Solution:   4 a  tanh ka  1
2

 ka 
2
 1 
ka   , tanh ka  1    4 a   1 2

 ka 
and ka   , lim  H  4 a 2
ka 

H
classically  c   a 2  4
c

NET/JRF (DEC-2015)

Q66. A Hermitian operator O has two normalized eigenstates 1 and 2 with eigenvalues 1

and 2 , respectively. The two states u  cos  1  sin  2 and v  cos  1  sin  2

are such that v O   7 / 4 and u v  0 . Which of the following are possible values of

 and  ?
   
(a)    and   (b)   and  
6 3 6 3
   
(c)    and   (d)   and   
4 4 3 6
Ans. : (a)
Solution: u  cos  1  sin  2 , v  cos  1  sin  2

7
it is given Oˆ 1  1 , Oˆ 2  2 2  v Oˆ v 
4
7 7
cos 2   2sin 2    cos 2   sin 2   1  sin 2    1
4 4
3 
sin    
2 3
u v  0  cos  cos   sin  sin   0  cos      0

      5 
    or        or      or   
2 2 2 3 3 2 6 6

H.No. 40-D, Ground Floor, Jia Sarai, Near IIT, Hauz Khas, New Delhi-110016
Phone: 011-26865455/+91-9871145498
Website: www.physicsbyfiziks.com | Email: fiziks.physics@gmail.com
218
fiziks
Institute for NET/JRF, GATE, IIT‐JAM, M.Sc. Entrance, JEST, TIFR and GRE in Physics

x
Q67. The ground state energy of a particle of mass m in the potential V  x   V0 cosh   ,
L
2
where L and V0 are constants (with V0  ) is approximately
2mL2

 2V0  V0  V0  V0
(a) V0  (b) V0  (c) V0  (d) V0 
L m L m 4L m 2L m
Ans. : (d)
x V
Solution: 
V0  cosh    0 e x / L  e  x / L
L 2

V  x 1  x 2  V0  x 1  x 2 
 0 1     ....  1      ....
2  L 2!  L   2  L 2!  L  
2
V V V x 1 V 
 0  0  0    V0   02  x 2
2 2 2 L 2 L 

V0 V0
K , 
L2 mL2
So, ground state energy is

  V0  V0
V0   V0  2
 V0 
2 2 mL 2L m
Q68. Let  nlm denote the eigenstates of a hydrogen atom in the usual notation. The state

1
2 200  3 211  7 210  5 211 
5
is an eigenstate of
(a) L2 , but not of the Hamiltonian or Lz (b) the Hamiltonian, but not of L2 or Lz

(c) the Hamiltonian, L2 and Lz (d) L2 and Lz , but not of the Hamiltonian
Ans. : (b)
1
Solution:    2 200  3 211  7 2 10  5 2 1 1 
5
13.6
H   
4
So  is eigen state of H

H.No. 40-D, Ground Floor, Jia Sarai, Near IIT, Hauz Khas, New Delhi-110016
Phone: 011-26865455/+91-9871145498
Website: www.physicsbyfiziks.com | Email: fiziks.physics@gmail.com
219
fiziks
Institute for NET/JRF, GATE, IIT‐JAM, M.Sc. Entrance, JEST, TIFR and GRE in Physics

But L2     and Lz    

So  is not eigen state of L2 and Lz

1
Q69. The Hamiltonian for a spin- particle at rest is given by H  E0  z   x  , where  x
2
and  z are Pauli spin matrices and E0 and  are constants. The eigenvalues of this
Hamiltonian are

(a)  E0 1   2 (b)  E0 1   2

 1 
(c) E0 (doubly degenerate) (d) E0 1   2 
 2 
Ans. : (a)
 1 0   0 1  1 
Solution: H  E0  z   x   E0        H  E0  
  0 1   1 0   1 

if  is eigen value, then


 1     
H   I  0  E0  0,    E0 1   2
   1    

Q70. A hydrogen atom is subjected to the perturbation


2r
V pert  r   cos
a0

where a0 is the Bohr radius. The change in the ground state energy to first order in 

   
(a) (b) (c) (d)
4 2 2 4
Ans. : (d)
Solution: For First order perturbation
r
1  2r 
E11  100 V p 100 , 100  e a , V p  cos  
 a03  a0 
 2 r  2 r
1  2r  4  2r 
E   3 e a0  cos   4 r 2 dr  3  e a0 cos   r 2 dr
1
1
0
 a0  a0  a0 0  a0 

H.No. 40-D, Ground Floor, Jia Sarai, Near IIT, Hauz Khas, New Delhi-110016
Phone: 011-26865455/+91-9871145498
Website: www.physicsbyfiziks.com | Email: fiziks.physics@gmail.com
220
fiziks
Institute for NET/JRF, GATE, IIT‐JAM, M.Sc. Entrance, JEST, TIFR and GRE in Physics

 ia2 r i 2 r

2   a0 1i  2 
 2 r  2 r  2 r 1 i 
4 e  e 0
0 a
 2
 3 e
a03  0 0
a0
  r dr   e r dr  e a0
r 2
dr 
a0 0

2
 

 
 
2 2! 2!   1 1 
     
a03   2 
3
 
3
2  1  i 3 1  i 3 
 1  i   1  i  
2  
  a0   0
a  

 
   
 1 1    1  1 
 
2 3
3  1 i  
3
4 2 4
 i 3 i 3

   1 i 
 
3
 2   2     e e 4

  2  2  

  i 34  i 3
    3  
  e  e 4
  2 cos  4  
4 2   4 2   

   1   
 2      E11 
4 2   2  4 4

Q71. The product of the uncertainties  Lx   Ly  for a particle in the state a 1,1  b 1, 1

where l , m denotes an eigenstate of L2 and Lz will be a minimum for

(a) a  ib (b) a  0 and b  1

3 1
(c) a  and b  (d) a  b
2 2
Ans. : (d)
Solution:   a 1,1  b 1, 1 , L   2b 1, 0 , L2   2 2b 1,1

L   2a 1, 0 , L2   2 2 a 1, 1

 L2   a 2 2  b 2 2  a  b 2 2 2
2 2
 2


 L2z   a  b 2  2
2

Lx  0, Ly  0

L2x 
1  2
4 
L  L2
  2 L2
 L2
2  4 
 
  1  a*b  b*a 2 2  2 2 2   2    a 2
b
2

H.No. 40-D, Ground Floor, Jia Sarai, Near IIT, Hauz Khas, New Delhi-110016
Phone: 011-26865455/+91-9871145498
Website: www.physicsbyfiziks.com | Email: fiziks.physics@gmail.com
221
fiziks
Institute for NET/JRF, GATE, IIT‐JAM, M.Sc. Entrance, JEST, TIFR and GRE in Physics

2  *
L2x 
2 

a b  b* a  a  b 
2 2
 
L2y 
 
2 L2  L22  L2  L2
4
  2
2
L2y 
2 
a  b  a*b  b*a 
2
  
2  2
    a b  b a  
2 2
Lx Ly   a b  a  b 1
2 * * 2 2

2 
2
 
2
Lx Ly  1  a*b  b*a (i)
2
1 i 2
Now check option (a) a  ib  a  ,b   Lx Ly 
2 2 2
 2
Option (b) a  0, b  1  Lx Ly 
2
3 1 2
Option (c) a  ,b   Lx Ly 
2 2 4
1 1
Option (d) a  b  a  ,b   Lx Ly  0 option (d) is correct
2 2
Q72. The ground state energy of a particle in potential V  x   g x , estimated using the trail
wavefunction
 c 2

  x    a5
a  x2 ,   x a
0, x a

(where g and c are constants) is
1/ 3 1/ 3 1/ 3 1/ 3
15   2 g 2  5  2 g 2  3  2 g 2  7  2 g 2 
(a)   (b)   (c)   (d)  
16  m  6 m  4 m  8 m 
Ans. : (a)
a
15
   dx  1  c  16
*
Solution:
a

 2  15  2 2
a
10 2
T 
2m  16a 2  a
   a 2
 x 2
 x 2
 a  x 2
 dx  T 
4ma 2

15  2 g
a
V 
16a 05   5
x a 2  x 2 dx  V  ga
16

E T  V (i)

H.No. 40-D, Ground Floor, Jia Sarai, Near IIT, Hauz Khas, New Delhi-110016
Phone: 011-26865455/+91-9871145498
Website: www.physicsbyfiziks.com | Email: fiziks.physics@gmail.com
222
fiziks
Institute for NET/JRF, GATE, IIT‐JAM, M.Sc. Entrance, JEST, TIFR and GRE in Physics

10 2 5 ga
E 
4ma 2 16
1
dE 8  2  3
 0  a3   a  2 
da mg  mg 
put the value of a in equation (i)
1
15   2 g 2  3
E  
16  m 

NET/JRF (JUNE-2016)
Q73. The state of a particle of mass m in a one dimensional rigid box in the interval 0 to L is

2  3  2 x  4  4 x  
given by the normalized wavefunction   x    sin    sin    . If its
L  5  L  5  L 
energy is measured the possible outcomes and the average value of energy are,
respectively
h 2 2h 2 73 h 2 h2 h2 19 h 2
(a) , and (b) , and
2mL2 mL2 50 mL2 8mL2 2mL2 40 mL2
h 2 2h 2 19 h 2 h 2 2h 2 73 h 2
(c) , and (d) , and
2mL2 mL2 10 mL2 8mL2 mL2 200 mL2
Ans. : (a)

2  3  2 x  4  4 x  
Solution:   x    sin    sin  
L  5  L  5  L 
n 2 2  2
Measurement E 
2mL2
h2 2h 2
 n  2  E2  and n  4  E4 
2mL2 mL2
9 16
Probability p  E2   and p  E4  
25 25
Now, average value of energy is
9 h2 16 2h 2 73h 2
E   an p  an      
25 2mL2 25 mL2 50mL2

H.No. 40-D, Ground Floor, Jia Sarai, Near IIT, Hauz Khas, New Delhi-110016
Phone: 011-26865455/+91-9871145498
Website: www.physicsbyfiziks.com | Email: fiziks.physics@gmail.com
223
fiziks
Institute for NET/JRF, GATE, IIT‐JAM, M.Sc. Entrance, JEST, TIFR and GRE in Physics

Q74. If Lˆ x , Lˆ y , Lˆ z are the components of the angular momentum operator in three dimensions
the commutator  Lˆ x , Lˆ x Lˆ y Lˆ z  may be simplified to

 
(a) iLx Lˆ2z  Lˆ2y (b) iLˆ z Lˆ y Lˆ x

(c) iL  2 Lˆ  Lˆ 
x
2
z
2
y (d) 0

Ans. : (a)
Solution:  Lx , Lx Ly Lz   Lx  Lx , Ly Lz    Lx , Lx  Ly Lz

 Lx  Lx , Ly  Lz  Lx Ly  Lx , Lz   0  Lx iLz  Lz  Lx Ly  iLy 


 iLx L2z  iLx L2y  iLx L2z  L2y 
Q75. Suppose that the Coulomb potential of the hydrogen atom is changed by adding an
 Ze 2 g
inverse-square term such that the total potential is V  r     2 , where g is a
r r
constant. The energy eigenvalues Enlm in the modified potential
(a) depend on n and l , but not on m
(b) depend on n but not on l and m
(c) depend on n and m , but not on l
(d) depend explicitly on all three quantum numbers n , l and m
Ans. : (b)
ze 2 g
Solution: V  r     2 is central potential
r r
So angular momentum is conserve then eigen value En ,l ,m will depend only on n , which

is principal quantum number.


Q76. The eigenstates corresponding to eigenvalues E1 and E2 of a time independent

Hamiltonian are 1 and 2 respectively. If at t  0 , the system is in a state

  t  0   sin  1  cos  2 , then the value of   t    t  at time t will be

(a) 1 (b)
 E sin
1
2
  E2 cos 2  
E12  E22

(c) eiE1t /  sin   eiE2t /  cos  (d) e iE1t /  sin 2   eiE2t /  cos 2 
Ans. : (a)

H.No. 40-D, Ground Floor, Jia Sarai, Near IIT, Hauz Khas, New Delhi-110016
Phone: 011-26865455/+91-9871145498
Website: www.physicsbyfiziks.com | Email: fiziks.physics@gmail.com
224
fiziks
Institute for NET/JRF, GATE, IIT‐JAM, M.Sc. Entrance, JEST, TIFR and GRE in Physics

Solution:   t  0   sin  1  cos  2


 iE1t  iE2t
  t   sin  1 e 
 cos  2 e 

 i  E1  E2 t

  t    t   sin  1 1  cos  2 2  2 Re e
2 2 
sin   cos  1 2

 sin 2   cos 2   0  1  1 2  0

1
Q77. Consider a particle of mass m in a potential V  x   m 2 x 2  g cos kx . The change in
2
1
the ground state energy, compared to the simple harmonic potential m 2 x 2 , to first
2
order in g is

 k 2   k 2   2k 2    k 2 
(a) g exp    (b) g exp   (c) g exp    (d) g exp   
 2m   2m   m   4m 
Ans. : (d)
Solution: Ground state wavefunction
1
m x 2
 m  4 
 0  x    e
2

  
The perturbation term is H p  g cos kx

First order correction E    0*  x  H P 0  x  dx
1
0


1 1
 m  2
 m x 2
 eikx  e  ikx  g  m  2    m x ikx   m x 
2 2

 ge   e  .e dx   e  . e dx 
 ikx
  dx  

 
      2  2       
1 1
m x  m x 
g  m  2 g  m  2
2 2
  ikx ikx
    dx    
 
e e dx
2      2     
From 1st term, we have
m  2 2 ikx  ik    ik  
1 2 2 1
m 
2
  ik  
g  m  2  x     
2 m  2 m   2 m 

g  m  2   x  
k 2

e  
     2 m 
   dx   e e 4 m
dx
2    
2     

H.No. 40-D, Ground Floor, Jia Sarai, Near IIT, Hauz Khas, New Delhi-110016
Phone: 011-26865455/+91-9871145498
Website: www.physicsbyfiziks.com | Email: fiziks.physics@gmail.com
225
fiziks
Institute for NET/JRF, GATE, IIT‐JAM, M.Sc. Entrance, JEST, TIFR and GRE in Physics

1
m 
2
 ik  
g  4km  m  2
2
k 2
 x  
 e
  2 m 
 e  dx  e 4 m
2     
1
m x 
g  m  2
2
 ikx
 

Similarly, from term (ii),  e dx
2     
1
m 
2
 ik  
g  k h  m  2
2
 k 2
 x  
 e
  2 m 
 e 4 m  dx  e 4 m
2     

g  k  k  
2 2 2
k 
 
Hence, E   e 4 m  e 4 m   ge 4 m
1
0
2  
Q78. The energy levels for a particle of mass m in the potential V  x    x , determined in

the WKB approximation

 1
b
2m  E  V  x dx   n   
a  2

(where a, b are the turning points and n  0,1, 2... ), are


2 2
 h  1  3  3h  1  3
(a) En    n   (b) En    n  
4 m  2   4 2m  2 
2 2
 3h  1  3  h  1  3
(c) En      (d) En     
2   2  
n n
4 m   4 2m 
Ans. : (b)
Solution: V  x    x
V  x
  x, x0
 V  x  
   x, x0
E E
b
 1 
2m  E  V  x dx   n      
a  2 x
E

 E E  1
From figure, a     , b     2m  E  V  x dx   n    
    
E  2

H.No. 40-D, Ground Floor, Jia Sarai, Near IIT, Hauz Khas, New Delhi-110016
Phone: 011-26865455/+91-9871145498
Website: www.physicsbyfiziks.com | Email: fiziks.physics@gmail.com
226
fiziks
Institute for NET/JRF, GATE, IIT‐JAM, M.Sc. Entrance, JEST, TIFR and GRE in Physics

E E
 
 1  1
0
 2m  E   xdx   E   xdx   n      2 2m  E   x  dx    n    

E 0  2 0  2

dt
put E   x  t , dx  

E
limit x  0  t  E , x t 0

  dt   1
0
2 2m  t     n   
E     2
0
2 2m  2 32   1 2 2m 2 32  1
  t    n      .E   n    h
  3 E  2  3  2
2
 1  3 
3
 3  1  3
 E  n 
2
 En    n  
 2  4 2m  4 2m  2 
Q79. A particle of mass m moves in one dimension under the influence of the potential
V  x     x  , where  is a positive constant. The uncertainty in the product

 x  p  in its ground state is

 
(a) 2 (b) (c) (d) 2
2 2
Ans. : (c)
Solution: V  x     x 

For this potential wavefunction


  x
  e x , x0
  x  
 x
  e , x0

which evenfunction about x  0


so x  0, p  0 x

1 1
now x 2  2  x 2 e 2 x dx   x  x2  x 
2

0
2 2
2

H.No. 40-D, Ground Floor, Jia Sarai, Near IIT, Hauz Khas, New Delhi-110016
Phone: 011-26865455/+91-9871145498
Website: www.physicsbyfiziks.com | Email: fiziks.physics@gmail.com
227
fiziks
Institute for NET/JRF, GATE, IIT‐JAM, M.Sc. Entrance, JEST, TIFR and GRE in Physics

 0 
d2 x d
2
 x d
2
p 2  2   * 2
 dx    2
  e 2
 ex
dx   2
  e 2
 e x dx

dx 
dx 0
dx

 2 3  2 3
0

e e    2 2 , which is not possible


2 x 2 x
   2 3
dx   
2 3
dx   
 0
2 2
 2
d
  dx   2 2 , p  p2  p  
2 2 2
so, we will use the formula p

dx
1 
now, x.p  . 
2 2
2  4
Q80. The ground state energy of a particle of mass m in the potential V  x   x ,
6m
1
 x 2
  4
estimated using the normalized trial wavefunction   x     e 2
, is
 

  1   3
 dxx 2 e  x   dx x 4 e  x 
2 2
[use and ]
  2   4 2

3 2 13 8 2 13 2 2 13 3 2 13
(a)  (b)  (c)  (d) 
2m 3m 3m 8m
Ans. : (d)
1
x
  4   2
2

Solution: E  T  V , for   x     e 2 , T 
  4m
1 1
 
   2   4  x 2   2   2  3 2 
2 2
V    x e dx     x 4 e  x dx  . 2 
2

    6m    6m 
6m 4 8m 2

 2 2 
E   (i)
4m 8m 2
dE  2 2 2  2    1
   0   1    0       3
d 4m 8m 3 4m   3 
Putting the value of  in equation (i),
 1

2 1

2
  2 1
   3
 3 2 13
E     3    
3  
2
4m  2  8m
8m    3
4m
 

H.No. 40-D, Ground Floor, Jia Sarai, Near IIT, Hauz Khas, New Delhi-110016
Phone: 011-26865455/+91-9871145498
Website: www.physicsbyfiziks.com | Email: fiziks.physics@gmail.com
228
fiziks
Institute for NET/JRF, GATE, IIT‐JAM, M.Sc. Entrance, JEST, TIFR and GRE in Physics

NET/JRF (DEC-2016)
Q81. Consider the two lowest normalized energy eigenfunctions  0  x  and  1  x  of a one

d 0
dimensional system. They satisfy  0  x    0*  x  and  1  x    , where  is a real
dx
constant. The expectation value of the momentum operator in the state  1 is

  2
(a)  (b) 0 (c) (d)
 2
 2
2
Ans. : (b)
d 0
Solution:  1  x   
dx
  
   d 0 d 2
px    px dx    1  i 1  dx    *  i  20 dx
*
*

x 
1
   
dx dx

d 0 d 2 0
 i   
2
dx

dx dx 2
Integrate by parts

2  d d 0 d 2 0 d 0 
  
d 0 d 2 0
I  i   0   dx 2 dx dx   0   i    dx dx 2 dx
2

 dx dx
  

d 0 d 2 0
I  0   i    dx dx 2 dx
2



d 0 
 0 , 0  0, x  
dx 
I  0  I  2I  0  I  0  px  0

d
Q82. Consider the operator, a  x  acting on smooth function of x . Then commutator
dx
 , cos x  is
(a)  sin x (b) cos x (c)  cos x (d) 0
Ans. : (a)
d
Solution: a  x 
dx

H.No. 40-D, Ground Floor, Jia Sarai, Near IIT, Hauz Khas, New Delhi-110016
Phone: 011-26865455/+91-9871145498
Website: www.physicsbyfiziks.com | Email: fiziks.physics@gmail.com
229
fiziks
Institute for NET/JRF, GATE, IIT‐JAM, M.Sc. Entrance, JEST, TIFR and GRE in Physics

 a, cos x    x   d  d 
d
, cos x    x, cos x    , cos x   0   , cos x 
 dx   dx   dx 
d  d d
 dx , cos x   x   dx cos x  x   cos x dx

d cos xd
 cos x    sin x     sin x
dx dx
 a, cos x   x    sin x
 a, cos x    sin x
   
Q83. Consider the operator   p  qA , where p is the momentum operator,

A   Ax , Ay , Az  is the vector potential and q denotes the electric charge. If

B   Bx , By , Bz  denotes the magnetic field, the z -component of the vector operator
 
   is
(a) iqBz  q  Ax p y  Ay px  (b) iqBz  q  Ax p y  Ay px 

(c) iqBz (d) iqBz


Ans. : (d)
  
Solution:   p  qA
       
 
    
 
   

 p  qA  p  qA   p  p  qp  A  qA  p  q 2 A  A
 
p  p  0
    

 qp  A   q i  A   qiB 
   
 
qA  p  q A i   0 
 
q 2 A  A  0
  
    qiB
So, z component is given by qiBz

H.No. 40-D, Ground Floor, Jia Sarai, Near IIT, Hauz Khas, New Delhi-110016
Phone: 011-26865455/+91-9871145498
Website: www.physicsbyfiziks.com | Email: fiziks.physics@gmail.com
230
fiziks
Institute for NET/JRF, GATE, IIT‐JAM, M.Sc. Entrance, JEST, TIFR and GRE in Physics

Q84. The dynamics of a free relativistic particle of mass m is governed by the Dirac
   
Hamiltonian H  c . p   mc 2 , where p is the momentum operator and    x ,  y ,  z 

and  are four 4  4 Dirac matrices. The acceleration operator can be expressed as
2ic   
(a)  cp   H  (b) 2ic 2

ic  2ic  
(c) H (d)   cp   H 
 
Ans. : (a)
 
Solution: H  c . p   mc 2
If vx velocity of x direction
From the Ehrenfest theorem
dx 1 x 1
vx    x, H     x, c x px  c y p y  c z p z   mc 2   0
dt i t i 
c
  x,  x px   c x
i
Similarly, acceleration is given by
dvx 1 c
ax    c x , H    x , c x px  c y p y  c z pz   mc 2 
dt i i
Using relation  i j   j i  0 ,  i    i  0 and  i , p j   0

 x , c x px   0
 
 x , c y p y   c  x y   y x  p y   y c  x , p y   c x y  c x y  p y  0  2c x y p y
 
 x , c z pz   c x z  c z x  pz   z c x , pz   c x z   c x z  pz  0  2c x z pz
 x ,  mc 2    x    x  mc 2  2mc 2 x 
 
c
ax   2c x y p y  2c x z p z  2 x  mc 2 
i  

2 x c
ax  c y p y  c z pz   mc 2  c x p x  c x px 
i  

2 x c
ax  c x p x  c y p y  c z pz   mc 2  c x px 
i  

H.No. 40-D, Ground Floor, Jia Sarai, Near IIT, Hauz Khas, New Delhi-110016
Phone: 011-26865455/+91-9871145498
Website: www.physicsbyfiziks.com | Email: fiziks.physics@gmail.com
231
fiziks
Institute for NET/JRF, GATE, IIT‐JAM, M.Sc. Entrance, JEST, TIFR and GRE in Physics

ax 
2c
i
2ic
 x .H  c x x px   c x x px   x .H  ,

 2
x x 
  2ic    
a  ax iˆ  a y ˆj  az kˆ  
 
cp   .H 

 
Q85. A particle of charge q in one dimension is in a simple harmonic potential with angular
2
t
 
frequency  . It is subjected to a time- dependent electric field E  t   Ae  
, where A

and  are positive constants and   1 . If in the distant past t   the particle was
in its ground state, the probability that it will be in the first excited state as t   is
proportional to
1 1
  2  2 1
2
(a) e (b) e 2 (c) 0 (d)
 
2

Ans. : (a)
2
t2
  i fi t
Solution: Transition probability is proportional to Pif   e  2
e where


3 1
  
 fi  2 2 

2
 t2
Pif   exp  it dt
 2
1  2 2 2 
i 2  
2
 t2   
2  i 
Now calculate  exp   2  it  dt   exp  2 t  it     
      
  2   2  

  2 2   1  it 
2
 exp     exp 2 
t   dt
 4     2 
2
 t2
Pif   exp  it dt
 2
2
  2 2   1  it 
2
Pif  exp     exp 2  t   dt
 4     2 
 2 2
Pif  exp 
2

H.No. 40-D, Ground Floor, Jia Sarai, Near IIT, Hauz Khas, New Delhi-110016
Phone: 011-26865455/+91-9871145498
Website: www.physicsbyfiziks.com | Email: fiziks.physics@gmail.com
232
fiziks
Institute for NET/JRF, GATE, IIT‐JAM, M.Sc. Entrance, JEST, TIFR and GRE in Physics

Q86. A particle is scattered by a central potential V  r   V0 re   r , where V0 and  are positive


 
constants. If the momentum transfer q is such that q  q   , the scattering cross-

section in the Born approximation, as q   , depends on q as

dn
 x e dx  n 
n ax
[You may use e ax dx ]
da
(a) q 8 (b) q 2 (c) q 2 (d) q 6
Ans. : (a)
Solution: The form factor is given for high energy as q  
 
2 m 2m
f  ,     rV  r  sin qr dr  2  r 2V0 e   v sin qr dr
q 0
2
q 0

mV0  2  r   iq  
  
2 m 2  r e
iqr
 e  iqr
 2 V0  r e dr  2 i   r e dr   r 2 e   dr 
 r   iq

q 0 2i  q 0 0 

mV0i 
 2 
2

2
 2 0 

 2mV i     iq      iq 
3 3
 
 q     iq 3    iq 3   q     iq 3    iq 3 
   

2mV0 i    iq  3 iq  3 q      iq  3 iq  3 q  
3 3 2 2 3 3 2 2

 2
q   2  q2 
3

   3 2 
2mV0i  6  2iq  2iq 3  2mV0  2q  6  q 
 2  2
 q    2  q 2 3 
  
q   2  q2 3 
 

q3  6 2  1 1 1  2 
  2  2  3
 q2  6
 4  2  1
q q   2  q q  q 
q  2  1
6

q 
    f     q 4   q 8
2 2

Q87. A particle in one dimension is in a potential V  x   A  x  a  . Its wavefunction   x  is


d
continuous everywhere. The discontinuity in at x  a is
dx
(b) A   a    a  
2m
(a) A  a 
2
2
(c) A (d) 0
2m
H.No. 40-D, Ground Floor, Jia Sarai, Near IIT, Hauz Khas, New Delhi-110016
Phone: 011-26865455/+91-9871145498
Website: www.physicsbyfiziks.com | Email: fiziks.physics@gmail.com
233
fiziks
Institute for NET/JRF, GATE, IIT‐JAM, M.Sc. Entrance, JEST, TIFR and GRE in Physics

Ans. : (a)

2 d   x 
2

Solution:   A  x  a   x   E  x 
2m dx 2
Integrates both side within limit
a   to a 
a  a  a 
2 d 2
dx   A   x  a  dx  E    x  dx
2m a dx 2

a  a 

 2  d II d I 
     A  a   0
2m  dx dx 
d II d I 2mA
  2  a
dx dx 
d 2mA
so discontinues in at x  a is 2   a  .
dx 

NET/JRF (JUNE-2017)
Q88. If the root-mean-squared momentum of a particle in the ground state of a one-
dimensional simple harmonic potential is p0 , then its root-mean-squared momentum in
the first excited state is
(a) p0 2 (b) p0 3 (c) p0 2 / 3 (d) p0 3 / 2
Ans. : (b)

Solution: P  m  Pˆ  m 
 a  a† 
2i
m  2
P2  
2
 a  a†2   2 N  1 
m 
P2  
2
a 2  a †2  2 N  1 
For any state n ,

a 2  0, a †2  0 and 2 N  1  2n  1

m 
P 2   2n  1 and P  0
2

H.No. 40-D, Ground Floor, Jia Sarai, Near IIT, Hauz Khas, New Delhi-110016
Phone: 011-26865455/+91-9871145498
Website: www.physicsbyfiziks.com | Email: fiziks.physics@gmail.com
234
fiziks
Institute for NET/JRF, GATE, IIT‐JAM, M.Sc. Entrance, JEST, TIFR and GRE in Physics

m 
Prms  P2  P  Prms  2n  1
2

m 
For ground stat n  0, Prms   P0
2

m 
So, for n  1, Prms  3
2

Prms  3P0

Q89. Consider a potential barrier A of height V0 and width b , and another potential barrier B

of height 2V0 and the same width b . The ratio TA / TB of tunnelling probabilities TA and

TB , through barriers A and B respectively, for a particle of energy V0 /100 is best


approximated by

 
(a) exp  1.99  0.99  8 mV0b 2 /  2 
 
(b) exp  1.98  0.98
  8 mV0b 2 /  2 

(c) exp 
  2.99  0.99  8 mV0b 2 /  2 

(d) exp 
  2.98  0.98  8 mV0b 2 /  2 

Ans. : (a)
 2 m V  E  V0
Solution: T  e , where E 
100
For potential A, V  V0
2m  V0  2 m  99 
  V0     V0 
 2 m 0.99V0 
TA e 2  100 
 TA e  2  100 
e
V0
For Potential B, V  2V0 and E =
100
2m  V0  2 m  199V0 
  2V0       2 m 1.99V0 
TB e 2  100 
 TB e  2  100 
 e
 0.99V0
TA e

TB e  1.99V0

TA
TB
 e  1.99V0
e
 0.99V 0

H.No. 40-D, Ground Floor, Jia Sarai, Near IIT, Hauz Khas, New Delhi-110016
Phone: 011-26865455/+91-9871145498
Website: www.physicsbyfiziks.com | Email: fiziks.physics@gmail.com
235
fiziks
Institute for NET/JRF, GATE, IIT‐JAM, M.Sc. Entrance, JEST, TIFR and GRE in Physics

Q90. A constant perturbation H  is applied to a system for time  t (where H  t   )


leading to a transition from a state with energy Ei to another with energy E f . If the time

of application is doubled, the probability of transition will be


(a) unchanged (b) doubled (c) quadrupled (d) halved
Ans. : (c)
Solution: For constant potential transition probability
2
 f v i  2  fi ti 
pif  4  sin 
h 2 2
fi  2 

at ti  2ti ,
2
4  f v i  fi ti
pif  sin 2
h 2 2
fi 2

at ti  2ti ,
2 2
4  f v i   fi 2ti  4  f v  i
p ff  2
sin   sin  fi ti 
h 2 2fi  2  h 2 2fi

sin 2  fi ti 
 2fi ti2
pif sin  fi ti 
2
 t 2 2

  fi i
t1  0
p ff  t    t   t 
2
sin  fi i 
2
sin 2  fi i   fi i 
 2   2  2 
 2fi ti2
2
4 2fi ti2
 4
 2fi ti2

pif (2)
 4  pif (2)  4 pif (1)
pif (1)

a b
Q91. The two vectors   and   are orthonormal if
0 c
(a) a  1, b  1/ 2, c  1/ 2 (b) a  1, b  1, c  0
(c) a  1, b  0, c  1 (d) a  1, b  1/ 2, c  1/ 2

H.No. 40-D, Ground Floor, Jia Sarai, Near IIT, Hauz Khas, New Delhi-110016
Phone: 011-26865455/+91-9871145498
Website: www.physicsbyfiziks.com | Email: fiziks.physics@gmail.com
236
fiziks
Institute for NET/JRF, GATE, IIT‐JAM, M.Sc. Entrance, JEST, TIFR and GRE in Physics

Ans. : (c)
a b
Solution: 1    , 2   
0 c
1 1  1  a  1

2 2  1  b  c 1
2 2

1 2  0   a 0   bc   0

a.b  0  c  0  a  b  0
so b  0

c  1, c  1
2

a  1, b  0, c  1
Q92. Consider the potential
  
V  r    i V0 a 3    r  ri 
3


where ri are the position vectors of the vertices of a cube of length a centered at the

2
origin and V0 is a constant. If V0 a 2  , the total scattering cross-section, in the low-
m
energy limit, is
2
 mV0 a 2 
2 16a 2  mV0 a 2 
(a) 16a   (b)  
  
2
 2  2 
2
64a 2  mV0 a 2  64a 2  mV0 a 2 
(c)   (d)  
  2   2  2 
Ans. : (c)
 
Solution: V  r    V0 a 3 3  r  ri 
i

  V0 a 3  x  xi    y  yi    z  zi 
i

where xi , yi , zi are co-ordinate at 8 corner cube whose center is at origin.

m
f     2 
V  r d 3 r
2 

H.No. 40-D, Ground Floor, Jia Sarai, Near IIT, Hauz Khas, New Delhi-110016
Phone: 011-26865455/+91-9871145498
Website: www.physicsbyfiziks.com | Email: fiziks.physics@gmail.com
237
fiziks
Institute for NET/JRF, GATE, IIT‐JAM, M.Sc. Entrance, JEST, TIFR and GRE in Physics


m 8
       x  xi    y  yi   z  zi dxdydz
3
V a
2  2
0
i 1

m
 V a 3 1  1  1  1  1  1  1  1
2  2 0

8mV0 a 3 4mV0 a 3
 
2  2  2

total scattering cross section    f   sin  d d .


2

16m 2V02 a 6
Differential scattering cross section D    f   
2

 24
16m 2V02 a 6 64a 2  m 2V02 a 4 
 4   
 24   h4 
2
64a 2  mV0 a 2 
  
  2 

Q93. The Coulomb potential V  r   e 2 / r of a hydrogen atom is perturbed by adding

H   bx 2 (where b is a constant) to the Hamiltonian. The first order correction to the


ground state energy is
1
(The ground state wavefunction is  0  e  r / a0 )
a 3
0

(a) 2ba02 (b) ba02 (c) ba02 / 2 (d) 2ba02


Ans. : (b)
Solution: H '  bx 2 put x  r sin  cos 

H '  br 2 sin 2  cos 2  .


1
E11   1 H   1 ,  1  e  r / a0
a 3
0

  1* H  1r 2 sin  dr d d


2r  2
b
 3  r 2 e a0 r 2 dr  sin 3  d  cos 2  d  ba02
 a0 0 0 0

H.No. 40-D, Ground Floor, Jia Sarai, Near IIT, Hauz Khas, New Delhi-110016
Phone: 011-26865455/+91-9871145498
Website: www.physicsbyfiziks.com | Email: fiziks.physics@gmail.com
238
fiziks
Institute for NET/JRF, GATE, IIT‐JAM, M.Sc. Entrance, JEST, TIFR and GRE in Physics

Q94. Using the trial function


 A  a 2  x 2  , a  x  a
  x  
 0 otherwise

the ground state energy of a one-dimensional harmonic oscillator is

5 1 5
(a)  (b)  (c)  (d) 
14 2 7
Ans. : (b)

 A  a  x  , a  x  a
2 2

Solution:   x   
0 , otherwise

For normalization

  dx  1
*

15 15
A2  5
 A
16a 16a 5
 2 2   2 15
a a
T    * 2  dx   2  2    a 2  x 2 dx
5 
2m  a x 2m 16a 0

5 2
T 
4ma 2
a a
2 x 2  a 2  x 2  dx.
1 1 15
V    *V dx , where V  x   5 
2
m 2 x 2  m 2
a
2 2 16a 0

m 2 a 2
V 
14
5 2 m 2 a 2
E  T V  
4ma 2 14

dE 5   2   2 m 2 a 35   2 
0   0  a   2 2 .
4

da 4ma 3 7 2 m  
1/ 2
 35    
a  
2
 .
 2   m 

5  2 m 2 m 2 35 
E  .  .
4 m  35 14 2 m

H.No. 40-D, Ground Floor, Jia Sarai, Near IIT, Hauz Khas, New Delhi-110016
Phone: 011-26865455/+91-9871145498
Website: www.physicsbyfiziks.com | Email: fiziks.physics@gmail.com
239
fiziks
Institute for NET/JRF, GATE, IIT‐JAM, M.Sc. Entrance, JEST, TIFR and GRE in Physics

  5 2 1 35    5 5  5
         
2  2 35 7 2  2  14 14  14

NET/JRF (DEC - 2017)


Q95. Let x denote the position operator and p the canonically conjugate momentum operator
of a particle. The commutator
 1 2 2 1 2
 2m p   x , m p   x 
2

where  and  are constants, is zero if

(a)    (b)   2 (c)   2  (d) 2  


Ans. : (b)
 1 2 1  1 
Solution:  p   x2 , p2   x2   0    p 2 , x 2    x 2 , p 2   0
 2m m  2m m
  1   
  x 2 , p 2    x 2 , p 2   0   x 2 , p 2       0    2
2m m m  2 
Q96. The normalized wavefunction of a particle in three dimensions is given by
1
  r , ,   e  r / 2 a where a  0 is a constant. The ratio of the most probable
8 a 3

distance from the origin to the mean distance from the origin, is

[You may use  dx x n e  x  n ! ]
0

1 1 3 2
(a) (b) (c) (d)
3 2 2 3
Ans. : (d)
r
1
Solution:   r , ,    e 2a
8 a 3
3
 2a   3a
r   r * r 2 dr sin  d d 
2
one can compare the wave function at hydrogen atom with Bohr radius a0  2a
most probable distance,

H.No. 40-D, Ground Floor, Jia Sarai, Near IIT, Hauz Khas, New Delhi-110016
Phone: 011-26865455/+91-9871145498
Website: www.physicsbyfiziks.com | Email: fiziks.physics@gmail.com
240
fiziks
Institute for NET/JRF, GATE, IIT‐JAM, M.Sc. Entrance, JEST, TIFR and GRE in Physics

d 2 r / a
r e 0
dr
rP  2a
rp 2a 2
 
r 3a 3

Q97. The state vector of a one-dimensional simple harmonic oscillator of angular frequency  ,
1
at time t  0 , is given by   0    0  2  , where 0 and 2 are the normalized
2
ground state and the second excited state, respectively. The minimum time t after which
the state vector   t  is orthogonal to   0  , is

 2  4
(a) (b) (c) (d)
2   
Ans. : (a)
1
Solution:   0    0  2 
2
  t 5  t
5 1  
E2    t    0 e 2
 2 e 2 

2 2  
 
E0     0    t   0  t  cos 1  1
2 E2  E0
  
t cos 1  1   .
 5 1  2 / 2 2
   
 2 2 
Q98. The normalized wavefunction in the momentum space of a particle in one dimension is

  p  , where  and  are real constants. The uncertainty  x in measuring
p 2
2

its position is

    
(a)  (b)  (c) (d)
 2
 3
2  
Ans. : (c)

Solution:   p  
p 2
2

H.No. 40-D, Ground Floor, Jia Sarai, Near IIT, Hauz Khas, New Delhi-110016
Phone: 011-26865455/+91-9871145498
Website: www.physicsbyfiziks.com | Email: fiziks.physics@gmail.com
241
fiziks
Institute for NET/JRF, GATE, IIT‐JAM, M.Sc. Entrance, JEST, TIFR and GRE in Physics

From inverse Fourier transformation


x
 
Normalize,   x   e 

x  0,
 x
 2 2
x2 
h 

x2e 
dx 
2 2

x  x2  x 
2

2

Q99. A phase shift of 300 is observed when a beam of particles of energy 0.1MeV is scattered

by a target. When the beam energy is changed, the observed phase shift is 600 . Assuming
that only s -wave scattering is relevant and that the cross-section does not change with
energy, the beam energy is
(a) 0.4 MeV (b) 0.3 MeV (c) 0.2 MeV (d) 0.15 MeV
Ans. : (b)
4 
Solution:   2 
2l  1 sin 2  l 
k l 0
2mE
only s -wave scattering is relevant l  0 k
2

4 4  2
  2 sin  0 
2
sin 2  0
k 2mE

sin 2 30 sin 2 60 sin 2 60


According to problem  E  0.1MeV  0.3MeV
0.1MeV E sin 2 30
1 1 1 
Q100. The Hamiltonian of a two-level quantum system is H     possible initial
2 1 1
state in which the probability of the system being in that quantum state does not change
with time, is
       
 cos 4   cos 8   cos 2   cos 6 
(a)   (b)   (c)   (d)  
 sin    sin    sin    sin  
       
 4  8   2  6 
Ans. : (b)

H.No. 40-D, Ground Floor, Jia Sarai, Near IIT, Hauz Khas, New Delhi-110016
Phone: 011-26865455/+91-9871145498
Website: www.physicsbyfiziks.com | Email: fiziks.physics@gmail.com
242
fiziks
Institute for NET/JRF, GATE, IIT‐JAM, M.Sc. Entrance, JEST, TIFR and GRE in Physics

Q101. Consider a one-dimensional infinite square well


0 for 0  x  a,
V  x  
 otherwise
If a perturbation
V for 0  x  a / 3,
V  x    0
0 otherwise

is applied, then the correction to the energy of the first excited state, to first order in V ,
is nearest to
(a) V0 (b) 0.16 V0 (c) 0.2 V0 (d) 0.33 V0
Ans. : (d)
a/3
Solution: V   V   dx
*
x 2 2
0

 2 x  1 4 x 
a/3 a/3
2 2
V   V0 sin 2   dx  V0  1  cos dx
0
a  a  a 0
2 a 

 4 
sin 1
2 a 3  3
 V0  
4   V0     0.33 V0
a 6   3 4 
 a 
Q102. The energy eigenvalues En of a quantum system in the potential V  cx 6 (where c  0 is

a constant), for large values of the quantum number n , varies as


(a) n 4 / 3 (b) n3/ 2 (c) n5/ 4 (d) n6 / 5
Ans. : (b)
Solution: We can use Bohr Somerfield theory
V  x   cx 6
1/ 6
E 1/ 6
E
2m  E  cx 6 dx  nh  2mE  
 

t
 Pdx  nh  4 C
0
c 0
1  t 6 dt  nh

31
E1/ 21/ 6  n  E 6
 n , E  n3/ 2
Therefore, correct option is (b)

H.No. 40-D, Ground Floor, Jia Sarai, Near IIT, Hauz Khas, New Delhi-110016
Phone: 011-26865455/+91-9871145498
Website: www.physicsbyfiziks.com | Email: fiziks.physics@gmail.com
243
fiziks
Institute for NET/JRF, GATE, IIT‐JAM, M.Sc. Entrance, JEST, TIFR and GRE in Physics

1 1 1 
Q103. The Hamiltonian of a two-level quantum system is H     possible initial
2 1 1
state in which the probability of the system being in that quantum state does not change
with time, is
       
 cos 4   cos 8   cos 2   cos 6 
(a)   (b)   (c)   (d)  
 sin    sin    sin    sin  
       
 4  8   2  6 
Ans. : (b)

NET/JRF (JUNE-2018)
1   
Q104. The Hamiltonian of a spin particle in a magnetic field B is given by H    .B. ,
2

where  is a real constant and    x ,  y ,  z  are the Pauli spin matrices. If

B   B0 , B0 , 0  and the spin state at time t  0 is an eigenstate of  x , then of the

expectation values  x ,  y and  z

(a) only  x changes with time (b) only  y changes with time

(c) only  z changes with time (d) all three change with time

Ans. : (d)
1 1
Solution:  x ,  y and  z will changes with time because Eigen state of  x ie   and
2 1

1 1    0 1 i 
  and can be written in basis of eigen state of H    .B.   B0  
2  1 1  i 0 
Q105. A particle of mass m is constrained to move in a circular ring of radius R . When a
a
perturbation V   2
cos 2  (where a is a real constant) is added , the shift in energy of
R
the ground state, to first order in a , is
a 2a a a
(a) (b) (c) (d)
R2 R2 2R2  R 2 
Ans. : (c)
H.No. 40-D, Ground Floor, Jia Sarai, Near IIT, Hauz Khas, New Delhi-110016
Phone: 011-26865455/+91-9871145498
Website: www.physicsbyfiziks.com | Email: fiziks.physics@gmail.com
244
fiziks
Institute for NET/JRF, GATE, IIT‐JAM, M.Sc. Entrance, JEST, TIFR and GRE in Physics

a 1
Solution: V   cos 2  where 0 
2
2
R
2
a 1
0 V ' 0   2 cos 
2

R2 0

2
a 1 a 2 a

2 R 2  2 1  cos 2  d  2 R
0
2

2 2R2

Q106. A particle of mass m is confined in a three-dimensional box by the potential


0, 0  x, y, z  a
V  x, y , z   
 otherwise

9 2 2
The number of eigenstates of Hamiltonian with energy is
2ma 2
(a) 1 (b) 6 (c) 3 (d) 4
Ans. : (c)
9 2  2
Solution: Enx ,ny , nz 
2ma 2
nx ny nz 
1 2 2 

2 2 1
2 1 2 

 22
where Exx , xy , xz   nx2  n y2  nz2 
2ma 2
p2
Q107. The nth energy eigenvalues En of a one-dimensional Hamiltonian H    x 4 (where
2m
  0 is a constant) in the WEB approximation, is proportional to
4/3 4/3 5/3 5/ 3
 1  1  1  1
(a)  n    1/ 3
(b)  n    2/3
(c)  n    1/ 3
(d)  n    2/3
 2  2  2  2
Ans. : (a)
Solution: From W.K.B approximation
 1
x
4. Pdx   n   h
0  2

H.No. 40-D, Ground Floor, Jia Sarai, Near IIT, Hauz Khas, New Delhi-110016
Phone: 011-26865455/+91-9871145498
Website: www.physicsbyfiziks.com | Email: fiziks.physics@gmail.com
245
fiziks
Institute for NET/JRF, GATE, IIT‐JAM, M.Sc. Entrance, JEST, TIFR and GRE in Physics

1/ 4
E
 

 1
4  2m  E   x 4 dx    n   h
0  2

making the integration dimensional


1/ 4 1 4/3
E  1  1  1
4   2mE   1  t dt   n    E 3/ 4   n    1/ 4  E   n    1/ 3
1/ 2 4
 
 0  2  2  2

d 
Q108. The differential scattering cross-section for the central potential V  r   e   r ,
d r
where  and  are positive constants, is calculated in thee first Born approximation. Its
dependence on the scattering angle  is proportional to ( A is a constant below)
1
   
(a)  A2  sin 2  (b)  A2  sin 2 
 2  2
2
 
2
 
(c)  A2  sin 2  (d)  A2  sin 2 
 2  2
Ans. : (c)

Solution: f     V  r  sin krdr  D    f  
2


1 e r
f      r sin krdr
k0 r
  
1 e  r  eikr  e  ikr  1
f     r  dr   e e dr   e   r e  ikr dr
  r ikr

k0 r  2i  2ik 0 0

1   r   ik  

1    ik    ik  2ik 2
   k2 
1
dr   e 
 r   ikr 
 e dr     
2ik  0  2ik    k
2 2
 2ik
2
1  1 
f    2 , D     2 2 
  k2   k 


D      2  k 2  , where k  sin
2

2
2 2
   
D      2  sin 2  or D     A2  sin 2 
 2  2

H.No. 40-D, Ground Floor, Jia Sarai, Near IIT, Hauz Khas, New Delhi-110016
Phone: 011-26865455/+91-9871145498
Website: www.physicsbyfiziks.com | Email: fiziks.physics@gmail.com
246
fiziks
Institute for NET/JRF, GATE, IIT‐JAM, M.Sc. Entrance, JEST, TIFR and GRE in Physics

Q109. At t  0 , the wavefunction of an otherwise free particle confined between two infinite

2  x 3 x 
walls at x  0 and x  L is   x, t  0    sin  sin  . Its wave function at a
L L L 

mL2
later time t  is
4 h
2  x 3 x  i / 6 2  x 3 x   i / 6
(a)  sin  sin e (b)  sin  sin e
L L L  L L L 

2  x 3 x   i / 8 2  x 3 x   i / 6
(c)  sin  sin e (d)  sin  sin e
L L L  L L L 
Ans. : (d)
 2 x 2 3 x 
Solution:   x, t  0    sin  sin 
 L L L L 
  x, t  0   1  3
 iE1t  iE3t
  x, t   1 e 
 3 e 

 22 9 2  2 mL2
E1  E3  t
2mL2 2mL2 4 
 i 9 i  i
  x, t   1 e 8
 3 e 8
e 8

1  3 e  i 

 i  i
 2 x 3 x 
e 8
 1  3 e 8
 sin 
2
sin
L 

 L L L

H.No. 40-D, Ground Floor, Jia Sarai, Near IIT, Hauz Khas, New Delhi-110016
Phone: 011-26865455/+91-9871145498
Website: www.physicsbyfiziks.com | Email: fiziks.physics@gmail.com
247
fiziks
Institute for NET/JRF, GATE, IIT‐JAM, M.Sc. Entrance, JEST, TIFR and GRE in Physics

NET/JRF (DEC - 2018)


Q110. The ground state energy of an anisotropic harmonic oscillator described by the potential
1
V  x, y, z   m 2 x 2  2m 2 y 2  8m 2 z 2 (in units of  ) is
2
5 7 3 1
(a) (b) (c) (d)
2 2 2 2
Ans. : (b)
1 1 1
Solution: V  x, y, z   m 2 x 2  m  2  y 2  m  4  z 2
2 2

2 2 2
x    y  2  z  4

 1  1  1
Enx ,ny ,nz   nx    x   n y    y   nz    z
 2  2  2
For ground state
nx  0, ny  0, nz  0

1 1 1 1 7
    2  4   1  2  4   
2 2 2 2 2
Q111. The product x p of uncertainties in the position and momentum of a simple harmonic

oscillator of mass m and angular frequency  in the ground state 0 , is . The value
2
of the product x p in the state, e  ipˆ  /  0 (where  is a constant and p̂ is the

momentum operator) is

 m  2  2
(a) (b)  (c) (d)
2  2 m  2
Ans. : (c)
Q112. Let the wavefunction of the electron in a hydrogen atom be

 1  2  1 
 r   200  r   211  r   100  r 
6 3 6

where nlm  r  are the eigenstates of the Hamiltonian in the standard notation. The

expectation value of the energy in this state is


(a) 10.8 eV (b) 6.2 eV (c) 9.5 eV (d) 5.1 eV
Ans. : (d)

H.No. 40-D, Ground Floor, Jia Sarai, Near IIT, Hauz Khas, New Delhi-110016
Phone: 011-26865455/+91-9871145498
Website: www.physicsbyfiziks.com | Email: fiziks.physics@gmail.com
248
fiziks
Institute for NET/JRF, GATE, IIT‐JAM, M.Sc. Entrance, JEST, TIFR and GRE in Physics

1 2 1
Solution:   2,0,0  2,1,1  1,0,0
6 3 6
 13.6  1 2 1  4 5
P    
 4  6 3 6 6
5 1
P  3.4   and P  13.6  
6 6
5 1 1 30.60
E   3.4     13.6     17.00  13.6  eV    5.1 eV
6 6 6 6
1
Q113. Three identical spin particles of mass m are confined to a one-dimensional box of
2
length L , but are otherwise free. Assuming that they are non-interacting, the energies of
 22
the lowest two energy eigen states, in units of , are
2mL2
(a) 3 and 6 (b) 6 and 9 (c) 6 and 11 (d) 3 and 9
Ans. : (b)
 2 2
Solution: Put  E0
2mL2
For ground state configuration 2 particle has engine E0 and 1 particle has engine 4E0

Total energy is 2  E0  1 4 E0  6 E0

For first excited state configuration, 1 particles has engine E0 and 2 particle has engine

4E0

Total energy 1 E0  2  4 E0  9 E0

 22
Lowest two energy levels are 6 E0 ,9 E0 respectively, where E0 
2mL2
Q114. Consider the operator Ax  Ly pz  Lz p y , where Li and pi denote, respectively, the

components of the angular momentum and momentum operators. The commutator


 Ax , x  , where x is the

x - component of the position operator, is


(a) i  zpz  yp y  (b) i  zpz  yp y  (c) i  zpz  yp y  (d) i  zpz  yp y 

H.No. 40-D, Ground Floor, Jia Sarai, Near IIT, Hauz Khas, New Delhi-110016
Phone: 011-26865455/+91-9871145498
Website: www.physicsbyfiziks.com | Email: fiziks.physics@gmail.com
249
fiziks
Institute for NET/JRF, GATE, IIT‐JAM, M.Sc. Entrance, JEST, TIFR and GRE in Physics

Ans. : (a)
Solution: Ax  Ly pz  Lz p y , Ly  zpx  xpz , Lz  xp y  ypx

 Ax , x    Ly pz , x    Lz p y , x    Ly , x  pz   Lz , x  p y
  zpx , x  pz   ypx , x  p y  z  px , x  pz  y  px , x  p y

  i zpz    i yp y   i  zpz  yp y 

p2
Q115. A one-dimensional system is described by the Hamiltonian H    x (where   0 ).
zm
The ground state energy varies as a function of  as
(a)  5 / 3 (b)  2 / 3 (c)  4 / 3 (d)  1/ 3
Ans. : (a)
Solution: Using Bohr-Sommerfield theory,
E
x0 

 pdx  nh  4  2m  E   x  dx  nh
0

E E
where x0 is turning point x0  x0 
 
1
E
 4  2mE 
 
0
1  tdt  nh

E 3/ 2
 n  E  2/3

Q116. If the position of the electron in the ground state of a Hydrogen atom is measured, the
probability that it will be found at a distance r  a0 ( a0 being Bohr radius) is nearest to

(a) 0.91 (b) 0.66 (c) 0.32 (d) 0.13


Ans. : (b)

Solution: P  a0  r      r 2 R10 dr
2

a0
 r
2
R10 
a03/ 2
 2r a0 r
4 
P  a0  r     3 r e
2 a0
dr  0.66
a0 a0

H.No. 40-D, Ground Floor, Jia Sarai, Near IIT, Hauz Khas, New Delhi-110016
Phone: 011-26865455/+91-9871145498
Website: www.physicsbyfiziks.com | Email: fiziks.physics@gmail.com
250
fiziks
Institute for NET/JRF, GATE, IIT‐JAM, M.Sc. Entrance, JEST, TIFR and GRE in Physics

1
Q117. A system of spin particles is prepared to be in the eigenstate of  z with eigenvalue 1 .
2
The system is rotated by at angle of 600 about the x -axis. After the rotation, the fraction
of the particles that will be measured to be in the eigenstate of  z with eigenvalue 1 is

1 2 1 3
(a) (b) (c) (d)
3 3 4 4
Ans. : (d)
Solution: Rotation with angle  about x axis
 
U  R     exp  i . 
 2

   
U  R     cos   I  i sin   ˆ  
2 2
  
U  Rx     cos I  i sin   ˆ   x
2 2
  
 cos 2 i sin 2  
Rx     Put  
   3
 i sin cos 
 2 2
 3 i 
1  
2 1
    Rx       2
 
0  i 3 0
 
 2 2 
 3
  3 1 i 0
  2     
  i  2  0  2  1 
 2
3
If  z is measure on  , the measurement is 1 with probability and 1 with
4
1
probability
4

H.No. 40-D, Ground Floor, Jia Sarai, Near IIT, Hauz Khas, New Delhi-110016
Phone: 011-26865455/+91-9871145498
Website: www.physicsbyfiziks.com | Email: fiziks.physics@gmail.com
251
fiziks
Institute for NET/JRF, GATE, IIT‐JAM, M.Sc. Entrance, JEST, TIFR and GRE in Physics

THERMODYNAMICS AND STATISTICAL PHYSICS


NET/JRF (JUNE-2011)
Q1. Consider the transition of liquid water to steam as water boils at a temperature of 1000 C
under a pressure of 1 atmosphere. Which one of the following quantities does not change
discontinuously at the transition?
(a) The Gibbs free energy (b) The internal energy
(c) The entropy (d) The specific volume
Ans. : (a)
Solution: In first order transition Gibbs free energy is continuous.
Q2. A particle is confined to the region x  0 by a potential which increases linearly as
u x   u 0 x . The mean position of the particle at temperature T is

k BT k BT
(b) k B T  / u 0
2
(a) (c) (d) u 0 k B T
u0 u0

Ans. : (a)
p2 u x
1   0
Solution: Partition function Z   e 2 mkBT dp  e kBT dx and x   xp  x dxdpx
h
 0 x 2 
p2 u0 x   k BT 
 xe  te
k BT t
 
dx  u  dt
 xe dp  e
2 mk BT k BT
dx k BT
 x   0
 x  0 0



p2 u x
 0
  0  k BT  u0
e e
t
 e dt
k BT
2 mk BT
dp  e k BT
dx dx  u 
0 0 0

Q3. A cavity contains blackbody radiation in equilibrium at temperature T. The specific heat
per unit volume of the photon gas in the cavity is of the form CV  T 3 , where  is a
constant. The cavity is expanded to twice its original volume and then allowed to
equilibrate at the same temperature T. The new internal energy per unit volume is
T 4
(a) 4T 4
(b) 2T 4
(c) T 4
(d)
4
Ans. : (d)
T 4
Solution: du  C v dT   T 3dT  u 
4

H.No. 40-D, Ground Floor, Jia Sarai, Near IIT, Hauz Khas, New Delhi-110016
Phone: 011-26865455/+91-9871145498
Website: www.physicsbyfiziks.com | Email: fiziks.physics@gmail.com
252
fiziks
Institute for NET/JRF, GATE, IIT‐JAM, M.Sc. Entrance, JEST, TIFR and GRE in Physics

Q4. Consider a system of N non-interacting spins, each of which has classical magnetic
moment of magnitude  . The Hamiltonian of this system in an external magnetic field
 N
  
H is   i .H , where  i is the magnetic moment of the i th spin. The magnetization per
i 1

spin at temperature T is

 2H   H  k B T 
(a) (b)  coth    
k BT   k B T  H 
 H   H 
(c)  sinh  (d)  tanh 
 k BT   k BT 
Ans. : (b)
2 
 H cos 
   cos exp kT
sin  d d
Solution: For classical limit M  0 0
 H cos 
 exp kBT sin  d d
  H  k BT 
M   coth   
  k BT  H 
Q5. Consider an ideal Bose gas in three dimensions with the energy-momentum relation
  p s with s  0 . The range of s for which this system may undergo a Bose-Einstein
condensation at a non-zero temperature is
(a) 1  s  3 (b) 0  s  2 (c) 0  s  3 (d) 0  s  
Ans. : (a)
NET/JRF (DEC-2011)
bS 3
Q6. The internal energy E of a system is given by E  , where b is a constant and other
VN
symbols have their usual meaning. The temperature of this system is equal to
2
bS 2 3bS 2 bS 3 S
(a) (b) (c) 2 (d)  
VN VN V N N
Ans. : (b)
 E  3bS 2
Solution: TdS  dE  PdV  dE  TdS  PdV     T  T 
 S V VN

H.No. 40-D, Ground Floor, Jia Sarai, Near IIT, Hauz Khas, New Delhi-110016
Phone: 011-26865455/+91-9871145498
Website: www.physicsbyfiziks.com | Email: fiziks.physics@gmail.com
253
fiziks
Institute for NET/JRF, GATE, IIT‐JAM, M.Sc. Entrance, JEST, TIFR and GRE in Physics

Q7. Consider a Maxwellian distribution of the velocity of the molecules of an ideal gas. Let
Vmp and Vrms denote the most probable velocity and the root mean square velocity,

respectively. The magnitude of the ratio Vmp / Vrms is

(a) 1 (b) 2 / 3 (c) 2/3 (d) 3 / 2


Ans. : (c)

2kT 3kT V 2
Solution: For Maxwellian distribution Vmp  , Vrms   mb 
m m Vrms 3
Q8. If the number density of a free electron gas in three dimensions is increased eight times,
its Fermi temperature will
(a) increase by a factor of 4 (b) decrease by a factor of 4
(c) increase by a factor of 8 (d) decrease by a factor of 8
Ans. : (a)
2
 3N   2 3 N
Solution: Fermi energy E F    , where is number density and g is degeneracy
 4Vg  2m V
2 2

n n 
TF1 n1
EF  TF K  TF     TF  n  
3 2 3
3   1   4 since  8.
V  TF2  n2  n2
1
Q9. A system of N non-interacting spin - particles is placed in an external magnetic field H.
2
The behavior of the entropy of the system as a function of energy is given by
(a) S (b) S

E 
 BH BH  B H E  B H
S
(c) S (d)

 B H E BH  B H E

Ans. : (a)

H.No. 40-D, Ground Floor, Jia Sarai, Near IIT, Hauz Khas, New Delhi-110016
Phone: 011-26865455/+91-9871145498
Website: www.physicsbyfiziks.com | Email: fiziks.physics@gmail.com
254
fiziks
Institute for NET/JRF, GATE, IIT‐JAM, M.Sc. Entrance, JEST, TIFR and GRE in Physics

S  N  U  N  U  N  U  N  U 
Solution:  ln   ln   , where   H . S is symmetrical
Nk 2N  2  2 N  2N 
about E .
Q10. A gas of N non-interacting particles is in thermal equilibrium at temperature T . Each
particle can be in any of the possible non-degenerate states of energy 0, 2 and 4 . The
average energy per particle of the gas, when   1 , is
(a) 2 (b) 3 (c) 2 / 3 (d) 
Ans. : (a)
0  e  o  2 e2  4 e4
Solution: E1  0, E 2  2 , E 3  4 , Z  e 0   e 2   e 4   E 
e 0   e 2  e 4 1

2e 2   4e 4  2 1  2....  4 1   4  .... 2  4 6


 E      2
1  e  2   e  4  1  1  2....  1  4..... 111 3

where   1 .
Q11. A one-dimensional chain consists of a set of N rods each of length a . When stretched
by a load, each rod can align either parallel or perpendicular to the length of the chain.
The energy of a rod is  when perpendicular to it. When the chain is in thermal
equilibrium at temperature T , its average length is
(a) Na / 2 (b) Na 
(c) Na / 1  e 2 / k BT  
(d) Na 1  e 2 / k BT 
Ans. : (c)
Solution: Let n1 no. of rods are parallel and n2 no. of rods are perpendicular.
Energy of rod when it is perpendicular  
Energy of rod when it is parallel is  .
e      e e  
P     and P   
e      e   e  e  e   e  
n1ae   n2 ae  Nae  Na
Average length  n1aP     n2 aP       
   

e e e e 1  e 2 
Since P    P  so n2  N , n1  0 .

H.No. 40-D, Ground Floor, Jia Sarai, Near IIT, Hauz Khas, New Delhi-110016
Phone: 011-26865455/+91-9871145498
Website: www.physicsbyfiziks.com | Email: fiziks.physics@gmail.com
255
fiziks
Institute for NET/JRF, GATE, IIT‐JAM, M.Sc. Entrance, JEST, TIFR and GRE in Physics

Q12. The excitations of a three-dimensional solid are bosonic in nature with their frequency 
and wave-number k are related by   k 2 in the large wavelength limit. If the chemical
potential is zero, the behavior of the specific heat of the system at low temperature is
proportional to
(a) T 1/ 2 (b) T (c) T 3 / 2 (d) T 3
Ans. : (c)
Solution: If dispersion relation is   k s ,
At low temperature specific heat  T 3/ s
Q13. Gas molecules of mass m are confined in a cylinder of radius R and height L
(with R  L ) kept vertically in the Earth’s gravitational field. The average energy of the
gas at low temperatures (such that mgL  k BT ) is given by

(a) Nk B T / 2 (b) 3Nk B T / 2 (c) 2 Nk B T (d) 5 Nk B T / 2


Ans. : (d)
1 H
h3 
Solution: Z  e dpx dp y dpz dxdydz

  px2   p 2y   p z2 L mgz

Z e e e dpz  dx dy  e
2 mk BT 2 mk BT 2 mk BT k BT
dpx dp y dz
   0

 
mgL 
 
3 3
mgz
 mk T  1  e B
k T
 mk T  2 L  2
Z   R2  B 2   e k BT
dz  Z   R 2  B 2   
 2   0  2    mg 
 k T 
 B 
ZN  Z N ,

 ln z 5 Nk BT
 E  k BT 2  , since mgL  k BT
T 2

H.No. 40-D, Ground Floor, Jia Sarai, Near IIT, Hauz Khas, New Delhi-110016
Phone: 011-26865455/+91-9871145498
Website: www.physicsbyfiziks.com | Email: fiziks.physics@gmail.com
256
fiziks
Institute for NET/JRF, GATE, IIT‐JAM, M.Sc. Entrance, JEST, TIFR and GRE in Physics

NET/JRF (JUNE-2012)

Q14. Consider a system of non-interacting particles in d dimensional obeying the dispersion


relation   Ak s , where  is the energy, k is the wave vector; s is an integer and A is
constant. The density of states, N    , is proportional to
s d d s
1 1 1 1
(a)  d
(b)  s
(c)  s
(d)  d

Ans. : (b)
Solution: We can solve this problem with intuition for example   Ak 2
1 3
1
Density of state in 3-dimensional N(ε)    
2 2

2
1
Density of state in 2-dimensional N(ε)    
0 2

1 1
1
Density of state in 1-dimensional N(ε)    
2 2

d
1
Density of state in d-dimensional, where   Ak  N     
s s

Q15. The number of ways in which N identical bosons can be distributed in two energy levels,
is
N  N  1 N  N  1
(a) N  1 (b) (c) (d) N
2 2
Ans. : (a)
Solution: Number of boson  N , Number of energy level  g
N  g 1
So number of ways to distribute N boson into g level is, W  cN  N  1 since

g  2.
Q16. The free energy of the gas of N particles in a volume V and at a temperature T is

F  Nk B T ln a0V k B T 
5/ 2

/ N , where a 0 is a constant and k B denotes the Boltzmann
constant. The internal energy of the gas is
3 5
(a) Nk B T (b) Nk B T
2 2


(c) Nk B T ln a 0V k B T 
5/ 2

/N 
3
2
Nk B T 
(d) Nk B T ln a0V / k BT 
5/ 2

Ans. : (b)

H.No. 40-D, Ground Floor, Jia Sarai, Near IIT, Hauz Khas, New Delhi-110016
Phone: 011-26865455/+91-9871145498
Website: www.physicsbyfiziks.com | Email: fiziks.physics@gmail.com
257
fiziks
Institute for NET/JRF, GATE, IIT‐JAM, M.Sc. Entrance, JEST, TIFR and GRE in Physics


Solution: F  Nk B T ln a0V k B T 
5/ 2

/ N , F  U  TS , U  F  TS

 F   F   F 
dF   SdT  PdV      S or S     U  F T 
 T V  T V  T V

F  Nk B T ln C T  a 0Vk B5 / 2
5/ 2
where C 
N
 F   F 
   Nk B ln CT
5/ 2

 Nk B T 
C 5 3/ 2
T  T   Nk B T ln CT
5/ 2 5
 Nk B T  
 T V CT  T V
5/ 2
2 2

 F  5  F  5
T   F  Nk B T  U  F  T     Nk B T .
 T V 2  T V 2

Q17. A system has two normal modes of vibration, with frequencies 1 and  2  21 . What

is the probability that at temperature T , the system has an energy less than 41 ?

[In the following x  e   1 and Z is the partition function of the system.]
(a) x 3 / 2 x  2 x 2  / Z (b) x 3 / 2 1  x  x 2  / Z

(c) x 3 / 2 1  2 x 2  / Z (d) x 3 / 2 1  x  2 x 2  / Z
Ans. : (d)
Solution: There is two normal mode so there is two degree of freedom.
 1  1
Energy of harmonic oscillator is E   n1  1   n2   2 .
 2  2
 1  1
E   n1  1   n2   21 where n1  0,1,2,3.... and n 2  0,1,2,3....
 2  2
31 51
Ground state energy E  , first excited state energy E  . Second excited state
2 2
71
energy E  which is doubly degenerate state so g  2 , other state have more
2
energy than 41 .
3  1 5  1 7  1

PE  41  
e

2
e

2
 2e

2

x
3
2
1  x  2 x 
2
where x  e   1 .
Z Z
Q18. Bose condensation occurs in liquid He 4 kept at ambient pressure at 2.17 K . At which

temperature will Bose condensation occur in He 4 in gaseous state, the density of which
is 1000 times smaller than that of liquid He 4 ? (Assume that it is a perfect Bose gas.)
(a) 2.17 mK (b) 21.7 mK (c) 21.7  K (d) 2.17  K
H.No. 40-D, Ground Floor, Jia Sarai, Near IIT, Hauz Khas, New Delhi-110016
Phone: 011-26865455/+91-9871145498
Website: www.physicsbyfiziks.com | Email: fiziks.physics@gmail.com
258
fiziks
Institute for NET/JRF, GATE, IIT‐JAM, M.Sc. Entrance, JEST, TIFR and GRE in Physics

Ans. : (b)
2
 N 3
Solution: For bosons T   
V 
Q19. Consider black body radiation contained in a cavity whose walls are at temperature T .
The radiation is in equilibrium with the walls of the cavity. If the temperature of the walls
is increased to 2T and the radiation is allowed to come to equilibrium at the new
temperature, the entropy of the radiation increases by a factor of
(a) 2 (b) 4 (c) 8 (d) 16
Ans. : (c)

 8 5 k B4T 4  F   32 5 k B4  3
Solution: For Black Body, energy is given by F  V , S       VT .
45 2C 3  T V  45 C 
3 3

 S  T 3 , If temperate increase from T to 2T then entropy will incase S to 8S .

NET/JRF (DEC-2012)

Q20. The entropy of a system,  S  , is related to the accessible phase space volume  by

S  k B ln E , N , V  where E , N and V are the energy, number of particles and volume
respectively. From this one can conclude that 
(a) does not change during evolution to equilibrium
(b) oscillates during evolution to equilibrium
(c) is a maximum at equilibrium
(d) is a minimum at equilibrium
Ans. : (c)
Solution: Entropy is maximum at equilibrium.
Q21. Let W be the work done in a quasistatic reversible thermodynamic process. Which of
the following statements about W is correct?
(a) W is a perfect differential if the process is isothermal
(b) W is a perfect differential if the process is adiabatic
(c) W is always a perfect differential
(d) W cannot be a perfect differential

H.No. 40-D, Ground Floor, Jia Sarai, Near IIT, Hauz Khas, New Delhi-110016
Phone: 011-26865455/+91-9871145498
Website: www.physicsbyfiziks.com | Email: fiziks.physics@gmail.com
259
fiziks
Institute for NET/JRF, GATE, IIT‐JAM, M.Sc. Entrance, JEST, TIFR and GRE in Physics

Ans. : (b)
Solution: Work done is perfect differential in adiabatic process.
Q22. The free energy difference between the superconducting and the normal states of a

material is given by F  f S  f N      where  is an order parameter and
2 4

2
 and  are constants s.t.   0 in Normal and   0 in the super conducting state,
while   0 always, minimum value of F is

2 2 3 2 5 2
(a)  (b)  (c)  (d) 
 2 2 2
Ans. : (b)
 F 4
Solution: F        2   
2 4 3

2  2

2   2   0   
3 2


 2
 2 2
Putting the value, F     2  F 
 2  min
2
Q23. A given quantity of gas is taken from the state A  C reversibly, by two P
A
paths, A  C directly and A  B  C as shown in the figure.
During the process A  C the work done by the gas is 100 J and the heat
absorbed is 150 J . If during the process A  B  C the work done by the B C
gas is 30 J , the heat absorbed is V
(a) 20 J (b) 80 J (c) 220 J (d) 280 J
Ans. : (b)
Solution: During path AC , dU  dQ  dW  150  100  50 J
Since, internal energy is point function, so dU will same in all path
In path ABC , dQ  dU  dW  50  30  80 J .

H.No. 40-D, Ground Floor, Jia Sarai, Near IIT, Hauz Khas, New Delhi-110016
Phone: 011-26865455/+91-9871145498
Website: www.physicsbyfiziks.com | Email: fiziks.physics@gmail.com
260
fiziks
Institute for NET/JRF, GATE, IIT‐JAM, M.Sc. Entrance, JEST, TIFR and GRE in Physics

Q24. Consider a one-dimensional Ising model with N spins, at very low temperatures when
almost all spins are aligned parallel to each other. There will be a few spin flips with each
flip costing an energy 2 J . In a configuration with r spin flips, the energy of the system
is E   NJ  2rJ and the number of configuration is N C r ; r varies from 0 to N . The
partition function is
N N N
 J   J   J 
(a)   (b) e  NJ / k BT
(c)  sinh  (d)  cosh 
 k BT   k B T   k B T 

Ans. : (d)
Solution: Let us consider only three energy levels, E r  2 J  2rJ i.e. E 0  2 J , E1  0 and

E2  2J .

Q2 
Ce
2
0
  E0
 2C1e  E1  2C 2 e  E2   e  2J
 2e 0  e  2 J  

e J  e J  2


4 4
2
Cr
r 0

2
 e J  e J 
Q2     cosh  J 2  cosh  J 2  Q N  cosh  J  N .
 2 
Q25. Consider a system of three spins S1, S2 and S3 each of which can take values +1 and -1.
The energy of the system is given by E   J S1 S 2  S 2 S 3  S 3 S1  where J is a positive
constant. The minimum energy and the corresponding number of spin configuration are,
respectively,
(a) J and 1 (b) 3 J and 1 (c) 3 J and 2 (d) 6 J and 2
Ans. : (c)
Solution: If we take S1  S2  S3  1 i.e.   
S1 S2 S3

Then energy, E   J 1  1  1  1  1  1  3J

Again S1  S2  S3  1 , then   

Energy  E   3J

So, minimum energy is  3J  and there are two spin configuration.

If we take   
S1 S2 S3

Then we get Maximum energy E  J .


H.No. 40-D, Ground Floor, Jia Sarai, Near IIT, Hauz Khas, New Delhi-110016
Phone: 011-26865455/+91-9871145498
Website: www.physicsbyfiziks.com | Email: fiziks.physics@gmail.com
261
fiziks
Institute for NET/JRF, GATE, IIT‐JAM, M.Sc. Entrance, JEST, TIFR and GRE in Physics

NET/JRF (JUNE-2013)

Q26. Ten grams of ice at 00 C is added to a beaker containing 30 grams of water at 250 C .
What is the final temperature of the system when it comes to thermal equilibrium? (The
specific heat of water is 1 cal / gm / 0 C and latent heat of melting of ice is 80 cal / gm )

(a) 00 C (b) 7.50 C (c) 12.50 C (d) 1.250 C


Ans. : (a)
Solution: The amount of heat required to melt the ice of mass 10 gm at 00 C is
Q  m  L  10  80  800Cal , where L is the latent heat of melting of ice and m is the

mass of the ice. The amount of heat available in water of mass 30 gm at 250 C is

Q  m  Cv  T  30 1 25  750Cal
Since the heat available is less than the heat required to melt the ice therefore ice will not
melt as a result the temperature of the system will be at 00 C only.
Q27. A vessel has two compartments of volume V1 and V2 , containing an ideal gas at pressures
p1 and p 2 , and temperatures T1 and T2 respectively. If the wall separating the
compartments is removed, the resulting equilibrium temperature will be
p1T1  p 2T2 V1T1  V2T2 p1V1  p 2V2
(d) T1T2 
1/ 2
(a) (b) (c)
p1  p 2 V1  V2  p1V1 / T1    p 2V2 / T2 
Ans. : (c)
p1V1 p2V2
Solution: V  V1  V2 , n  n1  n2   , U1  U 2  U , n1CvT1  n2CvT2  nCvT ,
T1 T2
p1V1  p2V2
n1T1  n2T2  nT  T 
p1V1 p2V2

T1 T2

H.No. 40-D, Ground Floor, Jia Sarai, Near IIT, Hauz Khas, New Delhi-110016
Phone: 011-26865455/+91-9871145498
Website: www.physicsbyfiziks.com | Email: fiziks.physics@gmail.com
262
fiziks
Institute for NET/JRF, GATE, IIT‐JAM, M.Sc. Entrance, JEST, TIFR and GRE in Physics

Q28. For temperature T1  T2 , the qualitative temperature dependence of the probability


distribution F v  of the speed v of a molecule in three dimensions is correctly
represented by the following figure:

T2
T1
F(v)

F(v)
(a) T1 (b)
T2

v v

T1
T2 T1
F(v)

F(v)
(c) (d)
T2

Ans. : (a)
v v
Solution: Area under the F  v  is conserve and the mean velocity shift towards right for higher
temperature.
Q29. A system of non-interacting spin- 1/ 2 charged particles are placed in an external magnetic
field. At low temperature T , the leading behavior of the excess energy above the ground
state energy, depends on T as: ( c is a constant)
(a) cT (b) cT 3 (c) e  c / T (d) c (is independent of T )
Ans. : (c)
 kTB H  H
 B 
 H  e  e kT 
Solution: U    B H tanh B    B H   B H BH 
kT  e kT  e  kT 
 
Excess energy from the ground level

 kTB H  H
 B    B H  H
 B    H
 B 
e  e   (  H )   H 1   e  e    H  e
   B H  B H 
kT kT kT kT
2
  BH  B B   B  H  H  B  B H B H 
 e kT  e kT  
  e kT
e
 B
kT   e kT  e kT 
    
C
At low temperature, the lower value, U  e T
, where C   B H .

H.No. 40-D, Ground Floor, Jia Sarai, Near IIT, Hauz Khas, New Delhi-110016
Phone: 011-26865455/+91-9871145498
Website: www.physicsbyfiziks.com | Email: fiziks.physics@gmail.com
263
fiziks
Institute for NET/JRF, GATE, IIT‐JAM, M.Sc. Entrance, JEST, TIFR and GRE in Physics

Q30. Consider a system of two Ising spins S1 and S 2 taking values  1 with interaction energy
given by    JS1 S 2 , when it is in thermal equilibrium at temperature T . For large T , the
average energy of the system varies as C / k B T , with C given by

(a)  2 J 2 (b)  J 2 (c) J 2 (d) 4 J


Ans. : (b)
Solution: The interaction energy is given by E   J S1 S 2 where S1 and S 2 taking values  1 .
Possible values of the Energy of the system are
E1   J 11   J , E 2   J  1 1   J

E3   J 1  1   J , E 4   J  1  1   J
E
 r  J  J 
E g e r r
kT
2 Je
J
kT
 2 Je

J
kT
 kTJ
 e  e kT

J
 1  kT
  J 
 1 
 kT  

U  r
  J
E J J  J J   J   J 
 r 
 e kT  e  kT 
 gr e
r
kT
2e kT
 2e kT
  1 
 kT
 1 


 kT 
J2 J
 U   C   J 2 (For large T ,  1 )
kT kT
Q31. Consider two different systems each with three identical non-interacting particles. Both
have single particle states with energies  0 ,3 0 and 5 0 ,  0  0 . One system is populated

1
by spin  fermions and the other by bosons. What is the value of E F  E B where E F
2
and EB are the ground state energies of the fermionic and bosonic systems respectively?

(a) 6 0 (b) 2 0 (c) 4 0 (d)  0


Ans. : (b)
Solution: Energy of Fermion = 2 1 0  3 0  5 0
Energy of boson = 3 1 0  3 0
E F  E B = 5 0  3 0  2 0

H.No. 40-D, Ground Floor, Jia Sarai, Near IIT, Hauz Khas, New Delhi-110016
Phone: 011-26865455/+91-9871145498
Website: www.physicsbyfiziks.com | Email: fiziks.physics@gmail.com
264
fiziks
Institute for NET/JRF, GATE, IIT‐JAM, M.Sc. Entrance, JEST, TIFR and GRE in Physics

NET/JRF (DEC-2013)
1
Q32. Three identical spin- fermions are to be distributed in two non-degenerate distinct
2
energy levels. The number of ways this can be done is
(a) 8 (b) 4 (c) 3 (d) 2
Ans. : (b)
Solution: Total number of degeneracy
g  (Number of energy state (n))  (Number of degeneracy due to spin ( 2 s  1 ))
1 1
n  2, s  , g  2  (2.  1)  4
2 2
Number of particle, N  3 . So number of ways, g cN  4 c3  4
Q33. Consider the melting transition of ice into water at constant pressure. Which of the
following thermodynamic quantities does not exhibit a discontinuous change across the
phase transition?
(a) Internal energy (b) Helmholtz free energy (c) Gibbs free energy (d) Entropy
Ans. : (c)
Solution: Ice to water: 1st order phase transition.
So Gibbs free energy is continuous, so it doesn’t exhibit discontinuous change.
Q34. Two different thermodynamic systems are described by the following equations of state:
1 3RN 1 1 5RN 2 
1  1 and 2    2  where T
1, 2 
, N 1, 2  and U 1, 2  are respectively, the
T 2U T 2U
temperatures, the mole numbers and the internal energies of the two systems, and R is
the gas constant. Let U tot denote the total energy when these two systems are put in

U 1
contact and attain thermal equilibrium. The ratio is
U tot

5 N 2  3N 1 N 1 N 2 
(a) (b) (c) 1 (d) 1
3N 1  5 N 2  3N 1  5 N 2  N  N 2  N  N 2 
Ans. : (b)
1 3RN 1 1 5RN 2 
Solution:  and 
T 1 2U 1 T 2  2U 2 
3 5
Now U tot  U (1)  U  2  RN 1T 1  RN  2T  2
2 2

H.No. 40-D, Ground Floor, Jia Sarai, Near IIT, Hauz Khas, New Delhi-110016
Phone: 011-26865455/+91-9871145498
Website: www.physicsbyfiziks.com | Email: fiziks.physics@gmail.com
265
fiziks
Institute for NET/JRF, GATE, IIT‐JAM, M.Sc. Entrance, JEST, TIFR and GRE in Physics

1
3RN 1T 1 
U 1 2  3 N 1T 1
  
U tot 1 3N 1T 1  5 RN  2T  2  3N 1T 1  5 N  2T  2
2 
3N 1
At thermal equilibrium T 1  T  2 , thus
3N 1  5 N 2 
Q35. The speed v of the molecules of mass m of an ideal gas obeys Maxwell’s velocity
distribution law at an equilibrium temperature T . Let  vx , v y , vz  denote the components

 
2
of the velocity and k B the Boltzmann constant. The average value of  vx   v y , where

 and  are constants, is



(a)  2   2 k B T / m   
(b)  2   2 k B T / m

(c)     k B T / m (d)     k B T / m
2 2

Ans. : (b)
Solution: Ideal gas obeys Maxwell velocity distribution law at equilibrium temperature. Then

 
2
average value of  vx   v y

 v 
2
Now x  v y   2 v x2   2 v y2  2  v x v y

k BT
v x  0, v y  0 and vx2   v y2  vz2
m

 v 
2
Then x  v y   2 v x2   2 v y2  2 v x v y

 v    2 B   2   2 
2 k BT kT kT
x  v y  2
m m m
Q36. The entropy S of a thermodynamic system as a function of energy S
C
E is given by the following graph. The temperatures of the phases B
A
A, B and C denoted by T A , TB and TC respectively.
E
Satisfy the following inequalities:
(a) TC  TB  T A (b) T A  TC  TB (c) TB  TC  T A (d) TB  T A  TC
Ans. : (c)
Solution: Temperatures of phase are: TA , TB , TC

H.No. 40-D, Ground Floor, Jia Sarai, Near IIT, Hauz Khas, New Delhi-110016
Phone: 011-26865455/+91-9871145498
Website: www.physicsbyfiziks.com | Email: fiziks.physics@gmail.com
266
fiziks
Institute for NET/JRF, GATE, IIT‐JAM, M.Sc. Entrance, JEST, TIFR and GRE in Physics

 dS  1 S
Since,   C
 dE  T B
dS A
Hence, will be slope, then it will be zero for B - phase
dE E
So TB   and in C and A phases, internal energy of C phase is more, so TC  T A

Now TB  TC  T A

Q37. A system of N classical non-interacting particles, each of mass m , is at a temperature T


and is confined by the external potential V r   Ar 2 (where A is a constant) in three
1
2
dimensions. The internal energy of the system is
A  k BT 
(c) N 2mA k B T
3
ln 
3/ 2
(a) 3 Nk B T (b) Nk B T (d) N
2 m  m 
Ans. : (a)

Solution: V r  
1 2 1
2
 
Ar  A x 2  y 2  z 2 it is harmonic oscillator.
2
3N
1  kT 
So its partition function will be z N   
N   
 ln Z N
Internal energy, U  kT 2  3 NkT
T
Q38. A Carnot cycle operates as a heat engine between two bodies of equal heat capacity until
their temperatures become equal. If the initial temperatures of the bodies are T1 and T2 ,
respectively and T1  T2 , then their common final temperature is

(a) T12 / T2 (b) T22 / T1 (c) T1T2 (d)


1
T1  T2 
2
Ans. : (c)
Solution: For heat Carnot engine the change in entropy for source and sink
TF dT  T TF dT T 
dS1   and dS 2  
 log  F
  log  F 
T1 T   T1 T2 T
 T2 
T T
S  dS1  dS 2  log F  log F .
T1 T2
T 
2

Since, Carnot engine is reversible in nature, so log F  0  TF  T1T2


T1T2

H.No. 40-D, Ground Floor, Jia Sarai, Near IIT, Hauz Khas, New Delhi-110016
Phone: 011-26865455/+91-9871145498
Website: www.physicsbyfiziks.com | Email: fiziks.physics@gmail.com
267
fiziks
Institute for NET/JRF, GATE, IIT‐JAM, M.Sc. Entrance, JEST, TIFR and GRE in Physics

NET/JRF (JUNE-2014)
Q39. Which of the graphs below gives the correct qualitative behaviour of the energy density
E r   of blackbody radiation of wavelength  at two temperatures T1 and T2 T1  T2  ?
(a) (b)
T2
T2
Er  
Er  
T1

 
(c) (d)
T2
T2
Er   Er  
T1
T1

Ans. : (c)  
Q40. A system can have three energy levels: E  0,   . The level E  0 is doubly degenerate,
while the others are non-degenerate. The average energy at inverse temperature  is

 e   e      
(a)   tanh   (d)   tanh  
(b)
1  e 
e  
 (c) 0
 2 
Ans. : (d)
Solution: E  0,   , E  0 doubly degenerate

z   gi e   Ei  2  e   0  e    e 


z  2  e   e   ln z  ln 2  e   e  
   
Now E  

ln  z   


ln 2  e   e     
1

2 e  e

 
   e    e    

      

   
  e 2
 e 2

 e e
E   

         tanh   
 2    
  
  2 
  e 2  e 2     e 2
 e 2
 
  
   
  

H.No. 40-D, Ground Floor, Jia Sarai, Near IIT, Hauz Khas, New Delhi-110016
Phone: 011-26865455/+91-9871145498
Website: www.physicsbyfiziks.com | Email: fiziks.physics@gmail.com
268
fiziks
Institute for NET/JRF, GATE, IIT‐JAM, M.Sc. Entrance, JEST, TIFR and GRE in Physics

Q41. The free energy F of a system depends on a thermodynamic variable  as

F  a 2  b 6
with a, b  0 . The value of  , when the system is in thermodynamic equilibrium, is

(b)  a / 6b  (c)  a / 3b  (d)  a / b 


1/ 4 1/ 4 1/ 4
(a) zero
Ans. : (c)
Solution: Frequency F  a 2  b 6 , a, b  0

2 F F
F is equilibrium i.e.  0 , now  2a  6b 5
 2


F
1/ 4
a  a 
 0  2a  6b 5   4    
 3b  3b 
Q42. For a particular thermodynamic system the entropy S is related to the internal energy U
and volume V by
S  cU 3 / 4V 1 / 4
where c is a constant. The Gibbs potential G  U  TS  PV for this system is
3PU cU US
(a) (b) (c) zero (d)
4T 3 4V
Ans. : (c)
Solution: S  cU 3/ 4V 1/ 4 , dU  TdS  PdV

 S  1  S  1 c  3 1/ 4 1/ 4 4 U 1/ 4
        U V  T 
 U V T  U V T 4 3c V 1/ 4
1
5 / 4
 U  SV U S V 5 / 4 1/ 4
4

    P    P  U
 V  S c 3 c 3

4 U 1/4 S V 5/4 1/4 4 1


G U   cU V
3/4 1/4
 U V  U  U  U  0
3c V 1/4
c 3 3 3
Q43. The pressure of a non-relativistic free Fermi gas in three-dimensions depends, at T  0 ,
on the density of fermions n as
(a) n 5 / 3 (b) n1 / 3 (c) n 2 / 3 (d) n 4 / 3
Ans. : (a)

H.No. 40-D, Ground Floor, Jia Sarai, Near IIT, Hauz Khas, New Delhi-110016
Phone: 011-26865455/+91-9871145498
Website: www.physicsbyfiziks.com | Email: fiziks.physics@gmail.com
269
fiziks
Institute for NET/JRF, GATE, IIT‐JAM, M.Sc. Entrance, JEST, TIFR and GRE in Physics

2
Solution: Pressure P  nEF , EF  n 2 / 3 , at T  0
3
2 2
P  n  n 2 / 3  n 5 / 3  P  n5 / 3
3 3
Q44. The vander Waals’ equation of state for a gas is given by
 a 
 P  2 V  b   RT
 V 
where P, V and T represent the pressure, volume and temperature respectively, and a
and b are constant parameters. At the critical point, where all the roots of the above
cubic equation are degenerate, the volume is given by
a a 8a
(a) (b) (c) (d) 3b
9b 27b 2 27bR
Ans. : (d)

 a   P   2 P 
Solution:  P  2  V  b   RT , for critical volume    0,  2   0
 V   V   V 
a ab
PV   Pb  2  RT
V V
P a 2ab 2 P 2a 6ab 2a 6ab
 0  P  2  3  0,  0  3  4  0  3  4  Vc  3b
V V V V 2
V V V V

H.No. 40-D, Ground Floor, Jia Sarai, Near IIT, Hauz Khas, New Delhi-110016
Phone: 011-26865455/+91-9871145498
Website: www.physicsbyfiziks.com | Email: fiziks.physics@gmail.com
270
fiziks
Institute for NET/JRF, GATE, IIT‐JAM, M.Sc. Entrance, JEST, TIFR and GRE in Physics

NET/JRF (DEC-2014)
Q45. The pressure P of a fluid is related to its number density  by the equation of state
P  a  b 2
where a and b are constants. If the initial volume of the fluid is V0 , the work done on
the system when it is compressed, so as to increase the number density from an initial
value of  0 to 2  0 is

(a) a 0V0 (b) a  b 0  0V0

 3a 7  b 
(c)   0   0V0 (d) a ln 2  b 0  0V0
 2 3 
Ans. : (d)
n n2 n
Solution: P  a  b 2  P  a b 2  
V V V
V2 dV V2 dV n n
W   P  dV  an  bn 2  , where V1  , V2 
V1 V V1 V 2 0 2 0

 W   n  a ln 2  b 0    0V0  a ln 2  b0  ,  n  0V0

Work done on the system  W   a ln 2  b0  0V0

Q46. An ideal Bose gas is confined inside a container that is connected to a particle reservoir.
Each particle can occupy a discrete set of single-particle quantum states. If the probability
that a particular quantum state is unoccupied is 0.1 , then the average number of bosons
in that state is
(a) 8 (b) 9 (c) 10 (d) 11
Ans. : (b)
Q47. In low density oxygen gas at low temperature, only the translational and rotational modes
of the molecules are excited. The specific heat per molecule of the gas is
1 3 5
(a) kB (b) k B (c) kB (d) kB
2 2 2
Ans. : (d)
Solution: Total D.O.F. = 3 transition + 2 rotation i.e. f  5
k B T 5k B T U 5
U f   CV   kB
2 2 T 2

H.No. 40-D, Ground Floor, Jia Sarai, Near IIT, Hauz Khas, New Delhi-110016
Phone: 011-26865455/+91-9871145498
Website: www.physicsbyfiziks.com | Email: fiziks.physics@gmail.com
271
fiziks
Institute for NET/JRF, GATE, IIT‐JAM, M.Sc. Entrance, JEST, TIFR and GRE in Physics

Q48. When a gas expands adiabatically from volume V1 to V2 by a quasi-static reversible


process, it cools from temperature T1 to T2 . If now the same process is carried out
adiabatically and irreversibly, and T2 is the temperature of the gas when it has
equilibrated, then
V V  T2V1
(a) T2  T2 (b) T2  T2 (c) T2  T2  2 1  (d) T2 
 V2  V2
Ans. : (b)
Q49. A random walker takes a step of unit length in the positive direction with probability 2 / 3
and a step of unit length in the negative direction with probability 1 / 3 . The mean
displacement of the walker after n steps is
(a) n / 3 (b) n / 8 (c) 2n / 3 (d) 0
Ans. : (a)
2 1
Solution: P  1   P  1 
3 3
2 1 1 n
For one step  1    , for n step 
3 3 3 3
Q50. A collection N of non-interacting spins S i , i  1, 2, ....., N , S i  1 is kept in an
external magnetic field B at a temperature T . The Hamiltonian of the system is
B
H   B i S i . What should be the minimum value of for which the mean value
k BT
1
Si  ?
3
1 1
(a) N ln 2 (b) 2 ln 2 (c) ln 2 (d) N ln 2
2 2
Ans. : (c)
B B

e kT
e kT
Solution: P  Si  1  B B
, P  Si  1  B B
 
e kT
e kT e kT
 e kT
B B
 
1e kT
e kT
 B 
Si   Si    tanh 

B B
 kT 
e kT
 e kT
B
For N particle Si   N tanh
kT
S 1  B  1 B 1
According to question, i    tanh     ln 2
N 3  kT  3 kT 2

H.No. 40-D, Ground Floor, Jia Sarai, Near IIT, Hauz Khas, New Delhi-110016
Phone: 011-26865455/+91-9871145498
Website: www.physicsbyfiziks.com | Email: fiziks.physics@gmail.com
272
fiziks
Institute for NET/JRF, GATE, IIT‐JAM, M.Sc. Entrance, JEST, TIFR and GRE in Physics

NET/JRF (JUNE-2015)
Q51. A system of N non-interacting classical particles, each of mass m is in a two
dimensional harmonic potential of the form V  r    x 2  y 2 where  is a positive  
 1 
constant. The canonical partition function of the system at temperature T is    :
 k BT 
N N
   2    2m 
2N
  
N
 2m 2 
(a)    (b)   (c)   (d)  2 
 2m        2m     
Ans. (d)


Solution: V  r    x 2  y 2 
 px2  p 2y  x 2  y 2
1    
z1  2
h 
e 2 mkT dpx  e 2 mkT dp y  e
 
kT
dx  e

kT
dy

2 mkT 2 mkT 1 1 1 
 z1  2  2
h2 h 2
2  2 
kT kT
N
 2 2 m   2 2 m 
z1   2   kT   z N   2 2 
2

 h   h  
Q52. A system of N distinguishable particles, each of which can be in one of the two energy
levels 0 and  , has a total energy n  , where n is an integer. The entropy of the system
is proportional to

 N !  N! 
(a) N ln n (b) n ln N (d) ln 
 n ! N  n  ! 
(c) ln  
 n!   
Ans. : (d)
Solution: No of ways for above configuration is  N Cn

N N
W   Entropy=k ln
n N n n N n

H.No. 40-D, Ground Floor, Jia Sarai, Near IIT, Hauz Khas, New Delhi-110016
Phone: 011-26865455/+91-9871145498
Website: www.physicsbyfiziks.com | Email: fiziks.physics@gmail.com
273
fiziks
Institute for NET/JRF, GATE, IIT‐JAM, M.Sc. Entrance, JEST, TIFR and GRE in Physics

Q53. The condition for the liquid and vapour phases of a fluid to be in equilibrium is given by
dP Q
the approximate equation  1 (Clausius-Clayperon equation) where vvap is the
dT Tvvap

volume per particle in the vapour phase, and Q1 is the latent heat, which may be taken to
be a constant. If the vapour obeys ideal gas law, which of the following plots is correct?

ln P ln P
(a) (b) (c) ln P (d) ln P
O T O T O T O T

Ans. (c)
dP Q RT dP Ql P dP Ql dT C
Solution:  l , vap 
dT Tvap P
 
dT RT 2

P

R T 2
 ln P  
T


Q54. Consider three Ising spins at the vertices of a triangle which interact with each other with
a ferromagnetic Ising interaction of strength J . The partition function of the system at
 1 
temperature T is given by    :
 k BT 

(a) 2e3 J  6e   J (b) 2e 3 J  6e  J

(c) 2e3 J  6e 3 J  3e  J  3e   J (d)  2 cosh  J 


3

Ans. (b)
Solution: H  J  S1S 2  S1S3  S 2 S3 

S1 S2 S3 E
1 1 1 3J

1 1
1

1 1 1   J
1 1 1 
1 11

1 1 1  J
1 1 1
1 1 1 3J
 z  2e3 J  6e  J

H.No. 40-D, Ground Floor, Jia Sarai, Near IIT, Hauz Khas, New Delhi-110016
Phone: 011-26865455/+91-9871145498
Website: www.physicsbyfiziks.com | Email: fiziks.physics@gmail.com
274
fiziks
Institute for NET/JRF, GATE, IIT‐JAM, M.Sc. Entrance, JEST, TIFR and GRE in Physics

Q55. A large number N of Brownian particles in one dimension start their diffusive motion
from the origin at time t  0 . The diffusion coefficient is D . The number of particles
crossing a point at a distance L from the origin, per unit time, depends on L and time t
as
 L2 4 Dt  L2 4 Dt
N NL N
(a) e  4 Dt  (b) e L2
(c) e  4 Dt 
(d) Ne L2
4 Dt 4 Dt 16 Dt 3
Ans. (a)
Solution: From Einstein Smoluchowski theory

dx   x2 
p  x  dx  exp  
4 Dt  4 Dt 

N   L2 
Number of particle passing from point L at origin  .exp  
4 Dt  4 Dt 

Q56. An ideal Bose gas in d -dimensions obeys the dispersion relation  k  Ak s , where A  
and s are constants. For Bose-Einstein condensation to occur, the occupancy of excited
states
d s

 s
Ne  c  d
0 e 
   
1 
where c is a constant, should remain finite even for   0 . This can happen if
d 1 1 d 1 d 1 d
(a)  (b)   (c) 1 (d)  1
s 4 4 s 2 s 2 s
Ans. (c)
d s
  s
Solution: Ne  c  d
0
e      1
B.E. condensation is possible in 3-D
d s 1 d 3
1
For materlistic particle g  2    
s 2 s 2
d  s d
For massless particle g  2  2 3
s s
d
In both cases  1
s

H.No. 40-D, Ground Floor, Jia Sarai, Near IIT, Hauz Khas, New Delhi-110016
Phone: 011-26865455/+91-9871145498
Website: www.physicsbyfiziks.com | Email: fiziks.physics@gmail.com
275
fiziks
Institute for NET/JRF, GATE, IIT‐JAM, M.Sc. Entrance, JEST, TIFR and GRE in Physics

NET/JRF (DEC-2015)
Q57. The heat capacity of the interior of a refrigerator is 4.2 kJ / K . The minimum work that

must be done to lower the internal temperature from 18o C to 17o C , when the outside

temperature is 27o C will be


(a) 2.20 kJ (b) 0.80 kJ (c) 0.30 kJ (d) 0.14 kJ
Ans. : (b)
Q58. For a system of independent non interacting one-dimensional oscillators, the value of the
free energy per oscillator, in the limit T  0 , is
1 3
(a)  (b)  (c)  (d) 0
2 2
Ans. : (a)
N
       
Solution: For the given system Z N   2sinh   F  kT ln Z N  NkT ln  2sinh  
 2kT    2kT  
   


 
2 e 2 kT
 e 2 kT

       

 
 


 NkT ln  NkT ln  e 1  e    NkT ln e  NkT ln 1  e kT 
2 kT kT 2 kT
 

2
      
 

F      
  kT ln 1  e kT   0   kT  0 
N 2   2 2

Q59. The partition function of a system of N Ising spins is Z  1N  2N where 1 and 2 are

functions of temperature, but are independent of N . If 1  2 , the free energy per spin

in the limit N   is
 
(a)  k BT ln  1  (b)  k BT ln 2 (c)  k BT ln  12  (d)  k BT ln 1
 2 
Ans. : (d)

 
Solution: Z  1N  2N , F   kT ln 1N  2N , it is given 1  2

     N  2
 F  kT ln 1N  1   2    , 0
   1    1
 
F
F   kT ln 1N   NkT ln 1   kT ln 1
N
H.No. 40-D, Ground Floor, Jia Sarai, Near IIT, Hauz Khas, New Delhi-110016
Phone: 011-26865455/+91-9871145498
Website: www.physicsbyfiziks.com | Email: fiziks.physics@gmail.com
276
fiziks
Institute for NET/JRF, GATE, IIT‐JAM, M.Sc. Entrance, JEST, TIFR and GRE in Physics

1
Q60. The Hamiltonian of a system of N non interacting spin particles is H   0 B  Siz ,
2 i

where Siz  1 are components of i th spin along an external magnetic field B . At a


0 B

temperature T such that e kBT  2 . the specific heat per particle is


16 8 16
(c) k B  ln 2  k B  ln 2 
2 2
(a) kB (b) k B ln 2 (d)
25 25 25
Ans. : (d)
0 B
Solution: For the given system E   0 B tanh
kT

 B B
2

CV   0  Nk sec 2 h 0
 kT  kT
0 B
CV  0 B  0 B
2
4
  k e kT  2   ln 2
N  kT   0 B  B
 0 
2
kT
 e kT
 e kT

 

CV  0 B    B  16
2 2
4 2 16k 16
  ln 2  k  ln 2 
2
  k  0  k 
N  kT   1
2
 kT  25 25 25
 2  
 2
1
Q61. An ensemble of non-interacting spin - particles is in contact with a heat bath at
2
temperature T and is subjected to an external magnetic field. Each particle can be in one
of the two quantum states of energies  0 . If the mean energy per particle is  0 / 2 ,
then the free energy per particle is

(a) 2 0

ln 4 / 3  (b)  0 ln  3 / 2  (c) 2 0 ln 2 (d)  0
ln 2
ln 3 ln 3
Ans. : (a)
0
Solution: For the given system, partition function, Z n  2 N cosh
kT
   
Mean energy per unit particle    0    0 tanh 0
 2 kT
 
 1 e e 1
put 0    tanh     

kT 2 e e 2

H.No. 40-D, Ground Floor, Jia Sarai, Near IIT, Hauz Khas, New Delhi-110016
Phone: 011-26865455/+91-9871145498
Website: www.physicsbyfiziks.com | Email: fiziks.physics@gmail.com
277
fiziks
Institute for NET/JRF, GATE, IIT‐JAM, M.Sc. Entrance, JEST, TIFR and GRE in Physics

1  1 2 0
 e 2  3    ln 3  0  ln 3  kT 
2 kT 2 ln 3
F   
 kT ln  2 cosh 0    kT ln  2 cosh   b  4ac
2
It is given,
N  kT 
 e  e  
  kT ln  2 2
 
  kT ln e e  1   kT ln  e  3  1   kT ln e 4 
  

 2 
0 2
 kT ln e   ln 4   kT   kT ln 4  kT  kT ln 4  0  0 ln 4
kT ln 3
  3    16  
 ln     ln  3  
 ln 3  2 ln 4   16 
0   0     0    
 ln 3   ln 3   ln 3 
   
  4 2   4   4 
 ln    ln   ln  
F   3   3  3
   0  2 0  2 0
N  ln 3  ln 3 ln 3
 
 
Q62. Which of the following graphs shows the qualitative dependence of the free energy
f  h, T  of a ferromagnet in an external magnetic field h , and at a fixed temperature

T  TC , where TC is the critical temperature?


f f f f
h
(a) (b) (c) (d)
h h h
Ans. : (c)
Solution: For super conductor state one will find two local minima
f
Option (c) is correct.
h

H.No. 40-D, Ground Floor, Jia Sarai, Near IIT, Hauz Khas, New Delhi-110016
Phone: 011-26865455/+91-9871145498
Website: www.physicsbyfiziks.com | Email: fiziks.physics@gmail.com
278
fiziks
Institute for NET/JRF, GATE, IIT‐JAM, M.Sc. Entrance, JEST, TIFR and GRE in Physics

NET/JRF (JUNE-2016)
Q63. The specific heat per molecule of a gas of diatomic molecules at high temperatures is
(a) 8k B (b) 3.5k B (c) 4.5 k B (d) 3k B
Ans. : (b)
Solution: For high temperature all number are excited so degree of freedom for diatomic
molecule is 7 .
fk BT 7k T  U 
Internal energy is , U  B , CV     3.5k B
2 2  T V
Q64. When an ideal monoatomic gas is expanded adiabatically from an initial volume V0 to
T
3V0 , its temperature changes from T0 to T . Then the ratio is
T0
2 1
1  1 3  1 3
(a) (b)   (c)   (d) 3
3 3 3
Ans. : (b)
Solution: For adiabatic process PV   k , T0V0 1  k
 1 v 1
V  1
 T  3V0 
 1  1
T0V0  T  T0  0   T  T0  
 3V0  3
5
For monoatomic gas  
3
5 2
1 2/3
 1 3  1 3 T 1
T  T0    T0      
3 3 T0  3 
Q65. A box of volume V containing N molecules of an ideal gas, is divided by a wall with a
V
hole into two compartments. If the volume of the smaller compartment is , the
3
variance of the number of particles in it, is
N 2N N
(a) (b) (c) N (d)
3 9 3
Ans. : (b)
V 1
Solution: Probability that one particle is in smaller compartment having volume , so p 
3 3

H.No. 40-D, Ground Floor, Jia Sarai, Near IIT, Hauz Khas, New Delhi-110016
Phone: 011-26865455/+91-9871145498
Website: www.physicsbyfiziks.com | Email: fiziks.physics@gmail.com
279
fiziks
Institute for NET/JRF, GATE, IIT‐JAM, M.Sc. Entrance, JEST, TIFR and GRE in Physics

There are only two options either particle is in left half or right half, so for one particle
distribution is Bernoulli for Bernoulli’s distribution  2  p 1  p  . For N particle

distribution is
1  1 2N
 2  Np 1  p   N  1   ,  2 
3  3 9
Q66. A gas of non-relativistic classical particles in one dimension is subjected to a potential
 1 
V  x    x (where  is a constant). The partition function is    
 k BT 
4m 2m 8m 3m
(a) (b) (c) (d)
 h
3 2 2
 h
3 2 2
 h
3 2 2
 3 2 h 2
Ans. : (c)
 p2  x  x
1  x  1 
Solution: z   e 2 mkT dpx  e kT dx   2 mkT   e kT dx
1/ 2

h  
h 

1/ 2  x
 2 mkT  
 z 
 h
2 
 
e kT
dx

 x 0 x  x
   kT kT 2kT
e kT
dx  e kT
dx   e kT
dx   
  0
  
1
 2 mkT   2kT   8 m  2
1/ 2
1
z      2 3 2  , put  
 h     h    kT
2

Q67. The internal energy E T  of a system at a fixed volume is found to depend on the

temperature T as E T   aT 2  bT 4 . Then the entropy S T  , as a function of temperature,

is
1 2 1 4 4
(a) aT  bT (b) 2aT 2  4bT 4 (c) 2aT  bT 3 (d) 2aT  2bT 3
2 4 3
Ans. : (c)
Solution: From first law of thermodynamics,
TdS  dE  PdV , dE  TdS  PdV , it is given dV  0
1
dE  TdS  dS  dE
T
E  aT 2  bT 4  dE  2aTdT  4bT 3 dT
4bT 3
dS 
1
T

2aTdT  4bT dT  2adT  4bT dT  2aT 
3 2

3

H.No. 40-D, Ground Floor, Jia Sarai, Near IIT, Hauz Khas, New Delhi-110016
Phone: 011-26865455/+91-9871145498
Website: www.physicsbyfiziks.com | Email: fiziks.physics@gmail.com
280
fiziks
Institute for NET/JRF, GATE, IIT‐JAM, M.Sc. Entrance, JEST, TIFR and GRE in Physics

Q68. Consider a gas of Cs atoms at a number density of 1012 atoms/cc. when the typical inter-
particle distance is equal to the thermal de Broglie wavelength of the particles, the
temperature of the gas is nearest to (Take the mass of a Cs atom to be 22.7 1026 kg )

(a) 1 109 K (b) 7  105 K (c) 1 103 K (d) 2 108 K


Ans. : (d)
Solution: When de Broglie wavelength = thermal wavelength
g3 / 2  z   2.61
2/3
N h3 N h2
 2 mkT 3 / 2   2 mkT   
V 2.61 V   2.612 / 3

   
2/3 2
1 h2 1 1012 6.6  1034
T n 2/3

2 mk  2.612 / 3 2  3.14  22.7 1026 1.38 1023   2.612 / 3
 6.6  108 1064  1049
2
 6.6 2  107
 
6.28  22.7  1.38   2.61 6.28  22.7  1.38   2.61
2/3 2/3

0.221 107 0.221


   107  0.116  107  1.16 108
 2.61 2/3
1.895

NET/JRF (DEC-2016)
Q69. The partition function of a two-level system governed by the Hamiltonian
  
H 
 
is
 

(a) 2sinh   2   2
(b) 2 cosh       2 2

(c) cosh        sinh   


 2   2 
1 2 2

2 
(d) cosh        sinh   
1 2 2
 2  2
2 
Ans. : (b)
  
Solution: H  
   

H.No. 40-D, Ground Floor, Jia Sarai, Near IIT, Hauz Khas, New Delhi-110016
Phone: 011-26865455/+91-9871145498
Website: www.physicsbyfiziks.com | Email: fiziks.physics@gmail.com
281
fiziks
Institute for NET/JRF, GATE, IIT‐JAM, M.Sc. Entrance, JEST, TIFR and GRE in Physics

The eigen value is given by E1    2   2 and E2    2   2

Z  trace  e   H   e   E1  e  E2  e    2  2
e

    2  2   2 cosh   2  2

Q70. Consider a gas of N classical particles in a two-dimensional square box of side L . If the
total energy of the gas is E , the entropy (apart from an additive constant) is

 L2 E   LE  L E  E
(a) Nk B ln   (b) Nk B ln   (c) 2 Nk B ln   (d) L2 k B ln  
 N   N   N  N

Ans. : (c)
N
1  2 mkTL2  E
Solution: Z N    kT 
N h2  N
N
1  L2 E  2 m
ZN  1
N  N 
Assume
h2

 L2 E   L2 E 
ln Z   ln N  N ln     N ln N  N  N ln  
 N   N 
 L2 E 
 F  kT ln Z  NkT ln N  NkT  NkT ln  
 N 
E  NkT

U F EF  L2 E 
S   Nk  Nk ln N  Nk  Nk ln  
T T  N 
2
 L2 E  L E L E 
 Nk ln  2   Nk ln    2 Nk ln  
 N   N   N 
Q71. Consider a random walk on an infinite two-dimensional triangular lattice, a part of which
is shown in the figure below.

If the probabilities of moving to any of the nearest neighbour sites are equal, what is the
probability that the walker returns to the starting position at the end of exactly three steps?
1 1 1 1
(a) (b) (c) (d)
36 216 18 12

H.No. 40-D, Ground Floor, Jia Sarai, Near IIT, Hauz Khas, New Delhi-110016
Phone: 011-26865455/+91-9871145498
Website: www.physicsbyfiziks.com | Email: fiziks.physics@gmail.com
282
fiziks
Institute for NET/JRF, GATE, IIT‐JAM, M.Sc. Entrance, JEST, TIFR and GRE in Physics

Ans. : (c)
Solution: For walker to return to starting position it must move along an equivalent triangle in
three steps.
For steps one any movement can result in equilateral triangle.
For step two, two out of six options will form equilateral triangle.
For step three, only one out of six options will form equilateral triangle
6 2 1 1
Total probability    
6 6 6 18
Q72. An atom has a non-degenerate ground-state and a doubly-degenerate excited state. The
energy difference between the two states is  . The specific heat at very low temperatures
   1 is given by
(a) k B    (b) k B e   (c) 2k B    e  
2
(d) k B

Ans. : (c)
Solution: Assume energy at ground state is 0 and energy at first excited state is  . The partition
function is Z  1  2e   
2 e   
Energy 
1  2e  
 
 1 2
 U 
2e kT
   2  e kT  2
kT 
2
kT
Specific heat, CV    
 T V  
  2
 
1  2e 
kT
1  2e 
kT

   
 

2   1  2e
kT

2    1  2e 
 
   kT  
 2k   e  2k    e
 kT  1  2e    
2 2
 

 1  2e 
kT

 

CV  2k    e    ,   
2

Q73. The electrons in graphene can be thought of as a two-dimensional gas with a linear
 
energy-momentum relation E  p v , where p   px , p y  and v is a constant. If  is the

number of electrons per unit area, the energy per unit area is proportional to
(a)  3/ 2 (b)  (c)  1/ 3 (d)  2

H.No. 40-D, Ground Floor, Jia Sarai, Near IIT, Hauz Khas, New Delhi-110016
Phone: 011-26865455/+91-9871145498
Website: www.physicsbyfiziks.com | Email: fiziks.physics@gmail.com
283
fiziks
Institute for NET/JRF, GATE, IIT‐JAM, M.Sc. Entrance, JEST, TIFR and GRE in Physics

Ans. : (a)
Solution: The number of k state in range k to k  dk :
2
 L 
In 2D , it is given by g  k  dk    2 kdk
 2 
Since, dispersion relation is E  P v  kv
2
 L  EdE L2
g  E  dE  2    2 
 2   v    v
2 2 2

The number of electron at T  00 K is


EF EF
L2 L2 N
N  g Ed E   EdE  EF2  2  2 v 2 . 2  EF2
0
 2v2 0
2  v
2 2
L

 N
EF2  2  2 v 2    2 
 L 
The average energy at T  0 K is
EF

 E  g  E  dE L2
EF
L2 EF3
Eav  0

N

N  2v 2 
0
E 2 dE 
3N  2v 2
L 2
2 L2
Eav   2  v  2  v  
2 2 2 2 1/ 2
2 v  3/ 2
3N  v
2 2
3N
E NEa v 2 E
 2  2 v  3/ 2    3/ 2
L2
L 3 L2

NET/JRF (JUNE-2017)
Q74. A thermodynamic function
G T , P, N   U  TS  PV

is given in terms of the internal energy U , temperature T , entropy S , pressure P ,


volume V and the number of particles N . Which of the following relations is true? (In
the following  is the chemical potential.)

G G G G
(a) S   (b) S  (c) V   (d)   
T N ,P T N ,P P N ,T N P ,T

Ans. : (a)
Solution: G  U  TS  PV

H.No. 40-D, Ground Floor, Jia Sarai, Near IIT, Hauz Khas, New Delhi-110016
Phone: 011-26865455/+91-9871145498
Website: www.physicsbyfiziks.com | Email: fiziks.physics@gmail.com
284
fiziks
Institute for NET/JRF, GATE, IIT‐JAM, M.Sc. Entrance, JEST, TIFR and GRE in Physics

dG  dU  Tds  sdT  PdV  VdP  TdS  PdV  TdS  SdT  PdV  VdP
dG   SdT  VdP
 G  G
    S and V
 T  N , P P N ,T

Q75. A box, separated by a movable wall, has two compartments filled by a monoatomic gas
CP
of   . Initially the volumes of the two compartments are equal, but the pressures are
CV

3P0 and P0 respectively. When the wall is allowed to move, the final pressures in the two
compartments become equal. The final pressure is
  
2 2  31/  
(c) 1  31/   P0
1 
(a)   P0 (b) 3   P0 (d)  1/  
P0
3 3 2  1 3 
Ans. : (c)
Solution: V1  V2  2V , V2  2V  V1 ,

3PV
0  PV1 , PV
0

 PV2

 P  2V  V1 
 
PV
0

From (i) and (ii)



 V1  1/  V1 1 2V  V1 2V 2V
3  3   1/     1  V1 
 2V  V1  2V  V1 3 V1 V1 1  1/ 31/  
put the value of V1 in (i)

 2V  3P0 P0
3PV
0

 P 1/ 
 1  1/ 3 
  P 
2
1  1/ 3 
1/  

2
1  3 
1/  

Q76. A gas of photons inside a cavity of volume V is in equilibrium at temperature T . If the


temperature of the cavity is changed to 2T , the radiation pressure will change by a factor
of
(a) 2 (b) 16 (c) 8 (d) 4
Ans. : (b)
Solution: For 3 dimensional system P  T 4
4
P2  2T 
   P2  P1 2  p1 16  16 P1
4

P1  T 

H.No. 40-D, Ground Floor, Jia Sarai, Near IIT, Hauz Khas, New Delhi-110016
Phone: 011-26865455/+91-9871145498
Website: www.physicsbyfiziks.com | Email: fiziks.physics@gmail.com
285
fiziks
Institute for NET/JRF, GATE, IIT‐JAM, M.Sc. Entrance, JEST, TIFR and GRE in Physics

Q77. In a thermodynamic system in equilibrium, each molecule can exist in three possible
states with probabilities 1/ 2, 1/ 3 and 1/ 6 respectively. The entropy per molecule is
1 2
(a) k B ln 3 (b) k B ln 2  k B ln 3
2 3
2 1 1 1
(c) k B ln 2  k B ln 3 (d) k B ln 2  k B ln 3
3 2 2 6
Ans. : (c)
Solution: S  k B  Pi ln Pi
i

1
P1  , P2  1/ 3 and P3  1/ 6 
2
1 
S  k B  ln1/ 2  1/ 3ln1/ 3  1/ 6 ln1/ 6  
2 
1 1 1
 kb   ln1  ln 2    ln1  ln 3   ln1  ln 6 
2 3 6

1 1 1 1  1 1 1 1 
 k B  ln 2  ln 3  ln 2  ln 3  k B  ln 2  ln 2  ln 3  ln 3
2 3 6 6  2 6 3 6 
 3ln 2  ln 2 2ln 3  ln 3   4 ln 2 3ln 3  2 1 
S  kB     kB     k B  ln 2  ln 3
 6 6   6 6  3 2 
Q78. The single particle energy levels of a non-interacting three-dimensional isotropic system,
labelled by momentum k , are proportional to k 3 . The ratio P /  of the average pressure
P to the energy density  at a fixed temperature, is
(a) 1/ 3 (b) 2 / 3 (c) 1 (d) 3
Ans. : (c)
Solution: E  p s , where p is momentum

sE
P    , where P is pressure
3V 
P s
 .
E 3
In problem, E  k 3 , so, s  3
3 E  E
pressure P     P    at fixed T .
3V  V 

H.No. 40-D, Ground Floor, Jia Sarai, Near IIT, Hauz Khas, New Delhi-110016
Phone: 011-26865455/+91-9871145498
Website: www.physicsbyfiziks.com | Email: fiziks.physics@gmail.com
286
fiziks
Institute for NET/JRF, GATE, IIT‐JAM, M.Sc. Entrance, JEST, TIFR and GRE in Physics

Q79. The Hamiltonian for three Ising spins S0 , S1 and S2 , taking values 1 , is
H   JS0  S1  S 2 
If the system is in equilibrium at temperature T , the average energy of the system, in

terms of    k BT  , is
1

1  cosh  2 J 
(a)  (b) 2 J 1  cosh  2 J  
2  sinh  2 J 
sinh  2 J 
(c) 2 /  (d) 2 J
1  cosh  2  J 
Ans. : (d)
Solution: H   JS0  S1  S 2    J  S0 S1  S0 S2  S0  1 S1   S 2  1

S0 S1 S2 E
1 1 1 2J
1 1 1 2J
1 1 1 0
1 1 1 0
1 1 1 0
1 1 1 0
1 1 1 2J
1 1 1 2 J
E1  2 J g1  2

E2  2 J g2  2

E3  0 g3  4
E g e
  Ei

U i i

g e  i
 Ei

0  2 J 2e 2  J   2 J  2e 2  J 4 J  e 2  J  e 2  J  8 J   sinh 2 J 
U  
4  2 e e 
2  J 2 J
4  2e  2e 2  J 2 J
4  4 cosh 2  J

2 J sinh 2 J
U 
1  cosh 2 J

H.No. 40-D, Ground Floor, Jia Sarai, Near IIT, Hauz Khas, New Delhi-110016
Phone: 011-26865455/+91-9871145498
Website: www.physicsbyfiziks.com | Email: fiziks.physics@gmail.com
287
fiziks
Institute for NET/JRF, GATE, IIT‐JAM, M.Sc. Entrance, JEST, TIFR and GRE in Physics

NET/JRF (DEC - 2017)


Q80. A monoatomic gas ‘of volume V is in equilibrium in a uniform vertical cylinder, the
lower end of which is closed by a rigid wall and the other by a frictionless piston. The
piston is pressed lightly and released. Assume that the gas is a poor conductor of heat and
the cylinder and piston are perfectly insulating. If the cross-sectional area of the cylinder
is A , the angular frequency of small oscillations of the piston about the point of
equilibrium, is
5
(a) 5 gA /  3V  (b) 4 gA /  3V  (c) gA / V (d) 7 gA /  5V 
3
Ans. : (a)

 P    V 
Solution: PV    P  P V  V   PV   P 1 

V 1  
 P   V 
 
 P  V   P  V   P  V 
 PV   PV  1  1    1  1    1  1  1   1
 P  V   P  V   P  V 
V P P V
1    1 (i)
V P P V
P V
For small oscillation, also neglect
P V
F mg
From equilibrium P  
A A
P V mg A dx
From (i), we get   , P  A   mg. V , P  A   . A.
P V A V
mgdx  mg
P  A   A F  Adx
V V
 gA 5 5 gA
 and      .
V 3 3 V
Q81. The relation between the internal energy U , entropy S , temperature T , pressure p ,
volume V , chemical potential  and number of particles N of a thermodynamic system
is dU  TdS  pdV   dN . That U is an exact differential implies that

p T U U
(a)   (b) p S
S V , N V S ,N T S ,N V S ,

U 1 U p T
(c) p  (d) 
T S ,N T V S , S V , N V S ,N

Ans. : (a)

H.No. 40-D, Ground Floor, Jia Sarai, Near IIT, Hauz Khas, New Delhi-110016
Phone: 011-26865455/+91-9871145498
Website: www.physicsbyfiziks.com | Email: fiziks.physics@gmail.com
288
fiziks
Institute for NET/JRF, GATE, IIT‐JAM, M.Sc. Entrance, JEST, TIFR and GRE in Physics

Solution: df  Adx  Bdy  Cdz


If f is perfect differential then, A  T , B   p, C  
x  S y V, z  N

 A   B 
   
 y  x , z  x  y , z

 A   C   B   C 
    ,   
 z  x , y  x  y , z  z  x , y  y  x , z

 T   P 
    
 V  S , N  S V , N
Q82. The number of microstates of a gas of N particles in a volume V and of internal energy
U , is given by
3N / 2
 aU 
 U , V , N   V  Nb  
N

 N 
(where a and b are positive constants). Its pressure P , volume V and temperature T ,
are related by
 aN   aN 
(a)  P   V  Nb   Nk BT (b)  P  2  V  Nb   Nk BT
 V   V 

(c) PV  Nk BT (d) P V  Nb   Nk BT

Ans. : (d)
3N
 aU  2
Solution:   u , v, N   V  Nb  
N

 N 
 3  aU 
S  k ln   Nk ln V  Nb   ln  
 2  N 
1 P
dS  dU  dV
T T
 S  1 3
    U  NkT
 U V T 2

 S  P P Nk
    
 V U T T V  Nb 

P V  Nb   NkT

H.No. 40-D, Ground Floor, Jia Sarai, Near IIT, Hauz Khas, New Delhi-110016
Phone: 011-26865455/+91-9871145498
Website: www.physicsbyfiziks.com | Email: fiziks.physics@gmail.com
289
fiziks
Institute for NET/JRF, GATE, IIT‐JAM, M.Sc. Entrance, JEST, TIFR and GRE in Physics

Q83. A closed system having three non-degenerate energy levels with energies E  0,   , is
at temperature T . For  2k BT , the probability of finding the system in the state with

energy E  0 , is
1 1 1 1
(a) (b) (c) cosh 2 (d)
1  2 cosh 2   2 cosh 2  2 cosh 2

Ans. : (a)
  

 kT
 
 e e
kT kT
  1  2 cosh 
Solution: Partition function is z  1   e  e kT   z  1  2 
  kT
   2 
 
Probability that system has energy, E  0
1
P  E  0 

1  2 cosh
kT
put  2kT
1
P  E  0 
1  2 cosh 2
Q84. Two non-degenerate energy levels with energies 0 and  are occupied by N non-
interacting particles at a temperature T . Using classical statistics, the average internal
energy of the system is
N N 3
(a) (b) (c) N  e/ kBT (d) Nk BT

1  e/ kBT  
1  e/ kBT  2

Ans. : (a)
Solution: For one particle
 
exp   
Quantum mechanical energy is, U   kT 

1 e kT

 
N exp   
 kT   U  N
For N particle, U 
   
1  exp    1  exp  
 kT   kT 

H.No. 40-D, Ground Floor, Jia Sarai, Near IIT, Hauz Khas, New Delhi-110016
Phone: 011-26865455/+91-9871145498
Website: www.physicsbyfiziks.com | Email: fiziks.physics@gmail.com
290
fiziks
Institute for NET/JRF, GATE, IIT‐JAM, M.Sc. Entrance, JEST, TIFR and GRE in Physics

Q85. Consider a quantum system of non-interacting bosons in contact with a particle bath. The
probability of finding no particle in a given single particle quantum state is 106 . The
average number of particles in that state is of the order of
(a) 103 (b) 106 (c) 109 (d) 1012
Ans. : (b)

NET/JRF (JUNE-2018)
Q86. Which of the following statements concerning the coefficient of volume expansion 
and the isothermal compressibility  of a solid is true?
(a)  and  are both intensive variables
(b)  is an intensive and  is an extensive variable
(c)  is an extensive and  is an intensive variable
(d)  and  are both extensive variables
Ans. : (a)
1  dV  1  P 
Solution:    ,      both are intensive property
V  dT  V  V T
Q87. The number of ways of distributing 11 indistinguishable bosons in 3 different energy
levels is

(a) 311 (b) 113 (c)


13! (d)
11!
2!11 ! 3!8!
Ans. : (c)
Solution: n  11 g  3
n  g 1 11  3  1 13
w  
n g 1 11 2 11 2
 a 
Q88. The van der Waals equation for one mole of a gas is  p  2  V  b   RT . The
 V 
corresponding equation of state for n moles of this gas at pressure P , volume V and
temperature T , is
 an 2   a 
(a)  P  2  V  nb   nRT (b)  P  2  V  nb   nRT
 V   V 
 an 2   a 
(c)  P  2  V  nb   nRT (d)  P  2  V  nb   nRT
 V   V 

H.No. 40-D, Ground Floor, Jia Sarai, Near IIT, Hauz Khas, New Delhi-110016
Phone: 011-26865455/+91-9871145498
Website: www.physicsbyfiziks.com | Email: fiziks.physics@gmail.com
291
fiziks
Institute for NET/JRF, GATE, IIT‐JAM, M.Sc. Entrance, JEST, TIFR and GRE in Physics

Ans. : (a)
Solution: For n mole gas van der Wall equation is
 an 2 
 p   V  nb   nRT
 V 

Q89. In a system of N distinguishable particles, each particle can be in one of two states with
energies 0 and  E , respectively. The mean energy of the system at temperature T is
1

(a)  N 1  e / kBT
2
 (b)  NE 1  e / kBT  
1
(c)  NE
2
(d)  NE 1  e  / kBT 
Ans. : (d)
Solution: For one particle system
0

0  e k BT    E  e  E / k BT  E e E / k BT E  NE
E     E / k BT
E 
e 0 / k BT
e E / k BT
1  e E / k BT e 1 1  e  E / k BT
Q90. The pressure P of a system of N particles contained in a volume V at a temperature T
1 1
is given by P  nk B T  an 2  bn3 , where n is the number density and a and b are
2 6
temperature independent constants. If the system exhibits a gas-liquid transition, the
critical temperature is
a a a2 a2
(a) (b) 2 (c) (d) 2
bk B 2b k B 2bk B b kB
Ans. : (c)
1 1 N
Solution: P  nk BT  an 2  bn3 n
2 6 V
P 2P
For critical condition  0 and 0
V V 2
N 1 N2 1 N3
P k BT  a 2  b 3
V 2 V 6 V
P aN 2 bN 3
 0  Nk BT   2 (i)
V V 2V

H.No. 40-D, Ground Floor, Jia Sarai, Near IIT, Hauz Khas, New Delhi-110016
Phone: 011-26865455/+91-9871145498
Website: www.physicsbyfiziks.com | Email: fiziks.physics@gmail.com
292
fiziks
Institute for NET/JRF, GATE, IIT‐JAM, M.Sc. Entrance, JEST, TIFR and GRE in Physics

2 P 3aN 2 2bN 3
 0  2 Nk B T   2 (ii)
V 2 V V
From equation (i) and (ii)
bN
VC 
a
b
put the value of VC  N in equation (i)
a
a2
T
2k B b
Q91. Consider a particle diffusing in a liquid contained in a large box. The diffusion constant
of the particle in the liquid is 1.0  102 cm 2 / s . The minimum time after which the root-
mean-squared displacement becomes more than 6 cm is

(a) 10 min (b) 6 min (c) 30 min (d) 6 min


Ans. : (a)
Solution: r 2  6 Dt

r 2   r.m.s    6 c.m  D  1102 c.m 2 / sec


2 2

r2 6
2

t   600 sec  10 min


60 6 1102
Q92. A thermally insulated chamber of dimensions  L, L, 2 L  is partitioned in the middle. One

side of the chamber is filled with n moles of an ideal gas at a pressure P and
temperature T , while the other side is empty. At t  0 , the partition is removed and the
gas is allowed to expand freely. The time to reach equilibrium varies as

L P, T

2L
(a) n1/ 3 L1T 1/ 2 (b) n 2 / 3 LT 1/ 2 (c) n0 LT 1/ 2 (d) n L1T 1/ 2
Ans. : (c)

H.No. 40-D, Ground Floor, Jia Sarai, Near IIT, Hauz Khas, New Delhi-110016
Phone: 011-26865455/+91-9871145498
Website: www.physicsbyfiziks.com | Email: fiziks.physics@gmail.com
293
fiziks
Institute for NET/JRF, GATE, IIT‐JAM, M.Sc. Entrance, JEST, TIFR and GRE in Physics

m v2 m 3kT
Solution: From kinetic theory of gases F   MLt 2  where nM  m M is
L L M
molecular mass
t 2  L2T  t  LT 1/ 2
o
Q93. The maximum intensity of solar radiation is at the wavelength of s u n ~ 5000 A and

corresponds to its surface temperature Tsun ~ 104 K . If the wavelength of the maximum
o
intensity of an X -ray star is 5 A , its surface temperature is of the order of
(a) 1016 K (b) 1014 K (c) 1010 K (d) 107 K
Ans. : (d)
Solution: From Vein’s law, Tmax sun  constant
0
5000 A00 104  5 A  T

5000  104
T T  107 K
5

NET/JRF (DEC - 2018)


Q94. The heat capacity CV at constant volume of a metal, as a function of temperature, is

 T   T 3 , where  and  are constants. The temperature dependence of the entropy at


constant volume is
1
(a)  T   T 3 (b)  T   T 3
3
1 1 1 1
(c) T   T 3 (d) T   T 3
2 3 2 4
Ans. : (a)
Solution: CV   T   T 3

dQ CV dT
dS  
T T

 dS      T dT
2

1
S  T   T 3
3
H.No. 40-D, Ground Floor, Jia Sarai, Near IIT, Hauz Khas, New Delhi-110016
Phone: 011-26865455/+91-9871145498
Website: www.physicsbyfiziks.com | Email: fiziks.physics@gmail.com
294
fiziks
Institute for NET/JRF, GATE, IIT‐JAM, M.Sc. Entrance, JEST, TIFR and GRE in Physics

2
Q95. The rotational energy levels of a molecule are E      1 , where   0,1, 2,... and
2I0

I 0 is its moment of inertia. The contribution of the rotational motion to the Helmholtz
free energy per molecule, at low temperatures in a dilute gas of these molecules, is
approximately
2
 2  
(a)  k BT 1   (b) k BTe I 0 k BT

 I 0 k BT 
2

(c) k BT (d) 3k BTe I 0 k BT

Ans. : (d)
2
Solution: E      1   0,1, 2,...
2I0
   2    1

z    2  1 e 2 I0

 0

  2    1

z  1    2  1 e 2 I 0 k BT

 0

     1  2

F  k BT ln z   k BT ln  1    2  1 e 2 I0 kBT 
  1 
 
x2
ln 1  x   x   ...
2
For low temperature, higher temperature can be neglected
  2    1   2
  2
  
F   k BT   2  1 e 2 I 0 k BT
  k B T 3 e I 0 k BT
 ...  3k B T e I 0 k BT

 1  
 
Q96. The vibrational motion of a diatomic molecule may be considered to be that of a simple
harmonic oscillator with angular frequency  . If a gas of these molecules is at
temperature T , what is the probability that a randomly picked molecule will be found in
its lowest vibrational state?
 
     1   
(a) 1  e k BT
(b) e 2 k BT
(c) tanh   (d) cosec h  
 k BT  2  2 k BT 
H.No. 40-D, Ground Floor, Jia Sarai, Near IIT, Hauz Khas, New Delhi-110016
Phone: 011-26865455/+91-9871145498
Website: www.physicsbyfiziks.com | Email: fiziks.physics@gmail.com
295
fiziks
Institute for NET/JRF, GATE, IIT‐JAM, M.Sc. Entrance, JEST, TIFR and GRE in Physics

Ans. : (a)
 1
Solution: E   n    n  0,1, 2,..
 2
  3  5 
 k BT
ze 2 k BT
e 2 k BT
e
   
 
e 2 k BT e 2 k BT 1
P  G.S .    3 
    
  1  e k BT
  1
e 2 k BT
e 2 k BT
 ... e 2 kBT 1  e kBT  ...   
 
  1 e k BT

Q97. Consider an ideal Fermi gas in a grand canonical ensemble at a constant chemical
potential. The variance of the occupation number of the single particle energy level with
mean occupation number n is
1
(a) n 1  n  (b) n (c) n (d)
n
Ans. : (a)
1  z  1
Solution: n  k BT       / kBT
z   V ,T e 1

 dn 
Variance  k BT    n 1  n 
 d  V ,T
Note: This may also be divided using simple Bernoulli distribution.

Q98. The Hamiltonian of a one-dimensional Ising model of N spins ( N large) is


N
H   J   i i 1
i 1

1
where the spin  i  1 and J is a positive constant. At inverse temperature   ,
k BT

the correlation function between the nearest neighbor spins  i i 1  is

e  J
(a) (b) e 2  J
 e  J  e  J 
(c) tanh   J  (d) coth   J 

Ans. : (c)

H.No. 40-D, Ground Floor, Jia Sarai, Near IIT, Hauz Khas, New Delhi-110016
Phone: 011-26865455/+91-9871145498
Website: www.physicsbyfiziks.com | Email: fiziks.physics@gmail.com
296
fiziks
Institute for NET/JRF, GATE, IIT‐JAM, M.Sc. Entrance, JEST, TIFR and GRE in Physics

Solution:  i   i 1 
 i   i 1

 i   i 1
N  1 
11
(i)
N 1 N  JN
For such an Ising model for N  1

z   cosh  J 
N


Average Energy  ln z

1
 N sinh  J  J
cosh  J
  NJ tanh  J (ii)
 Nj tanh  j
 i   i 1   tanh  j
 jN
Q99. A particle hops on a one-dimensional lattice with lattice spacing a . The probability of
the particle to hop to the neighboring site to its right is p , while the corresponding
probability to hop to the left is q  1  p . The root-mean squared deviation

x  x2  x
2
in displacement after N steps, is
q p

O
(a) a Npq (b) aN pq (c) 2a Npq (d) a N
Ans. : (c)
Solution: The standard deviation of Binomial distribution  Npq

Step size  2a (L & R)


Mean square displacement  2a Npq

Q100. The energy levels accessible to a molecule have energies E1  0, E2   and E3  2


(where  is a constant). A gas of these molecules is in thermal equilibrium at
temperature T . The specific heat at constant volume in the high temperature limit
 k BT    varies with temperature as
1 1 1 1
(a) (b) (c) (d)
T 3/ 2
T3 T T2

H.No. 40-D, Ground Floor, Jia Sarai, Near IIT, Hauz Khas, New Delhi-110016
Phone: 011-26865455/+91-9871145498
Website: www.physicsbyfiziks.com | Email: fiziks.physics@gmail.com
297
fiziks
Institute for NET/JRF, GATE, IIT‐JAM, M.Sc. Entrance, JEST, TIFR and GRE in Physics

Ans. : (d)

Solution: z  e0  e  / kBT  e 2  / kBT  1
kBT

z  1  e  / kBT  e 2  / kBT
A  k BT ln z  k BT ln 1  e  / kBT  e 2  / kBT 

  2 
A   k B T ln 1  1  ....  1  ...
 kBT kBT 

 3 
  k BT ln 3  
 k BT 

 
   3  1 3 
  k B 1ln 3    T   
T   kBT  3 k B T 2 
3
 kBT 
 

 
  2  1 3 1 3 3  3  3 
2

  k      1  3   
 3  3 k BT 3 k BT 2 k BT
B
T 2 2
3  k B T  k B T 2

 k BT k BT 

kB  2 kB  2
 
T  k BT    T  k B2T 2
2

2 A 1
CV  T  2
T 2
T

H.No. 40-D, Ground Floor, Jia Sarai, Near IIT, Hauz Khas, New Delhi-110016
Phone: 011-26865455/+91-9871145498
Website: www.physicsbyfiziks.com | Email: fiziks.physics@gmail.com
298
fiziks
Institute for NET/JRF, GATE, IIT‐JAM, M.Sc. Entrance, JEST, TIFR and GRE in Physics

ELECTRONICS AND EXPERIMENTAL METHODS


NET/JRF (JUNE-2011)

Q1. A signal of frequency 10 k Hz is being digitalized by an A/D converter. A possible


sampling time which can be used is
(a) 100  s (b) 40  s (c) 60  s (d) 200  s
Ans. : (b)
1 1
Solution: f S  2 f  TS    50 s  TS  50 s
2 f 20  103
Q2. Consider the digital circuit shown below in which the input C is always high (1).
A
B Z

C
(high)

The truth table for the circuit can be written as


A B Z
0 0 1
0 1 0
1 0 1
1 1 1

The entries in the Z column (vertically) are


(a) 1010 (b) 0100 (c) 1111 (d) 1011
Ans. : (d)
Solution: Z  A.B  B  1

H.No. 40-D, Ground Floor, Jia Sarai, Near IIT, Hauz Khas, New Delhi-110016
Phone: 011-26865455/+91-9871145498
Website: www.physicsbyfiziks.com | Email: fiziks.physics@gmail.com
298
fiziks
Institute for NET/JRF, GATE, IIT‐JAM, M.Sc. Entrance, JEST, TIFR and GRE in Physics

Q3. A time varying signal Vin is fed to an op-amp circuit with output signal Vo as shown in
10 K
the figure below.
The circuit implements a
(a) high pass filter with cutoff frequency 16 Hz 1K

Vo
(b) high pass filter with cutoff frequency 100 Hz V in 
1K 10 K
10 K
(c) low pass filter with cutoff frequency 16 Hz 1 F

(d) low pass filter with cutoff frequency 100 Hz 


Ans. : (c)
Solution: Since circuit has R and C combination, its a Low Pass filter and cutoff frequency
1
  16 Hz.
2RC

NET/JRF (DEC-2011)
Q4. In the operational amplifier circuit below, the voltage at point A is
5V
1K A
1V 
1V 
1K
1K 5V

(a) 1.0 V (b) 0.5V (c) 0 V (d) 5.0V


Ans. : (b)
1
Solution: V A   1  0.5V .
11

H.No. 40-D, Ground Floor, Jia Sarai, Near IIT, Hauz Khas, New Delhi-110016
Phone: 011-26865455/+91-9871145498
Website: www.physicsbyfiziks.com | Email: fiziks.physics@gmail.com
299
fiziks
Institute for NET/JRF, GATE, IIT‐JAM, M.Sc. Entrance, JEST, TIFR and GRE in Physics

Q5. A counter consists of four flip-flops connected as shown in the figure:


A0 A1 A2 A3

J Q J Q J Q J Q
CLK
K Q K Q K Q K Q

If the counter is initialized as A0 A1 A2 A3  0110 , the state after the next clock pulse is

(a) 1000 (b) 0001 (c) 0011 (d) 1100


Ans. : (b)
A 0 0 A1 1 A 2 1 A 3 0
Solution:

0 J Q J Q 0 J Q J Q
CLK
1 K Q K Q 0 1 K Q K Q
1 0 1

Q6. The pins 0, 1, 2 and 3 of part A of a microcontroller are connected with resistors to drive
an LED at various intensities as shown in the figure. For V CC
VCC = 4.2 V and a voltage drop of 1.2 V across the LED, 
the range (maximum current) and resolution (step size)
A3
of the drive current are, respectively, 0 . 75 k
A2
(a) 4.0 mA and 1.0 mA C 1 .5 k
A1
(b) 15.0 mA and 1.0 mA 3k
(c) 7.5 mA and 0.5 mA A0
6k
(d) 4.0 mA and 0.5 mA
Ans. : (c)
A3 , A2 , A1 , A0
Solution: For Maximum current
0, 0, 0, 0
4.2  1.2 4.2  1.2 4.2  1.2 4.2  1.2
Thus, I max      7.5mA
0.75k 1.5k 3k 6k

H.No. 40-D, Ground Floor, Jia Sarai, Near IIT, Hauz Khas, New Delhi-110016
Phone: 011-26865455/+91-9871145498
Website: www.physicsbyfiziks.com | Email: fiziks.physics@gmail.com
300
fiziks
Institute for NET/JRF, GATE, IIT‐JAM, M.Sc. Entrance, JEST, TIFR and GRE in Physics

A3 , A2 , A1 , A0 4.2  1.2
For Step size . Thus I 0   0.5mA
0, 0, 0, 1 6k
Q7. The figure below shows a voltage regulator utilizing a Zener diode of breakdown voltage
5 V and a positive triangular wave input of amplitude 10 V.
500 12
Vi 10
i 8

i(mA)
6
1K 4
2
0
0 1 2 3 4 5 6 7 8
t (s)
For Vi > 5V, the Zener regulates the output voltage by channeling the excess current
through itself. Which of the following waveforms shows the current i passing through the
Zener diode?
(a) 12 (b) 12
10
10
8
i(mA)

8
6
i(mA)

6
4
4
2
2
0
0 1 2 3 4 5 6 7 8 0
0 1 2 3 4 5 6 7 8
t (s) t (s)
(c) 12 (d) 12
10 10
8 8
i(mA)
i(mA)

6 6
4 4
2 2
0 0
0 1 2 3 4 5 6 7 8 0 1 2 3 4 5 6 7 8
t (s) t (s)
Ans. : (a)
Solution: When zener is OFF zener current is zero when zener is ON zener current will flow.

H.No. 40-D, Ground Floor, Jia Sarai, Near IIT, Hauz Khas, New Delhi-110016
Phone: 011-26865455/+91-9871145498
Website: www.physicsbyfiziks.com | Email: fiziks.physics@gmail.com
301
fiziks
Institute for NET/JRF, GATE, IIT‐JAM, M.Sc. Entrance, JEST, TIFR and GRE in Physics

NET/JRF (JUNE-2012)
Q8. In the op-amp circuit shown in the figure below, the input voltage is 1V. The value of the
1K
output V0 is

1K
 1K Vo
Vi  1V

1K

(a) -0.33 V (b) -0.50 V (c) -1.00 V (d) -0.25 V


Ans. : (b)
R F Vin 1 11 1
Solution: V0     V  0.05 where R F   K and R1  1K .
R1 2 11 2
Q9. An LED operates at 1.5 V and 5 mA in forward bias. Assuming an 80% external
efficiency of the LED, how many photons are emitted per second?
(a) 5.0 x 1016 (b) 1.5 x 1016 (c) 0.8 x 1016 (d) 2.5 x 1016
Ans. : (d)
i Pin i 5  103
Solution: Pin  ext hf , number of photon   ext  0.8  19
 2.5  1016
e hf e 1.6  10

Q10. The transistor in the given circuit has hfe = 35Ω and hie = 1000Ω. If the load resistance
RL = 1000Ω, the voltage and current gain are, respectively.
(a) -35 and + 35 VO
RL
(b) 35 and - 35

(c) 35 and – 0.97
VI 

(d) 0.98 and - 35

Ans. : (a)

H.No. 40-D, Ground Floor, Jia Sarai, Near IIT, Hauz Khas, New Delhi-110016
Phone: 011-26865455/+91-9871145498
Website: www.physicsbyfiziks.com | Email: fiziks.physics@gmail.com
302
fiziks
Institute for NET/JRF, GATE, IIT‐JAM, M.Sc. Entrance, JEST, TIFR and GRE in Physics

Q11. The output, O, of the given circuit in cases I and II, where
Case I: A, B = 1; C, D = 0; E, F = 1 and G = 0
Case II: A, B = 0; C, D = 0: E, F = 0 and G = 1
are respectively 

(a) 1, 0
(b) 0, 1 C
D
(c) 0, 0
(d) 1, 1
Ans. : (d) O

 
Solution: O  AB  CD E  F G  E F
G

NET/JRF (DEC-2012)
Q12. A live music broadcast consists of a radio-wave of frequency 7 MHz, amplitude-
modulated by a microphone output consisting of signals with a maximum frequency of
10 kHz. The spectrum of modulated output will be zero outside the frequency band
(a) 7.00 MHz to 7.01 MHz (b) 6.99 MHz to 7.01 MHz
(c) 6.99 MHz to 7.00 MHz (d) 6.995 MHz to 7.005 MHz
Ans. : (b)
Solution: Spectrum consists of f c  f m and f c  f m .
Q13. In the op-amp circuit shown in the figure, Vi is a sinusoidal input signal of frequency 10
Hz and V0 is the output signal. The magnitude of the gain and the phase shift, respectively,
close to the values 0.01F

(a) 5 2 and  / 2
(b) 5 2 and   / 2 10 K
1K
(c) 10 and zero
Vi 
Vo
(d) 10 and π 
Ans. : (d)
v0 X C RF v
Solution:   0  10
vin R1 R1  R F  vin

H.No. 40-D, Ground Floor, Jia Sarai, Near IIT, Hauz Khas, New Delhi-110016
Phone: 011-26865455/+91-9871145498
Website: www.physicsbyfiziks.com | Email: fiziks.physics@gmail.com
303
fiziks
Institute for NET/JRF, GATE, IIT‐JAM, M.Sc. Entrance, JEST, TIFR and GRE in Physics

Q14. The logic circuit shown in the figure below Implements the Boolean expression
A

HIGH y

(a) y  A  B (b) y  A  B (c) y  A  B (d) y  A  B


Ans. : (a)

Solution: Output of each Ex-OR gate is A and B . Thus y  A  B  A  B


Q15. A diode D as shown in the circuit has an i-v relation that can be approximated by
v 2  2v D , for v D  0
iD   D
0, for v D  0 1

The value of v D in the circuit is iD




(a)  1  11 V  (b) 8 V 10 V

D vD

(c) 5 V (d) 2 V
Ans. : (d)
Solution:  10  v D2  2v D  1  v D  0  v D  2V
Q16. Band-pass and band-reject filters can be implemented by combining a low pass and a
high pass filter in series and in parallel, respectively. If the cut-off frequencies of the low
pass and high pass filters are  0LP and  0HP , respectively, the condition required to
implement the band-pass and band-reject filters are, respectively,
(a)  0HP   0LP and  0HP   0LP (b) 0HP  0LP and  0HP   0LP

(c)  0HP   0LP and 0HP  0LP (d)  0HP   0LP and 0HP  0LP
Ans. : (b)

H.No. 40-D, Ground Floor, Jia Sarai, Near IIT, Hauz Khas, New Delhi-110016
Phone: 011-26865455/+91-9871145498
Website: www.physicsbyfiziks.com | Email: fiziks.physics@gmail.com
304
fiziks
Institute for NET/JRF, GATE, IIT‐JAM, M.Sc. Entrance, JEST, TIFR and GRE in Physics

NET/JRF (JUNE-2013)

Q17. A silicon transistor with built-in voltage 0.7 V is used in the circuit shown, with
V BB  9.7V , R B  300k, VCC  12V and RC  2k . Which of the following figures

correctly represents the load line and quiescent Q point?

RC

RB 
 
V CC
V BB

iC iC
    mA  
32 35 6 35
(a) 32 (b) 32
Q
30 Q
30
0 9 .7 VCE V  0 12 VCE V 

iC iC
mA   A  
6 35 32 Q 35
(c) Q
32 (d) 32
30 30
0 12 VCE V  0 9 .7 VCE V 

Ans. : (b)
VBB  VBE 9.7  0.7 V 12
Solution: I B    30 A and I C , sat  CC   6mA
RB 300  10 3
RC 2  10 3
Q18. If the analog input to an 8-bit successive approximation ADC is increased from 1.0 V to
2.0 V, then the conversion time will
(a) remain unchanged (b) double
(c) decrease to half its original value (d) increase four times
Ans. : (a)

H.No. 40-D, Ground Floor, Jia Sarai, Near IIT, Hauz Khas, New Delhi-110016
Phone: 011-26865455/+91-9871145498
Website: www.physicsbyfiziks.com | Email: fiziks.physics@gmail.com
305
fiziks
Institute for NET/JRF, GATE, IIT‐JAM, M.Sc. Entrance, JEST, TIFR and GRE in Physics

Q19. The input to a lock-in amplifier has the form Vi t   Vi sin  t   i  where Vi ,  ,  i are the
amplitude, frequency and phase of the input signal respectively. This signal is multiplied
by a reference signal of the same frequency  , amplitude Vr and phase  r . If the
multiplied signal is fed to a low pass filter of cut-off frequency  , then the final output
signal is
1  1 
(a) ViVr cos i   r  (b) ViVr cos i   r   cos  t   i   r 
2  2 

 1 
(c) ViVr sin  i   r  (d) ViVr cos i   r   cos  t   i   r 
 2 
Ans. : (a)
Vi V r
Solution: V  Vr sin  t   r   Vi sin  t   i   cos i   r   cos2 t   i   r 
2
Vi V r
Output of low pass filter= cos i   r 
2
Q20. Four digital outputs V , P, T and H monitor the speed v , tyre pressure p , temperature t
and relative humidity h of a car. These outputs switch from 0 to 1 when the values of the
parameters exceed 85 km/hr, 2 bar, 40 0 C and 50%, respectively. A logic circuit that is
used to switch ON a lamp at the output E is shown below.
Which of the following condition will not switch the lamp ON?
(a) v  85km / hr , p  2 bar , t  40 0 C , h  50%
V
(b) v  85km / hr , p  2 bar , t  40 C , h  50%
0

P
(c) v  85km / hr , p  2 bar , t  40 0 C , h  50% E

(d) v  85km / hr , p  2 bar , t  400 C , h  50% T

H
Ans. : (d)

H.No. 40-D, Ground Floor, Jia Sarai, Near IIT, Hauz Khas, New Delhi-110016
Phone: 011-26865455/+91-9871145498
Website: www.physicsbyfiziks.com | Email: fiziks.physics@gmail.com
306
fiziks
Institute for NET/JRF, GATE, IIT‐JAM, M.Sc. Entrance, JEST, TIFR and GRE in Physics

JRF/NET-(DEC-2013)
Q21. Consider the op-amp circuit shown in the figure.
If the input is a sinusoidal wave Vi  5 sin 1000t  , then 1 F

the amplitude of the output V0 is


1K
5 1K
(a) (b) 5 Vi  Vo
2 
5 2
(c) (d) 5 2
2
Ans. : (c)
vo X RF X C 10 3 1
Solution:   F , XF   where R F  1  10 3 , X C 
vin R1 RF  X C 1  j  j  10  10 6
3

vo 10 3 1 1 5 5 2
  3   vo  sin t  sin t
vin 2 10 2 2 2

Q22. If one of the inputs of a J-K flip flop is high and the other is low, then the outputs Q and

Q
(a) oscillate between low and high in race around condition
(b) toggle and the circuit acts like a T flip flop
(c) are opposite to the inputs
(d) follow the inputs and the circuit acts like an R  S flip flop
Ans. : (d)
Q23. A sample of Si has electron and hole mobilities of 0.13 and 0.05 m 2 /V- s respectively at
300 K. It is doped with P and Al with doping densities of 1.5  10 21 / m 3 and
2.5  10 21 / m 3 respectively. The conductivity of the doped Si sample at 300 K is
(a) 8  1 m 1 (b) 32  1 m 1 (c) 20.8  1 m 1 (d) 83.2  1 m 1
Ans. : (a)
Solution: Resulting doped crystal is p-type and p p  2.5  1.5  10 21 / m 3  1  10 21 / m 3

  en p  n  p p  p   ep p  p  1.6  10 19  1  10 21  0.05  8  1 m 1

H.No. 40-D, Ground Floor, Jia Sarai, Near IIT, Hauz Khas, New Delhi-110016
Phone: 011-26865455/+91-9871145498
Website: www.physicsbyfiziks.com | Email: fiziks.physics@gmail.com
307
fiziks
Institute for NET/JRF, GATE, IIT‐JAM, M.Sc. Entrance, JEST, TIFR and GRE in Physics

Q24. Two identical Zener diodes are placed back to back in series and are connected to a
variable DC power supply. The best representation of the I-V characteristics of the circuit
is
(a) I (b) I

V V

(c) I (d) I

V V

Ans. : (d)
Q25. A 4-variable switching function is given by f   5, 7, 8, 10, 13, 15  d 0, 1, 2  , where

d is the do-not-care-condition. The minimized form of f in sum of products (SOP) form


is
(a) A C  B D (b) AB  CD (c) AD  BC (d) B D  BD

Ans. : (d) CD CD CD CD
AB    BD

AB 1 1 BD
AB 1 1

AB 1 1

H.No. 40-D, Ground Floor, Jia Sarai, Near IIT, Hauz Khas, New Delhi-110016
Phone: 011-26865455/+91-9871145498
Website: www.physicsbyfiziks.com | Email: fiziks.physics@gmail.com
308
fiziks
Institute for NET/JRF, GATE, IIT‐JAM, M.Sc. Entrance, JEST, TIFR and GRE in Physics

NET/JRF (JUNE-2014)
Q26. The inner shield of a triaxial conductor is driven by an (ideal) op-amp follower circuit as
shown. The effective capacitance between the signal-carrying conductor and ground is

Signal


(a) unaffected (b) doubled (c) halved (d) made zero


Ans. : (a)
Q27. An op-amp based voltage follower
(a) is useful for converting a low impedance source into a high impedance source.
(b) is useful for converting a high impedance source into a low impedance source.
(c) has infinitely high closed loop output impedance
(d) has infinitely high closed loop gain
Ans. : (b)
Q28. An RC network produces a phase-shift of 30 o . How many such RC networks should be
cascaded together and connected to a Common Emitter amplifier so that the final circuit
behaves as an oscillator?
(a) 6 (b) 12 (c) 9 (d) 3
Ans. : (a)
Solution: Total phase shift must be 0 or 3600. Common Emitter amplifier has phase change of
1800 so we need 6 RC network for next 1800 phase shift.

H.No. 40-D, Ground Floor, Jia Sarai, Near IIT, Hauz Khas, New Delhi-110016
Phone: 011-26865455/+91-9871145498
Website: www.physicsbyfiziks.com | Email: fiziks.physics@gmail.com
309
fiziks
Institute for NET/JRF, GATE, IIT‐JAM, M.Sc. Entrance, JEST, TIFR and GRE in Physics

Q29. For the logic circuit shown in the below

X
B

A simplified equivalent circuit is A


(a) A (b) B
B X X
C
C

A A
(c) (d)
B B
X X
C C
Ans. : (d)
Solution:

A A
A.B  A  B  AC
B X
B

C
C
ABC

X   A  B  AC  ABC  AC  ABC  ABC  AC  AB  A  B  C 

H.No. 40-D, Ground Floor, Jia Sarai, Near IIT, Hauz Khas, New Delhi-110016
Phone: 011-26865455/+91-9871145498
Website: www.physicsbyfiziks.com | Email: fiziks.physics@gmail.com
310
fiziks
Institute for NET/JRF, GATE, IIT‐JAM, M.Sc. Entrance, JEST, TIFR and GRE in Physics

NET/JRF (DEC-2014)
Q30. Consider the amplifier circuit comprising of the two op-amps A1 and A2 as shown in the
figure. 1M
R
 10 K

 r
A1  V0
A2

If the input ac signal source has an impedance of 50 k  , which of the following


statements is true?
(a) A1 is required in the circuit because the source impedance is much greater than r
(b) A1 is required in the circuit because the source impedance is much less than R
(c) A1 can be eliminated from the circuit without affecting the overall gain
(d) A1 is required in the circuit if the output has to follow the phase of the input signal
Ans. : (a)
Solution: A1 is required in the circuit because the source impedance is much greater than r
Q31. The I  V characteristics of the diode in the circuit below is given by
V  0.7  / 500 for V  0.7
I  
 0 for V  0.7
where V is measured in volts and I is measured in amperes.
1K I

10 V

The current I in the circuit is


(a) 10.0 mA (b) 9.3 mA (c) 6.2 mA (d) 6.7 mA
Ans. : (c)
Solution: Applying K.V.L. 10  1000  I  V  0  10  1000  V  0.7  / 500  V  0

 10  2 V  0.7   V  0  3V  11.4  V  3.8 Volts

Thus I  V  0.7  / 500   3.8  0.7  / 500  3.1/ 500  6.2 mA

H.No. 40-D, Ground Floor, Jia Sarai, Near IIT, Hauz Khas, New Delhi-110016
Phone: 011-26865455/+91-9871145498
Website: www.physicsbyfiziks.com | Email: fiziks.physics@gmail.com
311
fiziks
Institute for NET/JRF, GATE, IIT‐JAM, M.Sc. Entrance, JEST, TIFR and GRE in Physics

Q32. In a measurement of the viscous drag force experienced by spherical particles in a liquid,
the force is found to be proportional to V 1 / 3 where V is the measured volume of each
particle. If V is measured to be 30 mm 3 , with an uncertainty of 2.7 mm 3 , the resulting
relative percentage uncertainty in the measured force is
(a) 2.08 (b) 0.09 (c) 6 (d) 3
Ans. : (b)
2
 F  2
Solution: The relative percentage uncertainty in the measure force is    2

 V  V
F

 F 
F     V where  V is the uncertainty in the measurement of volume.
 V 
F 1 2 / 3
 F  V 1/3   V
V 3
1 1 1 1
F   V   2.7   2.7   2.7   F  0.09
3  30 3   900 3  9.7
2/3 2/3 1/ 3
3V

Q33. Consider a Low Pass (LP) and a High Pass (HP) filter with cut-off frequencies f LP and
f HP , respectively, connected in series or in parallel configurations as shown in the
Figures A and B below. 
fHP
(A) Input  L Output (B) Input Output
fHP fLP
L
Which of the following statements is correct? fLP
(a) For f HP  f LP , A acts as a Band Pass filter and B acts as a band Reject filter
(b) For f HP  f LP , A stops the signal from passing through and B passes the signal
without filtering
(c) For f HP  f LP , A acts as a Band Pass filter and B passes the signal without filtering
(d) For f HP  f LP , A passes the signal without filtering and B acts as a Band Reject filter
Ans. : (c)
Q34. The power density of sunlight incident on a solar cell is 100 mW / cm 2 . Its short circuit

current density is 30 mA / cm 2 and the open circuit voltage is 0.7 V . If the fill factor of
the solar cell decreases from 0.8 to 0.5 then the percentage efficiency will decrease from
(a) 42.0 to 26.2 (b) 24.0 to 16.8 (c) 21.0 to 10.5 (d) 16.8 to 10.5

H.No. 40-D, Ground Floor, Jia Sarai, Near IIT, Hauz Khas, New Delhi-110016
Phone: 011-26865455/+91-9871145498
Website: www.physicsbyfiziks.com | Email: fiziks.physics@gmail.com
312
fiziks
Institute for NET/JRF, GATE, IIT‐JAM, M.Sc. Entrance, JEST, TIFR and GRE in Physics

Ans. : (d)
Solution: The efficiency of a solar cell is determined as the fraction of incident power which is
converted to electricity and is defined as
Voc I sc FF
 and Pmax  Voc I sc FF
Pin
where Voc is the open circuit voltage, I sc is the short circuit current density , FF is the

Fill factor, Pin is the input power and  is the efficiency of the solar cell.

Given Pin  100 mW / cm2 , I sc  30 mA / cm 2 , Voc  0.7 V

Let 1 is the efficiency of solar cell when FF  0.8

 0.7 V    30  103 A / cm2   0.8 16.8


 1  3
  1  0.168
100  10 W / cm 2
100
Let 2 is the efficiency of solar cell when FF  0.5

 0.7 V    30  103 A / cm2   0.5 10.5


 2  3
  2  0.105
100  10 W / cm 2
100
Thus efficiency decreases from 1  16.8% to 2  10.5%

NET/JRF (JUNE-2015)
Q35. The concentration of electrons, n and holes p , for an intrinsic semiconductor at a
3
 E 
temperature T can be expressed as n  p  AT 2 exp   g  , where Eg is the band
 2 k BT 
3
gap and A is a constant. If the mobility of both types of carrier is proportional to T 2
,
then the log of the conductivity is a linear function of T 1 , with slope
Eg Eg  Eg  Eg
(a) (b) (c) (d)
 2k B  kB  2k B  kB
Ans. (c)
3
  Eg  23   Eg 
Solution:  i  ni e  e   p   T 2 exp    T   i  C exp  
 2 k BT   2k B T 
E E
ln  i   g  ln C  slope is g
2 k BT 2k B

H.No. 40-D, Ground Floor, Jia Sarai, Near IIT, Hauz Khas, New Delhi-110016
Phone: 011-26865455/+91-9871145498
Website: www.physicsbyfiziks.com | Email: fiziks.physics@gmail.com
313
fiziks
Institute for NET/JRF, GATE, IIT‐JAM, M.Sc. Entrance, JEST, TIFR and GRE in Physics

 Pa 4
Q36. The viscosity  of a liquid is given by Poiseuille’s formula   . Assume that
8lV
l and V can be measured very accurately, but the pressure P has an rms error of 1% and
the radius a has an independent rms error of 3% . The rms error of the viscosity is
closest to
(a) 2% (b) 4% (c) 12% (d) 13%
Ans. (c)
Solution:   kPa 4
2
     
     p2     a2   a 4   p2   4 Pa 3   a2
2 2 2
n
 P   a 
2
 
2 2
   
  n  100    p 100   16  a  100   1  16  3  1  144  145
2 2

 n   p   a 

 
  n 100   12%
 n 
Q37. Consider the circuits shown in figures (a) and (b) below
2K 1K

10 K 10 K
10V 10V

10.7V 5V

(a) (b)
If the transistors in Figures (a) and (b) have current gain   dc  of 100 and 10 respectively,

then they operate in the


(a) active region and saturation region respectively
(b) saturation region and active region respectively
(c) saturation region in both cases
(d) active region in both cases
Ans. (b)
Solution: In both case input section is F.B.
10.7  0.7
For figure (a) I B   1 mA  I C  BI B  100 mA
10

H.No. 40-D, Ground Floor, Jia Sarai, Near IIT, Hauz Khas, New Delhi-110016
Phone: 011-26865455/+91-9871145498
Website: www.physicsbyfiziks.com | Email: fiziks.physics@gmail.com
314
fiziks
Institute for NET/JRF, GATE, IIT‐JAM, M.Sc. Entrance, JEST, TIFR and GRE in Physics

Thus VCB  VC  VB  10  2  100   0.7  ve

 output section is F.B.


since both section are F.B. so it is in saturation region.
5  0.7
For Figure (b) I B   0.43 mA  I C  BI B  4.3 mA
10
Thus VCB  VC  VB  10  4.3  0.7)   ve

 out put section is R.B.


Thus it is in active region
Q38. In the circuit given below, the thermistor has a resistance 3 k at 250 C . Its resistance

decreases by 150 per 0 C upon heating. The output voltage of the circuit at 300 C is
T

1V

1K
 Vout

(a) 3.75 V (b) 2.25 V (c) 2.25 V (d) 3.75 V


Ans. (c)
Solution: At 300 C , Resistance  3000  150  5  2250 
RF 2250
 V0   vi   1  V0  2.25 volts
R1 1000

H.No. 40-D, Ground Floor, Jia Sarai, Near IIT, Hauz Khas, New Delhi-110016
Phone: 011-26865455/+91-9871145498
Website: www.physicsbyfiziks.com | Email: fiziks.physics@gmail.com
315
fiziks
Institute for NET/JRF, GATE, IIT‐JAM, M.Sc. Entrance, JEST, TIFR and GRE in Physics

NET/JRF (DEC-2015)
Q39. If the reverse bias voltage of a silicon varactor is increased by a factor of 2 , the
corresponding transition capacitance
(a) increases by a factor of 2 (b) increases by a factor of 2
(c) decreases by a factor of 2 (d) decreases by a factor of 2
Ans. : (c)

1 CT V C V 1
Solution: CT     T   CT  CT
V CT V CT 2V 2

Q40. If the parameters y and x are related by y  log  x  , then the circuit that can be used to

produce an output voltage V0 varying linearly with x is

(a) y (b) y
 
Vo Vo
 

(c) (d)
y  y 
Vo Vo
 

Ans. : (c)
Solution: (1) Integrator
(2) Logarithmic Ampere V0  log y 

(3) Anti-log V0  e y  x 

(4) Differentiator

H.No. 40-D, Ground Floor, Jia Sarai, Near IIT, Hauz Khas, New Delhi-110016
Phone: 011-26865455/+91-9871145498
Website: www.physicsbyfiziks.com | Email: fiziks.physics@gmail.com
316
fiziks
Institute for NET/JRF, GATE, IIT‐JAM, M.Sc. Entrance, JEST, TIFR and GRE in Physics

Q41. The state diagram corresponding to the following circuit is

x D A
y
CLOCK

Flip Flop
00, 01,10 01,11
(a) 11 00 (b) 00,10 00,10

0 1 0 1

01,10,11 01,11

00,11 00, 01,10


(c) 01,10 00,10 (d) 11 11

0 1 0 1

00,11 00, 01,10


Ans. : (d)
Solution: DA  xy  A

Input Present Flip-Flop Next State


x y State A Input DA A

0 0 0 1 1
0 0 1 0 0
0 1 0 1 1
0 1 1 0 0
1 0 0 1 1
1 0 1 0 0
1 1 0 0 0
1 1 1 1 1

H.No. 40-D, Ground Floor, Jia Sarai, Near IIT, Hauz Khas, New Delhi-110016
Phone: 011-26865455/+91-9871145498
Website: www.physicsbyfiziks.com | Email: fiziks.physics@gmail.com
317
fiziks
Institute for NET/JRF, GATE, IIT‐JAM, M.Sc. Entrance, JEST, TIFR and GRE in Physics

Q42. A sinusoidal signal of peak to peak amplitude 1V and unknown time period is input to

the following circuit for 5 second’s duration. If the counter measures a value  3E 8  H in

hexadecimal, then the time period of the input signal is


0.1  F
 10 bit
Vi  counter
1K 10 K

(a) 2.5 ms (b) 4 ms (c) 10 ms (d) 5 ms


Ans. : (d)
Solution:  3E 8  H  3 162  14 16  8 1  1000 10

In 5 sec, number of counts is 1000 .


Then count per sec is  200 count/sec
1
So, T  sec  5ms
200

NET/JRF (JUNE-2016)
Q43. The dependence of current I on the voltage V of a certain device is given by
2
 V 
I  I 0 1  
 V0 
where I 0 and V0 are constants. In an experiment the current I is measured as the voltage

V applied across the device is increased. The parameters V0 and I 0 can be graphically

determined as
(a) the slope and the y -intercept of the I  V 2 graph
(b) the negative of the ratio of the y -intercept and the slope, and the y -intercept of the

I  V 2 graph
(c) the slope and the y -intercept of the I  V graph
(d) the negative of the ratio of the y -intercept and the slope, and the y -intercept of the

I  V graph
Ans. : (d)

H.No. 40-D, Ground Floor, Jia Sarai, Near IIT, Hauz Khas, New Delhi-110016
Phone: 011-26865455/+91-9871145498
Website: www.physicsbyfiziks.com | Email: fiziks.physics@gmail.com
318
fiziks
Institute for NET/JRF, GATE, IIT‐JAM, M.Sc. Entrance, JEST, TIFR and GRE in Physics

 V 
2
 V   I0 I
Solution: I  I 0 1    I  I 0 1    I  V  I0
 V0   V0  V0
 I0  I0
Slope    V0
V0  I0 V
V0
Intercept on y -axis  I 0
Q44. In the schematic figure given below, assume that the propagation delay of each logic gate
is t . 5 V
gate

The propagation delay of the circuit will be maximum when the logic inputs A and B
make the transition
(a)  0,1  1,1 (b) 1,1   0,1
(c)  0, 0   1,1 (d)  0, 0    0,1
Ans. : (d)
Solution:
Input Output
A B NOT OR AND OR
0 1 0 0 0 0
    3t
1 1 0 1 1 1
1 1 0 1 1 1
    3t
0 1 0 0 0 0

0 0 1 1 1 1
    t
1 1 0 1 1 1
0 0 1 1 1 1
    4t
0 1 0 0 0 0

H.No. 40-D, Ground Floor, Jia Sarai, Near IIT, Hauz Khas, New Delhi-110016
Phone: 011-26865455/+91-9871145498
Website: www.physicsbyfiziks.com | Email: fiziks.physics@gmail.com
319
fiziks
Institute for NET/JRF, GATE, IIT‐JAM, M.Sc. Entrance, JEST, TIFR and GRE in Physics

Q45. Given the input voltage Vi , which of the following waveforms correctly represents the

output voltage V0 in the circuit shown below?

0.5 10 K

5K
Vi 0 Vi 
t
V0
0.5V 
5K 10 K
0.5
0

3.0 3.0
(a) 2.5 (b) 2.5
2.0 t V0 2.0
V0
1.5 1.5
1.0 1.0 t
0.5 0.5
0.0 0.0
0 0

3.0 3.0
(c) 2.5 (d) 2.5
2.0 2.0 t
V0 1.5 V0 1.5
1.0 t 1.0
0.5 0.5
0.0 0.00
0
Ans. : (b)
 10  10 10
Solution: V0   1     0.5   Vi  V0  1  2Vi
 5  15 5
When Vi  0  V0  1V , when Vi  0.1V  V0  0.8 V , when Vi  0.5V  V0  0V

Q46. The decay constants f p of the heavy pseudo-scalar mesons, in the heavy quark limit, are

a
related to their masses m p by the relation f p  , where a is an empirical parameter
mp

to be determined. The values m p   6400  160  MeV and f p  180  15  MeV

correspond to uncorrelated measurements of a meson. The error on the estimate of a is


3 3 3 3
(a) 175  MeV  2 (b) 900  MeV  2 (c) 1200  MeV  2 (d) 2400  MeV  2

Ans. : (c)

H.No. 40-D, Ground Floor, Jia Sarai, Near IIT, Hauz Khas, New Delhi-110016
Phone: 011-26865455/+91-9871145498
Website: www.physicsbyfiziks.com | Email: fiziks.physics@gmail.com
320
fiziks
Institute for NET/JRF, GATE, IIT‐JAM, M.Sc. Entrance, JEST, TIFR and GRE in Physics

Solution: a  f p m1/p 2
2 2
 a  2  a  2 a a f
 
2
  f p     m p   m1/p 2 and  p1
a f p
   m p  f p m p
2m p2
1

f 2
  f
2 
2
 m 
2
  f 2
   mp 
2
2
  a2  m p 2f p   m2    p    a  a  p 
p
 p
a
    
4m p p
a  fp

2

 2m p 
   f p   2m p  
 

 a  f p m1/p 2  180 MeV  6400 MeV   180  80  MeV 


1/ 2 3/ 2

2 2
  fp   15  2 3
  mp   160  2 4
      6.9  10 and       1.56  10
 fp   180   2m p   2  6400 

   MeV 
1/ 2
 a  180  80  MeV 
3/ 2 1/ 2 3/ 2
6.9  103  1.56  104   180  80  7  103

  a  1204  MeV 
3/ 2

NET/JRF (DEC-2016)
Q47. Which of the following circuits implements the Boolean function
F  A, B, C    1, 2, 4, 6  ?

C I0 C I0
(a) I1 4  1 (b) I1 4  1

I MUX
2
F I MUX
2
F

I 3 S1 S0 I 3 S1 S0

A B A B

C I0 0 I0
(c) I1 4  1 (d) I1 4  1
1
I MUX
2
F I MUX
2
F

I 3 S1 S0 C I 3 S1 S0

A B A B
Ans. : (b)
H.No. 40-D, Ground Floor, Jia Sarai, Near IIT, Hauz Khas, New Delhi-110016
Phone: 011-26865455/+91-9871145498
Website: www.physicsbyfiziks.com | Email: fiziks.physics@gmail.com
321
fiziks
Institute for NET/JRF, GATE, IIT‐JAM, M.Sc. Entrance, JEST, TIFR and GRE in Physics

Solution:
A B C F
0 0 0 0
0 0 1 1 F C
0 1 0 1
0 1 1 0 F C
1 0 0 1
1 0 1 0 F C
1 1 0 1
1 1 1 0 F C
  aV  
Q48. The I  V characteristics of a device can be expressed as I  I s exp    1 , where T
  T  
is the temperature and a and I s are constants independent of T and V . Which one of

the following plots is correct for a fixed applied voltage V ?


3 2
(a) (b) 1
2
0
log I
log I

1 1
2
0 3 3
1 2 3 0 1 2
aV / T aV / T

2 4
1 3
(c) (d)
2
0 1
log I

log I

1 0
1
2
2
3 3 3
0 1 2 0 1 2 3
aV / T aV / T
Ans. : (d)
av
Solution: Let  x For large x ; I  I s e x  log e I  log e Is  x  log e I  x
T

H.No. 40-D, Ground Floor, Jia Sarai, Near IIT, Hauz Khas, New Delhi-110016
Phone: 011-26865455/+91-9871145498
Website: www.physicsbyfiziks.com | Email: fiziks.physics@gmail.com
322
fiziks
Institute for NET/JRF, GATE, IIT‐JAM, M.Sc. Entrance, JEST, TIFR and GRE in Physics

Q49. The active medium in a blue LED (light emitting diode) is a Gax In1 x N alloy. The band

gaps of GaN and InN are 3.5 eV and 1.5 eV respectively. If the band gap of Gax In1 x N

varies approximately linearly with x , the value of x required for the emission of blue
light of wavelength 400 nm is (take hc  1200 eV -nm )
(a) 0.95 (b) 0.75 (c) 0.50 (d) 0.33
Ans. : (b)
Solution: EgGaN  3.5eV and EgInN  1.5eV E  eV 

Band Gap energy of Gax In1 x N is E  x . 3.5


3.0
For blue light of wavelength 400nm , the band gap
1.5
hc 1200 eV .nm
energy is    3.0 eV .
 400nm x
0 x? 1
 3.5 1.5   3.0 1.5 
Thus equating slopes we get;     2x  1.5  x  0.75
 1 0   x  0 
Q50. In the circuit below, the input voltage Vi is 2V ,Vcc  16 V , R2  2k  and RL  10 k 
VCC


Vi  R1 RL
R2

The value of R1 required to deliver 10 mW of power across RL is

(a) 12k (b) 4k (c) 8k (d) 14 k


Ans. : (c)
0  vi vi  v0 v0  0 
Solution: Apply kCL ; I 2  I1  I L    Vo
R2 R1 RL 
Vi I1
2
Vi
v R1 RL
pL  0
 10mW  v0  10V I2 R2
RL IL
0  2 2  10 10V 8
    1   R1  8 k 
2 R1 10k R1

H.No. 40-D, Ground Floor, Jia Sarai, Near IIT, Hauz Khas, New Delhi-110016
Phone: 011-26865455/+91-9871145498
Website: www.physicsbyfiziks.com | Email: fiziks.physics@gmail.com
323
fiziks
Institute for NET/JRF, GATE, IIT‐JAM, M.Sc. Entrance, JEST, TIFR and GRE in Physics

Q51. Two sinusoidal signals are sent to an analog multiplier of scale factor 1V 1 followed by a
low pass filter (LPF).

V1  5cos 100t 

LPF
Multiplier fC  5Hz
Vout

V2  20 cos 100t   / 3

If the roll-off frequency of the LPF is f c  5 Hz , the output voltage Vout is

(a) 5V (b) 25V (c) 100 V (d) 50V


Ans. : (b)
Solution: After multiplying
  1     
5cos 100t   20 cos 100t    1V 1  100  cos  200t    cos  
 3 2  3  3 

    1
 50 cos  200t    
  3  2
1
After pass L.P.F. v0  50   25V
2
Q52. The resistance of a sample is measured as a function of temperature, and the data are
shown below.
T  0C 2 4 6 8

R  90 105 110 115

The slope of R vs T graph, using a linear least-squares fit to the data, will be
6 4 2 8
(a) 0
(b) 0
(c) 0
(d) 0
C C C C
Ans. : (b)

H.No. 40-D, Ground Floor, Jia Sarai, Near IIT, Hauz Khas, New Delhi-110016
Phone: 011-26865455/+91-9871145498
Website: www.physicsbyfiziks.com | Email: fiziks.physics@gmail.com
324
fiziks
Institute for NET/JRF, GATE, IIT‐JAM, M.Sc. Entrance, JEST, TIFR and GRE in Physics

NET/JRF (JUNE-2017)
Q53. In the n -channel JFET shown in figure below, Vi  2V , C  10 pF , VDD  16 V and

RD  2k  . VDD
RD
D C
VO

Vi
G
S

If the drain D - source S saturation current I DSS is 10 mA and the pinch-off voltage VP is

8V , then the voltage across points D and S is


(a) 11.125 V (b) 10.375 V (c) 5.75 V (d) 4.75 V
Ans. : (d)
Solution: VGSQ  VGG  2V
2 2
 V   2 
I DQ  I DSS 1  GS   10mA 1    5.63mA
 VP   8 

VDS  VDD  I D RD  16  5.63  z  4.8V

1
Q54. The gain of the circuit given below is  .
 RC
C

R V
Vin 
a Vout
 b
V

ground
The modification in the circuit required to introduce a dc feedback is to add a resistor
(a) between a and b
(b) between positive terminal of the op-amp and ground
(c) in series with C
(d) parallel to C
H.No. 40-D, Ground Floor, Jia Sarai, Near IIT, Hauz Khas, New Delhi-110016
Phone: 011-26865455/+91-9871145498
Website: www.physicsbyfiziks.com | Email: fiziks.physics@gmail.com
325
fiziks
Institute for NET/JRF, GATE, IIT‐JAM, M.Sc. Entrance, JEST, TIFR and GRE in Physics

Ans. : (d)
Q55. A 2  4 decoder with an enable input can function as a
(a) 4 1 multiplexer (b) 1 4 demultiplexer
(c) 4  2 encoder (d) 4  2 priority encoder
Ans. : (b)
Q56. The experimentally measured values of the variables x and y are 2.00  0.05 and
3.00  0.02 respectively. What is the error in the calculated value of z  3 y  2 x from the
measurements?
(a) 0.12 (b) 0.05 (c) 0.03 (d) 0.07
Ans. : (a)
Solution: z  3 y  2 x
2 2
 z   z 
     y2     x2  9 y2  4 x2  0.12
2
z
 y   x 
Q57. Let I 0 be the saturation current,  the ideality factor and vF and vR the forward and

reverse potentials respectively, for a diode. The ratio RR / RF of its reverse and forward

resistances RR and RF , respectively, varies as (In the following k B is the Boltzmann

constant, T is the absolute temperature and q is the charge.)

vR  qv  vF  qv 
(a) exp  F  (b) exp  F 
vF   k BT  vR   k BT 

vR  qv  vF  qv 
(c) exp   F  (d) exp   F 
vF   k BT  vR   k BT 
Ans. : (a)
KT
Solution: I  I 0  eV /VT  1 , VT 
q
RR VR / I R VR I F
  
RF VF / I F VF I R

RR VR I 0 eVF /VT VR  qV 
   exp  F 
RF VF I0 VF  KT 

H.No. 40-D, Ground Floor, Jia Sarai, Near IIT, Hauz Khas, New Delhi-110016
Phone: 011-26865455/+91-9871145498
Website: www.physicsbyfiziks.com | Email: fiziks.physics@gmail.com
326
fiziks
Institute for NET/JRF, GATE, IIT‐JAM, M.Sc. Entrance, JEST, TIFR and GRE in Physics

Q58. In the figures below, X and Y are one bit inputs. The circuit which corresponds to a one
bit comparator is

X
X Y
(a) Y
X Y

X Y

X
X Y
(b)
X Y

Y X Y

X
(c) X Y

X Y

Y X Y

X
X Y
(d)
Y
X Y

Y X Y

Ans. : (c)
Solution: (a) 01  XY , 02  XY , 03  0

(b). 01  XY , 02  XY , 03  Y

(c) 01  XY , 02  XY , 03  XY  XY  X  Y (Equality comparator)

(d) 01  XY , 02  X  Y , 03  XY

H.No. 40-D, Ground Floor, Jia Sarai, Near IIT, Hauz Khas, New Delhi-110016
Phone: 011-26865455/+91-9871145498
Website: www.physicsbyfiziks.com | Email: fiziks.physics@gmail.com
327
fiziks
Institute for NET/JRF, GATE, IIT‐JAM, M.Sc. Entrance, JEST, TIFR and GRE in Physics

Q59. Both the data points and a linear fit to the current vs voltage of a resistor are shown in the
graph below. 1

I (amps)
0 V  volts  25

If the error in the slope is 1.255  103  1 , then the value of resistance estimated from the
graph is
(a)  0.04  0.8   (b)  25.0  0.8  

(c)  25  1.25   (d)  25  0.0125  

Ans. : (b)
I max  I min 1 0 1
Solution: Slope     m (let)
Vmax  Vmin 25  0 25
V 1 R 1
I   mV  R   25 where  2
R m m m
2
 R  2 1 2
Error in R is  R2    m  4 m  R m
4 2

 m  m

  R  R 2 m   25  1.255 103  0.8  R   25.0  0.8  


2

Q60. In the following operational amplifier circuit Cin  10 nF , Rin  20 k , RF  200 k  and
RF
CF  100 pF .
CF

Ri Ci

Vin
Vout

The magnitude of the gain at a input signal frequency of 16 kHz is


(a) 67 (b) 0.15 (c) 0.3 (d) 3.5
H.No. 40-D, Ground Floor, Jia Sarai, Near IIT, Hauz Khas, New Delhi-110016
Phone: 011-26865455/+91-9871145498
Website: www.physicsbyfiziks.com | Email: fiziks.physics@gmail.com
328
fiziks
Institute for NET/JRF, GATE, IIT‐JAM, M.Sc. Entrance, JEST, TIFR and GRE in Physics

Ans. : (d)
1  1 
RF  /  RF  
Vo z RF  X CF J  cF  J  cF 
Solution:  F  
Vin zi Ri  X Ci  1 
 Ri  
 J  ci 

Vo  RF /  J  cF RF  1  RF j ci
  
Vin  jci Ri  1 / jci  jcF RF  1 1  j Ri ci 
Vo  ci RF
  ,   2 f
Vin 1   cF RF  1   Ri ci 
2 2


V0

 2  16 103 10 109  200  103 
Vln 1  4 2 16 103   200 103  100  1012  1  4 2 16  103   20  103  10  109 
2 2 2 2 2 2

V0 64
   4.45
Vln 2.4  20.12

NET/JRF (DEC - 2017)


Q61. The full scale voltage of an n -bit Digital-to-Analog Convener is V . The resolution that
can be achieved in it is
V V V V
(a) (b) (c) (d)
 2  1
n
 2  1
n
22 n n

Ans. : (a)
Q62. A Zener diode with an operating voltage of 10 V at 250 C has a positive temperature

coefficient of 0.07 % per 0 C of the operating voltage. The operating voltage of this

Zener diode at 1250 C is


(a) 12.0 V (b) 11.7 V (c) 10.7 V (d) 9.3 V
Ans. : (c)
Solution: With increase of 1000 C , the voltage increases by  7% of operating voltage
 7 
Thus at 1250 C , the operating voltage is   10  10   V  10.7 V
 100 

H.No. 40-D, Ground Floor, Jia Sarai, Near IIT, Hauz Khas, New Delhi-110016
Phone: 011-26865455/+91-9871145498
Website: www.physicsbyfiziks.com | Email: fiziks.physics@gmail.com
329
fiziks
Institute for NET/JRF, GATE, IIT‐JAM, M.Sc. Entrance, JEST, TIFR and GRE in Physics

Q63. In the circuit below the voltages VBB and VCC are kept fixed, the
VCC
voltage measured at B is a constant, but that measured at A
fluctuates between a few V to a few mV . 9V

From these measurements it may be inferred that the RC

(a) base is open internally B


(b) emitter is open internally VBB RB
(c) collector resistor is open 3V A
(d) base resistor is open
Ans. : (d)
Q64. In the circuit below, D1 and D2 are two silicon diodes with the same characteristics. If

the forward voltage drop of a silicon diode is 0.7 V then the value of the current I1  I D1
I1
is
1k  I D1 ID2
10V D1 D2 Vo

(a) 18.6 mA (b) 9.3 mA (c) 13.95 mA (d) 14.65 mA


Ans. : (c)
10  0.7
Solution: I1   9.3 mA
1k
I I 3
 
I D1  I D2  1  I1  I D1  I1  1  I1  13.95 mA
2 2 2
Q65. The sensitivity of a hot cathode pressure gauge is 10 mbar 1 . If the ratio between the
numbers of the impinging charged particles to emitted electrons is 1:10 , then the
pressure
(a) 10 mbar (b) 101 mbar (c) 102 mbar (d) 102 mbar
Ans. : (c)
I 1
Solution: Pressure, P   
I S
I
where  ratio between the number of the impinging charge particles to emitted
I
electrons
H.No. 40-D, Ground Floor, Jia Sarai, Near IIT, Hauz Khas, New Delhi-110016
Phone: 011-26865455/+91-9871145498
Website: www.physicsbyfiziks.com | Email: fiziks.physics@gmail.com
330
fiziks
Institute for NET/JRF, GATE, IIT‐JAM, M.Sc. Entrance, JEST, TIFR and GRE in Physics

S  Sensitivity of Gauge.

1 1 
 P  1 
 102 m bar
10  10 m bar 

Thus correct option is (c)


Q66. The circuit below comprises of D -flip flops. The output is taken from Q3 , Q2,Q1 and Q0 as
shown in the figure.

LSB Q Q1 Q2
MSB Q
0 3

D Q D Q D Q D Q

CLR CLR CLR CLR

CLK

RST
the binary number given by the string Q3 , Q2,Q1Q0 changes for every clock pulse that is

applied to the CLK input. If the output is initialized at 0000 , the the corresponding
sequence of decimal numbers that repeats itself, is
(a) 3, 2,1, 0 (b) 1,3, 7,14,12,8
(c) 1,3, 7,15,12,14, 0 (d) 1,3, 7,15,14,12,8, 0
Ans. : (d)
Solution:

Clock Q3 Q2 Q1 Q0
0 0 0 0
1 0 0 0 1 1
2 0 0 1 1 3
3 0 1 1 1 7
4 1 1 1 1 15
5 1 1 1 0 14
6 1 1 0 0 12
7 1 0 0 0 8
8 0 0 0 0 0

H.No. 40-D, Ground Floor, Jia Sarai, Near IIT, Hauz Khas, New Delhi-110016
Phone: 011-26865455/+91-9871145498
Website: www.physicsbyfiziks.com | Email: fiziks.physics@gmail.com
331
fiziks
Institute for NET/JRF, GATE, IIT‐JAM, M.Sc. Entrance, JEST, TIFR and GRE in Physics

NET/JRF (JUNE-2018)
Q67. A sinusoidal signal with a peak voltage V p and average value zero, is an input to the

following circuit.

Vp
Input voltage RL Output
voltage
C

Assuming ideal diodes, the peak value of the output voltage across the load resistor RL is

Vp
(a) V p (b) (c) 2V p (d) 2V p
2
Ans. : (c)
Solution: It’s a voltage doubler circuit
Peak value  2V p

Q68. In the following circuit, the value of the common-emitter forward current amplification
factor  for the transistor is 100 and VBE is 0.7 V .

VCC 20.7 V

500 k R1 R2
5k
Ib Output
Input
Ik
The base current I B is

(a) 40  A (b) 30  A (c) 44  A (d) 33  A


Ans. : (d)
Vcc  VBE 20.7  0.7 20 20 1000
Solution: I B      A  33.3 A
RB   RE 500  100 1 600 K 600

H.No. 40-D, Ground Floor, Jia Sarai, Near IIT, Hauz Khas, New Delhi-110016
Phone: 011-26865455/+91-9871145498
Website: www.physicsbyfiziks.com | Email: fiziks.physics@gmail.com
332
fiziks
Institute for NET/JRF, GATE, IIT‐JAM, M.Sc. Entrance, JEST, TIFR and GRE in Physics

Q69. In the following JK flip-flop circuit, J and K inputs are tied together to VCC . If the

input is a clock signal of frequency f , the frequency of the output Q is


VCC
I Q

Clock signal
K Q

f
(a) f (b) 2 f (c) 4 f (d)
2
Ans. : (d)
Solution: It divides clock frequency by 2
Q70. Which of the following gates can be used as a parity checker?
(a) an OR gate (b) a NOR gate
(c) an exclusive OR (XOR) gate (d) an AND gate
Ans. : (c)
Q71. Two signals A1 sin  t  and A2 cos t  are fed into the input and the reference channels,

respectively, of a lock-in amplifier. The amplitude of each signal is 1 V . The time


constant of the lock-in amplifier is such that any signal of frequency larger than  is
filtered out. The output of the lock-in amplifier is
(a) 2 V (b) 1 V (c) 0.5 V (d) 0 V
Ans. : (d)
A1 A2
Solution: v  A1 sin t. A2 cos t  sin t  t   sin t  t  
2 
A1 A2
v sin 2t
2
This signal will be filtered out, so output is 0V .

H.No. 40-D, Ground Floor, Jia Sarai, Near IIT, Hauz Khas, New Delhi-110016
Phone: 011-26865455/+91-9871145498
Website: www.physicsbyfiziks.com | Email: fiziks.physics@gmail.com
333
fiziks
Institute for NET/JRF, GATE, IIT‐JAM, M.Sc. Entrance, JEST, TIFR and GRE in Physics

Q72. The full scale of a 3 -bit digital-to-analog (DAC) converter is 7 V . Which of the following
tables represents the output voltage of this 3 -bit DAC for the given set of input bits?

Input bits Output voltage Input bits Output voltage


(a) 000 0 (b) 000 0
001 1 001 1.25
010 2 010 2.5
011 3 011 3.75

Input bits Output voltage Input bits Output voltage


(c) 000 1.25 (d) 000 1
001 2.5 001 2
010 3.75 010 3
011 5 011 4
Ans. : (a)
Solution: (111)  7V ,  001  1V ,  010   2V ,  011  3V , 100   4V
Q73. The input Vi to the following circuit is a square wave as shown in the following figure
Vi
T /2 T C R
 0, 0  t 
V

Which of the waveforms V0 best describes the output?


Vo Vo
T /2 T
(a) 0, 0 (b)
  t  0, 0  T
T /2 t

Vo Vo
T /2 T T /2 T
(c) (d)
 0, 0  t
 0, 0  t
Ans. : (b)
Solution: It’s a differentiator circuit

H.No. 40-D, Ground Floor, Jia Sarai, Near IIT, Hauz Khas, New Delhi-110016
Phone: 011-26865455/+91-9871145498
Website: www.physicsbyfiziks.com | Email: fiziks.physics@gmail.com
334
fiziks
Institute for NET/JRF, GATE, IIT‐JAM, M.Sc. Entrance, JEST, TIFR and GRE in Physics

NET/JRF (DEC - 2018)


Q42. Consider the following circuit, consisting of an RS flip-flop and two AND gates.

J 1 A S Q
CLK
K 2 B R Q

Which of the following connections will allow the entire circuit to act as a JK flip-flop?
(a) connect Q to pin 1 and Q to pin 2

(b) connect Q to pin 2 and Q to pin 1

(c) connect Q to K input and Q to J input

(d) connect Q to J input and Q to K input


Ans. : (b)
Q43. The truth table below gives the value Y  A, B, C  where A, B and C are binary variables.

The output Y can be represented by A B C Y


(a) Y  ABC  ABC  ABC  ABC 0 0 0 1
0 0 1 0
(b) Y  ABC  ABC  ABC  ABC
0 1 0 0
(c) Y  ABC  ABC  ABC  ABC 0 1 1 1
1 0 0 1
(d) Y  ABC  ABC  ABC  ABC 1 1 0 0
Ans. : (b) 1 1 1 1

Solution: Y  ABC  ABC  ABC  ABC


Q44. A sinusoidal signal is an input to the following circuit
VCC

v R R
Input Vo
C
 0, 0  t C
R

H.No. 40-D, Ground Floor, Jia Sarai, Near IIT, Hauz Khas, New Delhi-110016
Phone: 011-26865455/+91-9871145498
Website: www.physicsbyfiziks.com | Email: fiziks.physics@gmail.com
335
fiziks
Institute for NET/JRF, GATE, IIT‐JAM, M.Sc. Entrance, JEST, TIFR and GRE in Physics

Which of the following graphs best describes the output wave function?
V V
(a) (b)

 0, 0 t  0, 0 t

V V
(c) (d)
 0, 0 t  0, 0 t

Ans. : (a)
Solution: In CE transistor output has phase charge of 
Q45. A sinusoidal voltage having a peak value of V p is an input to the following circuit, in

which the DC voltage is Vb


R
V
Vp
Input D Output
t
Vb

Assuming an ideal diode which of the following best describes the output waveform?

Vp
(a) (b) Vb

t  0, 0  t
Vb

(c) (d)
Vb Vb

 0, 0  t  0, 0  t
Vb
Vp

H.No. 40-D, Ground Floor, Jia Sarai, Near IIT, Hauz Khas, New Delhi-110016
Phone: 011-26865455/+91-9871145498
Website: www.physicsbyfiziks.com | Email: fiziks.physics@gmail.com
336
fiziks
Institute for NET/JRF, GATE, IIT‐JAM, M.Sc. Entrance, JEST, TIFR and GRE in Physics

Ans. : (c)
Q64. The input Vi to the following circuit is a square wave as shown in the following figure.

Vi
T /2 T
Vi C R
 0,0  t 
Vo

which of the waveforms best describes the output?


Vo Vo
(a) (b)
 0, 0   0, 0  T /2 3T /2 5T /2 t
T /2 3T /2 5T /2 t

Vo
(c) (d) Vo

 0, 0  T /2 3T /2 t  0, 0  T /2 3T /2 5T /2 t

Ans. : (c)
Solution: Differentiator circuit.
Q65. The amplitude of a carrier signal of frequency f 0 is sinusoidally modulated at a

frequency f   f 0 . Which of the following graphs best describes its power spectrum?

Power Power
(a) (b)

f f
(f0 - 3f ')
(f0 - 2f ')
(f0 - 4f')

(f0 + 2f ')

(f0 + 4f')
(f0 + 3f ')
(f0 + f ')

(f0 + f ')
(f0 - f ')
f0

(f0 - f ')
f0

H.No. 40-D, Ground Floor, Jia Sarai, Near IIT, Hauz Khas, New Delhi-110016
Phone: 011-26865455/+91-9871145498
Website: www.physicsbyfiziks.com | Email: fiziks.physics@gmail.com
337
fiziks
Institute for NET/JRF, GATE, IIT‐JAM, M.Sc. Entrance, JEST, TIFR and GRE in Physics

Power Power
(c) (d)

(f0 - 2f ') f f

(f0 + 2f ')
(f0 + f ')
(f0 - f ')
f0

(f0 + f ')
f0
Ans. : (b)
Solution: 2sin A cos B  sin  A  B   sin  A  B 

C  t   A sin  2 ft  - Carrier wave

M  t   M cos  2 f 0t  -Modulation waveform

 M  t   Am cos  2 f 0t 
 M t  
Amplitude modulated wave y  t   1   C t 
 A 
Am Am
y  t   A sin  2 ft   sin  2  f  f 0    sin  2  f  f 0  
2 2
Q66. The standard deviation of the following set of data
10.0,10.0,9.9,9.9,9.8,9.9,9.9,9.9,9.8,9.9 is nearest to
(a) 0.10 (b) 0.07 (c) 0.01 (d) 0.04
Ans. : (b)
 xi  x 
2
Solution: x xi  x

10.0 0.1 0.01


10.0 0.1 0.01
9.9 0 0
9.9 0 0
9.8 0.1 0.01
9.9 0 0
9.9 0 0
9.9 0 0
9.8 0.1 0.01
9.9 0 0
99 0.04
H.No. 40-D, Ground Floor, Jia Sarai, Near IIT, Hauz Khas, New Delhi-110016
Phone: 011-26865455/+91-9871145498
Website: www.physicsbyfiziks.com | Email: fiziks.physics@gmail.com
338
fiziks
Institute for NET/JRF, GATE, IIT‐JAM, M.Sc. Entrance, JEST, TIFR and GRE in Physics

where x 
 x  99  9.9
N 10
and standard deviation is

  x  x
2
0.04
 i
  0.066
N 1 9
   0.07

H.No. 40-D, Ground Floor, Jia Sarai, Near IIT, Hauz Khas, New Delhi-110016
Phone: 011-26865455/+91-9871145498
Website: www.physicsbyfiziks.com | Email: fiziks.physics@gmail.com
339
fiziks
Institute for NET/JRF, GATE, IIT‐JAM, M.Sc. Entrance, JEST, TIFR and GRE in Physics

ATOMIC AND MOLECULAR PHYSICS


NET/JRF (JUNE-2011)

Q1. Consider the energy level diagram (as shown in the figure below) of a typical three level
ruby laser system with 1.6  1019 Chromium ions per cubic centimeter. All the atoms
excited by the 0.4 μm radiation decay rapidly to level E2 , which has a lifetime  = 3 ms.
E3
0.4  m

E2
0.7  m
E1

A. Assuming that there is no radiation of wavelength 0.7 μm present in the pumping


cycle and that the pumping rate is R atoms per cm3, the population density in the level N2
builds up as:
(a) N2 (t) = R(et/ –1) (b) N2 (t) = R(1 – e-t/)

(c) N 2 t  
Rt 2

1  e 
t / 
(d) N2 (t) = R t

Ans. : (b)
Solution: N 2  t   R 1  et / 

B. The minimum pump power required (per cubic centimeter) to bring the system to
transparency, i.e. zero gain, is
(a) 1.52 kW (b) 2.64 kW (c) 0.76 kW (d) 1.32 kW
Ans. : (c)
Solution: The Minimum Power required to achieve zero gain is
N hv N hc 1.6  1019 6.6  10 34  3  10 8
P     754 W cm 3
2  2  2 6
0.7  10  3  10 3

P  0.76kW per cubic centimeter

H.No. 40-D, Ground Floor, Jia Sarai, Near IIT, Hauz Khas, New Delhi-110016
Phone: 011-26865455/+91-9871145498
Website: www.physicsbyfiziks.com | Email: fiziks.physics@gmail.com
340
fiziks
Institute for NET/JRF, GATE, IIT‐JAM, M.Sc. Entrance, JEST, TIFR and GRE in Physics

NET/JRF (DEC-2011)
Q2. Given that the ground state energy of the hydrogen atom is –13.6 eV, the ground state
energy of positronium (which is a bound state of an electron and a positron) is
(a) + 6.8 eV (b) – 6.8 eV (c) – 13.6 eV (d) – 27.2 eV
Ans. : (b)
13.6
Solution: The energy expression for Positronium atom is E n   eV 
2n 2
 13.6
For n = 1, E1  eV   6.8eV ,  E1  6.8 eV
2
Q3. A laser operating at 500 nm is used to excite a molecule. If the Stokes line is observed at
770 cm-1, the approximate positions of the Stokes and the anti-Stokes lines are
(a) 481.5 nm and 520 nm (b) 481.5 nm and 500 nm
(c) 500 nm and 520 nm (d) 500 nm and 600 nm
Ans. :
Solution: Given 0  500 nm  5  105 cm , v stoke  770 cm 1   0  20,000cm 1

Raman shift v  v 0  v stoke  19230 cm 1

Wave number of anti-stokes line is vanti -stoke  v  v0 = 39,230 cm-1

In wavelength term  anti  stoke  2.549  10 7  254.9 nm and  stoke  12987 nm

Q4. If the hyperfine interaction in an atom is given by H  a S e  S p where S e and S p denote

the electron and proton spins, respectively, the splitting between the 3 S1 and 1 S 0 state is

(a) a 2 / 2 (b) a 2 (c) a 2 / 2 (d) 2a 2


Ans. : (b)
       1

Solution: Total spin is S  S e  S p  S 2  S e2  S p2  2 S e  S p  S e  S p  S 2  S e2  S p2
2

 
 
 H  aS e  S p  S  S e2  S p2 where S e2  S p2  S S  1 2   2
a 2
2
3
4
a  2 3 2 3 2  a  2 3 2
H   S       S   
2 4 4  2 2 

For 3 S1 : S  1  S 2  S S  1 2  2 2 , For 1 S 0 : S  0  S 2  S S  1 2  0 2

a 3 a a 3 3 2
 H1   2   2   2 for 3 S1 and H 2   0   2  a for 1 S 0
2 2 4 2 2 4

H.No. 40-D, Ground Floor, Jia Sarai, Near IIT, Hauz Khas, New Delhi-110016
Phone: 011-26865455/+91-9871145498
Website: www.physicsbyfiziks.com | Email: fiziks.physics@gmail.com
341
fiziks
Institute for NET/JRF, GATE, IIT‐JAM, M.Sc. Entrance, JEST, TIFR and GRE in Physics

1 3 2
 The splitting between 3
S1 and 1 S 0 is H  H 1  H 2    a  a
2

4 4
Q5. The ratio of intensities of the D1 and D2 lines of sodium at high temperature is
(a) 1:1 (b) 2:3 (c) 1:3 (d) 1:2
Ans. : (d)
Solution: The electronic transition for D2 and D1 line is
3
2 1
I  D2  2 J 2  1 2 4 2
D2 : 2P3/ 2  2S1/ 2 , D1 : 2P1/ 2  2S1/ 2     
I D1  2 J 1  1 1 2 1
2 1
2
Q6. An atom of mass M can be excited to a state of mass M    by photon capture. The
frequency of a photon which can cause this transition is
c 2 c 2 2 c 2 c 2
(a) (b) (c) (d)   2M 
2h h 2Mh 2Mh
Ans. : (d)
Solution: The conservation law of energy and momentum give


Mc 2  h  M    c 4  p 2 c 2
2
1/ 2
and
h
c
 p

M 2 c 4  h 2 2  2 Mc 2 h  M 2 c 4  2 c 4  2 Mc 4  h 2 2  2Mc 2 h  2 c 4  2Mc 4

   c 2    c 2
 2Mc h  2Mc 1 
2
  
4
1       2M  .
 2M  h  2M  2Mh

NET/JRF (JUNE-2012)
12
Q7. The first absorption spectrum of C16O is at 3.842 cm-1 while that of 13
C16O is at
3.673 cm-1. The ratio of their moments of inertia is
(a) 1.851 (b) 1.286 (c) 1.046 (d) 1.038
Ans. : (c)
Solution: For 12 C 16O : 2 B1  3.842 cm 1  B1  1.921 cm 1

For 13
2 B 2  3.673 cm 1
C 16 O :  B 2  1.8365 cm 1
h I 2 B1 1.921
Where, B  2     1.046
8 IC I1 B2 1.8365

H.No. 40-D, Ground Floor, Jia Sarai, Near IIT, Hauz Khas, New Delhi-110016
Phone: 011-26865455/+91-9871145498
Website: www.physicsbyfiziks.com | Email: fiziks.physics@gmail.com
342
fiziks
Institute for NET/JRF, GATE, IIT‐JAM, M.Sc. Entrance, JEST, TIFR and GRE in Physics

Q8. The spin-orbit interaction in an atom is given by H  aL  S , where L and S denote the
orbital and spin angular momenta, respectively, of the electron. The splitting between the
levels 2P3/2 and 2P1/2 is
3 2 1 2 5 2
(a) a (b) a (c) 3a 2 2 (d) a
2 2 2
Ans. : (a)
    
Solution: Given H  aL  S where J  L  S .
  1
 
 
a
 J 2  L2  S 2  2 L  S  L  S  J 2  L2  S 2  H  J 2  L2  S 2
2 2
 
1 3
For 2P3/ 2 : S  which gives S 2  S  S  1  2   2
2 4

L = 1 which gives L2  LL  1 2  2 2


3 15 a 15 3
which gives J 2  J  J  1 2   2  H1    2    2   2
a
J
2 4 2 4 4 2

1 3
For 2P1/ 2 : S  which gives S 2  S S  1 2   2
2 4
L = 1 which gives L2  LL  1 2  2 2

1 3 a 3 3 
J which gives J 2  J  J  1 2   2  H 2    2   2    a 2
2 4 2 4 4 
a 2 3
H  H1  H 2    a 2  a 2
2 2
Q9. The spectral line corresponding to an atomic transition from J = 1 to J = 0 states splits in
a magnetic field of 0.1 Tesla into three components separated by 1.6  10-3 Ǻ. If the zero
field spectral line corresponds to 1849 Ǻ, what is the g-factor corresponding to the J = 1
hc
state? (You may use  2  10 4 cm)
0
(a) 2 (b) 3/2 (c) 1 (d) 1/2
Ans. : (c)
Solution: The Zeeman splitting is E   gM J  B B   g B B for MJ = +1

2
Given, Zeeman splitting separations,     1.6  10 3 Å
c

H.No. 40-D, Ground Floor, Jia Sarai, Near IIT, Hauz Khas, New Delhi-110016
Phone: 011-26865455/+91-9871145498
Website: www.physicsbyfiziks.com | Email: fiziks.physics@gmail.com
343
fiziks
Institute for NET/JRF, GATE, IIT‐JAM, M.Sc. Entrance, JEST, TIFR and GRE in Physics

c 3  10 8
      1.6  10 1  0.1404  1010
 2
1849  10 
10 2

E 6.625  10 34  0.1404  1010


g    1.00
B B 9.27  10  24  0.1

NET/JRF (DEC-2012)
Q10. Consider the energy level diagram shown below, which corresponds to the molecular
nitrogen laser. 2
R  21

1
1
0
20 -3 -1
If the pump rate R is 10 atoms cm s and the decay routes are as shown with
 21  20 ns and  1  1s , the equilibrium populations of states 2 and 1 are, respectively,
(a) 1014 cm-3 and 2  1012 cm-3 (b) 21012 cm-3 and 1014 cm-3.
(c) 21012 cm-3 and 2  10 6 cm-3 (d) zero and 1020 cm-3
Ans. : (b)
dN 2 N dN1 N 2 N 1
Solution:  R  2 and   .
dt  21 dt  21  1
dN 2 dN1
Under equilibrium condition  0
dt dt
 N 2   21 R  1020  20  109  2  1012 cm3

 1 N 2 10 6  2  1012 cm 3
 N1    1014 cm 3
 21 20  10 9

Q11. Consider a hydrogen atom undergoing a 2 P  1S transition. The lifetime tsp of the 2P
state for spontaneous emission is 1.6 ns and the energy difference between the levels is
10.2 eV. Assuming that the refractive index of the medium n0 = 1, the ratio of Einstein
coefficients for stimulated and spontaneous emission B21   / A21   is given by
(a) 0.683 × 1012 m3J-1s-1 (b) 0.146 × 10-12 Jsm-3 .
(c) 6.83 × 1012 m3J-1s-1 (d) 1.463 × 10-12 Jsm-3 .

H.No. 40-D, Ground Floor, Jia Sarai, Near IIT, Hauz Khas, New Delhi-110016
Phone: 011-26865455/+91-9871145498
Website: www.physicsbyfiziks.com | Email: fiziks.physics@gmail.com
344
fiziks
Institute for NET/JRF, GATE, IIT‐JAM, M.Sc. Entrance, JEST, TIFR and GRE in Physics

Ans. : (a)
B21  2 c 3  2 2 c 3
Solution: n 0  1 , E  10.2 eV and    0.67  1012 .
A21  3 n03  E 3 n03

Q12. Consider a He-Ne laser cavity consisting of two mirrors of reflectivities R1 = 1 and
R2 = 0.98. The mirrors are separated by a distance d = 20 cm and the medium in between
has a refractive index n0 = 1 and absorption coefficient α = 0. The values of the
separation between the modes  and the width  p of each mode of the laser cavity are:

(a)   75kHz,  p  24kHz (b)   100kHz,  p  100kHz

(c)   750MHz,  p  2.4 MHz (d)   2.4MHz,  p  750MHz

Ans. : (c)
c
Solution: Mode separation    750 MHz
2dn0

where c  3  10 8 m / sec , d  20  102 m and n0  1 .

1 2n 0 d
Width of each mode  p  where t c  .
2t c  1 
c ln  2d

 R1 R2 e 
Note: In this question, there is no need to calculate  p . Since in the given options there

is only one option with   750 MHz and i.e., option (c). You can calculate  p without

calculator so use your common sense.

NET/JRF (JUNE-2013)
Q13. A muon  
 from cosmic rays is trapped by a proton to form a hydrogen-like atom.
Given that a muon is approximately 200 times heavier than an electron, the longest
wavelength of the spectral line (in the analogue of the Lyman series) of such an atom will
be
o o o o
(a) 5.62 A (b) 6.67 A (c) 3.75 A (d) 13.3 A
Ans. : (b)
m m p
Solution: In case of muonic atom, the reduce mass is m '   180me
m  m p

H.No. 40-D, Ground Floor, Jia Sarai, Near IIT, Hauz Khas, New Delhi-110016
Phone: 011-26865455/+91-9871145498
Website: www.physicsbyfiziks.com | Email: fiziks.physics@gmail.com
345
fiziks
Institute for NET/JRF, GATE, IIT‐JAM, M.Sc. Entrance, JEST, TIFR and GRE in Physics

 m'  E E
E n'    21  180 21 where, E1  13.6eV
 me  n n

For ground state of muonic atom n=1, E1'  180 E1

For first excited state of muonic atom n=2, E 2'  45 E1


The longest wavelength of the photon corresponds to the transition between first and ground state
of muonic atom.
The energy difference between first excited and ground state is
E  E 2'  E1'  135E1  1836eV  2938  10 19 J
In term of wavelength
hc hc 6.63  10 34  3  10 8 o
 
10
E  hv    6. 67  10 m  6.67 A
 E 2938  10 19
Q14. Consider the hydrogen-deuterium molecule HD. If the mean distance between the two
atoms is 0.08 nm and the mass of the hydrogen atom is 938 MeV / c 2 , then the energy
difference E between the two lowest rotational states is approximately
(a) 10 1 eV (b) 10 2 eV (c) 2  10 2 eV (d) 10 3 eV
Ans. : (b)
h2
Solution: Rotational energy expression E  J  J  1  AJ  J  1
8 2 I
Difference between two lowest energy levels is E  2 A
h2 2
where A   and I  r 2
8 I
2
2I
MHMD M  2M H 2 2
Here,    H  MH ,    938 MeV/c2
M H  M D M H  2M H 3 3
1
  1.01  1034  J -sec   1.01  1034   1019  eV -sec   6.3  1016  eV -sec 
1.6

 E 
2


6.3  10 16 eV  sec 2

 9.2  103 eV  102 eV


I 2
3

 938  10 6 eV / c 2  0.08  10 9 m 2

Q15. The electronic energy levels in a hydrogen atom are given by E n  13.6 / n 2 eV. If a

selective excitation to the n  100 level is to be made using a laser, the maximum
allowed frequency line-width of the laser is
(a) 6.5 MHz (b) 6.5 GHz (c) 6.5 Hz (d) 6.5 kHz
H.No. 40-D, Ground Floor, Jia Sarai, Near IIT, Hauz Khas, New Delhi-110016
Phone: 011-26865455/+91-9871145498
Website: www.physicsbyfiziks.com | Email: fiziks.physics@gmail.com
346
fiziks
Institute for NET/JRF, GATE, IIT‐JAM, M.Sc. Entrance, JEST, TIFR and GRE in Physics

Ans. : (b)
2  13.6 2  13.6
Solution: E n  13.6 / n 2  E n  3
eV  h   1.6  10 19    6.5 GHz .
n n3
Q16. Consider the laser resonator cavity shown in the figure.
If I1 is the intensity of the radiation at R1  1 R2  R
l
mirror M 1 and  is the gain coefficient of the
medium between the mirrors, then the energy
x
density of photons in the plane P at a
P
distance x from M 1 is M1 M2

(a)  I1 / c  e  x (b)  I1 / c  e x

(c)  I1 / c   e x  e x  (d)  I1 / c  e2 x


Ans. : (c)
NET/JRF (DEC-2013)
Q17. A perturbation V pert  aL2 is added to the Hydrogen atom potential. The shift in the

energy level of the 2 P state, when the effects of spin are neglected up to second order in
a , is
3 2 4
(a) 0 (b) 2a 2  a 2  4 (c) 2a 2 (d) a 2  a 
2
Ans. : (c)
Solution: For 2 P state, L=1
The shift in the energy due to perturbation V pert  aL2 is

E  aL( L  1) 2  a1(1  1) 2  2a 2


Q18. A gas laser cavity has been designed to operate at   0.5 m with a cavity length of 1 m .
With this set-up, the frequency is found to be larger than the desired frequency by 100 Hz.
The change in the effective length of the cavity required to retune the laser is
(a)  0.334  10 12 m (b) 0.334  10 12 m (c) 0.167  10 12 m (d)  0.167  10 12 m
Ans. : (d)
c c  c  c
Solution:   m    m 2 L   2 L  2 L   L  L
2L 2L c 2L L L
 L 0.5  106  1  100
 L     0.167  1012 m
c 3  108
H.No. 40-D, Ground Floor, Jia Sarai, Near IIT, Hauz Khas, New Delhi-110016
Phone: 011-26865455/+91-9871145498
Website: www.physicsbyfiziks.com | Email: fiziks.physics@gmail.com
347
fiziks
Institute for NET/JRF, GATE, IIT‐JAM, M.Sc. Entrance, JEST, TIFR and GRE in Physics

Q19. The spectroscopic symbol for the ground state of 13 Al is 2 P1/ 2 . Under the action of a

strong magnetic field (when L  S coupling can be neglected) the ground state energy
level will split into
(a) 3 levels (b) 4 levels (c) 5 levels (d) 6 levels
Ans. : (c)
Solution: In extremely strong magnetic field coupling between L-S breaks down. J is no longer a
valid quantum number.
The ground state energy level will have  2S  1 2 L  1  6 states, but 2 states are

degenerate. Therefore, total energy levels will be 5 .

NET/JRF (JUNE-2014)
Q20. A spectral line due to a transition from an electronic state p to an s state splits into three
Zeeman lines in the presence of a strong magnetic field. At intermediate field strengths
the number of spectral lines is
(a) 10 (b) 3 (c) 6 (d) 9
Ans. : (a)
Solution: For p state: l  1, s  1/ 2 : j  1/ 2 & 3 / 2 .
This gives two spectral terms 2P3/2 & 2P1/2
For s state: l= 0, s = 1/2: j = 1/2 : This gives spectral terms 2S1/2
The terms 2P3/2 and 2S1/2 corresponding to J = 3/2 & J = 1/2 will break into 2J+1 Zeeman
levels, which is 4 and 2 respectively.
For 2P3/2 Mj = - 3/2 -1/2 +1/2 +3/2
For 2S1/2 Mj = -1/2 +1/2
The selection rule is ∆MJ = 0, ± 1 (MJ = 0 → MJ = 0 If ∆J = 0)
∆MJ = 0 gives π component, ∆MJ = ±1 gives σ component
Number of π component = 2, Numbers of + components = 2
Number of - components = 2
The terms 2P1/2 and 2S1/2 corresponding to J = 1/2 & J = 1/2 will break into 2J+1 Zeeman
levels, which is 2 & 2 respectively.
For 2P1/2 Mj = -1/2 +1/2, For 2S1/2 Mj = -1/2 +1/2

H.No. 40-D, Ground Floor, Jia Sarai, Near IIT, Hauz Khas, New Delhi-110016
Phone: 011-26865455/+91-9871145498
Website: www.physicsbyfiziks.com | Email: fiziks.physics@gmail.com
348
fiziks
Institute for NET/JRF, GATE, IIT‐JAM, M.Sc. Entrance, JEST, TIFR and GRE in Physics

The selection rule is ∆MJ = 0, ± 1 (MJ = 0 → MJ = 0 If ∆J = 0)


∆MJ = 0 gives π component, ∆MJ = ±1 gives σ component
Number of π component = 2, Numbers of + components = 1
Number of - components = 1
Thus, total number of Zeeman component = 10
Q21. A double slit interference experiment uses a laser emitting light of two adjacent
frequencies v1 and v 2 v1  v 2  . The minimum path difference between the interfering
beams for which the interference pattern disappears is
c c c c
(a) (b) (c) (d)
v 2  v1 v 2  v1 2v 2  v1  2v 2  v1 
Ans. : (c)
Solution: The condition of maximum intensity for interfering laser beam is: d sin   n
 1
The condition of dark intensity for interfering laser beam is: d sin    n  
 2
For interference pattern to vanish, the minimum path difference should be /2
c c 2
The spectral bandwidth of laser is defined as, v  2  
 v 
For two closely spaced line of wavelength 1 and 2
c 2  1 c c
  1 2   
v  1  2 1 1

c c  2  1

2 1 2 1
Since, for interference pattern to vanish for two closely spaced line of wavelength 1 and
 c c
2, the minimum path difference should be =  
2 2v 2 2   1 

Q22. How much does the total angular momentum quantum number J change in the transition
of Cr 3d 6  atom as it ionize to Cr 2 3d 4 ?
(a) Increases by 2 (b) Decreases by 2 (c) Decreases by 4 (d) Does not change
Ans. : (c)

 
Solution: In Cr 3d 6 state

M L  2  1 0  1  2

H.No. 40-D, Ground Floor, Jia Sarai, Near IIT, Hauz Khas, New Delhi-110016
Phone: 011-26865455/+91-9871145498
Website: www.physicsbyfiziks.com | Email: fiziks.physics@gmail.com
349
fiziks
Institute for NET/JRF, GATE, IIT‐JAM, M.Sc. Entrance, JEST, TIFR and GRE in Physics

1
In this configuration, S  4   2 and L  2
2
This is the case of more than half filled subshell, thus state with highest J value will
have the lowest energy.
The highest J - value is J  L  S  y

Thus, the ground state spectral term for this configuration is 5 D4 .

 
Now in Cr 2 3d 4
M L  2  1 0  1  2

1
In this configuration, S  4   2 and L  2
2
Since this is the case of less than half filled subshell, thus, state with lowest J value will
have the lowest energy.
The lowest J - value is J  L  5  2  2  0 .
Thus the ground state spectral term for this configuration is the J - value decreases from
J  4 to J  0 .
Thus correct answer is option (c).
NET/JRF (DEC-2014)
Q23. An atomic transition 1 P  1S in a magnetic field 1 Tesla shows Zeeman splitting. Given
that the Bohr magneton  B  9.27  10 24 J / T , and the wavelength corresponding to the
transition is 250 nm, the separation in the Zeeman spectral lines is approximately
(a) 0.01 nm (b) 0.1 nm (c) 1.0 nm (d) 10 nm
Ans. : (a)
Solution: This is the case of Normal Zeeman effect. The Zeeman separation in terms of
 B
frequency,   B , where  B is Bohr magneton
h
2  2 B B
In terms of wavelength,     
c c h

 250 10 m    9.27 10 J / T  1T   3 10


2
9 24

  12
m  0.003 nm
 3 10 m / s  6.625 10 Js 
8 34

None of the answer is matching correctly. But best suitable answer is option (a)
H.No. 40-D, Ground Floor, Jia Sarai, Near IIT, Hauz Khas, New Delhi-110016
Phone: 011-26865455/+91-9871145498
Website: www.physicsbyfiziks.com | Email: fiziks.physics@gmail.com
350
fiziks
Institute for NET/JRF, GATE, IIT‐JAM, M.Sc. Entrance, JEST, TIFR and GRE in Physics

Q24. If the leading anharmonic correction to the energy of n th vibrational level of a diatomic
2
 1
molecule is  xe  n    with xe  0.001 , the total number of energy levels possible
 2
is approximately
(a) 500 (b) 1000 (c) 250 (d) 750
Ans. : (a)
2
 1  1
Solution: The energy of anharmonic oscillator is Ev   v     xe  v   
 2  2
where v  0, 1, 2, .....vmax is vibrational quantum number v  vmax
dEv  1 
Now,  0    2 xe  vmax     0
dv v  vmax
 2 E

 1 1 1 1 1
 1  2 xe  vmax    vmax      500
 2  2 xe 2 2 xe 2  0.001 
 
Q25. The effective spin-spin interaction between the electron spin S e and the proton spin S p
 
in the ground state of the Hydrogen atom is given by H   aS e  S p . As a result of this

interaction, the energy levels split by an amount


1 2 3 2
(a) a (b) 2a 2 (c) a 2 (d) a
2 2
Ans. : (c)
 
Solution: The Hamiltonian is given as H  aSe  S p where Se and S p are electron and proton spin.

The coupling between Se and S p gives net resultant spin angular momentum
       1
S  Se  S p , S 2  Se2  S p2  2Se  S p  Se  S p   S 2  Se2  S p2 
2 Sp
S
 H
2

a 2
S  Se2  S p2 
Se
where S 2  S  S  1  2 , Se2  Se  Se  1  2 , S p2  S p  S p  1  2 F 1
1 2
a
1 1 4
Since Se  and S p   S  0, 1 12 s1/2
2 2 3
 a 2
4
F 0
H.No. 40-D, Ground Floor, Jia Sarai, Near IIT, Hauz Khas, New Delhi-110016
Phone: 011-26865455/+91-9871145498
Website: www.physicsbyfiziks.com | Email: fiziks.physics@gmail.com
351
fiziks
Institute for NET/JRF, GATE, IIT‐JAM, M.Sc. Entrance, JEST, TIFR and GRE in Physics

a 3 3 2 3 2
For S  0 (singlet state), H1   0       a
2 4 4 4
a 3 2 3 2 1 2
For S  1 (Triplet state), H 2   2       a
2 4 4  4
1 2 3 2
 H  H 2  H1  a  a  H  a 2
4 4

NET/JRF (JUNE-2015)
Q26. Of the following term symbols of the np 2 atomic configurations, 1 S0 ,3 P0 ,3 P1 ,3 P2 and
1
D2 which is the ground state?
(a) 3 P0 (b) 1 S0 (c) 3 P2 (d) 3 P1
Ans. (a)
Solution: According to Hund’s rules
(i) State with highest multiplicity has lowest energy
(ii) State with same multiplicity, the state with highest L will have lowest energy
(iii) State with same multiplicity and L value. The state with lowest J has lowest energy
(only if subshell is less than half filled) from the given states 1 S0 ,3 P0 ,3 P1 ,3 P21 D2

The 3 P0 will have the lowest energy


 1
Q27. A diatomic molecule has vibrational states with energies Ev    v   and rotational
 2

states with energies E j  Bj  j  1 , where v and j are non-negative integers . consider

the transitions in which both the initial and final states are restricted to v  1 and j  2
and subject to the selection rules v  1 and j  1 . Then the largest allowed energy
of transition is
(a)   3B (b)   B (c)   4 B (d) 2  B
Ans. (c)
 1
Solution: E    v    BJ  J  1
 2
For vibrational transition with v  1 and rotational transition with J   we get
E  Einitial  Efinal    2 B  J  1

H.No. 40-D, Ground Floor, Jia Sarai, Near IIT, Hauz Khas, New Delhi-110016
Phone: 011-26865455/+91-9871145498
Website: www.physicsbyfiziks.com | Email: fiziks.physics@gmail.com
352
fiziks
Institute for NET/JRF, GATE, IIT‐JAM, M.Sc. Entrance, JEST, TIFR and GRE in Physics

where J is lowest quantum number


According to question in rotational states, two transition is possible, one for
J  0  J  1 and second for J  1  J  2
But, second transition will have photon of higher energy
 For J  1  J  2
E    2 B 1  1    4 B

NET/JRF (DEC-2015)
Q28. The LS configurations of the ground state of 12Mg , 13
Al , 17Cl and 18
Ar are respectively,

(a) 3 S1 , 2 P1/ 2 , 2 P1/ 2 and 1 S0 (b) 3 S1 , 2 P3 / 2 , 2 P3 / 2 and 3 S1

(c) 1 S0 , 2 P1/ 2 , 2 P3 / 2 and 1 S0 (d) 1 S0 , 2 P3 / 2 , 2 P1/ 2 and 3 S1


Ans. : (c)
Solution: 12 Mg :1s 2 2s 2 2 p 6 3s 2

Ground state: 1 S0
13
Al :1s 2 2s 2 2 p 6 3s 2 3 p1
the terms are 2 p1/ 2 and 2 p3/ 2 . Since its less than half filled. Thus

Ground state: 2 p1/ 2


17
Cl :1s 2 2s 2 2 p 6 3s 2 3 p 5
The terms are 2 p1/ 2 and 2 p3/ 2 . Since it shell is more than half filled.

Thus Ground state: 2 p3/ 2


18
Ar :1s 2 2s 2 2 p 6 3s 2 3 p 6

Ground state: 1 S0

Q29. For a two level system, the population of atoms in the upper and lower levels are 3 1018
and 0.7 1018 , respectively. If the coefficient of stimulated emission is
3.0  105 m3 / W -s 3 and the energy density is 9.0 J / m3 -Hz , the rate of stimulated
emission will be
(a) 6.3 1016 s 1 (b) 4.1  1016 s 1 (c) 2.7  1016 s 1 (d) 1.8  1016 s 1
Ans. : None of the answer is matching.

H.No. 40-D, Ground Floor, Jia Sarai, Near IIT, Hauz Khas, New Delhi-110016
Phone: 011-26865455/+91-9871145498
Website: www.physicsbyfiziks.com | Email: fiziks.physics@gmail.com
353
fiziks
Institute for NET/JRF, GATE, IIT‐JAM, M.Sc. Entrance, JEST, TIFR and GRE in Physics

Solution: The rate of stimulated emission is


dN 2
 N 2 B21 u  w 
dt
m3 m3
where N 2  3 1018 , B21  3  105  3  10 5

W  s3 J  s2
J s
and u  w   9.0 J / m3  Hz  9.0
m3
dN 2 m3 J s
 3 1018  3 105  9 3  8.11024 s 1
dt J s 2
m
Q30. The first ionization potential of K is 4.34 eV , the electron affinity of Cl is 3.82 eV
and the equilibrium separation of KCl is 0.3 nm . Then energy required to dissociate a
KCl molecule into a K and a Cl atom is
(a) 8.62 eV (b) 8.16 eV (c) 4.28 eV (d) 4.14 eV
Ans. : (c)
Solution: Energy required to dissociate KCl is  KCl  K   Cl  

 
2
1 q1q2  9 Nm 
2 1.6  1019 c
V   9  10 2 
 7.7  1019 J  4.79 eV
4 0 r12  c  0.3 10 m9

The band dissociation energy is the energy required to dissociate a molecule into its
component atom KCl  K  Cl
To find the energy required to dissociate KCl into K and Cl , we must add an electron
to the K  ion, which releases the atomic potassium ionization energy. Remove one
electron from Cl  ion which requires the atomic chlorine electron affinity energy
Given ionization energy of K  Eie  4.34 eV

Electron affinity of Cl  Eai  3.82 eV


Thus the energy gained in the transformation from ion to atom is
 Eie  Eai  4.34  3.82  0.52 eV

Thus dissociation energy  4.79  0.52  4.27 eV

H.No. 40-D, Ground Floor, Jia Sarai, Near IIT, Hauz Khas, New Delhi-110016
Phone: 011-26865455/+91-9871145498
Website: www.physicsbyfiziks.com | Email: fiziks.physics@gmail.com
354
fiziks
Institute for NET/JRF, GATE, IIT‐JAM, M.Sc. Entrance, JEST, TIFR and GRE in Physics

NET/JRF (JUNE-2016)
Q31. The ground state electronic configuration of 22 Ti is  Ar  3d 2 4s 2 . Which state, in the

standard spectroscopic notations, is not possible in this configuration?


(a) 1 F3 (b) 1 S0 (c) 1 D2 (d) 3 P0
Ans. : (a)
Solution: The spectroscopic term for d 2 are obtained as
l1  2, l2  2  L  4,3, 2,1, 0

1 1
s1  , s2   S  0,1
2 2
Now, S  0, L  4  J  4, 1G4

S  0, L  2  J  2, 1 D2

S  0, L  0  J  0, 1S0

S  1, L  3  J  4,3, 2, 3
F4 , 3F3 , 3F2

S  1, L  1  J  2,1, 0, 3 P2 , 3 P1 ,3 P0
Thus 1 F3 is not possible spectroscopic term of  Ar  3d 2 4 S 2

Q32. In a normal Zeeman Effect experiment using a magnetic field of strength 0.3 T , the
splitting between the components of a 660 nm spectral line is
(a) 12 pm (b) 10 pm (c) 8 pm (d) 6 pm
Ans. : (d)
 2 eB  660 10 
2
9
1.6  1019  0.3
Solution:      6.09  1012 m  6 pm
c 4 m 3  108 4  9.110 31

Q33. The separation between the energy levels of a two-level atom is 2 eV . Suppose that

4  1020 atoms are in the ground state and 7  1020 atoms are pumped into the excited state
just before lasing starts. How much energy will be released in a single laser pulse?
(a) 24.6 J (b) 22.4 J (c) 98 J (d) 48 J
Ans. : (d)
Solution: N 2  N1  3  1020

 N  N1  3  1020
Energy of laser pulse, E   2  h   2 1.6 1019 J  E  48 J
 2  2
H.No. 40-D, Ground Floor, Jia Sarai, Near IIT, Hauz Khas, New Delhi-110016
Phone: 011-26865455/+91-9871145498
Website: www.physicsbyfiziks.com | Email: fiziks.physics@gmail.com
355
fiziks
Institute for NET/JRF, GATE, IIT‐JAM, M.Sc. Entrance, JEST, TIFR and GRE in Physics

NET/JRF (DEC-2016)
Q34. In the L  S coupling scheme, the terms arising from two non-equivalent p -electrons are

(a) 3S , 1P, 3P, 1D, 3D (b) 1S , 3S , 1P, 1D

(c) 1S , 3S , 3P, 3D (d) 1S , 3S , 1P, 3P, 1D, 3D


Ans. : (d)
Solution: For pp configuration
1 1
s1  , s2   S  0,1
2 2
l1  1, l2  1  L  0,1, 2
Thus
S  0, L  0  J  0 ; 1S0 or 1S

S  0, L  1  J  1 ; 1 P1 or 1 P

S  0, L  2  J  2 ; 1 D2 or 1 D

S  1, L  0  J  1 ; 3 S1 or 3 S

S  1, L  1  J  0,1, 2 ; 3 P0,1,2 or 3 P

S  1, L  2  J  1, 2,3 ; 3 D1,2,3 or 3 D

Thus spectroscopic terms are 1 S ,3 S ,1 P,3 P 1 D, 3 D


Q35. The total spin of a hydrogen atom is due to the contribution of the spins of the electron
and the proton. In the high temperature limit, the ratio of the number of atoms in the spin-
1 state to the number in the spin- 0 state is
1 1
(a) 2 (b) 3 (c) (d)
2 3
Ans. : (b)
Solution: F  1 (when spins are parallel)
2
S1/ 2
F  0 (when spins are anti-parallel)

The degeneracy of quantum level is 2 F  1 .


2 F1  1 3
Thus,  3
2 F0  1 1

H.No. 40-D, Ground Floor, Jia Sarai, Near IIT, Hauz Khas, New Delhi-110016
Phone: 011-26865455/+91-9871145498
Website: www.physicsbyfiziks.com | Email: fiziks.physics@gmail.com
356
fiziks
Institute for NET/JRF, GATE, IIT‐JAM, M.Sc. Entrance, JEST, TIFR and GRE in Physics

Q36. A two level system in a thermal (black body) environment can decay from the excited
state by both spontaneous and thermally stimulated emission. At room temperature
 300 K  , the frequency below which thermal emission dominates over spontaneous

emission is nearest to
(a) 1013 Hz (b) 108 Hz (c) 105 Hz (d) 1011 Hz
Ans. : (d)
Solution: At thermal equilibrium, the ratio of the number of spontaneous to stimulated emission
is given by

A21  1.054  1034 J .S
 e kT  1 ; where   2.551014 sec
B21 u   23
kT 1.38  10 J / K  300 K

Now, for v  1013 Hz    6.3 1013 rad / sec ,


A21
 e1.6  1  5  1  4
B21u  
For v  1011 Hz    6.3 1011 rad / sec ,
A21 3
  e1.610  1  1.03  1  0.03
B21u  
This ratio is less than one, thus stimulated thermal emission dominates over spontaneous
emission below all frequency of 1011 Hz
NET/JRF (JUNE-2017)
Q37. An atomic spectral line is observed to split into nine components due to Zeeman shift. If
the upper state of the atom is 3D2 then the lower state will be

(a) 3F2 (b) 3F1 (c) 3P1 (d) 3P2


Ans. : (c)
Solution: The possible zeeman component for different transitions are:
(i) 3D2  3 F2 : Total zeeman components are 12

(ii) 3D2  3P1 : Total zeeman components are 9

(iii) 3D2  3P2 : Total zeeman components are 12


3
F1 state is not allowed because for 3F , possible ‘ J ’ values are 2, 3 and 4.

Only 3 D2  3 P1 transition gives 9 components. Therefore, ground state is 3P1 .


Thus, correct option is (c).
H.No. 40-D, Ground Floor, Jia Sarai, Near IIT, Hauz Khas, New Delhi-110016
Phone: 011-26865455/+91-9871145498
Website: www.physicsbyfiziks.com | Email: fiziks.physics@gmail.com
357
fiziks
Institute for NET/JRF, GATE, IIT‐JAM, M.Sc. Entrance, JEST, TIFR and GRE in Physics

Q38. If the coefficient of stimulated emission for a particular transition is 2.1 1019 m3W 1s 3
0
and the emitted photon is at wavelength 3000 A , then the lifetime of the excited state is
approximately
(a) 20 ns (b) 40 ns (c) 80 ns (d) 100 ns
Ans. : (c)
A21 8 h 3 1 c3 3
Solution:  . Life time     
B21 c3 A21 8 h 3 B21 8 hB21

 
 3000 10  10 3

8  6.6 10 J .s. 2.1 10


34 19
m3 w1s 3 

2.7  1020
  7.7 108 sec  80n sec
3.5  1013
Q39. If the binding energies of the electron in the K and L shells of silver atom are 25.4 keV
and 3.34 keV , respectively, then the kinetic energy of the Auger electron will be
approximately
(a) 22 keV (b) 9.3keV (c) 10.5 keV (d) 18.7 keV
Ans. : (d)
Solution: K .E. Auger electron is K .E   EK  EL   EL

 EK  2 EL Auger electorn
e
  25.4  2  3.34  keV
Vacuum level
 18.7keV
L

h

H.No. 40-D, Ground Floor, Jia Sarai, Near IIT, Hauz Khas, New Delhi-110016
Phone: 011-26865455/+91-9871145498
Website: www.physicsbyfiziks.com | Email: fiziks.physics@gmail.com
358
fiziks
Institute for NET/JRF, GATE, IIT‐JAM, M.Sc. Entrance, JEST, TIFR and GRE in Physics

NET/JRF (DEC - 2017)


Q40. Consider a system of identical atoms in equilibrium with blackbody radiation in a cavity
at temperature T . The equilibrium probabilities for each atom being in the ground state
0 and an excited state 1 are P0 and P1 respectively. Let n be the average number of

photons in a mode in the cavity that causes transition between the two states. Let W01

and W10 denote, respectively, the squares of the matrix elements corresponding to the

atomic transitions 0  1 and 1  0 . Which of the following equations hold in

equilibrium?
(a) P0 nW01  PW
1 1 0 (b) P0 nW01  PnW
1 1 0

(c) P0 nW01  PW
1 10  PnW
1 10 (d) P0 nW01  PW
1 10  PnW
1 10

Ans. : (d)
Solution: In equilibrium condition the number of upward transitions must be equal to the number
of downward transition.
Rate of upward transition  P0 nW01
Rate of downward transition = Rate of spontaneous transitions + Rate of stimulated
transition  PW
1 10  PnW
1 10 1

Thus, at thermal equilibrium W01 W10


P0 nW01  PW
1 10  PnW
1 10
0

 Correct option is (d)


Q41. The Zeeman shift of the energy of a state with quantum numbers L, S , J and mJ is

mJ  B B  
Hz 
J  J  1
 L.J  g s S .J 
where B is the applied magnetic field, g s is the g -factor for the spin and

B
 1.4 MHz  G 1 , where h is the Planck constant. The approximate frequency shift of
h
the S  0, L  1 and mJ  1 state, at a magnetic field of 1G , is

(a) 10 MHz (b) 1.4 MHz (c) 5 MHz (d) 2.8 MHz
Ans. : (b)

H.No. 40-D, Ground Floor, Jia Sarai, Near IIT, Hauz Khas, New Delhi-110016
Phone: 011-26865455/+91-9871145498
Website: www.physicsbyfiziks.com | Email: fiziks.physics@gmail.com
359
fiziks
Institute for NET/JRF, GATE, IIT‐JAM, M.Sc. Entrance, JEST, TIFR and GRE in Physics

eB e B  B J 2
Solution: v   .  B E2
4 m 2m h h J 1

 1.4 MH z G 1
 1G   1.4 MHz E1
J
Q42. The separations between the adjacent levels of a normal multiplet are found to be
22 cm 1 and 33 cm 1 . Assume that the multiplet is described well by the L  S coupling

scheme and the Lande’s interval rule, namely E  J   E  J  1  AJ , where A is a

constant. The term notaions for this multiplet is


(a) 3 P0,1,2 (b) 3 F2,3,4 (c) 3 G3,4,5 (d) 3 D1,2,3

Ans. : (d)
Solution: E1  E  J  1  E  J   A  J  1  22 cm 1

E2  E  J  2   E  J  1  A  J  2   33 cm 1

A  J  1 22
   J 1
A  J  2 33

Thus, the J values for all those three levels are 1, 2,3 . The, corresponding terms are
3
D1,2,3 .

Q43. If the fine structure splitting between the 2 2 P3/ 2 and 2 2 P3/ 2 levels in the hydrogen atom

is 0.4 cm 1 , the corresponding splitting in Li 2 will approximately be

(a) 1.2 cm 1 (b) 10.8 cm 1 (c) 32.4 cm 1 (d) 36.8 cm 1


Ans. : (c)
Solution: E  z 4

 E  Li  zLi 4 34
    81
 E  H  zH 4 14

  E  Li  81 EH   81 0.4 cm 1

 E Li  32.4 cm1


Thus, correct option is (c).

H.No. 40-D, Ground Floor, Jia Sarai, Near IIT, Hauz Khas, New Delhi-110016
Phone: 011-26865455/+91-9871145498
Website: www.physicsbyfiziks.com | Email: fiziks.physics@gmail.com
360
fiziks
Institute for NET/JRF, GATE, IIT‐JAM, M.Sc. Entrance, JEST, TIFR and GRE in Physics

NET/JRF (JUNE-2018)
Q44. Two Stern-Gerlach apparatus S1 and S2 are kept in a line ( x -axis). The directions of

their magnetic fields are along the positive z and y -axes, respectively. Each apparatus
only transmits particles with spins aligned in thee direction of its magnetic field. If an
1
initially unpolarized beam of spin particles passes through this configuration, the ratio
2
of intensities l0 : l f of the initial and final beams is

l0 lf
k̂ ĵ

(a) 16 :1 (b) 2 :1 (c) 4 :1 (d) 1: 0


Ans. : (c)
I0 I0 4
Solution: I f  
4 If 1

Q45. A photon of energy 115.62 keV ionizes a K -shell electron of a Be atom. One L -shell
electron jumps to the K -shell to fill this vacancy and emits a photon of energy
109.2 keV in the process. If the ionization potential for the L -shell is 6.4 keV , the
kinetic energy of the ionized electron is
(a) 6.42 keV (b) 12.82 keV (c) 20 eV (d) 32 eV
Ans. : (c)
Solution: Binding energy of K -shell electron 6.4 KeV L
I p  6.4 KeV  109.2 KeV  115.6 KeV 109.2 KeV
Thus, K.E. of ionized electron is Ip K
= 115.62 KeV  115.6 KeV  0.02 KeV  20 eV
115.6 KeV e

Q46. The value of the Lande g -factor for a fine-structure level defined by the quantum number
L  1, J  2 and S  1 , is
11 4 8 3
(a) (b) (c) (d)
6 3 3 2
Ans. : (d)

H.No. 40-D, Ground Floor, Jia Sarai, Near IIT, Hauz Khas, New Delhi-110016
Phone: 011-26865455/+91-9871145498
Website: www.physicsbyfiziks.com | Email: fiziks.physics@gmail.com
361
fiziks
Institute for NET/JRF, GATE, IIT‐JAM, M.Sc. Entrance, JEST, TIFR and GRE in Physics

J  J  1  S  S  1  L  L  1 2  2  1  2  2 1 3
Solution: g  1   1  1 
2 J  J  1 2  2  2  1 2 2

Q47. The electronic energy level diagram of a molecule is shown in the following figure,
2
23

3
 21 34 Laser light
4

41
1
Let  ij denote the decay rate for a transition from the level i to j . The molecules are

optically pumped from level 1 to 2 . For the transition from level 3 to level 4 to be a
lassing transition, the decay rates have to satisfy
(a)  21   23   41  34 (b)  21   41   23  34

(c)  41   23   21  34 (d)  41   21  34   23


Ans. : (c)
Solution: The state 3 is metastable state, therefore 34 would be lowest to enhance population

inversion  41   23 . Thus correct sequence is  41   23   21  34

NET/JRF (DEC - 2018)


Q48. At low temperatures, in the Debye approximation, the contribution of the phonons to the
heat capacity of a two dimensional solid is proportional to
(a) T 2 (b) T 3 (c) T 1/ 2 (d) T 3/ 2
Ans. : (a)
Solution: The dispersion relation of phonons is   AK
The phonon specific heat in d -dimension is CV  T d

For 2 dimensional solid d  2


CV  T 2

H.No. 40-D, Ground Floor, Jia Sarai, Near IIT, Hauz Khas, New Delhi-110016
Phone: 011-26865455/+91-9871145498
Website: www.physicsbyfiziks.com | Email: fiziks.physics@gmail.com
362
fiziks
Institute for NET/JRF, GATE, IIT‐JAM, M.Sc. Entrance, JEST, TIFR and GRE in Physics

Q49. Barium Titanate  BaTiO3  crystal has a cubic perovskite

structure, where the Ba 2 ions are at the vertices of a unit


cube, the O 2 ions are at the centres of the faces while the
Ti 2 is at the centre. The number of optical phonon modes of
the crystal is
(a) 12 (b) 15
Ti 4 Ba 2  O 2
(c) 5 (d) 18
Ans. : (a)
Solution: Effective number of atoms per unit cell is
1 1
neff   8   6  11  1  3  1  5
8 2
Total degree of freedom 5  3  15
The number of Acoustical phonon modes  3
The number of optical phonon modes 15  3  12
Q50. The dispersion relation of optical phonons in a cubic crystal is given by   k   0  ak 2

where 0 and a are positive constants. The contribution to the density of states due to

these phonons with frequencies just below 0 is proportional to

(a) 0    (b) 0    (c) 0    (d) 0   


1/ 2 3/ 2 2

Ans. : (a)
3 3
 L   L 
Solution: g  k  dk    4 k dk  
2
 4 k  kdk
 2   2 
1
0    and d  2akdk
1/ 2
Given   0  ak 2  k 
a
3
 L  4
 g   d    3/ 2  0
 
1/ 2

 2  2a

Thus,     0   
1/ 2

H.No. 40-D, Ground Floor, Jia Sarai, Near IIT, Hauz Khas, New Delhi-110016
Phone: 011-26865455/+91-9871145498
Website: www.physicsbyfiziks.com | Email: fiziks.physics@gmail.com
363
fiziks
Institute for NET/JRF, GATE, IIT‐JAM, M.Sc. Entrance, JEST, TIFR and GRE in Physics

Q51. A silicon crystal is doped with phosphorus atoms. (The binding energy of a H atom
is 13.6 eV , the dielectric constant of silicon is 12 and the effective mass of electrons in
the crystal is 0.4 me ). The gap between the donor energy level and the bottom of the
conduction band is nearest to
(a) 0.01 eV (b) 0.08 eV (c) 0.02 eV (d) 0.04 eV
Ans. : (d)
13.6 M *
Solution: Ed    eV 
 2 Me
13.6
 Ed   0.4  0.04 eV
12 
2

H.No. 40-D, Ground Floor, Jia Sarai, Near IIT, Hauz Khas, New Delhi-110016
Phone: 011-26865455/+91-9871145498
Website: www.physicsbyfiziks.com | Email: fiziks.physics@gmail.com
364
fiziks
Institute for NET/JRF, GATE, IIT‐JAM, M.Sc. Entrance, JEST, TIFR and GRE in Physics

CONDENSED MATTER PHYSICS


NET/JRF (JUNE-2011)
0
Q1. A narrow beam of X - rays with wavelength 1.5 A is reflected from an ionic crystal with

an fcc lattice structure with a density of 3.32 gcm 3 . The molecular weight is 108 amu

( 1 amu  1.66 1024 g )


A. The lattice constant is
0 0 0 0
(a) 6.00 A (b) 4.56 A (c) 4.00 A (d) 2.56 A
Ans. : (a)
Solution: Given ne f f  4 , M = 108 kg,   3.32gm cm -3 = 3320 kgm-3,

N A  6.023 1026 atoms/kmol

ne f f  M 4  108
a3    6.00  10 30 m 3  6.00  10 10  6.00 A 0
NA   6.023  10 26  3320
B. The sine of the angle corresponding to (111) reflection is

3 3 1 1
(a) (b) (c) (d)
4 8 4 8
Ans. : (b)
Solution: According to Bragg’s law
 a a
2d sin    , sin   where d   for (111) plane
2d h2  k 2  l 2 3

3 3  1.5 A0 3 3 3
 sin      .
2a 2 6A 0
2 6 2 8
Q2. A flux quantum (fluxoid) is approximately equal to 210-7 gauss-cm2. A type II
superconductor is placed in a small magnetic field, which is then slowly increased till the
field starts penetrating the superconductor. The strength of the field at this point is
2
 105 gauss.

A. The penetrating depth of this superconductor is
0 0 0 0
(a) 100 A (b) 10 A (c) 1000 A (d) 314 A

H.No. 40-D, Ground Floor, Jia Sarai, Near IIT, Hauz Khas, New Delhi-110016
Phone: 011-26865455/+91-9871145498
Website: www.physicsbyfiziks.com | Email: fiziks.physics@gmail.com
365
fiziks
Institute for NET/JRF, GATE, IIT‐JAM, M.Sc. Entrance, JEST, TIFR and GRE in Physics

Ans. : (a)
Solution: Given Fluxoid  0  2  10 7 gauss –cm2

2
First Critical field H c1    10 5 gauss

The relation between first critical field and penetration depth is
0 0 2  107 0
H c1    2
   1012 cm 2    10 6 cm  100 A
2
 H c1   2  105

B. The applied field is further increased till superconductivity is completely destroyed.
8
The strength of the field is now  105 gauss. The correlation length of the

superconductor is
0 0 0 0
(a) 20 A (b) 200 A (c) 628 A (d) 2000 A
Ans. : None of the options is matched.
8
Solution: Given second critical field H c 2    10 5 gauss. The relation between second critical

0
field and correlation length is H c 2  2.

0 2  10 7 1 1 100 0
 2     10 12 cm 2     106 cm   1010 m  50 A
H c 2 8
   10 5 4
2 2

Q3. The two dimensional lattice of graphene is an arrangement of Carbon atoms forming a
honeycomb lattice of lattice spacing a, as shown below. The Carbon atoms occupy the
vertices.
1

c1
1

(A). The Wigner-Seitz cell has an area of d2


1

c2
1

3 2
1

a1
1 1

(a) 2a 2 (b) a d1 b1
2
1

a2 b2
a
2 3 3 2
(c) 6 3a (d) a
2
Ans. : (d)
 
Solution: Primitive lattice vectors are b1 and b2

H.No. 40-D, Ground Floor, Jia Sarai, Near IIT, Hauz Khas, New Delhi-110016
Phone: 011-26865455/+91-9871145498
Website: www.physicsbyfiziks.com | Email: fiziks.physics@gmail.com
366
fiziks
Institute for NET/JRF, GATE, IIT‐JAM, M.Sc. Entrance, JEST, TIFR and GRE in Physics


b1  3a cos 300 iˆ  3a cos 600 ˆj 
2
3
a  3iˆ  ˆj 
   3 3 2
b2 
2
3
a  
3i  ˆj ,  A  b2  b1 
2
a

(B). The Bravais lattice for this array is a


 
(a) rectangular lattice with basis vectors d1 and d 2
 
(b) rectangular lattice with basis vectors c1 and c2
 
(c) hexagonal lattice with basis vectors a1 and a2
 
(d) hexagonal lattice with basis vectors b1 and b2

Ans. : (c)
 
Solution: The Bravaiss lattice for this array is the Hexagonal lattice with basis vectors a1 and a 2

NET/JRF (DEC-2011)
Q4. The potential of a diatomic molecule as a function of the distance r between the atoms is
a b
given by V r    6
 12 . The value of the potential at equilibrium separation between
r r
the atoms is:
(a)  4a 2 / b (b)  2a 2 / b (c)  a 2 / 2b (d)  a 2 / 4b
Ans. : (d)
a b dV r 
Solution: Given V r    6
 12 . At equilibrium radius, 0
r r dr r r0

dV r  6a 12b r013 12b 2b 2b


   7  13  0  7    r06 
dr r0 r0 r0 6a a a

a b a2 a2  a2
 The value of potential at equilibrium is V r0         .
r06 r012 2b 4b 4b
Q5. If the number density of a free electron gas in three dimensions is increased eight times,
its Fermi temperature will
(a) increase by a factor of 4 (b) decrease by a factor of 4
(c) increase by a factor of 8 (d) decrease by a factor of 8

H.No. 40-D, Ground Floor, Jia Sarai, Near IIT, Hauz Khas, New Delhi-110016
Phone: 011-26865455/+91-9871145498
Website: www.physicsbyfiziks.com | Email: fiziks.physics@gmail.com
367
fiziks
Institute for NET/JRF, GATE, IIT‐JAM, M.Sc. Entrance, JEST, TIFR and GRE in Physics

Ans. : (a)
Solution: The relation between Fermi energy and electron density is E F 
2
2m
3 2 n  
2/3
.

E  '
F
2
2m

3 2  8n
2/3
 4E
 4 E F  TF'  F TF  4TF
EF
Q6. The excitations of a three-dimensional solid are bosonic in nature with their frequency ω
and wave-number k are related by ω  k2 in the large wavelength limit. If the chemical
potential is zero, the behaviour of the specific heat of the system at low temperature is
proportional to
(a) T 1/ 2 (b) T (c) T 3/ 2 (d) T 3
Ans. : (c)
Solution: If the dispersion relation is   k s in large wavelength. Then the specific heat is
C v  T 3 / s . Given   k 2  C v  T 3 / 2

NET/JRF (JUNE-2012)
Q7. Consider a system of non-interacting particles in d dimensional obeying the dispersion
relation   Ak s , where ε is the energy, k is the wavevector, s is an integer and A is
constant. The density of states, N(ε), is proportional to
s d d s
1 1 1 1
(a)  d (b)  s
(c)  s
(d)  d
Ans. : (b)
Q8. The experimentally measured transmission spectra of metal, insulator and semiconductor
thin films are shown in the figure. It can be inferred that I, II and III correspond,
respectively, to

 
Transmission (%)

(a) insulator, semiconductor and metal


(b) semiconductor, metal and insulator
(c) metal, semiconductor and insulator
(d) insulator, metal and semiconductor 

Ans. : (a) 300 1000


Wavelength (nm)

H.No. 40-D, Ground Floor, Jia Sarai, Near IIT, Hauz Khas, New Delhi-110016
Phone: 011-26865455/+91-9871145498
Website: www.physicsbyfiziks.com | Email: fiziks.physics@gmail.com
368
fiziks
Institute for NET/JRF, GATE, IIT‐JAM, M.Sc. Entrance, JEST, TIFR and GRE in Physics

Q9. The energy required to create a lattice vacancy in a crystal is equal to 1 eV. The ratio of
the number densities of vacancies n(1200 K)/n(300 K) when the crystal is at equilibrium
at 1200 K and 300 K, respectively, is approximately
(a) exp 30  (b) exp 15 (c) exp15 (d) exp30
Ans. : (d)
Solution: The equation for number density of vacancies n  Ne  E / 2 k BT where E: Energy required
to form vacancies, N: density of lattice sites
E  1 1
n1200K 
E  1 1  E  1 
n e  E / 2 k BT1   
2k T T     
 1   E / 2 k BT2  e B  2 1  ,  e 2 k B  300 1200   e 2 k B  400   e 30
n2 e n300 K 
Q10. The dispersion relation of phonons in a solid is given by
 2 k    02 3  cos k x a  cos k y a  cos k z a 
The velocity of the phonons at large wavelength is
(a)  0 a / 3 (b)  0 a (c) 3 0 a (d)  0 a / 2
Ans. : (d)
Solution: For large  , k x a, k y a, k z a  are small.
  k 2 a 2   k y2 a 2   k z2 a 2   02 a 2 2
 k    3  1  x   1 
2 2
0
2   2
  1 
  2
 
2

k x  k y2  k z2 
    
 2a2 a d  0 a
 2 k   0 k 2    0 k  v g   .
2 2 dk 2

NET/JRF (DEC-2012)
Q11. A magnetic field sensor based on the Hall Effect is to be fabricated by implanting As into
a Si film of thickness 1 μm. The specifications require a magnetic field sensitivity of
500 mV/Tesla at an excitation current of 1 mA. The implantation dose is to be adjusted
such that the average carrier density, after activation, is
(a) 1.25 × 1026 m-3 (b) 1.25 × 1022 m-3
(c) 4.1 × 1021 m-3 (d) 4.1 × 1020 m-3
Ans. : (b)
IB 103 1 V
Solution: n   6 19
 3
 1.25 1022 m 3 where H  500  10 3 V / T .
teVH 10 1.6 10 500 10 B

H.No. 40-D, Ground Floor, Jia Sarai, Near IIT, Hauz Khas, New Delhi-110016
Phone: 011-26865455/+91-9871145498
Website: www.physicsbyfiziks.com | Email: fiziks.physics@gmail.com
369
fiziks
Institute for NET/JRF, GATE, IIT‐JAM, M.Sc. Entrance, JEST, TIFR and GRE in Physics

Q12. In a band structure calculation, the dispersion relation for electrons is found to be
 k   cos k x a  cos k y a  cos k z a ,
where β is a constant and a is the lattice constant. The effective mass at the boundary of
the first Brillouin zone is
2 2 4 2 2 2
(a) (b) (c) (d)
5 a 2 5 a 2 2 a 2 3 a 2
Ans. : (d)
2
Solution:  k   cos k x a  cos k y a  cos k z a , Effective mass m  
 d 2 k 
 2 
 d k 
 
  
Brilliouin zone boundary is at k x   ,ky   ,kz   .
a a a
 d 2 k  2
Hence  2   3 a 2  m   .
 d k   , , 3 a 2
a a a

Q13. The radius of the Fermi sphere of free electrons in a monovalent metal with an fcc
structure, in which the volume of the unit cell is a3, is
1/ 3 1/ 3 1/ 3
 12 2   3 2   2  1
(a)  3  (b)  3  (c)  3  (d)
 a   a  a  a

Ans. : (a)
1/ 3
 3 2 N   2    2 k F2 
  3 n   
2/3
Solution: Radius of Fermi sphere is k F    , EF   2

 V   2m   2m 
1/ 3
N 4  12 2 
For fcc solid  3  k F   3  .
V a  a 

H.No. 40-D, Ground Floor, Jia Sarai, Near IIT, Hauz Khas, New Delhi-110016
Phone: 011-26865455/+91-9871145498
Website: www.physicsbyfiziks.com | Email: fiziks.physics@gmail.com
370
fiziks
Institute for NET/JRF, GATE, IIT‐JAM, M.Sc. Entrance, JEST, TIFR and GRE in Physics

NET/JRF (JUNE-2013)

Q14. Using the frequency-dependent Drude formula, what is the effective kinetic inductance of
a metallic wire that is to be used as a transmission line? [In the following, the electron
mass is m , density of electrons is n , and the length and cross-sectional area of the wire
 and A respectively.]


(a) mA / ne 2   (b) zero 
(c) m / ne 2 A  
(d) m A / ne 2  2 
Ans. : (c)
Q15. The phonon dispersion for the following one-dimensional diatomic lattice with masses
M 1 and M 2 (as shown in the figure)
K

M1 M2 M1 M2
is given by

 1 1  4M 1 M 2 2  qa 

 2 q   K    1  1  sin   
 M 1 M 2   M 1  M 2 2  2  

where a is the lattice parameter and K is the spring constant. The velocity of sound is
K M 1  M 2  K
(a) a (b) a
2M 1 M 2 2M 1  M 2 

K M 1  M 2  KM 1 M 2
(c) a (d) a
2M 1  M 2 
3
M 1M 2

Ans. : (b)
Solution: For small value of q (i.e. long wavelength approximation limit).

 qa  qa
We have sin   
 2  2

 1 1  4M 1M 2 2  qa 

2 q      1  1  sin  2 
 M1  M 2 
2
 M 1 M 2     

1  4M 1 M 2  qa  
2
 1
  2 q       1  1  2   
M
 1 M 2  

 M 1  M 2   2  

H.No. 40-D, Ground Floor, Jia Sarai, Near IIT, Hauz Khas, New Delhi-110016
Phone: 011-26865455/+91-9871145498
Website: www.physicsbyfiziks.com | Email: fiziks.physics@gmail.com
371
fiziks
Institute for NET/JRF, GATE, IIT‐JAM, M.Sc. Entrance, JEST, TIFR and GRE in Physics

 1 1    1 4 M 1 M 2 q 2 a 2 
  2 q       1  1  



 M 1 M 2    2 M 1  M 2  4 
2

 1 1   M 1 M 2 q 2 a 2 
  2 q       1  1 



   M 1  M 2  2 
2
 M1 M 2

 1 1   M 1 M 2 q 2 a 2 
For Acoustical branch:  2 q       1  1 



   M 1  M 2  2 
2
 M1 M 2

 M  M2  M 1 M 2 q 2 a 2  a 2
  2 q     1   
 M  M 2 2  2M  M  q
2

 M 1M 2  1 2  1 2


  q   aq
2M 1  M 2 

 
Velocity of sound is v g   a
q 2M 1  M 2 

Q16. The electron dispersion relation for a one-dimensional metal is given by


 ka 1 2 
 k  2 0 sin 2  sin ka 
 2 6 
where k is the momentum, a is the lattice constant,  0 is a constant having dimensions of

energy and ka   . If the average number of electrons per atom in the conduction band

is 1/3, then the Fermi energy is


(a)  0 / 4 (b)  0 (c) 2 0 / 3 (d) 5 0 / 3
Ans. : (a)
Q17. If the energy dispersion of a two-dimensional electron system is E  uk where u is the
velocity and k is the momentum, then the density of states D E  depends on the energy
as
(a) 1 / E (b) E (c) E (d) constant
Ans. : (c)
Solution: In two dimensional system, the number of allowed k-states in range k and k  dk is
2
 L 
g k dk    2kdk .
 2 

H.No. 40-D, Ground Floor, Jia Sarai, Near IIT, Hauz Khas, New Delhi-110016
Phone: 011-26865455/+91-9871145498
Website: www.physicsbyfiziks.com | Email: fiziks.physics@gmail.com
372
fiziks
Institute for NET/JRF, GATE, IIT‐JAM, M.Sc. Entrance, JEST, TIFR and GRE in Physics

E dE
Given dispersion relation is E  uk  k   dk 
u u
2 2
 L  E dE  L  2
 g E dE    2     EdE
 2   2  u 
2
u u

g E dE 1 L2
  E    E.
dE u 2 2
NET/JRF (DEC-2013)
Q18. The physical phenomenon that cannot be used for memory storage applications is
(a) large variation in magnetoresistance as a function of applied magnetic field
(b) variation in magnetization of a ferromagnet as a function of applied magnetic field
(c) variation in polarization of a ferroelectric as a function of applied electric field
(d) variation in resistance of a metal as a function of applied electric field

Ans. : (d)
Q19. The energy of an electron in a band as a function of its wave vector k is given
by E k   E 0  B cos k x a  cos k y a  cos k z a  , where E0 , B and a are constants. The

effective mass of the electron near the bottom of the band is


2 2 2 2 2
(a) (b) (c) (d)
3Ba 2 3Ba 2 2Ba 2 Ba 2
Ans. : (d)
Solution: Near the bottom of the band the k  0
1 1 1
 k x a  , cos k y a  1   k y a  , cos k z a  1   kz a 
2 2 2
cos k x a  1 
2 2 2
 1 1 1 2
E  k   E0  B  cos k x a  cos k y a  cos k z a   E0  B 1   k x a   1   k y a   1   k z a  
2 2

 2 2 2 
 1 2 1
 E0  B  3  a 2  k x  k x  k x    E0  3B  Ba 2 k 2
 2  2
2
 2
Effective mass of the electron is m *  2 
d E / dk 2 Ba 2

H.No. 40-D, Ground Floor, Jia Sarai, Near IIT, Hauz Khas, New Delhi-110016
Phone: 011-26865455/+91-9871145498
Website: www.physicsbyfiziks.com | Email: fiziks.physics@gmail.com
373
fiziks
Institute for NET/JRF, GATE, IIT‐JAM, M.Sc. Entrance, JEST, TIFR and GRE in Physics

Q20. A DC voltage V is applied across a Josephson junction between two superconductors


with a phase difference 0 . If I 0 and k are constants that depend on the properties of the
junction, the current flowing through it has the form
 2eVt   2eVt 
(a) I 0 sin   0  (b) kV sin   0 
     
(c) kV sin  0 (d) I 0 sin  0  kV
Ans. : (a)
Q21. A uniform linear monoatomic chain is modeled by a spring-mass system of masses m
separated by nearest neighbour distance a and spring constant m 02 . The dispersion
relation for this system is
  ka    ka 
(a)  k   2 0 1  cos   (b)  k   2 0 sin 2  
  2   2 
 ka   ka 
(c)  k   2 0 sin   (d)  k   2 0 tan  
 2   2
Ans. : (c)
Solution: The dispersion relation for uniform linear mono-atomic chain of atoms is
 ka 
 k   2 0 sin  
 2 

H.No. 40-D, Ground Floor, Jia Sarai, Near IIT, Hauz Khas, New Delhi-110016
Phone: 011-26865455/+91-9871145498
Website: www.physicsbyfiziks.com | Email: fiziks.physics@gmail.com
374
fiziks
Institute for NET/JRF, GATE, IIT‐JAM, M.Sc. Entrance, JEST, TIFR and GRE in Physics

NET/JRF (JUNE-2014)
Q22. The pressure of a nonrelativistic free Fermi gas in three-dimensions depends, at T  0 ,
on the density of fermions n as
(a) n 5 / 3 (b) n1 / 3 (c) n 2 / 3 (d) n 4 / 3
Ans. : (a)
Solution: The Fermi energy in three dimension is defined as
2/3
 2  3 2 N 
EF  
2m  V 
 
2
2m

3 2 n 2/3

Where, n is the electron concentration or density of free Fermi gas.


The total energy of free Fermi gas in 3D is
2/3
3 3  2  3 2 N 
E  NE F  N   
5 5 2m  V 

The pressure of a nonrelativistic free Fermi gas is defined as


 E 
p F  
3
  N
2
3 2 N     V 5 / 3
2/3  2
 V  N 5 2m  3

2
5
2
 nEF  n 
5
2
2m
3 n  
2 2/3 2 2
5 2m
3 2  n 5 / 3
2/3

Q23. Consider an electron in bcc lattice with lattice constant a . A single particle wavefunction


  
that satisfies the Bloch theorem will have the form f r  exp ik .r , with f r  being

 2   2   2 
(a) 1  cos   x  y  z   cos   x  y  z   cos   x  y  z 
 a   a   a 
 2   2   2 
(b) 1  cos   x  y   cos   y  z   cos   z  x 
 a   a   a 
     
(c) 1  cos   x  y   cos   y  z   cos   z  x 
a  a  a 
     
(d) 1  cos   x  y  z   cos   x  y  z   cos   x  y  z 
a  a  a 
Ans. : (b)
Solution: The primitive translational vector for BCC is

H.No. 40-D, Ground Floor, Jia Sarai, Near IIT, Hauz Khas, New Delhi-110016
Phone: 011-26865455/+91-9871145498
Website: www.physicsbyfiziks.com | Email: fiziks.physics@gmail.com
375
fiziks
Institute for NET/JRF, GATE, IIT‐JAM, M.Sc. Entrance, JEST, TIFR and GRE in Physics

 a
 a
2
  2
  a
 
a '  iˆ  ˆj  kˆ , b '  iˆ  ˆj  kˆ , c '  iˆ  ˆj  kˆ
2

Bloch function defined as
  
 
 k r   u k r e ik .r  f r e ik .r

Here f r  is atomic wavefunction, which has the periodicity of the lattice i.e.
 
u k r  a   u k r 
Given Bloch function
  2   2   2 
f (r )  1  cos   x  y    cos   y  z    cos   z  x  
 a   a   a 
   2  a a   2  a a   2  a a 
f (r  a ' )  1  cos   x  y     cos   y  z     cos   z  x   
 a  2 2   a  2 2   a  2 2 

   2   2   2 
f (r  a ' )  1  cos   x  y   cos   y  z   2   cos   z  x 
 a   a   a 

   2   2   2  
f (r  a ' )  1  cos   x  y   cos   y  z   cos   z  x   f (r )
 a   a   a 
  
f (r  a ' )  f (r )
Similarly,
     
f (r  b ' )  f (r ) and f ( r  c ' )  f ( r )
Other functions do not satisfy the periodicity
Q24. The dispersion relation for electrons in an f.c.c. crystal is given, in the tight binding
approximation, by
 kxa kya kya k a k a k a
 k   4 0 cos cos  cos cos z  cos z cos x 
 2 2 2 2 2 2 

where a is the lattice constant and  0 is a constant with the dimension of energy. The x -

 
component of the velocity of the electron at  , 0, 0  is
a 
(a)  2 0 a /  (b) 2 0 a /  (c)  4 0 a /  (d) 4 0 a / 

H.No. 40-D, Ground Floor, Jia Sarai, Near IIT, Hauz Khas, New Delhi-110016
Phone: 011-26865455/+91-9871145498
Website: www.physicsbyfiziks.com | Email: fiziks.physics@gmail.com
376
fiziks
Institute for NET/JRF, GATE, IIT‐JAM, M.Sc. Entrance, JEST, TIFR and GRE in Physics

Ans. : (d)
Solution: Group velocity of electron in dispersive medium is expressed as

 1 d 1  d ˆ d ˆ d    
v   i j kˆ   v x iˆ  v y ˆj  v z kˆ
 dk   dk x dk y dk z 
 kx a kya ka k a  ka kya kya k a 
 sin cos  cos z sin x  iˆ   cos x sin  sin cos z  ˆj  
 2 a  2 2 2 2   2 2 2 2  
v 0  

  sin k z a cos y  cos k x a sin k z a  kˆ
k a

  2 2 2 2  

 
At  , 0, 0 
a 
 2 a         
v  0  sin cos 0  cos 0sin  iˆ   cos sin 0  sin 0cos 0  ˆj   cos 0sin 0  sin 0cos  kˆ 
  2 2  2   2 

 4 a   
v  0 iˆ  0 ˆj  0kˆ    0iˆ  0 ˆj  0kˆ   vx iˆ  v y ˆj  vz kˆ
    

 4 a  
vx  0 , v y  0, vz  0

4 0 a
The x - component of velocity is vx 

NET/JRF (DEC-2014)
Q25. When laser light of wavelength  falls on a metal scale with 1 mm engravings at a
grazing angle of incidence, it is diffracted to form a vertical chain of diffraction spots on
a screen kept perpendicular to the scale. If the wavelength of the laser is increased by 200
nm, the angle of the first-order diffraction spot changes from 5 0 to
(a) 6.60 0 (b) 5.14 0 (c) 5.018 0 (d) 5.210
Ans. : (c)
Solution: The condition of maxima peak in grating is
b sin   m ; m  0,1, 2,3,....
where b is the width of slit or width of engraving, whereas ‘ m ’ is the order of
diffraction and  is the angle of diffraction
For 1st order diffraction: b sin    (i)

H.No. 40-D, Ground Floor, Jia Sarai, Near IIT, Hauz Khas, New Delhi-110016
Phone: 011-26865455/+91-9871145498
Website: www.physicsbyfiziks.com | Email: fiziks.physics@gmail.com
377
fiziks
Institute for NET/JRF, GATE, IIT‐JAM, M.Sc. Entrance, JEST, TIFR and GRE in Physics

When wavelength of incident light increased to    200 nm , let’s assume the 1st order

peak appears at    b sin      200 (ii)


Subtracting equation (i) from equation (ii), we get
b sin   b sin    200
200 200  109
 sin    sin    sin     sin 
b 10 3
 sin    2  10 4  sin   2  10 4  sin 50  2  10 4  0.087196  0.08736
    sin 1  0.08736     5.010
Q26. Consider the crystal structure of sodium chloride which is modeled as a set of touching
spheres. Each sodium atom has a radius r1 and each chlorine atom has a radius r2 . The
centres of the spheres from a simple cubic lattice. The packing fraction of this system is
 r 3
  r2 
3
 2 r13  r23
(a)   1      (b)
 r1  r2   r1  r2   3 r1  r2 3

r13  r23 r13  r23


(c) (d) 
r1  r2 3 2r1  r2 
3

Ans. : (b)
Solution: This question can only be solved by solving each option by assuming r1  r2 and

comparing result with the packing fraction of simple cubic which is .
6
  r 3  r 3    1 3  1 3  
Option (a):   1
 
2
          
 r1  r2   r1  r2    2   2   4
2 r13  r23 2 2r 3 2 1 
Option (b):     
3  r1  r2 3 3 8r 3 3 4 6
r13  r23 2r 3 1
Option (c):  3 
 r1  r2 
3
8r 4
 r13  r23 2r 3 
Option (d):  
2  r1  r2 3 2  8r 3
8

Thus, correct option is (b)

H.No. 40-D, Ground Floor, Jia Sarai, Near IIT, Hauz Khas, New Delhi-110016
Phone: 011-26865455/+91-9871145498
Website: www.physicsbyfiziks.com | Email: fiziks.physics@gmail.com
378
fiziks
Institute for NET/JRF, GATE, IIT‐JAM, M.Sc. Entrance, JEST, TIFR and GRE in Physics

Q27. Consider two crystalline solids, one of which has a simple cubic structure, and the other
has a tetragonal structure. The effective spring constant between atoms in the c -direction
is half the effective spring constant between atoms in the a and b directions. At low
temperatures, the behaviour of the lattice contribution to the specific heat will depend as
a function of temperature T as
(a) T 2 for the tetragonal solid, but as T 3 for the simple cubic solid
(b) T for the tetragonal solid, and as T 3 for the simple cubic solid
(c) T for both solids
(d) T 3 for both solids
Ans. : (d)
Solution: The specific heat of solid in three dimensions is proportional to T 3 and it is
independent of crystal structure.
In 3D : CV  T 3

In 2D : CV  T 2

In 1D : CV  T

Q28. A superconducting ring carries a steady current in the presence of a magnetic field B
normal to the plane of the ring. Identify the INCORRECT statement.
(a) The flux passing through the superconductor is quantized in units of hc / e
(b) The current and the magnetic field in the superconductor are time independent.
   
(c) The current density j and B are related by the equation   j  2 B  0 , where 
is a constant
(d) The superconductor shows an energy gap which is proportional to the transition
temperature of the superconductor
Ans. : (a)
Solution: The flux quantization in superconducting ring is   no

hc h
where o  in CGS units and o  in MKS units.
2e 2e

H.No. 40-D, Ground Floor, Jia Sarai, Near IIT, Hauz Khas, New Delhi-110016
Phone: 011-26865455/+91-9871145498
Website: www.physicsbyfiziks.com | Email: fiziks.physics@gmail.com
379
fiziks
Institute for NET/JRF, GATE, IIT‐JAM, M.Sc. Entrance, JEST, TIFR and GRE in Physics

NET/JRF (JUNE-2015)
Q29. X -ray of wavelength   a is reflected from the 111 plane of a simple cubic lattice. If

the lattice constant is a , the corresponding Bragg angle (in radian) is


   
(a) (b) (c) (d)
6 4 3 8
Ans. (c)
Solution: According to Bragg’s Law 2d sin   
a a a
where d    for 111 plane
h k l
2 2 2
111 3

 a 3  3 
 sin        sin 1   
2d a 2  2  3
2
3
Q30. The critical magnetic fields of a superconductor at temperatures 4 K and 8 K are
11 mA / m and 5.5 mA / m respectively. The transition temperature is approximately
(a) 8.4 K (b) 10.6 K (c) 12.9 K (d) 15.0 K
Ans. (b)
Solution: The relation between critical field and critical temperature is
  T 2 
H C T   H 0 1    
  TC  

Let at T  T1 , H C T1  , T  T2 , H C T   H C T2 

  T 2    T 2 
Thus we get H C T1   H 0 1     , H C T2   H 0 1   2  
1

  TC     TC  

H C T1  2
2
T 
1  1  T2  T12
H C T1  C  2 2 8   4 
2 2
T H T
  C  T   TC   10.6
H C T2  T 
2 C
H C T1  2 1
1  2  1
H C T2 
 TC
where T1  4 k , T2  8 k , H C T1   11 mA / m and H C T2   5.5 mA / m

H.No. 40-D, Ground Floor, Jia Sarai, Near IIT, Hauz Khas, New Delhi-110016
Phone: 011-26865455/+91-9871145498
Website: www.physicsbyfiziks.com | Email: fiziks.physics@gmail.com
380
fiziks
Institute for NET/JRF, GATE, IIT‐JAM, M.Sc. Entrance, JEST, TIFR and GRE in Physics

Q31. The low-energy electronic excitations in a two-dimensional sheet of grapheme is given



 
by E k  vk , where v is the velocity of the excitations. The density of states is

proportional to
3 1
(a) E (b) E 2 (c) E 2 (d) E 2
Ans. (a)
Solution: The number of k - states in range k and k  dk in two dimension is
2
 L 
g  k  dk    2 kdk
 2 
2 2
 L  E dE  L  2
 E   k  dE   dk  g  E  dE    2     EdE
 2     2    
2

The density of state is


g  E  dE
2
 L  2
 E    E   E  E
 2    
2
dE
Q32. A He  Ne laser operates by using two energy levels of Ne separated by 2.26 eV .
Under steady state conditions of optical pumping, the equivalent temperature of the
system at which the ratio of the number of atoms in the upper state to that in the lower
1
state will be , is approximately (the Boltzmann constant k B  8.6 105 eV / K )
20
(a) 1010 K (b) 108 K (c) 106 K (d) 104 K
Ans. (d)
Solution: According to Boltzmann relation
E2 N2
N2  E  N1  kT  E E
 exp     exp  T 
N1  kT  N2  E  N  E1 N1
k ln  2 
 N1 
N1 2.26
E  2.26 eV , k B  8.6 105 eV / K ,  20  T   0.877 104 K
N2  1 
8.6 105 ln  
 20 
 T  104 K

H.No. 40-D, Ground Floor, Jia Sarai, Near IIT, Hauz Khas, New Delhi-110016
Phone: 011-26865455/+91-9871145498
Website: www.physicsbyfiziks.com | Email: fiziks.physics@gmail.com
381
fiziks
Institute for NET/JRF, GATE, IIT‐JAM, M.Sc. Entrance, JEST, TIFR and GRE in Physics

NET/JRF (DEC-2015)
Q33. The first order diffraction peak of a crystalline solid occurs at a scattering angle of 300
when the diffraction pattern is recorded using an x-ray beam of wavelength 0.15 nm . If

the error in measurements of the wavelength and the angle are 0.01nm and 10
respectively, then the error in calculating the inter-planar spacing will approximately be
(a) 1.1 102 nm (b) 1.3  104 nm (c) 2.5  102 nm (d) 2.0  103 nm
Ans. : (a)
 d 1 d  cos 
Solution: Bragg’s Law for n  1,   2d sin   d    , 
2sin   2sin   2sin 2 
Error in d can be calculated as
2 2 2
 d   d  2  1  2   cos   2
     2  
2
        
      2sin    2sin  
d 2

2 2 2
 d2
1  2sin   2   cos    2sin   2
 2           
d 4sin    
2
 
 2sin  sin     
1

 2
 2
 2        2 2 2
 d
  2   d  d        
d 2
 2
tan      tan   

where   30o ,   1.5  1010 m,    0.11010 m,    1o

 1.5 1010
d   1.5  1010 m
2sin  2sin 30o
1
    
2 2 1
 0.11010   2
    3  
2 2

  180    1.5  1010  0.067   


2
Thus,  d  1.5 1010  10   
 1.5 10   tan 30     180  
     
  
1 1
 1.5  10  0.067    0.03  2  1.5 1010  0.005389 2
102 2

 
 d  1.5 10  0.0734  0.111010  1.11011 m  1.1102 nm
10

H.No. 40-D, Ground Floor, Jia Sarai, Near IIT, Hauz Khas, New Delhi-110016
Phone: 011-26865455/+91-9871145498
Website: www.physicsbyfiziks.com | Email: fiziks.physics@gmail.com
382
fiziks
Institute for NET/JRF, GATE, IIT‐JAM, M.Sc. Entrance, JEST, TIFR and GRE in Physics

Q34. The dispersion relation of electrons in a 3-dimensional lattice in the tight binding
approximation is given by,
 k   cos k x a   cos k y a   cos k z a
where a is the lattice constant and  ,  ,  are constants with dimension of energy. The

   
effective mass tensor at the corner of the first Brillouin zone  , ,  is
a a a

 1/  0 0   1/  0 0 
2   2  
(a) 2  0 1/  0  (b) 2  0 1/  0 
a  a 
 0 0 1/    0 0 1/  

 1/  0 0  1/  0 0 
2   2  
(c) 2  0 1/  0  (d) 2  0 1/  0 
a  a 
 0 0 1/    0 0 1/  

Ans. : (c)
Solution: The effective mass as a tensor quantity can be written as
 m*xx m*xy m*xz 
 *  2
mij*   m*yx m*yy m yz  where mij 
*

 2E 
 m*zx m*zy m*zz   

 ki k j 
since  k   cos k x a   cos k y a   cos k z a

2  2 2 2
 m*xx   , m*yy  
  2   a 2 cos k x a   2   a 2 cos k y a
   2 
 k x k x   k y 

2 2
m  2  2
*
, other terms are zero
     a cos k z a
zz

 2
 k z 

1/  0 0 
    * 2 2 2 2 
Now, at  , ,  ; mxx  , m yy 
*
, mzz  2  mij  2  0 1/ 
* *
0 
a a a a 2
a 2
a a
 0 0 1/  

H.No. 40-D, Ground Floor, Jia Sarai, Near IIT, Hauz Khas, New Delhi-110016
Phone: 011-26865455/+91-9871145498
Website: www.physicsbyfiziks.com | Email: fiziks.physics@gmail.com
383
fiziks
Institute for NET/JRF, GATE, IIT‐JAM, M.Sc. Entrance, JEST, TIFR and GRE in Physics

Q35. A thin metal film of dimension 2 mm  2 mm contains 4  1012 electrons. The magnitude
of the Fermi wavevector of the system, in the free electron approximation, is
(a) 2   107 cm 1 (b) 2  107 cm 1 (c)   107 cm 1 (d) 2  107 cm 1
Ans. : (b)
Solution: This is the case of two dimensional metal box. The Fermi wave vector of electron in
2  D is
1
1
 N 2
k F   2 n  2   2 2  ; L2  2mm  2mm  4  102 cm 2 ,
 L 
1
 4  10  12 1
 2 10 cm   2  107 cm 1
2
 k F  2  2 2 
14 2 2

 4 10 cm 
Q36. For an electron moving through a one-dimensional periodic lattice of periodicity a ,
which of the following corresponds to an energy eigenfunction consistent with Bloch’s
theorem?
  x   x     x  2 x   
(a)   x   A exp  i   cos    (b)   x   A exp  i   cos   
  a  2a      a  a  

  2 x  2 x      x  x 
(c)   x   A exp  i   i cosh     (d)   x   A exp  i  i 
  a  a     a 2a  

Ans. : (b)
Solution: According to block theorem,   x  a     x 

   2    x 2 x   
  x  a   A exp i   x  a   cos   x  a      A exp i      cos   2   
  a  a      a   a   

  2 x   x 2 x  
 A exp i   x  a   cos   A exp i   cos 
 a a    a a  

H.No. 40-D, Ground Floor, Jia Sarai, Near IIT, Hauz Khas, New Delhi-110016
Phone: 011-26865455/+91-9871145498
Website: www.physicsbyfiziks.com | Email: fiziks.physics@gmail.com
384
fiziks
Institute for NET/JRF, GATE, IIT‐JAM, M.Sc. Entrance, JEST, TIFR and GRE in Physics

NET/JRF (JUNE-2016)
Q37. Consider electrons in graphene, which is a planar monoatomic layer of carbon atoms. If
the dispersion relation of the electrons is taken to be   k   ck (where c is constant)

over the entire k -space, then the Fermi energy  F depends on the number density of

electrons  as
1 2 1
(a)  F   2 (b)  F   (c)  F   3 (d)  F   3
Ans. : (a)
Solution: In 2 D , density of state is
 L   d
g  k  dk    2 kdk , where   ck  k  and dk 
 2  c c
2
 L   d L2
 g   d     2  .   d
 2  c c 2 c 2
Now, number electrons at T  0 K is
F L2 F L2 2 N
N   g   d     d     F2  4 c 2 2  4 c 2 
0 2 c 2 0 4 c 2 F
L

  F  4 c 2  1/ 2   F   1/ 2
Q38. Suppose the frequency of phonons in a one-dimensional chain of atoms is proportional to
the wave vector. If n is the number density of atoms and c is the speed of the phonons,
then the Debye frequency is
 cn
(a) 2 cn (b) 2 cn (c) 3 cn (d)
2
Ans. : (d)
Solution: Given   k    ck ( c is velocity of phonon)
L d L
Now g   d   d
 d / dk c
D L D L
Also N   g   d    d  N  D
0 c 0 c
N  N cn
 D  c  c n,  n    f D  . Best answer is (d).
L  L 2

H.No. 40-D, Ground Floor, Jia Sarai, Near IIT, Hauz Khas, New Delhi-110016
Phone: 011-26865455/+91-9871145498
Website: www.physicsbyfiziks.com | Email: fiziks.physics@gmail.com
385
fiziks
Institute for NET/JRF, GATE, IIT‐JAM, M.Sc. Entrance, JEST, TIFR and GRE in Physics

Q39. The band energy of an electron in a crystal for a particular k -direction has the form
  k   A  B cos 2ka , where A and B are positive constants and 0  ka   . The
electron has a hole-like behaviour over the following range of k :
 3    3
(a)  ka  (b)  ka   (c) 0  ka  (d)  ka 
4 4 2 4 2 4
Ans. : (a)
d d 2
Solution:   k   A  B cos 2ka ,  2 Ba sin 2ka , 2
 4 Ba 2 cos 2ka
dk dk
2 2
 
Effective mass m*  2 
d  / dk 2 2
4 Ba cos 2ka

   
Effective mass of electron me* and effective mass of holes mh* are opposite in sign i.e.,
m *
h  m . *
e 

Now, in the range 0  ka  , m* is positive
4
3 
While in the range  ka  , m* is negative
4 4
 3
Thus, electron has hole like behaviour in the region  ka 
4 4
NET/JRF (DEC-2016)
Q40. Consider a hexagonal lattice with basis vectors as shown in the figure below.
y

x

a2 a
1

a
If the lattice spacing is a  1 , the reciprocal lattice vectors are

 4   2 2   4   2 2 
(a)  ,0,   ,  (b)  ,0,  , 
 3   3 3  3   3 3

 4   2   2 2   2 
(c)  0,  ,  ,  (d)  ,  ,  2 , 
 3   3  3 3  3
Ans. : (a)
H.No. 40-D, Ground Floor, Jia Sarai, Near IIT, Hauz Khas, New Delhi-110016
Phone: 011-26865455/+91-9871145498
Website: www.physicsbyfiziks.com | Email: fiziks.physics@gmail.com
386
fiziks
Institute for NET/JRF, GATE, IIT‐JAM, M.Sc. Entrance, JEST, TIFR and GRE in Physics

Solution: From the figure, we can write


a1 
3a
2
   
3 xˆ  yˆ , a2  3a yˆ , a3  azˆ (let us assume)

  
Now V  a1.  a2  a3  
3a
2
 3 xˆ  yˆ . 3ayˆ  azˆ 

3a
2
 
3 xˆ  yˆ . 3a 2 xˆ  
3 3 3
2
a

 
Also, a3  a1  azˆ 
3a
2

3x  y 
ˆ ˆ
3a 2
2
 3 yˆ  xˆ  
Reciprocal lattice vectors are
 
* a2  a3 3a 2 xˆ 4 4
a1  2  2  xˆ  0 yˆ  xˆ  0 yˆ
V 3 3 3 3a 3
a
2

 
a3  a1
3 2
a 
3 yˆ  xˆ
2   2 2
*
a2  2
V
 2 2
3 3 3

3a

 xˆ  3 yˆ  
 3
xˆ 
3
 
yˆ 

a
2
* 4  2 2
for a  1 : a1  xˆ  0 yˆ , a2*   xˆ  yˆ
3 3 3
Q41. Consider a one-dimensional chain of atoms with lattice constant a . The energy of an
electron with wave-vector k is   k      cos  ka  , where  and  are constants. If

an electric field E is applied in the positive x -direction, the time dependent velocity of
an electron is
(In the following B is the constant)
 eE 
(a) Proportional to cos  B  at  (b) proportional to E
  

 eE 
(c) independent of E (d) proportional to sin  B  at 
  
Ans. : (d)
Solution: In the presence of electric field E , we can write
   
dp dk
F  eE   eE    eE
dt dt

H.No. 40-D, Ground Floor, Jia Sarai, Near IIT, Hauz Khas, New Delhi-110016
Phone: 011-26865455/+91-9871145498
Website: www.physicsbyfiziks.com | Email: fiziks.physics@gmail.com
387
fiziks
Institute for NET/JRF, GATE, IIT‐JAM, M.Sc. Entrance, JEST, TIFR and GRE in Physics

eE
Integration gives, k  t   k  0   t

d  1   k 
The group velocity v  
dk  dk
  k 
Since,   k      cos  ka  ,    a sin ka
k
a
Thus, v  sin  ka 

Time dependent velocity of electron is
a a  eE  
v t   sin  k  t  a   sin  k  0   ta
     

a  eE  a  eE 
 sin  k  0  a  at   v  t   sin  B  at
       
Q42. A thin rectangular conducting plate of length a and width b is placed in the xy -plane in
two different orientations as shown in the figures below. In both cases a magnetic field B
is applied in the z -direction and a current flows in the x direction due to the applied
voltage V . b
y B

B
x a
V1 a
V1 b

   
V V
If the Hall voltage across the y -direction in the two cases satisfy V2  2V1 the ratio a : b
must be
(a) 1: 2 (b) 1: 2 (c) 2 :1 (d) 2 :1
Ans. : (d)
IB
Solution: Since, Hall voltage is given by VH  , where w is width of conducting plate.
w

H.No. 40-D, Ground Floor, Jia Sarai, Near IIT, Hauz Khas, New Delhi-110016
Phone: 011-26865455/+91-9871145498
Website: www.physicsbyfiziks.com | Email: fiziks.physics@gmail.com
388
fiziks
Institute for NET/JRF, GATE, IIT‐JAM, M.Sc. Entrance, JEST, TIFR and GRE in Physics

 l1 a 
Since, in case (I), V  I1 R1 and R1   
A1 ab b

I1  bV
V  I1 
b 
I1 B bVB bVB
Then, VH  V1     w  a 
w 2w 2a
 l2 b 
And also in case (II), R2   
A2 ab a

V Va
V  I 2 R2  I 2  
R2 
I 2 B VaB
Then, VH  V2  
 w  2b

a2 2
Since, V2  2V1    a : b  2 :1
b2 1

NET/JRF (JUNE-2017)
Q43. The energy gap and lattice constant of an indirect band gap semiconductor are 1.875 eV
and 0.52 nm , respectively. For simplicity take the dielectric constant of the material to be
unity. When it is excited by broadband radiation, an electron initially in the valence band
at k  0 makes a transition to the conduction band. The wavevector of the electron in the
conduction band, in terms of the wavevector kmax at the edge of the Brillouin zone, after
the transition is closest to
(a) kmax /10 (b) kmax /100 (c) kmax /1000 (d) 0
Ans. : (a)
Solution: The K  value of electron in C.B. is

2mE  2  9.1 10 kg 1.875 1.6 10 J  


1/ 2
31 19

K   K  7 109 m 1
 1.05  1034 J .S .
2 2  3.14 K
K max at the Brillouin Zone is K max   9
 1.2 1010 m 1  K  max
a 0.52 10 m 10

H.No. 40-D, Ground Floor, Jia Sarai, Near IIT, Hauz Khas, New Delhi-110016
Phone: 011-26865455/+91-9871145498
Website: www.physicsbyfiziks.com | Email: fiziks.physics@gmail.com
389
fiziks
Institute for NET/JRF, GATE, IIT‐JAM, M.Sc. Entrance, JEST, TIFR and GRE in Physics

Q44. The electrical conductivity of copper is approximately 95% of the electrical conductivity
of silver, while the electron density in silver is approximately 70 % of the electron
density in copper. In Drude’s model, the approximate ratio  Cu /  Ag of the mean collision

time in copper  Cu  to the mean collision time in silver  Ag  is

(a) 0.44 (b) 1.50 (c) 0.33 (d) 0.66


Ans. : (d)
ne2  n    n
Solution:    cu  cu cu  cu  cu  Ag
m  Ag nAg  Ag  Ag  Ag ncu

 cu 0.95 Ag 0.7ncu
    0.66
 Ag  Ag ncu

NET/JRF (DEC - 2017)


1 
Q45. The dispersion relation of a gas of spin fermions in two dimensions is E  v k ,
2

where E is the energy, k is the wave vector and v is a constant with the dimension of
velocity. If the Fermi energy at zero temperature is F , the number of particles per unit
area is
F 3F  3/F 2 2F
(a) (b) (c) (d)
 4 v  
6 2 v33   3v  
3 3

2 v 2  2 
Ans. : (d)
 E dE
Solution: E  v k  k   dk 
v v
2 2
 L  E dE  L  4
g  E  dE  2   .2 . .   . E.dE.
 2  v v  2   v 2

at T  0 K ,
EF
L2 4 2F
N  g  E dE  .
0
4 2  v 2 2
L2
N 2F
2  v
2 2

N 2F
n 
L2 2 v 2  2
H.No. 40-D, Ground Floor, Jia Sarai, Near IIT, Hauz Khas, New Delhi-110016
Phone: 011-26865455/+91-9871145498
Website: www.physicsbyfiziks.com | Email: fiziks.physics@gmail.com
390
fiziks
Institute for NET/JRF, GATE, IIT‐JAM, M.Sc. Entrance, JEST, TIFR and GRE in Physics

Q46. A crystal of MnO has NaCl structure. It has a paramagnetic to anti- ferromagnetic
transition at 120 K . Below 120 K , the spins within a single 111 planes are parallel but

the spins in adjacent 111 planes are antiparallel. If neutron scattering is used to

determine the lattice constants, respectively, d and d  , below and above the transition
temperature of MnO then
d d
(a) d  (b) d  (c) d  2d  (d) d  2d 
2 2
Ans. : (c)

Solution: At 80 K there are extra reflection not present at 293 K .


The reflection at 80 K may be classified in term of a cubic unit cell at lattice constant
0
8.85 A .
0
At 293 K the reflection corresponds to an FCC unit cell of lattice constant 4.43 A

d  a / 3 4.43 1
     d  2d 
d a / 3 8.85 2
Q47. A metallic nanowire of length l is approximated as a one-dimensional lattice of N
atoms with lattice spacing a . If the dispersion of electrons in the lattice is given
as E  k   E0  2t cos ka , where E0 and t are constants, then the dnsity of states inside

the nanowire depends on E as


2
t2  E  E0 
(a) N 3
E  E0
(b)  2t   1
 

E  E0 N
(c) N 3 (d)
t2  2t    E  E0 
2 2

Ans. (d)
Solution: E  k   E0  2t cos ka

dE
dE  2ta sin ka dk  dk 
2 ta sin ka

H.No. 40-D, Ground Floor, Jia Sarai, Near IIT, Hauz Khas, New Delhi-110016
Phone: 011-26865455/+91-9871145498
Website: www.physicsbyfiziks.com | Email: fiziks.physics@gmail.com
391
fiziks
Institute for NET/JRF, GATE, IIT‐JAM, M.Sc. Entrance, JEST, TIFR and GRE in Physics

2L 2L dE
 g  E  dE  dk 
  2 ta 1  cos 2 ka

L dE 2L dE
 
 ta E E
2 a  2t    E  E0 
2 2

1  0 
 2t 
 Density of state is,

g  E  dE 2N
 E  
dE   2t    E  E0 
2 2

Q48. Consider a two-dimensional material of length l and width w subjected to a constant


magnetic field B applied perpendicular to it. The number of change carries per unit area
B
may be expressed as n  k q , where k is appositive real number and q is the
 2  
carrier charge. Then the Hall resistivity  xy is

2 k l 2  w 2  2 k
(a) (b) (c) (d)
q2 w kq 2 l kq 2 q2
Ans. : (c)
A VH A
Solution:  xy  R    A  lw 
l I l
IB
where, VH 
nqw
B A B B 2 
  xy      2
nqw l nq k  q B  q kq
2 

H.No. 40-D, Ground Floor, Jia Sarai, Near IIT, Hauz Khas, New Delhi-110016
Phone: 011-26865455/+91-9871145498
Website: www.physicsbyfiziks.com | Email: fiziks.physics@gmail.com
392
fiziks
Institute for NET/JRF, GATE, IIT‐JAM, M.Sc. Entrance, JEST, TIFR and GRE in Physics

NET/JRF (JUNE-2018)
Q49. Sodium Chloride  NaCl  crystal is a face-centered cubic lattice with a basis consisting of

Na  and Cl  ions separated by half the body diagonal of a unit cube. Which of the
planes corresponding to the Miller indices given below will not give rise to Bragg
reflection of X -rays?
(a)  220  (b)  242  (c)  221 (d)  311

Ans. : (c)
Solution: Mixed  h k l  are absent in NaCl .  221 is mixed number of even and odd therefore

this plane is absent.


Q50. The dispersion relation for the electrons in the conduction band of a semiconductor is
given by E  E0   k 2 where  and E0 are constants. If c is the cyclotron resonance

frequency of the conduction band electrons in a magnetic field B , the value of  is


c 2 2c  2c  2c
(a) (b) (c) (d)
4eB eB eB 2eB
Ans. : (d)
eB 2
Solution: c  * where m  2
*

m d E / dk 2
d 2E
Since E  E0   k  2  2 2

dk
eB 2  2c
 c  2  2 eB   
 / 2  2eB
Q51. Hard disc of radius R are arranged in a two-dimensional triangular lattice. What is the
fractional area occupied by the discs in the closest possible packing?

 3   2 2
(a) (b) (c) (d)
6 3 2 5 7
Ans. : (a)
neff   r 2
Solution: P.F 
A
1 3 2 3 3 2 3 3
 2r   6 3r 2
2
where neff   6  1  3 and A  6  a  a 
3 4 2 2
3 r 2   3
P.F .   
6 3r 2
2 3 6
H.No. 40-D, Ground Floor, Jia Sarai, Near IIT, Hauz Khas, New Delhi-110016
Phone: 011-26865455/+91-9871145498
Website: www.physicsbyfiziks.com | Email: fiziks.physics@gmail.com
393
fiziks
Institute for NET/JRF, GATE, IIT‐JAM, M.Sc. Entrance, JEST, TIFR and GRE in Physics

NET/JRF (DEC - 2018)


Q52. The diatomic molecule HF has an absorption line in the rotational band at 40 cm 1 for
18 19
the isotope F . The corresponding line for the isotope F will be shifted by
approximately
(a) 0.05 cm 1 (b) 0.11 cm 1 (c) 0.33 cm 1 (d) 0.01 cm 1
Ans. : (b)
1 18 18
Solution: For 1HF 18 : 2 B1  40 cm 1  B1  20 cm 1 and reduce mass is 1  
1  18 19
119 19
For 1 HF 19 : The reduce mass is 2   and rotational constant is B2 .
1  19 20
B2 1
Since, 
B1 2
1 18 20
 B2   B1    20 cm 1  19.945 cm 1
2 19 19

Thus, 2 B2  39.889 cm 1

Shift in spectral line  2 B1  2 B2  40  39.889  0.11 cm 1

Q53. The excited state  n  4, l  2  of an election in an atom may decay to one or more of the

lower energy levels shown in the diagram below.

n4
l2

n3
l  0 l 1 l  2

n2
l 1
1
Of the total emitted light, a fraction comes from the decay to the state ( n  2, l  1 ).
4
Based on selection rules, the fractional intensity of the emission line due to the decay to
the state  n  3, l  1

3 1 1
(a) (b) (c) (d) 0
4 2 4

H.No. 40-D, Ground Floor, Jia Sarai, Near IIT, Hauz Khas, New Delhi-110016
Phone: 011-26865455/+91-9871145498
Website: www.physicsbyfiziks.com | Email: fiziks.physics@gmail.com
394
fiziks
Institute for NET/JRF, GATE, IIT‐JAM, M.Sc. Entrance, JEST, TIFR and GRE in Physics

Ans. : (a)
Solution: According to the selection rule for electric dipole, transition is  l  1 , the transition

from  n  4, l  2  to  n  3, l  0  and  n  3, l  2  is forbidden. If I is the intensity of

I I 3I
the total emitted light and I1  , therefore, I 2  I   .
4 4 4
I2
I 3 4d
Thus, 2  n4
I 4 l2

n3 3s 3d
I1
l  0 l 1 l2
2p
n2
l 1

Q54. The volume of an optical cavity is 1 cm3 . The number of modes it can support within a
bandwidth of 0.1 nm , centered at   500 nm , is of the order of

(a) 103 (b) 105 (c) 1010 (d) 107


Ans. : (c)
Solution: Number of Laser modes
 0.1 109
N  8 V 
 8  10 
2 3
  4.02 1010
  5 10 
4 4
7

H.No. 40-D, Ground Floor, Jia Sarai, Near IIT, Hauz Khas, New Delhi-110016
Phone: 011-26865455/+91-9871145498
Website: www.physicsbyfiziks.com | Email: fiziks.physics@gmail.com
395
fiziks
Institute for NET/JRF, GATE, IIT‐JAM, M.Sc. Entrance, JEST, TIFR and GRE in Physics

NUCLEAR AND PARTICLE PHYSICS


NET/JRF (JUNE-2011)

Q1. The radius of a 64


29 Cu nucleus is measured to be 4.8  10-13 cm.
(A) The radius of a 27
12 Mg nucleus can be estimated to be
(a) 2.86  10-13 cm (b) 5.2 10-13 cm (c) 3.6 10-13 cm (d) 8.6 10-13 cm
Ans. : (c)
1/ 3
RMg  AMg 
1/ 3
 27 
Solution: Since R  R0  A      
1/ 3

RCu  ACu   64 
RMg 3 3
  RMg   4.8  10 13  3.6  10 13 cm.

RCu 4 4
(B) The root-mean-square (r.m.s) energy of a nucleon in a nucleus of atomic number A in
its ground state varies as:
(a) A4 / 3 (b) A1 / 3 (c) A1/ 3 (d) A2 / 3
Ans. : (c)
Q2. A beam of pions (π+) is incident on a proton target, giving rise to the process
  p  n  
(A) Assuming that the decay proceeds through strong interactions, the total isospin I and
its third component I3 for the decay products, are
3 3 5 5
(a) I  , I 3  (b) I  , I 3 
2 2 2 2
5 3 1 1
(c) I  , I 3  (d) I  , I 3  
2 2 2 2
Ans. : (c)
1 5 1 3
Solution:    p  n       ; I : 1 1  , I3 :  11 
2 2 2 2
(B) Using isospin symmetry, the cross-section for the above process can be related to that
of the process
(a)  n  p  (b)  p  n  
     

(c)  n  p  (d)  p  n 
     

Ans. : (c)

H.No. 40-D, Ground Floor, Jia Sarai, Near IIT, Hauz Khas, New Delhi-110016
Phone: 011-26865455/+91-9871145498
Website: www.physicsbyfiziks.com | Email: fiziks.physics@gmail.com
396
fiziks
Institute for NET/JRF, GATE, IIT‐JAM, M.Sc. Entrance, JEST, TIFR and GRE in Physics

NET/JRF (DEC-2011)
Q3. According to the shell model the spin and parity of the two nuclei 125
51 Sb and 89
38 Sr are,
respectively,
   
5 5 5 7
(a)   and   (b)   and  
2 2 2 2
   
7 5 7 7
(c)   and   (d)   and  
2 2 2 2
Ans. : (d)
Solution: 125
51 Sb ; Z  51 and N  74
Z  51

 s1/ 2   p3/ 2   p1/ 2   d5 / 2   s1/ 2   d3/ 2   f 7 / 2   p3/ 2   f5 / 2   p1/ 2   g9 / 2   g7 / 2 


2 4 2 6 2 4 8 4 6 2 10 1


7 7
 j  and l  4 . Thus spin and parity   
2 2
89
38 Sr ; Z  38 and N  51
N  51:

 s1/ 2   p3/ 2   p1/ 2   d5 / 2   s1/ 2   d3/ 2   f 7 / 2   p3/ 2   f5 / 2   p1/ 2   g9 / 2   g 7 / 2 


2 4 2 6 2 4 8 4 6 2 10 1


7 7
 j and l  4 . Thus spin and parity   
2 2
Q4. The difference in the Coulomb energy between the mirror nuclei 49
24 Cr and 49
25 Mn is

6.0 MeV . Assuming that the nuclei have a spherically symmetric charge distribution and

that e2 is approximately 1.0 MeV-fm, the radius of the 49


25 Mn nucleus is

(a) 4.9  10-13 m (b) 4.9  10-15 m


(c) 5.1  10-13 m (d) 5.1  10-15 m
3  1  10 15
Ans. : (b) R
3e 2
5  W
Z1  Z 2  
2 2

5 6
25 2  24 2   4.9  10 15 m .

H.No. 40-D, Ground Floor, Jia Sarai, Near IIT, Hauz Khas, New Delhi-110016
Phone: 011-26865455/+91-9871145498
Website: www.physicsbyfiziks.com | Email: fiziks.physics@gmail.com
397
fiziks
Institute for NET/JRF, GATE, IIT‐JAM, M.Sc. Entrance, JEST, TIFR and GRE in Physics

NET/JRF (JUNE-2012)
1
Q5. The ground state of 12 Pb nucleus has spin-parity J  , while the first excited state
207 p

2
5
has J  p
.The electromagnetic radiation emitted when the nucleus makes a transition
2
from the first excited state to ground state are
(a) E2 and E3 (b) M2 or E3 (c) E2 or M3 (d) M2 or M3
Ans. : (c)
Solution: No parity change; J  2,3

For El type,    1 , (for no parity change l  2 )


l

For M l type,    1 , (for no parity change l  3 )


l 1

J  2 , No parity change  E 2 ; J  3 , No parity change  M 3


Q6. The dominant interactions underlying the following processes
A. K   p      , B.       K   K  , C.    p   0 are

(a) A: strong, B: electromagnetic and; C: weak


(b) A: strong, B: weak and; C: weak
(c) A: weak, B: electromagnetic and; C: strong
(d) A: weak, B: electromagnetic and; C: weak
Ans. : (a)
(A) K   p      (Strong interaction)

1 1
I 3 :    1  1 (Conserved)
2 2
(B)       K   K  (Electromagnetic interaction)

(C)    p   0 (Weak interaction)


1
I3 :1  0 (Not conserved)
2

H.No. 40-D, Ground Floor, Jia Sarai, Near IIT, Hauz Khas, New Delhi-110016
Phone: 011-26865455/+91-9871145498
Website: www.physicsbyfiziks.com | Email: fiziks.physics@gmail.com
398
fiziks
Institute for NET/JRF, GATE, IIT‐JAM, M.Sc. Entrance, JEST, TIFR and GRE in Physics

NET/JRF (JUNE-2013)
Q7. The binding energy of a light nucleus Z , A in MeV is given by the approximate formula

B A, Z   16 A  20 A 2/3 3
 Z 2 A 1 / 3  30
N  Z  2

4 A
where N  A  Z is the neutron number. The value of Z of the most stable isobar for a
given A is
1 1 1
A A2 / 3  A A A2 / 3  A A4 / 3 
(a) 1   (b) (c) 1   (d) 1  
2 160  2 2 120  2 64 
Ans. : (a)
1
B A A2 / 3 
Solution:  0  Z   1  
Z Z Z  2 160 

Q8. A spin-1/2 particle A undergoes the delay A  B  C  D , where it is known


that B and C are also spin-1/2 particles. The complete set of allowed values of the spin of
the particle D is
1 3 5 1 1 3 5 7
(a) ,1, , 2, , 3, ... (b) 0, 1 (c) only (d) , , , ,....
2 2 2 2 2 2 2 2
Ans. : (c)
Solution: Spin of the left side and combined spin of the products must be same to conserve the
spin angular momentum conservation law.
Q9. Muons are produced through the annihilation of particle a and its anti-particle, namely
the process a  a       . A muon has a rest mass of 105 MeV/c2 and its proper life
time is 2 s . If the center of mass energy of the collision is 2.1 GeV in the laboratory
frame that coincides with the center-of-mass frame, then the fraction of muons that will
decay before they reach a detector placed 6 km away from the interaction point is
(a) e 1 (b) 1  e 1 (c) 1  e 2 (d) e 10
Ans. : (b)
t
 t N  2.1
Solution: N  N 0 e   e t  e  , where   2  10 6 s ,    10 3  20 and
N0 105

6  10 3
1
5 t 1 N 
t  2  10 sec . Thus    e 2
 1  e 1 .
3  10 8
 2 N0

H.No. 40-D, Ground Floor, Jia Sarai, Near IIT, Hauz Khas, New Delhi-110016
Phone: 011-26865455/+91-9871145498
Website: www.physicsbyfiziks.com | Email: fiziks.physics@gmail.com
399
fiziks
Institute for NET/JRF, GATE, IIT‐JAM, M.Sc. Entrance, JEST, TIFR and GRE in Physics

NET/JRF (DEC-2013)
A
Q10. The intrinsic electric dipole moment of a nucleus Z X
(a) increases with Z , but independent of A
(b) decreases with Z , but independent of A
(c) is always zero
(d) increases with Z and A
Ans. : (d)
Q11. According to the shell model, the total angular momentum (in units of  ) and the parity
of the ground state of the 37 Li nucleus is

3 3
(a) with negative parity (b) with positive parity
2 2
1 7
(c) with positive parity (d) with negative parity
2 2
Ans. : (a)
Solution: Z  3, N  4

For odd Z  3; s12/ 2  p31 / 2   j  3 / 2, l  1 and parity   11  1 .

NET/JRF (JUNE-2014)
Q12. The recently-discovered Higgs boson at the LHC experiment has a decay mode into a
photon and a Z boson. If the rest masses of the Higgs and Z boson are 125 GeV/c 2 and

90 GeV/c 2 respectively, and the decaying Higgs particle is at rest, the energy of the
photon will approximately be
(a) 35 3 GeV (b) 35 GeV (c) 30 GeV (d) 15 GeV
Ans. : (c)
Solution: Assume H is symbol of Higgs boson, H  Z 

E H2  E Z2 1252  902
E    30GeV
2EH 2  125

H.No. 40-D, Ground Floor, Jia Sarai, Near IIT, Hauz Khas, New Delhi-110016
Phone: 011-26865455/+91-9871145498
Website: www.physicsbyfiziks.com | Email: fiziks.physics@gmail.com
400
fiziks
Institute for NET/JRF, GATE, IIT‐JAM, M.Sc. Entrance, JEST, TIFR and GRE in Physics

Q13. In a classical model, a scalar (spin-0) meson consists of a quark and an antiquark bound
b
by a potential V r   ar  , where a  200 MeV fm -1 and b  100 MeV fm . If the
r
masses of the quark and antiquark are negligible, the mass of the meson can be estimated
as approximately
(a) 141 MeV/c 2 (b) 283 MeV/c 2 (c) 353 MeV/c 2 (d) 425 MeV/c 2
Ans. : (b)
Solution: At equilibrium separation the potential is minimum, thus the equilibrium separation
can be determined as
dV  r  b b 100MeVfm 1
a  0  r0   1
 fm
dr r  r0
r02
a 200MeVfm 2

The equilibrium separation between particles is also estimated by uncertainty principle



r0  ct  r0  c ( where, Et  )
E
Where, c is the velocity of the virtual meson
 200 MeV . fm
r0  c 
E E MeV 
200MeV . fm 1
Using above two relation  fm
E MeV  2

E  200 2  283MeV  E  m  c 2
E
the mass of the meson m   283MeV / c 2
c 2

NET/JRF (DEC-2014)
Q14. Consider the four processes
(i) p   n  e   ve (ii) 0  p   e   v e

(iii)    e   ve (iv)  0    
which of the above is/are forbidden for free particles?
(a) only (ii) (b) (ii) and (iv) (c) (i) and (iv) (d) (i) and (ii)
Ans. : (d)

H.No. 40-D, Ground Floor, Jia Sarai, Near IIT, Hauz Khas, New Delhi-110016
Phone: 011-26865455/+91-9871145498
Website: www.physicsbyfiziks.com | Email: fiziks.physics@gmail.com
401
fiziks
Institute for NET/JRF, GATE, IIT‐JAM, M.Sc. Entrance, JEST, TIFR and GRE in Physics

Solution: (i) p   n  e   e [Not allowed]


It violate energy conservation. The mass of proton is less than mass of neutron. Free
proton is stable and can not decay to neutron. Proton can decay to neutron only inside the
nucleus, where energy violation is taken care by Heisenberg uncertainty principle.
(ii)  0  p   e   e [Not allowed]. In this decay charge is not conserved

(iii)    e   e [allowed through Weak interaction]

(iv)  0     [allowed through Electromagnetic interaction]


Q15. In deep inelastic scattering electrons are scattered off protons to determine if a proton has
any internal structure. The energy of the electron for this must be at least
(a) 1.25  10 9 eV (b) 1.25  1012 eV (c) 1.25  10 6 eV (d) 1.25  10 8 eV
Ans. : (b)
Solution: The internal structure of proton can only be determined if the wavelength of the
incoming electron is nearly equal to the size of the proton
i.e.   R  1.2 A1/ 3  fm   1.2 fm  1.2  1015 m

h h
According to de-Broglie relation,   
p 2mE

 0 150
This can be also written as    
  E  eV 

150 150
 E  eV     1.04  1012  E  1.04 1012 eV
1.2  10 
2 5 2
  0

    
 
The bet suitable answer is option (b).
Q16. If the binding energy B of a nucleus (mass number A and charge Z ) is given by

 2Z  A
2
aC Z 2
B  aV A  aS A 2/3
 asym  1/ 3
A A
where aV  16 MeV , a S  16 MeV , a sym  24 MeV and aC  0.75 MeV , then for the most

stable isobar for a nucleus with A  216 is


(a) 68 (b) 72 (c) 84 (d) 92
Ans. : (c)

H.No. 40-D, Ground Floor, Jia Sarai, Near IIT, Hauz Khas, New Delhi-110016
Phone: 011-26865455/+91-9871145498
Website: www.physicsbyfiziks.com | Email: fiziks.physics@gmail.com
402
fiziks
Institute for NET/JRF, GATE, IIT‐JAM, M.Sc. Entrance, JEST, TIFR and GRE in Physics

dB 2  2Z  A   2 2aC Z
Solution: For the most stable isobar for a nucleus  0  asym  1/ 3  0
dZ A A
2  2Z  216   2 2Z 4  2Z  216  3 2Z
 24  0.75 0   0
 216 
1/ 3
216 9 4 6

4  2Z  216  Z
   0  16  2 Z  216   9 Z  0  41Z  216  16  Z  82.3
9 4

NET/JRF (JUNE-2015)
Q17. The reaction 2
1 D 12 D 42 He   0 cannot proceed via strong interactions because it
violates the conservation of
(a) angular momentum (b) electric charge
(c) baryon number (d) isospin
Ans. (d)
Solution: 1 D 2  1 D 2  2 He 4   0 (Not conserved)
I: 0 0  0 1
This isopin is not conserved in above reaction.
Q18. Let us approximate the nuclear potential in the shell model by a three dimensional
isotropic harmonic oscillator. Since the lowest two energy levels have angular momenta
l  0 and l  1 respectively, which of the following two nuclei have magic numbers of
protons and neutrons?
(a) 42 He and 16
8 O (b) 12 D and 84 Be (c) 42 He and 84 Be (d) 42 He and 12
6 C
Ans. (a)
Solution: 2 He 4 has Z  2, N  2

and 8 O16 has Z  8, N  8 magic numbers  2,8, 20, 28,50,82,126 

Q19. The charm quark S assigned a charm quantum number C  1 . How should the
Gellmann-Nishijima formula for electric charge be modified for four flavors of quarks?
1 1
(a) I 3  B  S C (b) I 3  B  S  C
2 2
1 1
(c) I 3  B  S C (d) I 3  B  S  C
2 2

H.No. 40-D, Ground Floor, Jia Sarai, Near IIT, Hauz Khas, New Delhi-110016
Phone: 011-26865455/+91-9871145498
Website: www.physicsbyfiziks.com | Email: fiziks.physics@gmail.com
403
fiziks
Institute for NET/JRF, GATE, IIT‐JAM, M.Sc. Entrance, JEST, TIFR and GRE in Physics

Ans. (d)
1
Solution: From Gell-Mann-Nishijima formula Q  I 3  B  S
2
1
For Quark it is generalized as Q  I 3  B  S  C
2
NET/JRF (DEC-2015)
Q20. Consider the following processes involving free particles
(i) n  p  e   ve (ii) p  n   

(iii) p  n      0   0 (iv) p  ve  n  e 
Which of the following statements is true?
(a) Process (i) obeys all conservation laws
(b) Process (ii) conserves baryon number, but violates energy-momentum conservation
(c) process (iii) is not allowed by strong interaction but is allowed by weak interactions
(d) Process (iv) conserves baryon number, but violates lepton number conservation
Ans. : (b)
Solution: (i) n  p  e   ve

q 0 1 1 0 (conserved)
1 1 1 1
spin     (not conserved)
2 2 2 2
Le 0 0  1  1 (not conserved)
(ii) Baryon number is conserved but energy and momentum conservation violated.
(iii) spin is not conserved
(iv) obeys all conservation laws.
Q21. Of the nuclei of mass number A  125 , the binding energy calculated from the liquid
drop model (given that the coefficients for the Coulomb and the asymmetry energy are
ac  0.7 MeV and asym  22.5 MeV respectively) is a maximum for

(a) 125
54 Xe (b) 124
53 I (c) 125
52 Te (d) 125
51 Sb
Ans. : (c)

H.No. 40-D, Ground Floor, Jia Sarai, Near IIT, Hauz Khas, New Delhi-110016
Phone: 011-26865455/+91-9871145498
Website: www.physicsbyfiziks.com | Email: fiziks.physics@gmail.com
404
fiziks
Institute for NET/JRF, GATE, IIT‐JAM, M.Sc. Entrance, JEST, TIFR and GRE in Physics

4  22.5 125  0.7  53 


2/3
4aa  ac A1/ 3 4aa A  ac A2 / 3
Solution: Z 0    Z0 
2ac A1/ 3  8aa A1 8aa  2ac A2 / 3 8  22.5  2  0.7  53 
2/3

11250  17.5 11267.5


 Z0    52.4  Z 0  52
180  35 215
NET/JRF (JUNE-2016)
Q22. A radioactive element X decays to Y , which in turn decays to a stable element Z . The
decay constant from X to Y is 1 , and that from Y to Z is 2 . If, to begin with, there are

1 1
only N 0 atoms of X , at short times ( t  as well as ) the number of atoms of Z
1 2
will be
1 12
(a) 12 N 0t 2 (b) N 0t
2 2  1  2 

(c)  1  2  N 0t 2 (d)  1  2  N 0t
2

Ans. : (a)
1 2
X   Y  Z
Solution: t  0 N0 0 0
t N1 N2 N3
dN dN
Rate equations N1  N 0 e  1t , 2  1 N1  2 N 2 , 3  2 N 2
dt dt
  e 2t  e  1 t 
N 3  N 0 1  1  2 
  2  1   2  1  
 1  22t 2  2  12t 2  
 N 0 1   1   t     1   t  
  2  1   2   2  1  
2 1
2  
 1  t 1 22t 2 2 2 1t 2 12t 2 
 N 0 1   1 2     
  2  1   2  1   2  1  2  2  1   2  1   2  1  2 
 1  2t 2 2  2t 2    t 2   1  1
 N0   2   1   1 2 N0  2    12 N 0t
2

  2 1 
   2  2 1
   2  2  
 2 1 2  1 2

H.No. 40-D, Ground Floor, Jia Sarai, Near IIT, Hauz Khas, New Delhi-110016
Phone: 011-26865455/+91-9871145498
Website: www.physicsbyfiziks.com | Email: fiziks.physics@gmail.com
405
fiziks
Institute for NET/JRF, GATE, IIT‐JAM, M.Sc. Entrance, JEST, TIFR and GRE in Physics

Q23. In the large hadron collider  LHC  , two equal energy proton beams traverse in opposite

directions along a circular path of length 27 km . If the total centre of mass energy of a
proton-proton pair is 14 TeV , which of the following is the best approximation for the
proper time taken by a proton to traverse the entire path?
(a) 12 ns (b) 1.2  s (c) 1.2 ns (d) 0.12  s
Ans. : (a)
Solution: The proton travel at nearly speed of light in LHC , therefore
d 27 103
t   9 105 sec
c 3 108
v2 t
Since, proton is relativistic, t0  t 1  2 
c 
1 m0 c 2 938 MeV 938  106 eV
 E   m0 c 2      1.34 104
 E 7 TeV 7 1012 eV
t
Thus, t0   9  105  1.34 104  1.2 108 sec  12 ns

Q24. Let ES denotes the contribution of the surface energy per nucleon in the liquid drop

model. The ratio ES  27


13 
Al : ES  64
30 
Zn is

(a) 2 : 3 (b) 4 : 3 (c) 5 : 3 (d) 3 : 2


Ans. : (b)
2 1 1
E  Al   27  3  64  3 4
1 
B A3 
Solution: ES   A3 S   
A A ES  Z n  1 1
 64  3  27  3 3

Q25. According to the shell model, the nuclear magnetic moment of the 27
13 Al nucleus is (Given

that for a proton gl  1, g s  5.586 , and for a neutron gl  0, g s  3.826 )

(a) 1.913  N (b) 14.414  N (c) 4.793  N (d) 0


Ans. : (c)
5
Solution: 13 Al 27 : Z  13, N  14 for Z  13, S1/2 2 , P3/4 2 , P1/22 , d55/ 2  j  , l  2
2

 2 j  1  g S   N   2   1  5.586  N    4.793  N
1 1 5
Magnetic moment,  
2 2 2 

H.No. 40-D, Ground Floor, Jia Sarai, Near IIT, Hauz Khas, New Delhi-110016
Phone: 011-26865455/+91-9871145498
Website: www.physicsbyfiziks.com | Email: fiziks.physics@gmail.com
406
fiziks
Institute for NET/JRF, GATE, IIT‐JAM, M.Sc. Entrance, JEST, TIFR and GRE in Physics

NET/JRF (DEC-2016)
Q26. What should be the minimum energy of a photon for it to split an  -particle at rest into a
tritium and a proton?
(The masses of 4
2 He, 13 H and 1
1 H are 4.0026 amu,3.0161 amu and 1.0073 amu

respectively, and 1 amu  938 MeV )


(a) 32.2 MeV (b) 3MeV (c) 19.3 MeV (d) 931.5 MeV
Ans. : (c)
Solution: From conservation of energy
E  m c 2  m1H 3 c 2  m1H 1 c 2

or E   m1H 3  m1H 1  m   938 MeV  19.5 MeV

Q27. Which of the following reaction(s) is/are allowed by the conservation laws?
(i)    n   0  K 
(ii)    p   0  K 0
(a) both (i) and (ii) (b) only (i)
(c) only (ii) (d) neither (i) nor (ii)
Ans. : (a)
Solution: (i)    n   0  K 
q :1  0  0  1
B : 0 1  1 0
S : 0  0  1  1
Reaction is allowed
(ii)    p   0  K 0
q : 1  1  0  0
B : 0 1  1 0
S : 0  0  1  1
Reaction is allowed

H.No. 40-D, Ground Floor, Jia Sarai, Near IIT, Hauz Khas, New Delhi-110016
Phone: 011-26865455/+91-9871145498
Website: www.physicsbyfiziks.com | Email: fiziks.physics@gmail.com
407
fiziks
Institute for NET/JRF, GATE, IIT‐JAM, M.Sc. Entrance, JEST, TIFR and GRE in Physics

Q28. A particle, which is a composite state of three quarks u , d and s , has electric charge,
spin and strangeness respectively, equal to
1 1 1
(a) 1, , 1 (b) 0, 0, 1 (c) 0, , 1 (d) 1,   1
2 2 2
Ans. : (c)
Solution: charge, spin and strangers of Quarks u , d & s are given as
U D S Total
Charge 2 1 1 0
3 3 3
Spin 1 1 1 1 3
or
2 2 2 2 2
Strangeness 0 0 1 1
If a particle x is a composite of u, d & s , then net charge, spin and strangeness on x is
net charge  0
1 3
net spin  or and net strangeness  1
2 2
NET/JRF (JUNE-2017)
Q29. If in a spontaneous  - decay of 232
92 U at rest, the total energy released in the reaction is

Q , then the energy carried by the  - particle is


(a) 57Q / 58 (b) Q / 57 (c) Q / 58 (d) 23Q / 58
Ans. : (a)
Solution: Energy carried by the   particle is
 A4 228 57
KE   Q  Q Q
 A  232 58
Q30. The range of the nuclear force between two nucleons due to the exchange of pions is
1.40 fm . If the mass of pion is 140 MeV / c 2 and the mass of the rho-meson is

770 MeV / c 2 , then the range of the force due to exchange of rho-mesons is
(a) 1.40 fm (b) 7.70 fm (c) 0.25 fm (d) 0.18 fm
Ans. : (c)

H.No. 40-D, Ground Floor, Jia Sarai, Near IIT, Hauz Khas, New Delhi-110016
Phone: 011-26865455/+91-9871145498
Website: www.physicsbyfiziks.com | Email: fiziks.physics@gmail.com
408
fiziks
Institute for NET/JRF, GATE, IIT‐JAM, M.Sc. Entrance, JEST, TIFR and GRE in Physics

c
Solution: Range for nuclear force between nucleon will be R  ct  and c  199MeVfm
mc 2
199 MeVfm
R  0.25 fm
MeV 2
770 2  c
c
Q31. A baryon X decays by strong interaction as X         0 , where   is a member
of the isotriplet    ,  0 ,    . The third component I 3 of the isospin of X is

(a) 0 (b) 1/ 2 (c) 1 (d) 3 / 2


Ans. : (a)
Solution: X        0

I 3 :1
1 0

 I 3 for X is 0 .

NET/JRF (DEC-2017)
Q32. 28 Ni , in
The spin-parity assignments for the ground and first excited states of the isotope 57
the single particle shell model, are
   
1 3 5 7
(a)   and   (b)   and  
2 2 2 2
   
3 5 3 5
(c)   and   (d)   and  
2 2 2 2
Ans. : (d)
Solution: Spin parity for 28 Ni 57 for ground state and first excited state

For 28 Ni 57 : P  28 , N  29  will decide the j P

So, for N  29 , ground state configuration,


1s1/2 21 p3/4 21 p1/2 21d56/ 2 2s1/2 21d3/4 21 f 78/ 2 2 p3/1 2

3
So, j  , l  1
2

3
Spin parity for ground state of 28 Ni
57
 
2
For first excited state,
H.No. 40-D, Ground Floor, Jia Sarai, Near IIT, Hauz Khas, New Delhi-110016
Phone: 011-26865455/+91-9871145498
Website: www.physicsbyfiziks.com | Email: fiziks.physics@gmail.com
409
fiziks
Institute for NET/JRF, GATE, IIT‐JAM, M.Sc. Entrance, JEST, TIFR and GRE in Physics

1s1/2 21 p3/4 21 p1/2 21d56/ 2 2s1/2 21d 3/4 21 f 78/ 2 2 p3/1 2  1 f 5 / 2



5 5
P  , l  3  spin parity   
2 2
Q33. The first excited state of the rotational spectrum of the nucleus 238
92 U has an energy

45 keV above the ground state. The energy of the second excited state (in keV) is
(a) 150 (b) 120 (c) 90 (d) 60
Solution: As per the shell model (Collective Model)
Rotational Energies,
2
Er  J  J  1 , I  is moment of inertia where only even value of J are allowed
2I
i.e., J  0 , 2 , 4 , 6 ,........
Now, for ground state J  0 , E  0 keV

For first excited stat, J  2 , E  45keV (given)


2  2 45
So, 45 keV   2  3 or,  keV (i)
2I 2I 6
Now, for second excited state, J  4
2 2
E2   4  5 (put value of from (i))
2I 2I
45 900
or, E2   20   150 keV .
6 6
Q34. Which of the following process is not allowed by the strong interaction but is allowed by
the weak interaction?
(a) K 0   0  K 0       (b) p  n  d  p  p

(c)    K 0  p  n (d) p     n   
Ans. : (a)
Solution: (1) K0  0  K0    
Charge 0 0 0 1 1 Conserved
Spin 0 0 0 0 0 Conserved
1 1
I 1 1 1 Not conserved
2 2

H.No. 40-D, Ground Floor, Jia Sarai, Near IIT, Hauz Khas, New Delhi-110016
Phone: 011-26865455/+91-9871145498
Website: www.physicsbyfiziks.com | Email: fiziks.physics@gmail.com
410
fiziks
Institute for NET/JRF, GATE, IIT‐JAM, M.Sc. Entrance, JEST, TIFR and GRE in Physics

1 1
I3 0  1 1  I3  1
2 2
S 1 0 1 0 0 S  1
This interaction is not allowed by strong interaction but allowed by weak interaction.

NET/JRF (JUNE-2018)
Q35. The reaction 63
Cu29  p  63
Zn30  n is followed by a prompt   decay of zinc
63
Zn30  63Cu29  e   ve . If the maximum energy of the position is 2.4 MeV , the Q -

value of the original reaction in MeV is nearest to


[Take the masses of electron, proton and neutron to be 0.5 MeV / c 2 ,938 MeV / c 2 and

939.5 MeV / c 2 ,respectively.]


(a) 4.4 (b) 2.4 (c) 4.8 (d) 3.4
Ans. : (a)
Solution: For 63
Zn30 63 Cu29  e   e

Q1   Zn  30e   Cu  29e  e   Zn  Cu  2e  2.4 MeV

For 63
Cu29  p  63
Zn30  n

Q0   Cu  29e   p    Zn  30e   n 

 Cu  Zn  e  p  n   Q1  2e   e  p  n  Q1  e  p  n 

 2.4   0.5  938  939.5   4.4. MeV

Q36. A deuteron d captures a charged pion   in the l  1 state, and subsequently decays into
a pair of neutrons  n  via strong interaction. Given that the intrinsic parities of   , d and

n are 1, 1 and 1 respectively, the spin wavefunction of the final state neutrons is
(a) linear combination of a singlet and a triplet
(b) singlet
(c) triplet
(d) doublet
Ans. : (b)

H.No. 40-D, Ground Floor, Jia Sarai, Near IIT, Hauz Khas, New Delhi-110016
Phone: 011-26865455/+91-9871145498
Website: www.physicsbyfiziks.com | Email: fiziks.physics@gmail.com
411
fiziks
Institute for NET/JRF, GATE, IIT‐JAM, M.Sc. Entrance, JEST, TIFR and GRE in Physics

Solution: Parity must conserve intersections


 d nn
The parity of the initial state is
1
 1 P Pd   1  1 1  1
l

The parity of the final state is

 1 Pn Pn   1  1 1   1  1


l l l
l  0, 2,....

because the nucleons are identical fermions, the allowed states of two nucleons are
1
S0 , 3 P0,1,2 corresponding to l  0 and l  1 . Thus only l  0 (singlet) is allowed.

Q37. Which of the following elementary particle processes does not conserve strangeness?
(a)  0  p  k    0 (b)    p  k 0   0

(c)  0   0  n (d) K 0      
Ans. : (d)
Solution: (a)
 0  p  k   0
Conserved
S: 0 0 1 1
(b)
  p  k 0  0
Conserved
S: 0 0 1 1
(c)
0  0 n
Conserved
S: 0 0 0
(d)
K0     
Not conserved
S : 1 0 0

H.No. 40-D, Ground Floor, Jia Sarai, Near IIT, Hauz Khas, New Delhi-110016
Phone: 011-26865455/+91-9871145498
Website: www.physicsbyfiziks.com | Email: fiziks.physics@gmail.com
412
fiziks
Institute for NET/JRF, GATE, IIT‐JAM, M.Sc. Entrance, JEST, TIFR and GRE in Physics

NET/JRF (DEC-2018)
Q38. Assume that pion-nucleon scattering at low energies, in which isospin is conserved is
 
described by the effective interaction potential Veff  F  r  I .I N , where F  r  is a
 
function of the radial separation r and I and I N denote, respectively, the isospin

 I 3/ 2
vectors of a pion and the nucleon. The ratio of the scattering cross-sections
 I 1/ 2
3 1
corresponding to total isospins I  and is
2 2
3 1 5 1
(a) (b) (c) (d)
2 4 4 2
Ans. : None of the options is matched.
Solution: The isospin of pion is I  1

1
The isospin of nucleon is I N 
2
3 1
 Total isospin is I  ,
2 2
There are three different  - mesons
1,1  1,   , 1, 0   0 , 1, 1   

and two nucleons, a proton and a neutron


1 1 1 1
,  P , ,  n
2 2 2 2
3
we can write the states corresponding I 
2
3 3 1 1
,  1,1 ,    p
2 2 2 2

3 1 2 1 1 1 1 1 2 0 1 
,  1, 0 ,  1,1 ,    p   n
2 2 3 2 2 3 2 2 3 3

3 1 1 1 1 2 1 1 1  2 0
,  1, 1 ,  1, 0 ,    p   n
2 2 3 2 2 3 2 2 3 3

H.No. 40-D, Ground Floor, Jia Sarai, Near IIT, Hauz Khas, New Delhi-110016
Phone: 011-26865455/+91-9871145498
Website: www.physicsbyfiziks.com | Email: fiziks.physics@gmail.com
413
fiziks
Institute for NET/JRF, GATE, IIT‐JAM, M.Sc. Entrance, JEST, TIFR and GRE in Physics

3 3 1 1
,  1, 1 ,    n
2 2 2 2
2 2
 2  1   1   2
2 2

  I 3 / 2  1             1
2 2

 3  3  3  3
2 1 1 2
 1    1  4
3 3 3 3
1
The states corresponding to I  are
3
1 1 2 1 1 1 1 1 2  1 0
,  1,1 ,   1, 0 ,   n   p
2 2 3 2 2 3 2 2 3 3

1 1 1 1 1 2 1 1 1 0 2 
,  1, 0 ,   1, 1 ,   n   p
2 2 3 2 2 3 2 2 3 3
2 2
 2  1   1   1   2
2 2 2

  I 1/ 2             
 3  3   3   3   3 

2 1 1 2
    2
3 3 3 3
 I 3 / 2 4 2
Thus  
 I 1/ 2 2 1
The best possible answer is option (d)
Q39. A nucleus decays by the emission of a gamma ray from an excited state of spin parity 2
to the ground state with spin-parity 0 what is the type of the corresponding radiation?
(a) magnetic dipole (b) electric quadrupole
(c) electric dipole (d) magnetic quadrupole
Ans. : (b)
Solution: I i  2 , I  0

 L  2 and parity change


 The transition is of electric quadrupole  E2  nature.

H.No. 40-D, Ground Floor, Jia Sarai, Near IIT, Hauz Khas, New Delhi-110016
Phone: 011-26865455/+91-9871145498
Website: www.physicsbyfiziks.com | Email: fiziks.physics@gmail.com
414
fiziks
Institute for NET/JRF, GATE, IIT‐JAM, M.Sc. Entrance, JEST, TIFR and GRE in Physics

Q40. The low lying energy levels due to the vibrational excitations of an even-even nucleus are
shown in the figure below.
0 E4  1.35 MeV
2 E3  1.25 MeV
4 E2  1.17 MeV
 2E

jp E1  0.56 MeV

E

0 E0  0 MeV
The spin-parity j p of the level E1 is

(a) 1 (b) 1 (c) 2 (d) 2


Ans. : (d)
Solution: Quadrupole oscillations are the lowest order nuclear vibrational mode. The quanta of
vibrational energy are called phonons. A quadrupole phonon carries 2 units of angular

momentum. Therefore, the parity is P   1   ve


2

Also, the even-even ground state is O  . The 1 phonon excited state is 2 . The 2 phonons
excited states are 0 , 2 , 4 . Thus correct option is (a)

1.35 ___ 0 

1.25 ____ 2  2 -phonons
1.17 ____ 4 

0.56 ____ 2 :1 -phonon

0 ____ 0 : Ground state


meV

H.No. 40-D, Ground Floor, Jia Sarai, Near IIT, Hauz Khas, New Delhi-110016
Phone: 011-26865455/+91-9871145498
Website: www.physicsbyfiziks.com | Email: fiziks.physics@gmail.com
415

You might also like